FAR Study Set

Ace your homework & exams now with Quizwiz!

The SEC's rulemaking procedures identified on their website include which of the following steps? A. Issue identification B. Commission deliberation C. Rule adoption D. None of the answer choices are correct.

C. Rule adoption Following steps in the website: Concept Release Rule Proposal Rule Adoption

How should the acquirer recognize a bargain purchase in a business acquisition? A. As a gain in earnings at the acquisition date B. As goodwill in the statement of financial position C. As negative goodwill in the statement of financial position D. As a deferred gain that is amortized into earnings over the estimated future periods benefited

A. As a gain in earnings at the acquisition date In a business acquisition, the acquiring corporation must recognize the assets and liabilities acquired at fair value. If the net assets acquired exceed the purchase price—a bargain purchase—the excess must be recognized as a gain in earnings at the date of the acquisition.

How should plan investments be reported in a defined benefit plan's financial statements? A. At fair value B. At actuarial present value C. At cost D. At net realizable value

A. At fair value

Within the context of the qualitative characteristics of accounting information, which of the following is a fundamental qualitative characteristic? A. Relevance B. Timeliness C. Comparability D. Feedback value

A. Relevance The fundamental qualitative characteristics of accounting information are relevance and faithful representation.

On the budgetary comparison schedule, a variance should be calculated between the final budget and: A. actual expenditures plus outstanding encumbrances. B. the original budget expressed on a GAAP basis. C. actual appropriations. D. the original budget.

A. actual expenditures plus outstanding encumbrances. "Actual expenditures plus outstanding encumbrances" is the correct answer. The goal of the variance column is to compare appropriations against both expenditures and encumbrances; in essence, to account for not only spending activities, but commitments made to purchase goods or services in the current period.

When the allowance method of recognizing uncollectible accounts is used, the entry to record the write-off of a specific account: A. decreases both accounts receivable and the allowance for uncollectible accounts. B. decreases accounts receivable and increases the allowance for uncollectible accounts. C. decreases both accounts receivable and net income. D. increases the allowance for uncollectible accounts and decreases net income.

A. decreases both accounts receivable and the allowance for uncollectible accounts.

The orientation of accounting and reporting for all proprietary funds of governmental units is: A. income determination. B. program. C. flow of funds. D. project.

A. income determination.

The diluting effect of options and warrants and their equivalents is reflected in diluted EPS by application of the treasury stock method, which assumes that proceeds from exercise are used to: A. purchase common stock at the average market price. B. retire treasury stock. C. issue treasury stock. D. retire convertible debentures that were issued at par.

A. purchase common stock at the average market price.

Statement of Financial Accounting Concepts 8 (SFAC 8), Chapter 8, lists all of the following examples of past events and current conditions potentially impacting the entity's future line items and cash flows except for: A. related party reporting. B. dependency on a few customers or suppliers. C. suspected regulatory violations. D. existing or potential litigation.

A. related party reporting.

The market price of a bond issued at a discount is equal to: A. the present value of its principal amount plus the present value of all future interest payments, using the market (effective) interest rate. B. the present value of its principal amount minus the present value of all future interest payments, using the coupon (stated) interest rate. C. the present value of its principal amount minus the present value of all future interest payments, using the market (effective) interest rate. D. the present value of its principal amount plus the present value of all future interest payments, using the coupon (stated) interest rate.

A. the present value of its principal amount plus the present value of all future interest payments, using the market (effective) interest rate.

A company that is a large accelerated filer must file its Form 10-Q with the U.S. Securities and Exchange Commission within how many days after the end of the period? A. 30 days C. 40 days C. 45 days D. 60 days

C. 40 days

The governmental fund measurement focus is on the determination of: A. income and flow of financial resources. B. income, economic resources, and flow of financial resources. C. flow of financial resources. D. economic resources.

C. flow of financial resources. Financial statements for governmental funds should be prepared and presented utilizing the flow of current financial resources measurement focus. This hybrid basis of accounting is called the modified accrual basis of accounting.

On January 1, Feld traded a delivery truck and paid $10,000 cash for a tow truck owned by Baker. The delivery truck had an original cost of $140,000, accumulated depreciation of $80,000, and an estimated fair value of $90,000. Feld estimated the fair value of Baker's tow truck to be $100,000. The transaction had commercial substance. What amount of gain should be recognized by Feld? A. $0 B. $3,000 C. $10,000 D. $30,000

D. $30,000 90000FV-60000CV=30000

Last year, Katt Co. reduced the carrying amount of its long-lived assets used in operations from $120,000 to $100,000, in connection with its annual impairment review. During the current year, Katt determined that the fair value of the same assets had increased to $130,000. What amount should Katt record as restoration of previously recognized impairment loss in the current year's financial statements? A. $10,000 B. $20,000 C. $30,000 D. $0

D. $0 After an impairment loss is recognized, the reduced carrying amount of the asset should be treated as the new cost and the restoration of the impairment is not recognized. IFRS (International Financial Reporting Standards) will allow the restoration of an impairment loss, but U.S. GAAP will not allow the restoration.

Universe Co. issued 500,000 shares of common stock in the current year. Universe declared a 30% stock dividend. The market value was $50 per share, the par value was $10, and the average issue price was $30 per share. By what amount will Universe decrease stockholders' equity for the dividend? A. $1,500,000 D. $7,500,000 C. $4,500,000 D. $0

D. $0 Stock dividends are accounted for by reclassifying a portion of retained earnings as contributed capital. They do not reduce assets or increase liabilities. Therefore, total stockholders' equity is not changed.

On January 1, year 1, a company grants 5,000 nonqualified stock options to an employee with a strike price of $3 per option and fair value of $8 per option. All of the options vest at the end of 5 years from the grant date. At the end of year 1, the company's stock price was $10 per share. What amount of annual stock compensation cost should the company report for year 1? A. $0 B. $5,000 C. $3,000 D. $8,000

D. $8,000 8*5000=40000 40000/5=8000

Which of the following assets or transactions is an element of comprehensive income? A. Investments by owners B. Distributions to owners C. Deferred revenue D. Sales revenue

D. Sales revenue Doesn't include sales revenue but would not include investments or distributions by owners or deferred revenue.

For a public business entity, the goodwill impairment test is required to be performed: A. only at the end of the fiscal year. B. only at the beginning of the fiscal year. C. any time during the last quarter of the fiscal year. D. any time during the fiscal year, provided that it is performed at the same time every year.

D. any time during the fiscal year, provided that it is performed at the same time every year.

Based on a physical inventory taken on December 31, 20X1, Chewy Co. determined its chocolate inventory on a FIFO basis at $26,000 with a replacement cost of $20,000. Chewy estimated that, after further processing costs of $12,000, the chocolate could be sold as finished candy bars for $40,000. Chewy's normal profit margin is 10% of sales. At what amount should Chewy report its chocolate inventory on its December 31, 20X1, balance sheet? A. $26,000 B. $28,000 C. $20,000 D. $24,000

Inventory measured using any method other than LIFO or the retail inventory method (e.g., FIFO or average cost) is measured at the lower of cost and net realizable value (NRV), which is defined to be the estimated selling price in the ordinary course of business, less reasonably predictable costs of completion, disposal, and transportation. If the NRV of inventory is lower than its cost, the difference is recognized as a loss in earnings in the period in which it occurs. Chewy should report chocolate inventory on December 31, 20X1 at cost of $26,000, which is lower than the NRV of $28,000 (selling price of $40,000 less processing costs of $12,000).

At the end of Year 1, Lane Co. held equity securities that cost $86,000 and which had a year-end market value of $92,000. During Year 2, all of these securities were sold for $104,500. At the end of Year 2, Lane had acquired additional equity securities that cost $73,000 and which had a year-end market value of $71,000. What is the net impact of these stock activities on Lane's Year 2 income statement? A. Net loss of $2,000 B. Net gain of $10,500 C. Net gain of $18,500 D. Net gain of $16,500

Investments in equity securities are recognized on the balance sheet at fair value. Unrealized holding gains and losses are included in earnings. Realized gains and losses are also included in earnings. Gain on sale of the first securities was$104,500 − $92,000 $12,500Unrealized loss of the second securities was $73,000 − $71,000 (2,000)Net gain on income statement $10,500

Jen Co. had 200,000 shares of common stock and 20,000 shares of 10%, $100 par value cumulative preferred stock. No dividends on common stock were declared during the year. Net income was $2,000,000. What was Jen's basic earnings per share? A. $9.00 B. $9.09 C. $10.00 D. $11.11

A. $9.00 EPS = (NI - Preferred Stock Dividends) / Weighted Average Common Shares EPS = ($2,000,000 - (20,000*100*10%)) / 200,000 EPS = ($2,000,000 - 200,000) / 200,000 EPS = $9.00

Redwood Co.'s financial statements had the following information at year-end: Cash $ 60,000 Accounts receivable 180,000 Allowance for uncollectible accounts 8,000 Inventory 240,000 Trading debt securities 90,000 Prepaid rent 18,000 Current liabilities 400,000 Long-term debt 220,000 What was Redwood's quick ratio? A. 0.81 to 1 B. 0.83 to 1 C. 0.94 to 1 D. 1.46 to 1

A. 0.81 to 1 Quick ratio = current assets (excluding inventories and prepaid assets) / current liabilities QR = (60,000 + (180,000 - 8,000) + 90,000) / 400,000 = .81 Account receivable is netted

On July 1, Year 1, Kay Corp. sold equipment to Mando Co. for $100,000. Kay accepted a 10% note receivable for the entire sales price. This note is payable in two equal installments of $50,000 plus accrued interest on December 31, Year 1 and Year 2. On July 1, Year 2, Kay discounted the note at a bank at an interest rate of 12%. Kay's proceeds from the discounted note were: $50,350. $49,350. $48,400. $51,887.

$51,887. When accounting for a discounted note and computing the cash proceeds, one must first find the maturity value of the note, what will be received by the holder of the note when it comes due. By the time of the discounting, some of the principal has already been paid. At the end of December, Year 2, the $50,000 will be received by the bank along with 10% interest (since the principal will have been outstanding for a whole year). Only the second installment, the final $50,000 principal plus interest of $5,000 ($50,000 outstanding balance × 0.10), will be paid to the bank when due, for a total payment at maturity of $55,000. The discounted proceeds will be based on this amount, the discount rate (0.12 × 6/12) over the discounting period (from July to December of Year 2, 6 months). The discount amount is thus: PV (Present value) = Maturity value/(1 + r)n PV = $55,000/(1 + .06)1 PV = $51,887 (rounded)

On May 18, 20X1, Sol Corp.'s board of directors declared a 10% dividend. The market price of Sol's 3,000 outstanding shares of $2 par value common stock was $9 per share on that date. The stock dividend was distributed on July 21, 20X1, when the stock's market price was $10 per share. What amount should Sol credit to additional paid-in capital for this stock dividend? A. $2,100 B. $2,400 C. $3,000 D. $2,700

A. $2,100 Consider the journal entry to record the declaration of the dividend: Dr. Cr.Retained earnings (3,000 x .10 x $9) 2,700Common stock dividenddistributable (3,000 x .10 x $2) 600Additional paid-in capital ($2,700 - $600) 2,100

Falton Co. had the following first-year amounts related to its $9,000,000 construction contract: Actual costs incurred and paid $2,000,000Estimated additional costs to complete 6,000,000Progress billings 1,800,000Cash collected 1,500,000 What amount should Falton recognize as a current liability at year-end, if revenue is recognized over time? A. $0 B. $200,000 C. $250,000 D. $300,000

A. $0 At year-end, Falton should record an account receivable of $300,000 ($1,800,000 - $1,500,000) and costs and profits in excess of billings of $450,000 ($2,250,000 - $1,800,000). However, no current liability exists because costs and profits exceed billings. Journal entries: Construction in Progress 2,250,000*Cash and profit 2,250,000Accounts Receivable 1,800,000Progress Billings 1,800,000Cash 1,500,000Accounts Receivable 1,500,000 * Includes $2,000,000 of costs incurred and $250,000 of gross profit (($2,000,000 ÷ $8,000,000) × $9,000,000).

During 20X1, Orca Corp. decided to change from the FIFO method of inventory valuation to the weighted-average method. Inventory balances under each method were as follows: FIFO Weighted-Average January 1, 20X1 $71,000 $77,000 December 31, 20X1 79,000 83,000 Orca's income tax rate is 30%. In its 20X1 financial statements, what amount should Orca report as the cumulative effect of this accounting change to be included in 20X1 net income? A. $0 B. $2,800 C. $6,000 D. $4,000

A. $0 FASB ASC 250-10-45-5 mandates that voluntary changes in accounting principle be recognized using the retrospective approach, in which the cumulative effect is reported as an adjustment of the beginning-of-year retained earnings of the earliest year presented. The only exception is when the FASB issues a new pronouncement and mandates in that pronouncement that a change in accounting principle made to comply with that pronouncement should be made by including the cumulative effect in net income of the year of change.

Oak County incurred the following expenditures in issuing long-term bonds: Issue cost $400,000Debt insurance 90,000 When Oak establishes the accounting for operating debt service, what amount should be deferred and amortized over the life of the bonds? A. $0 B. $490,000 C. $90,000 D. $400,000

A. $0 The GASB evaluated these debt issuance costs and concluded that, with the exception of prepaid insurance, the costs relate to services provided in the current period and thus they should be expensed in the current period.

Baker Co. issued 100,000 shares of common stock in the current year. On October 1, Baker repurchased 20,000 shares of its common stock on the open market for $50 per share. At that date, the stock's par value was $1 and the average issue price was $40 per share. Baker uses the cost method for treasury stock transactions. On December 1, Baker reissued the stock for $60 per share. What amount should Baker report as treasury stock gain at December 31? A. $0 B. $200,000 C. $980,000 D. $400,000

A. $0 Treasury stock transactions are equity transactions and result in no gain or loss. Treasury stock transactions affect additional paid-in capital accounts and retained earnings.

Arc Hospital received an unconditional pledge for $1 million, which will be paid in four installments of $250,000 over four years. What amount of pledge revenue should be recognized in the second year? A. $0 B. $500,000 C. $1,000,000 D. $250,000

A. $0 Unconditional promises are reported in the period in which they are promised or received, whichever is earlier. The full $1 million was promised in the first year, making the full $1 million pledge revenue at that time. It does not matter that the money will be received annually in installments.

Willem Co. reported the following liabilities at December 31, Year 1: Accounts payable trade $ 750,000Short-term borrowings 400,000Mortgage payable, current portion $100,000 3,500,000Other bank loan, matures June 30, Year 2 1,000,000 The $1,000,000 bank loan was refinanced with a 20-year loan on January 15, Year 2, with the first principal payment due January 15, Year 3. Willem's audited financial statements were issued February 28, Year 2. What amount should Willem report as current liabilities at December 31, Year 1? A. $1,250,000 B. $850,000 C. $2,250,000 D. $1,150,000

A. $1,250,000 The refinanced loan is not included in current liabilities. FASB ASC 470-10-45-13 and 45-14, "Short-Term Obligations Expected to Be Refinanced," addresses this refinanced loan: "Short-term obligations arising from transactions in the normal course of business that are due in customary terms shall be classified as current liabilities. A short-term obligation shall be excluded from current liabilities only if the conditions in the following paragraph are met. Funds obtained on a long-term basis before the balance sheet date would be excluded from current assets if the obligation to be liquidated is excluded from current liabilities. "A short-term obligation shall be excluded from current liabilities if the entity intends to refinance the obligation on a long-term basis (see [FASB ASC] 470-10-45-12B) and the intent to refinance the short-term obligation on a long-term basis is supported by an ability to consummate the refinancing demonstrated in either of the following ways: "Post-balance-sheet-date issuance of a long-term obligation or equity securities. After the date of an entity's balance sheet but before that balance sheet is issued or is available to be issued (as discussed in [FASB ASC] 855-10-25), a long-term obligation or equity securities have been issued for the purpose of refinancing the short-term obligation on a long-term basis. If equity securities have been issued, the short-term obligation, although excluded from current liabilities, shall not be included in owners' equity.

During Year 2, a former employee of Dane Co. began a suit against Dane for wrongful termination in November of Year 1. After considering all of the facts, Dane's legal counsel believes that the former employee will prevail and will probably receive damages of between $1,000,000 and $1,500,000, with $1,300,000 being the most likely amount. Dane's financial statements for the year ended December 31, Year 2, will not be issued until February of Year 3. In its December 31, Year 2, balance sheet, what amount should Dane report as a liability with respect to the suit? A. $1,300,000 B. $1,000,000 C. $0 D. $1,500,000

A. $1,300,000 A loss contingency must be accrued if it is probable that the loss will occur and the amount can be reasonably estimated. If a range of loss can be estimated, the best estimate within the range is accrued. Since legal counsel believes that $1,300,000 is the most likely amount to be collected, Dane should report this amount as a liability.

On September 22, 20X1, Yumi Corp. purchased merchandise from an unaffiliated foreign company for 10,000 units of the foreign company's local currency. On that date, the spot rate was $.55. Yumi paid the bill in full on March 20, 20X2, when the spot rate was $.65. The spot rate was $.70 on December 31, 20X1. What amount should Yumi report as a foreign currency transaction loss in its income statement for the year ended December 31, 20X1? A. $1,500 B. $0 C. $500 D. $1,000

A. $1,500 A foreign currency transaction loss occurred because it cost more to purchase the units of foreign currency on December 31 ($.70) than it cost when the transaction originated on September 22 ($.55). The amount of loss would be computed as follows: Transaction loss = Number of units x Change in rate= 10,000 x ($.70 - $.55)= 10,000 x $.15= $1,500

Charles Corporation acquires all of the outstanding stock of Harry Corporation. After the acquisition, the stock of Harry Corporation is retired and the Harry Corporation ceases to exist. This is an example of a: A. statutory merger. B. goodwill acquisition. C. statutory consolidation. D. None of the answer choices are correct.

A. statutory merger. A statutory merger occurs when one entity acquires another entity and the second entity ceases to exist. The acquiring entity remains in existence and continues the operations of the combined businesses.

Pine Co. purchased land for $450,000 as a factory site. An existing building on the site was razed before construction began. Additional information is as follows: Cost of razing old building $ 60,000Title insurance and legal fees to purchase land 30,000Architect's fees 95,000New building construction cost 1,850,000 What amount should Pine capitalize as the cost of the completed factory building? A. $1,945,000 B. $1,975,000 C. $1,910,000 D. $2,005,000

A. $1,945,000 When purchasing land intended for the addition of a new building that contains existing infrastructure, the cost of the land and new building must be allocated accordingly. The cost of purchasing the land, razing the old building, and title insurance and legal fees would all be allocated to the land account and not depreciated because land has an indefinite life. The architect's fee to design the building and the new building construction would be capitalized to the building account in the amount of $1,945,000 ($95,000 + $1,850,000) and depreciated over the life of the new building.

Kaplan Parish has a number of outstanding bond issues that include a $6,700,000 general obligation bond that financed a parish Justice Center, a $3,200,000 revenue bond that financed upgrades to the sewer treatment system, a $874,000 special assessment bond for sidewalks, and a $2,900,000 general obligation bond used for streets and roads. Revenues of the sewer fund, a proprietary fund, are expected to pay off the revenue bond. What should Kaplan Parish report as long-term liabilities in the governmental activities column of the government-wide statement of net position? A. $10,474,000 B. $9,800,000 C. $7,574,000 D. $12,800,000

A. $10,474,000 The amount to be reported as long-term liabilities in the governmental activities column of the government-wide statement of net position is $10,474,000 ($6,700,000 + $2,900,000 + $874,000). The revenue bonds of $3,200,000 should be shown as a fund liability.

A government contributed $100,000 to its defined benefit pension plan during the year and had no current amounts due to the plan at year-end. During the year, the net pension liability related to the plan increased by $200,000 and the total pension liability increased by $350,000. What amount of pension expenditure would the government recognize for the year? A. $100,000 B. $350,000 C. $0 D. $200,000

A. $100,000 While a government's full accrual financial statements recognize pension expense primarily based upon changes in the pension liability during the year, the modified accrual financial statements recognize pension expenditures primarily based upon a funding approach. Therefore, the government would recognize the full $100,000 contribution as pension expense for the year.

Taft Corp. uses the equity method to account for its 25% investment in Flame, Inc. During 20X1, Taft received dividends of $30,000 from Flame and recorded $180,000 as its equity in the earnings of Flame. Additional information follows: All the undistributed earnings of Flame will be distributed as dividends in future periods. The dividends received from Flame are eligible for the 65% dividends-received deduction. There are no other temporary differences. Enacted income tax rates are 21% for 20X1 and thereafter. In its December 31, 20X1, balance sheet, what amount should Taft report for deferred income tax liability? A. $11,025 B. $19,500 C. $31,500 D. $29,295

A. $11,025 The recognized but currently unreceived income will generate a future tax consequence, a future tax liability. However, dividends received by a corporation are eligible for a dividends-received deduction (a permanent difference), and thus only a smaller taxable amount (multiplied by the tax rate) will eventually be paid because the permanent difference lowers taxable income.

On January 1, 20X1, Jamin Co. had a credit balance of $260,000 in its allowance for uncollectible accounts. Based on past experience, 2% of Jamin's credit sales has been uncollectible. During 20X1, Jamin wrote off $325,000 of uncollectible accounts. Credit sales for 20X1 were $9,000,000. On its December 31, 20X1, balance sheet, what amount should Jamin report as allowance for uncollectible accounts? A. $115,000 B. $245,000 C. $180,000 D. $440,000

A. $115,000

A city government reported a $9,000 increase in net position in the motor pool internal service fund, a $12,000 increase in net position in the water enterprise fund, and a $7,000 increase in the employee pension fund. The motor pool internal service fund provides service primarily to the police department. What amount should the city report as the change in net position for business-type activities in its statement of activities? A. $12,000 B. $9,000 C. $28,000 D. $21,000

A. $12,000 Only the water enterprise fund is a business-type activity which provides services to the general public for a fee; the pension fund and the internal service fund are not business-type activities. The city should report $12,000 as the change in net position.

Asp Co. was organized on January 2, 20X1, with 30,000 authorized shares of $10 par common stock. During 20X1 the corporation had the following capital transactions: January 5: issued 20,000 shares at $15 per share. July 14: purchased 5,000 shares at $17 per share. December 27: reissued the 5,000 shares held in treasury at $20 per share. Asp used the par value method to record the purchase and re-issuance of the treasury shares. In its December 31, 20X1, balance sheet, what amount should Asp report as additional paid-in capital in excess of par? A. $125,000 B. $150,000 C. $140,000 D. $100,000

A. $125,000 The additional $2 (17-15) spent on the repurchase would be debited to retained earnings, the rest would fall under common stock or the APIC account

On January 2, 20X1, Randall Co. purchased a long-lived asset. The purchase price of the asset was $250,000, with no salvage value. The estimated useful life of the asset was 10 years. Randall used the straight-line method to calculate depreciation expense. An impairment loss on the asset of $30,000 was recognized on December 31, 20X4. The estimated useful life of the asset at December 31, 20X4, did not change. What amount should Randall report as depreciation expense in its income statement for the next year, 20X5? A. $20,000 B. $22,000 C. $25,000 D. $30,000

A. $20,000 Depreciation should be modified when an asset has been determined to be impaired. 250000-100000(AD)=150000 150000-30000=120000 120000/6=20000

A government's assets include inventory of $2 million, roads constructed for $25 million with accumulated depreciation of $10 million, and equipment acquired for $5 million with accumulated depreciation of $1 million. Its liabilities include an outstanding balance of $5 million for bonds payable issued to construct the roads and a $1 million short-term loan for inventory purchases. What amount should be reported as the net investment in capital assets in the government-wide statement of net assets position? A. $14 million B. $10 million C. $26 million D. $25 million

A. $14 million The government-wide statement of net position is one of two required government-wide statements (the other is the government-wide statement of activities). Remember that governmental activities and business-type activities are reported separately. Governmental activities typically include (1) all governmental fund assets and liabilities, (2) general capital assets (including infrastructure such as streets, roads, and bridges), (3) general long-term liabilities, and (4) the assets and liabilities of internal service activities. Therefore, net investment in capital assets is $14 million ($25 million − $10 million + $5 million − $1 million − $5 million). Short-term assets of $2 million and short-term liabilities of $1 million are not included.

A company calculated the following data for the period: Cash received from customers $25,000Cash received from sale of equipment 1,000Interest paid to bank on note 3,000Cash paid to employees 8,000 What amount should the company report as net cash provided by operating activities in its statement of cash flows? A. $14,000 B. $15,000 C. $18,000 D. $26,000

A. $14,000 Cash provided by operating activities is computed as follows: Cash received from customers $25,000Total cash received $25,000Interest paid to bank on note 3,000Cash paid to employees 8,000Total cash paid out 11,000Net cash provided 14,000 Cash received from sale of equipment is included in investing activities.

A company decided to sell an unprofitable division of its business. The company can sell the entire operation for $800,000, and the buyer will assume all assets and liabilities of the operations. The tax rate is 30%. The assets and liabilities of the discontinued operation are as follows: Buildings $5,000,000 Accumulated depreciation 3,000,000 Mortgage on buildings 1,100,000 Inventory 500,000 Accounts payable 600,000 Accounts receivable 200,000 What is the after-tax net loss on the disposal of the division? A. $140,000 B. $200,000 C. $1,540,000 D. $2,200,000

A. $140,000 NBV (net of assets and liabilities) = 1,000,000 loss = 200,000 200,000 - (1-.3) = 140,000

The following condensed balance sheet is presented for the partnership of Alfa and Beda, who share profits and losses in the ratio of 60:40, respectively: Cash $ 45,000 Other assets 625,000 Beda (loan) 30,000 $700,000======== Accounts payable $120,000 Alfa (capital) 348,000 Beda (capital) 232,000 $700,000======== The assets and liabilities are fairly valued on the balance sheet. Alfa and Beda decide to admit Capp as a new partner with a 20% interest. No goodwill or bonus is to be recorded. What amount should Capp contribute in cash or other assets? A. $145,000 B. $116,000 C. $140,000 D. $110,000

A. $145,000 Total capital prior to admission of Capp = $348,000 + $232,000 = $580,000. Since Capp is contributing an amount exactly equal to 20% of final capital, the $580,000 represents 80% of final capital. Final capital = $580,000 / .80 = $725,000Less Alfa and Beda capital 580,000Contribution to be made by Capp $145,000

Dari, Inc. guaranteed the debt of a related party. In December, Dari learned that it is probable it will be required to pay between $150,000 and $200,000 within the next six months in satisfaction of its guarantee, but no amount within that range is more likely. What amount of contingent liability should Dari accrue in its December 31 balance sheet? A. $150,000 B. $0 C. $200,000 D. $175,000

A. $150,000 Dari should accrue a contingent liability of $150,000 as of December 31. For a loss contingency to be accrued as a loss in the financial statements, the probability of occurrence must be probable and the amount of loss at least reasonably estimable. If no loss amount in the range of potential losses is more likely than the other possible amounts, the lower end of the range ($150,000) is accrued as a loss and contingent liability.

At year-end, Rim Co. held several investments with the intent of selling them in the near term. The investments consisted of $100,000, 8%, 5-year bonds purchased for $92,000, and equity securities purchased for $35,000. At year-end, the bonds were selling on the open market for $105,000 and the equity securities had a fair value of $50,000. What amount should Rim report as securities in its year-end balance sheet? A. $155,000 B. $127,000 C. $50,000 D. $142,000

A. $155,000 Investment in debt securities (other than those intended to be held until maturity) and investment in equity securities are reported at fair value in the balance sheet. The exceptions are marketable debt securities that a company plans to hold to maturity (reported at amortized cost) and marketable equity securities that provide the company the ability to significantly influence the investee (equity method required) or to control the investee (consolidation required). Therefore, Rim should report both at fair value of $155,000.

In Year 1, Lobo Corp. reported for financial statement purposes the following revenue and expenses that were not included in taxable income: Premiums on officers' life insurance under which thecorporation is the beneficiary $ 5,000Interest revenue on qualified state or municipal bonds 10,000Depreciation deducted for income tax purposes in excess ondepreciation reported for financial statement purposes 10,000Estimated future warranty costs to be paid in Year 2 and Year 3 60,000 Lobo's enacted tax rate for the current and future years is 30%. Lobo expects to operate profitably in the future. There were no temporary differences in prior years. The deferred tax benefit is: A. $18,000. B. $19,500. C. $21,000. D. $22,500.

A. $18,000. Deferred tax benefits only come about from temporary differences, like depreciation and warranty costs. All deferred tax assets and liabilities are classified as noncurrent, and apply to both financial accounting and to tax accounting, but are taken at different times for each. The other items are only taken into account in financial accounting, and are not income or expense items for taxes. When depreciation for tax purposes is in excess of depreciation for financial accounting, then it will not give rise to a benefit, but instead to a liability. This leaves only the warranty costs, which do give rise to a deferred tax benefit, since the warranty costs will defer to future years' additional tax deductions. The way to measure the deferred tax benefit is by multiplying the estimated warranty costs by the future tax rate of 30%: $60,000 × 0.30 = $18,000

Gray Co. was granted a patent on January 2, 20X1, and appropriately capitalized $45,000 of related costs. Gray was amortizing the patent over its estimated useful life of 15 years. No impairment losses were recognized. During 20X4, Gray paid $15,000 in legal costs in successfully defending an attempted infringement of the patent. After the legal action was completed, Gray sold the patent to the plaintiff for $75,000. Gray's policy is to take no amortization in the year of disposal. In its 20X4 income statement, what amount should Gray report as gain from sale of patent? A. $24,000 B. $15,000 C. $39,000 D. $27,000

A. $24,000 Patents are carried at the purchase price or legal filing costs (if internally generated) and are amortized over the shorter of legal or useful life. Legal costs of successful patent defense are added to the patent carrying cost. Initial capitalized amount $45,000amortization (3/15 x $45,000) (9,000)Infringement defense costs 15,000 Carrying value at time of sale $51,000========Sales price $75,000 Carrying value - 51,000Gain on sale $24,000

Plack Co. purchased 10,000 shares (2% ownership) of Ty Corp. on February 14, 20X1. Plack received a stock dividend of 2,000 shares on April 30, 20X1, when the market value per share was $35. Ty paid a cash dividend of $2 per share on December 15, 20X1. In its 20X1 income statement, what amount should Plack report as dividend income? A. $24,000 B. $20,000 C. $90,000 D. $94,000

A. $24,000 Receipt of a stock dividend does not result in income to the recipient and no journal entry is made by Plack Co. As a result of the stock dividend, Plack now owns 12,000 shares of stock and receipt of a $2 per share cash dividend yields income of $24,000.

Selected information from the accounts of Row Co. on December 31, 20X1, follows: Total income since incorporation $420,000Total cash dividends paid 130,000Total value of property dividendsdistributed 30,000Excess of proceeds over cost oftreasury stock sold, accountedfor using cost method 110,000 In its December 31, 20X1, financial statements, what amount should Row report as retained earnings? A. $260,000 B. $290,000 C. $400,000 D. $370,000

A. $260,000 420k - 160k (cash and property dividends) = 260k Note: The excess of proceeds over cost of treasury stock sold would be credited to "additional paid-in capital" under the cost method.

On January 1 of the current year, Wren Co. leased a building to Brill under an operating lease for 10 years at $50,000 per year, payable the first day of each lease year. Wren paid $15,000 to a real estate broker as a finder's fee. The building is depreciated $12,000 per year. For the year, Wren incurred insurance and property tax expense totaling $9,000. Wren's net rental income for the year should be: A. $27,500. B. $29,000. C. $35,000. D. $36,500.

A. $27,500. The revenue under the lease is the $50,000 each year, and the expenses include the depreciation and the property tax for the year. The broker's fee ($15,000) should be amortized equally based over the 10 years of the lease, or $1,500 a year.

At the beginning of the current year, a company held trading debt securities with a fair value of $250,000. During the year, the company received interest income of $25,000 from the securities and purchased an additional $50,000 of trading debt securities. At the end of the current year, the company recognized an unrealized loss of $20,000 on the trading debt securities held as of the end of the year. What amount should the company report for the trading debt securities in its statement of financial position at the end of the current year? A. $280,000 B. $325,000 C. $300,000 D. $305,000

A. $280,000 We are told these debt securities are carried as trading securities ("purchased an additional $50,000 of trading debt securities"). The other option, not used, would be held-to-maturity securities. Trading securities are carried at fair value with interest earned recognized in net income. Because the loss is unrealized, we can deduce that the company did not sell them and only saw a drop in value. The company would report the securities at their fair value as follows: $250,000 (Beginning balance) + $50,000 (Purchases) − $20,000 (Unrealized loss) = $280,000

Paprika Corporation issues $300,000 of 5% bonds dated April 1, Year 1. Interest is payable semiannually on September 30 and March 31. The bonds mature in eight years. The current market for similar bonds is 6%. The entire issue is sold on the date of issue. The following values are given: Present Value of Ordinary Annuity Present Value of $1 N=16; i=2.5% 13.0550 0.67362 N=16; i=3.0% 12.5611 0.62317 What amount of proceeds on the sale of bonds should Paprika report? A. $281,159 B. $281,369 C. $296,294 D. $300,000

A. $281,159 This question is about the computation of the issue price for a bond. The bonds will pay semiannually, and thus will pay $7,500 twice each year, computed as follows: Face amount of $300,000 × 5% coupon × 6/12 (half-year) = $7,500 The bond is priced to yield the market rate, 6% annually or 3% semiannually. The present value of the bonds is thus the $7,500 multiplied by the present value of the ordinary annuity for 16 periods and 3%, plus the $300,000 face value of the bonds multiplied by the present value of $1 at 16 periods, 3%: Issue price = ($7,500 × 12.5611) + ($300,000 × 0.62317) = $94,208 + $186,951 = $281,159

A company exchanged land with an appraised value of $50,000 and an original cost of $20,000 for machinery with a fair value of $55,000. The fair value of the land is more clearly evident than the fair value of the machine. Assuming that the transaction has commercial substance, what is the gain on the exchange? A. $30,000 B. $5,000 C. $35,000 D. $0

A. $30,000 Nonmonetary exchanges are generally recorded at fair value. Value of property exchanged $50,000Original cost 20,000Gain on exchange $30,000

Grant, Inc., acquired 30% of South Co.'s voting stock for $200,000 on January 2, 20X1. Grant's 30% interest in South gave Grant the ability to exercise significant influence over South's operating and financial policies. During 20X1, South earned $80,000 and paid dividends of $50,000. South reported earnings of $100,000 for the six months ended June 30, 20X2, and $200,000 for the year ended December 31, 20X2. On July 1, 20X2, Grant sold half of its stock in South for $150,000 cash. South paid dividends of $60,000 on October 1, 20X2. In its 20X2 income statement, what amount should Grant report as gain from the sale of half of its investment? A. $30,500 B. $35,000 C. $24,500 D. $45,500

A. $30,500 Since Grant purchased 30%, there is a presumption of significant influence, requiring the application of the equity method. Thus, Grant recognizes the percent share of the South income, adding to the investment balance, and the 30% of the dividends lower the carrying amount of the investment. Initial cost of investment $200,00030% of South's 20X1 income (30% x $80,000) 24,00030% of 20X1 dividends (30% x $50,000) (15,000)Carrying amount of investmenton December 31, 20X1 $209,00030% of South's Jan-June 20X2 income 30,000 (30% x $100,000) Carrying amount of investment on June 30, 20X2 $239,000Times 50% x 0.50 Equals carrying amount of one-half of investment $119,500========= $ 150,000 Sales prices- 119,500 Carrying amount$ 30,500 Gain on sale=========

A capital projects fund for a new city courthouse recorded a receivable of $300,000 for a state grant for which all eligibility requirements have been met and a $450,000 transfer from the general fund. What amount should be reported as revenue by the capital projects fund? A. $300,000 B. $0 C. $750,000 D. $450,000

A. $300,000 Transfers are other financing sources and are never recognized as revenues. If the receivable is associated with an unrestricted grant or with a restricted grant for which all eligibility requirements have been met, the state grant should be reported as revenues.

Wollongong Company decided to begin using dollar-value LIFO at the beginning of 20X5. The inventory value at January 1, 20X5, was $250,000. The current cost of the inventory at December 31, 20X5, was $306,000. At the end of 20X6, the current cost of the inventory was $288,750. The relevant index at the end of 20X5 was 1.02 and at the end of 20X6 was 1.05. The amounts Wollongong should report for inventory at the end of 20X5 and 20X6 are: A. $301,000 (20X5) and $275,500 (20X6). B. $300,000 (20X5) and $275,500 (20X6). C. $301,000 (20X5) and $276,250 (20X6). D. $306,000 (20X5) and $276,250 (20X6).

A. $301,000 (20X5) and $275,500 (20X6). Adjusting 20X5 ending inventory results in cost at base-year prices of $300,000 ($306,000 ÷ 1.02). The $300,000 cost at base-year prices includes two layers: beginning of 20X5 layer = $250,000 and end of 20X5 layer = $50,000. The adjustment of the end of 20X5 layer to current-year prices = $50,000 × 1.02 = $51,000. $250,000 + $51,000 = $301,000 is reported for inventory at the end of 20X5. Adjusting 20X6 ending inventory results in cost at base-year prices of $275,000 ($288,750 ÷ 1.05). The $275,000 cost at base-year prices includes two layers: beginning of 20X5 layer = $250,000 and end of 20X5 layer = $25,000. Because the cost at base-year prices at the end of 20X6 is less than the cost at base-year prices at the end of 20X5 ($275,000 vs. $300,000), the layer created in 20X5 is reduced to $25,000 ($50,000 − $25,000). The adjustment of the end of 20X5 layer to current-year prices = $25,000 × 1.02 = $25,500. $250,000 + $25,500 = $275,500 is reported for inventory at the end of 20X6.

Coriander Corporation acquired assets subject to unconditional retirement obligations measured at undiscounted cash flow estimates of $237,000 and discounted cash flow estimates of $153,000. Coriander reported a total asset retirement obligation of $196,000 in last year's financial statements. Coriander paid $102,000 toward the settlement of previously recorded asset retirement obligations and recorded an accretion expense of $61,000. What amount should Coriander report for the asset retirement obligation in this year's balance sheet? A. $308,000 B. $257,000 C. $349,000 D. $563,000

A. $308,000 An asset retirement obligation (ARO) refers to an obligation associated with the retirement of a tangible, long-lived asset, such as a nuclear power plant. An entity should originally recognize the fair value of an ARO in the period in which it is incurred if a reasonable estimate of fair value can be made. The amount is $308,000, computed as follows: Total obligation $196,000Add: Discounted cash flow 153,000*Accretion expense 61,000Deduct: Payment (102,000)$308,000======== * The original undiscounted cash flow of $237,000 should be adjusted to the discounted cash flows estimate.

Nect Corporation reported a retained earnings balance of $330,000 at December 31 of the previous year. In August of the current year, Nect determined that insurance premiums of $51,000 for the 2-year period beginning January 1 of the previous year had been paid and fully expensed in that year. Nect has a 40% income tax rate. What amount should Nect report as adjusted beginning retained earnings in its current-year statement of retained earnings? A. $345,300 B. $360,600 C. $319,800 D. $330,000

A. $345,300 Income was understated in the previous year because the full insurance premium was recorded as expense, leading to an understatement in beginning retained earnings the next year. The beginning retained earnings balance should reflect the after-tax effect of only half of the insurance premium because the premium covered two years. Nect should report adjusted beginning retained earnings of $345,300: Beginning retained earnings as originally reported $330,000 Overstated expense of $51,000/2 - Tax of 40% ($10,200) 15,300 Corrected beginning retained earnings $345,300

Which of the following should be disclosed in a summary of significant accounting policies? A. Basis of consolidation B. Concentration of credit risk of financial instruments C. Composition of plant assets D. Adequacy of pension plan assets in relation to vested benefits

A. Basis of consolidation Examples of accounting principles and methods for which disclosure of policy is frequently made include, but are not limited to, depreciation methods, consolidation basis, interperiod tax allocation, inventory pricing, and revenue recognition methods.

On March 1 of 20X6, Holly Co. began constructing a building it plans to use as its new headquarters office. On that same day, Holly borrowed $600,000 of specific construction debt at a rate of 8%. In addition to the construction-specific debt, Holly also incurred interest of $14,000 on other borrowings. Interest computed on the weighted-average amount of accumulated expenditures for the building during 20X6 was $36,500. What amount of interest cost should Holly capitalize? A. $36,500 B. $14,000 C. $40,000 D. $54,000

A. $36,500 Incorrect For qualifying assets being constructed for an entity's own use, FASB ASC 835-20-30-2 requires interest cost to be capitalized equal to the lesser of (a) the avoidable interest (based on the weighted-average amount of accumulated expenditures), or (b) the actual interest cost incurred. Holly's avoidable interest is given to be $36,500. Since the $54,000 actual interest cost incurred (($600,000 × .08 × 10/12) + $14,000) is greater than the avoidable interest of $36,500, the amount of interest that Holly can capitalize is $36,500.

West Co. paid $50,000 for an intangible asset other than goodwill. Fair value of the asset is $55,000. West signed a contract to sell the asset for $10,000 in 10 years. What amount of amortization expense should West record each year? A. $4,000 B. $4,500 C. $5,000 D. $5,500

A. $4,000 The amount of an intangible asset to be amortized is the amount initially assigned to that asset less any residual value. The residual value is the estimated fair value of the intangible asset at the end of its useful life to the reporting entity less any disposal costs. The residual value should be assumed to be zero unless at the end of its useful life the asset is expected to continue to have a useful life to another entity and the reporting entity has a commitment from a third party to purchase the asset at the end of its useful life. In this case, the amount to be amortized is $40,000 ($50,000 - $10,000) over 10 years, or $4,000 a year.

A county's balances in the general fund included the following: Appropriations $745,000 Encumbrances 37,250 Expenditures 298,000 Vouchers payable 55,875 What is the remaining amount available for use by the county? A. $409,750 B. $391,125 C. $353,875 D. $447,000

A. $409,750 The appropriations included in the adopted budget constitute the maximum authorized for expenditure during the period and cannot legally be exceeded unless subsequently amended by the legislative body. In this question, the appropriation was established at $745,000; expenditures incurred to date were $298,000 and $37,250 was encumbered. Only $409,750 remains available for spending. The vouchers payable represent past expenditures waiting only for cash payment. Appropriation $745,000Expenditures (298,000)Encumbrances (37,250)Funds available $409,750

Moss Corp. owns 20% of Dubro Corp.'s preferred stock and 80% of its common stock. Dubro's stock outstanding on December 31, 20X1, is as follows: 10% cumulative preferred stock $100,000 Common stock 700,000 Dubro reported net income of $60,000 for the year ending December 31, 20X1. What amount should Moss record as equity in earnings of Dubro for the year ending December 31, 20X1? A. $42,000 B. $48,000 C. $48,400 D. $50,000

A. $42,000 When computing the income share for an investor, first deduct the cumulative preferred dividends from net income. Moss' share of preferred dividends= 20% x 10% x $100,000= $2,000Earnings attributable to common shareholders= $60,000 - 10% x $100,000= $50,000Moss' share of common earnings= 80% x $50,000= $40,000Moss' total equity in Dubro earnings= $2,000 + $40,000= $42,000

On December 31 of the previous year, Jason Company adopted the dollar-value LIFO retail inventory method. Inventory data are as follows: LIFO Cost Retail Inventory, 12/31 previous year $360,000 $500,000 Inventory, 12/31 current year -- 660,000 Increase in price level for current year 10% Cost to retail ratio for current year 70% Under the LIFO retail method, Jason's inventory at December 31 of the current year should be: A. $437,000. B. $472,000. C. $462,000. D. $483,200.

A. $437,000. When applying the dollar-value LIFO retail method, you need to (as in dollar-value LIFO) restate ending-year retail to base-year prices: $660,000 ÷ 1.10 (1 + 10% increase) = $600,000 This is a $100,000 increase in the ending-year retail amount over the retail amount at the beginning of the year (in base-year prices). Now, determine the ending inventory using dollar-value LIFO retail directly, by adding to the beginning inventory of $360,000 the new layer of $100,000 multiplied by both the new layer's cost-to-retail percentage and the new layer price level of 1.1: $360,000 + ($100,000 × 0.7 × 1.1) = $437,000

On January 2, 20X1, Emme Co. sold equipment with a carrying amount of $480,000 in exchange for a $600,000 non-interest-bearing note due January 2, 20X4. There was no established exchange price for the equipment. The prevailing rate of interest for a note of this type on January 2, 20X1, was 10%. The present value of 1 at 10% for three periods is 0.75. In Emme's 20X1 income statement, what amount should be reported as interest income? A. $45,000 B. $9,000 C. $60,000 D. $50,000

A. $45,000 Interest is revenue charged in relation to money lent. One must recognize it based on the amount lent and the agreed-upon rate. Present value of note = 75 × $600,000 = $450,000 Interest income for 20X1 = 0.10 × $450,000 = $45,000

On December 31, 20X1, Byte Co. had capitalized software costs of $600,000 with an economic life of four years. Sales for 20X2 were 10% of expected total sales of the software. At December 31, 20X2, the software had a net realizable value of $480,000. In its December 31, 20X2, balance sheet, what amount should Byte report as net capitalized cost of computer software? A. $450,000 B. $480,000 C. $540,000 D. $432,000

A. $450,000 When software costs are capitalized, yearly amortization of these costs is based on the greater of the ratio of current sales to expected total sales or the straight-line method over the useful life of the asset (four years). Sales ratio: 10% (0.10) × $600,000 = $60,000 Straight-line: 25% (0.25) × $600,000 = $150,000 Since straight-line amortization is larger and is used, the remaining capitalized cost is $600,000 less $150,000, or $450,000. Since the net realizable value, $480,000, is greater than the $450,000, there is no need for an additional write-off. FASB ASC 350-40-35-4 states: "Amortization: The costs of computer software developed or obtained for internal use shall be amortized on a straight-line basis unless another systematic and rational basis is more representative of the software's use." FASB ASC 985-20-35-4 states: "Net Realizable Value of Capitalized Software Costs: At each balance sheet date, the unamortized capitalized costs of a computer software product shall be compared to the net realizable value of that product. The amount by which the unamortized capitalized costs of a computer software product exceed the net realizable value of that asset shall be written off."

Ace Co. sold to King Co. a $20,000, 8%, 5‑year note that required five equal annual year-end payments. This note was discounted to yield a 9% rate to King. The present value factors of an ordinary annuity of $1 for five periods are as follows: 8%: 3.993 9%: 3.890 What should be the total interest revenue earned by King on this note? A. $5,560 B. $5,050 C. $9,000 D. $8,000

A. $5,560 Thus, the annual payment is $20,000 ÷ 3.993, or $5,009 annually. (Uses the 8% rate as the note should be at discount) The present value of the note, however, and thus the initial discount is based on the yield percentage of 9%. Therefore, the note's initial present value is the payment amount multiplied by 3.89 ($5,009 × 3.89), or $19,485. The total amount to be received on this note is 5 years multiplied by the payment of $5,009, as specified, for a total of $25,045. Interest is generally the amount returned over and above the amount originally recognized, which was $19,485 originally. Thus, the total interest revenue is $25,045 − $19,485, or $5,560.

Ace Corp. entered into a troubled debt restructuring (TDR) agreement with National Bank. National agreed to accept land with a carrying amount of $75,000 and a fair value of $100,000 in payment and cancellation of a note (from Ace) with a carrying amount of $150,000. Disregarding income taxes, what amount should Ace report as a gain from the restructuring in its income statement? A. $50,000 B. $75,000 C. $25,000 D. $0

A. $50,000 In a transfer of assets to satisfy debt in a TDR, the debtor: recognizes a gain or loss on the transfer of assets (equal to the difference between the fair value of $100,000 and recorded value of $75,000 of the asset transferred), or $25,000, and recognizes a gain on the restructuring of a debt (equal to the difference between the $100,000 fair value of the asset and the $150,000 carrying value of the debt), or $50,000. Even though a total gain of $75,000 is recognized on the income statement, the portion attributable to the restructuring is $50,000.

Pine Corp.'s books showed pretax income of $800,000 for the current year ended December 31. In the computation of federal income taxes, the following data were considered: Gain on involuntary conversion (Pine haselected to replace the property withinthe statutory period using total proceeds.) $350,000Depreciation deducted for tax purposes in excessof depreciation deducted for book purposes $50,000Federal estimated tax payments $70,000Enacted federal tax rates 30% What amount should Pine report as its current federal income tax liability on its December 31 balance sheet? A. $50,000 B. $65,000 C. $120,000 D. $135,000

A. $50,000 The current income tax liability is based on the taxable income for the year and the tax rate for the year, so one needs to compute taxable income first. The pretax income was $800,000 and part of this amount is a gain that qualifies for tax deferral, the reinvested proceeds from the involuntary conversion gain. Also, the amount of tax depreciation is used to compute taxable income. Taxable income is $400,000 ($800,000 less the gain of $350,000, and less the additional tax depreciation of $50,000). This amount multiplied by the tax rate of 30% gives us the tax due of $120,000 ($400,000 × 0.30). Because some of this amount due has been paid in estimates already, only the remaining $50,000 is a liability yet to pay ($120,000 - $70,000).

Tam Co. reported the following items in its year-end financial statements: Capital expenditures $1,000,000 Sales-type lease payments 125,000 Income taxes paid 325,000 Dividends paid 200,000 Net interest payments 220,000 What amount should Tam report as supplemental disclosures in its statement of cash flows prepared using the indirect method? A. $545,000 B. $1,870,000 C. $1,125,000 D. $745,000

A. $545,000 Regardless of whether the direct or indirect method is used to determine cash flows from operating activities, the following items are required to be disclosed: Amount of income taxes paid during the period ($325,000) Amount of interest paid during the period ($220,000) $325,000 + $220,000 = $545,000

In 20X1, May Corp. acquired land by paying $75,000 down and signing a note with a maturity value of $1,000,000. On the note's due date, December 31, 20X6, May owed $40,000 of accrued interest and $1,000,000 principal on the note. May was in financial difficulty and was unable to make any payments. May and the bank agreed to amend the note as follows: The $40,000 of interest due on December 31, 20X6, was forgiven. The principal of the note was reduced from $1,000,000 to $950,000 and the maturity date extended one year to December 31, 20X7. May would be required to make one interest payment totaling $30,000 on December 31, 20X7. As a result of the troubled debt restructuring, May should report a gain, before taxes, in its 20X6 income statement of: A. $60,000. B. $50,000. C. $90,000. D. $40,000.

A. $60,000. Gain = Total due on debt - Required restructured payment = Principal + Accrued interest - (Restructured principal + Required interest) = ($1,000,000 + $40,000) - ($950,000 + $30,000) = $1,040,000 - $980,000 = $60,000

Cobb, Inc.'s inventory at May 1 consisted of 200 units at a total cost of $1,250. Cobb uses the periodic inventory method. Purchases for the month were as follows: Date No. of Units Unit Cost Total Cost May 4 20 $5.80 $116.00 May 17 80 5.50 440.00 Cobb sold 10 units on May 14 for $120. What is Cobb's weighted-average cost of goods sold for May? A. $60.20 B. $62.50 C. $62.10 D. $65.00

A. $60.20 Under a periodic system, units and costs are determined at the end of the period and are not constantly updated during the period. The weighted-average cost of goods sold is determined as follows: Computation of average cost per unit: Total cost of goods available for sale(beginning inventory + purchases) $1,806 ($1,250 + $116 + $440)Divided by total units for sale 300 (200 + 20 + 80)= Average cost per unit $6.02 Cost of goods sold = Average cost per unit × Units soldCost of goods sold = $6.02 × 10 units = $60.20

On August 18, 20X8, Gilles Inc.'s board of directors declared a 15% stock dividend. The market price of Gilles' 50,000 outstanding shares of $3 par value common stock was $12 per share on that date. The stock dividend was distributed on October 4, 20X8, when the stock's market price was $14 per share. What amount should Gilles credit to additional paid-in capital for this stock dividend? A. $67,500 B. $90,000 C. $22,500 D. $50,000

A. $67,500 Small stock dividends (stock dividends less than 20%-25%) are accounted for based on the fair market value of the shares when declared. Consider the journal entry to record the declaration of the dividend: Dr. Cr. Retained earnings (50,000 × .15 × $12) 90,000 Common stock dividend distributable(50,000 × .15 × $3) 22,500 Additional paid-in capital($90,000 − $22,500) 67,500

In the current year, a nongovernmental, not-for-profit entity incurred $630,000 in expenditures during the year. It also received donated legal services, which otherwise would have cost $40,000, and consumed donated supplies with a value of $15,000. What should the entity report as total expenses in its statement of activities for the current year? A. $685,000 B. $645,000 C. $630,000 D. $670,000

A. $685,000 The answer choice "$685,000" is correct: $630,000 + $40,000 + $15,000 = $685,000. Since the donated supplies of $15,000 represent an asset to the organization, as this asset is consumed, the fair value equivalent of consumption activities would need to be expensed. Meanwhile, legal services of $40,000 meets the three conditions for being recognized as a contributed service since the expense (1) required specialized skills to perform the activity, (2) the attorneys donating the service possess those skills, and (3) the organization would have to buy legal services if they were not donated. $630,000 is incorrect. This excludes donations of $15,000 supplies and $40,000 legal services. $645,000 is incorrect. This excludes donations of $40,000 legal services. $670,000 is incorrect. This excludes donations of $15,000 supplies.

At the beginning of the year, Cann Co. started construction on a new $2 million addition to its plant. Total construction expenditures made during the year were $200,000 on January 2, $600,000 on May 1, and $300,000 on December 1. On January 2, the company borrowed $500,000 for the construction at 12%. The only other outstanding debt the company had was a 10% interest rate, long-term mortgage of $800,000, which had been outstanding the entire year. What amount of interest should Cann capitalize as part of the cost of the plant addition? A. $72,500 B. $60,000 C. $140,000 D. $132,000

A. $72,500 The total interest cost during the year includes the $500,000 at 12% interest ($500,000 × 0.12 = $60,000) and the interest on the other debt ($800,000 × 0.10 = $80,000). There cannot be more than $140,000 interest capitalized (the total interest accrued of $60,000 and $80,000). The weighted-average accumulated expenditures take into account the amounts expended on the building during the year based on how much of the year occurred after the payment. The $200,000 paid in January was paid at the beginning of the year and was outstanding all year ($200,000 × 12/12 of the year, or $200,000). The May payment was only outstanding for May through December for 8/12 of the year, so $600,000 × 8/12 = $400,000 weighted-average expenditure. The December payment was made for only the last month, or 1/12 of the year, for an expenditure of $300,000 × 1/12 = $25,000. The total weighted-average accumulated expenditures were thus $200,000 + $400,000 + $25,000, for a total of $625,000. The capitalized interest cost on these expenditures is based on the interest rates of the debt outstanding during the year, first, to the extent of any specific construction debt, i.e., the $500,000 at 12%. The interest capitalized on the first $500,000 of expenditures is based on 12%, or $60,000, and the interest on the remaining $125,000 of expenditures ($625,000 - $500,000) is paid at the rate of 10% for another $12,500 of capitalized interest. Thus, the total of the capitalized interest is $60,000 and $12,500, for a total of $72,500.

The definition of a smaller reporting company with respect to market value, as established by the U.S. Securities and Exchange Commission, includes companies with less than exactly what amount in public equity float? A. $75 million B. $150 million C. $125 million D. $100 million

A. $75 million There are three categories for filers with the SEC. Entities with market values exceeding $700,000,000 are classified as large accelerated filers; entities with market values between $75,000,000 and $700,000,000 (and at least $100 million in annual revenues) are accelerated filers; and entities with market value below $75,000,000 and/or annual revenues less than $100,000,000 are non-accelerated (or small) filers. Each category has different filing requirements and deadlines.

Blue Township has a number of outstanding bond issues that include a $4,000,000 general obligation bond that financed City Hall, a $2,000,000 revenue bond that financed upgrades to the water treatment plant, a $1,000,000 special assessment bond for sidewalks, and a $3,000,000 general obligation bond used for streets and roads. Revenues of the water fund, a proprietary fund, are expected to pay off the revenue bond. What should Blue Township report as long-term liabilities in the governmental activities column of the government-wide statement of net position? A. $8,000,000 B. $4,000,000 C. $7,000,000 D. $9,000,000

A. $8,000,000 The obligations of the proprietary fund, the revenue bonds of $2,000,000, should be shown as a fund liability. The other bonds should be shown in the government-wide statements as a liability relating to governmental activities, but not shown in the fund statements.

The following changes in Vel Corp.'s account balances occurred during 20X1: Increase Assets $89,000 Liabilities 27,000 Capital stock 60,000 Additional paid-in capital 6,000 Except for a $13,000 dividend payment and the year's earnings, there were no changes in retained earnings for 20X1. What was Vel's net income for 20X1? A. $9,000 B. $17,000 C. $13,000 D. $4,000

A. $9,000 Increases in assets must equal increases in liabilities and equity (specifically increase in retained earnings in equity): Assets = Liabilities + Equity. 89k - 27k = 62 k increase add back dividend of 13k --> 75k subtract capital stock of 60k+6k = 9K

Tulip Co. owns 100% of Daisy Co.'s outstanding common stock. Tulip's cost of goods sold for the year totals $600,000 and Daisy's cost of goods sold totals $400,000. During the year, Tulip sold inventory costing $60,000 to Daisy for $100,000. By the end of the year, all transferred inventory was sold to third parties. What amount should be reported as cost of goods sold in the consolidated statement of income? A. $900,000 B. $960,000 C. $940,000 D. $1,000,000

A. $900,000 Under the acquisition method, a number of adjusting and eliminating entries are made during the consolidation process. Eliminations may be categorized as those related to the following: The investment account Current-year changes in the investment account Year-end reciprocal balance sheet accounts Reciprocal income statement accounts Intercompany profits and losses Since Daisy sold all of the inventory purchased from Tulip, Daisy would have recognized $100,000 in cost of goods sold (COGS). As Daisy is a 100%-owned subsidiary, 100% of the COGS from Tulip is eliminated (i.e., intercompany profits and losses). Total COGS on the consolidated statement of income is $900,000 ($600,000 + $400,000 − $100,000).

Although future-oriented information requires considerable judgment and tends to not be disclosed, the FASB recommends in Statement of Financial Accounting Concepts 8 (SFAC 8) disclosure for the following two future-oriented types of information: A. (1) Estimates and assumptions and (2) management's plans and strategies B. (1) Estimates and assumptions and (2) alternative accounting methods C. (1) Common knowledge and (2) management's plans and strategies D. (1) Terms and timing of cash flows and (2) nonperformance risk

A. (1) Estimates and assumptions and (2) management's plans and strategies

Baker Co. began its operations during the current year. The following is Baker's balance sheet at December 31: Cash $192,000 Accounts receivable 82,000 Total assets $274,000 Accounts payable $ 24,000 Common stock 200,000 Retained earnings 50,000 Total liabilities and stockholders' equity $274,000 Baker's net income for the current year was $78,000 and dividends of $28,000 were declared and paid. Common stock was issued for $200,000. What amount should Baker report as cash provided by operating activities in its statement of cash flows for the current year? A. 20,000 B. 50,000 C. 192,000 D. 250,000

A. 20,000 Baker should report $20,000 as net cash provided by operating activities: Net income $78,000 Increase in accounts receivable ($82,000) Increase in accounts payable 24,000 Net cash provided by operating activities = $20,000

On December 31, 20X1, in a strategic shift, Greer Co. entered into an agreement to sell its Hart segment's assets. On that date, Greer estimated the gain from the disposition of the assets in 20X2 would be $700,000 and Hart's 20X2 operating losses would be $200,000. Hart's actual operating losses were $300,000 in both 20X1 and 20X2, and the actual gain on disposition of Hart's assets in 20X2 was $650,000. Disregarding income taxes, what net gain (loss) should be reported for discontinued operations in Greer's comparative 20X2 and 20X1 income statements? A. 20X2: $350,000; 20X1: $(300,000) B. 20X2: $50,000; 20X1: $(300,000) C. 20X2: $0; 20X1: $50,000 D. 20X2: $(150,000) ; 20X1: $200,000

A. 20X2: $350,000; 20X1: $(300,000)

Fern Co. has net income, before taxes, of $200,000, including $20,000 interest revenue from municipal bonds and $10,000 paid for officers' life insurance premiums where the company is the beneficiary. The tax rate for the current year is 30%. What is Fern's effective tax rate? A. 28.5% B. 30.0% C. 31.5% D. 27.0%

A. 28.5% The first step is to know the taxable income. Net income for accounting purposes before tax would be the starting point. Adjust this number up or down for nontaxable or nondeductible items. The municipal interest income would be nontaxable, so this would be subtracted from accounting income and the insurance premiums would be nondeductible (since they relate to nontaxed income) and would need to be added. Thus, net income before taxes of $200,000 minus the municipal bond interest of $20,000, plus the insurance premiums $10,000 equals taxable income of $190,000: $200,000 - $20,000 + $10,000 = $190,000 Taxable income times the tax rate equals tax due of $57,000: $190,000 × 0.30 = $57,000 The effective tax rate would be the total tax due divided by the total income earned: $57,000 ÷ $200,000 = 0.285 (28.5%)

Host Co. has adopted FASB ASC 205-20 (Presentation of Financial Statements—Discontinued Operations). On October 1, 20X1, in a strategic shift, Host Co. approved a plan to dispose of a segment of its business. Host expected that the sale would occur on April 1, 20X2, at an estimated gain of $350,000. The segment had actual and estimated operating losses as follows: 01/01/X1 to 09/30/X1 $(300,000) 10/01/X1 to 12/31/X1 (200,000) 01/01/X2 to 03/31/X2 (400,000) Assuming that the segment qualified as a component under FASB ASC 205-20-45, in its 20X1 income statement, what should Host report as a loss from operation of a discontinued segment? A. 500,000 B. 600,000 C. 200,000 D. 250,000

A. 500,000 Under FASB ASC 205-20-45-3, the losses from a discontinued segment that qualifies as a component are reported in the period they occur. An anticipated loss on sale should be recognized by writing the component down to FMV. An anticipated gain on sale of the component should not be recognized until the day of the sale. Loss 01/01/X1 to 09/30/X1 ($200,000) Loss 10/01/X1 to 12/31/X1 ( 300,000) Loss ($500,000)

Which of the following statements regarding fiduciary funds is true? A. Fiduciary funds and fiduciary component units are not reported in the government-wide financial statements. B. Custodial funds are not required to report a statement of changes in net position. C. Private-purpose trust funds are governmental funds. D. Revenues and expenses are reported in the statement of changes in net position for investment trust funds.

A. Fiduciary funds and fiduciary component units are not reported in the government-wide financial statements.

When preparing a draft of its 20X1 balance sheet (statement of financial position), Mont, Inc., reported net assets totaling $875,000. Included in the asset section of the balance sheet were the following: Treasury stock of Mont, Inc., at cost, which approximates market value on December 31, 20X1 $24,000 Idle machinery 11,200 Cash surrender value of life insurance on corporate executives 13,700 Investments in equity securities 8,400 At what amount should Mont's net assets be reported in the December 31, 20X1, balance sheet (statement of financial position)? A. $851,000 B. $850,100 C. $842,600 D. $834,500

A. 851,000 Idle machinery, cash surrender value of life insurance on corporate executives, and investments in equity securities should appear in the asset section of the balance sheet (statement of financial position). Treasury stock should be presented as a reduction of stockholder's equity, not as an asset.

Which of the following types of agreement represents a split-interest arrangement? A. A charitable remainder trust B. A perpetual trust naming ABC as sole beneficiary C. A direct bequest to ABC in the donor's will D. A direct gift to ABC to be made by the donor next year

A. A charitable remainder trust "A charitable remainder trust" is correct. When trusts provide that the organization must share resources with another beneficiary, this agreement represents a split-interest arrangement. Split-interest agreement are categorized into five types: 1) perpetual trusts held by third parties, 2) charitable gift annuities, 3) charitable lead trusts, 4) pooled income funds, and 5) charitable remainder trusts. A charitable remainder trust represents a split-interest agreement because the donor has designated a specified percentage or dollar amount of the trust's principal and earnings to be paid to a third-party beneficiary, such as a surviving spouse or child (in addition to the money that is being retained by the recipient organization). The answer choices "a perpetual trust naming ABC as sole beneficiary," "a direct bequest to ABC in the donor's will," and "a direct gift to ABC to be made by the donor next year" are all incorrect. Under all these arrangements, ABC has exclusive rights to the entire portion of the donation. In other words, it is not having to divide (or share) the donation amongst other third parties, such as a surviving spouse or child; therefore, these arrangements do not represent split-interest agreements.

At its date of incorporation, Glean, Inc., issued 100,000 shares of its $10 par common stock at $11 per share. During the current year, Glean acquired 30,000 shares of its common stock at a price of $16 per share and accounted for them by the cost method. Subsequently, these shares were reissued at a price of $12 per share. There have been no other issuances or acquisitions of its own common stock. What effect does the reissuance of the stock have on the following accounts? A. A decrease in retained earnings and no effect on additional paid-in capital B. A decrease in both retained earnings and additional paid-in capital C. No effect on retained earnings or additional paid-in capital D. No effect on retained earnings and a decrease in additional paid-in capital

A. A decrease in retained earnings and no effect on additional paid-in capital Using the cost method of accounting, additional paid in capital from treasury stock is credited when treasury stock is reissued at excess. The sale of the treasury stock resulted in a $120,000 loss: $12 selling price − $16 cost to purchase = $4 30,000 shares × $4 = $120,000 The journal entry for reissuance of the treasury stock is as follows: Cash ($12 x 30,000) 360,000Retained Earnings 120,000Treasury Stock ($16 x 30,000) 480,000 There was only one treasury stock reissuance so there is no additional paid in capital transaction to be made. Therefore, under the cost method, Glean would record the loss (decrease or debit) to retained earnings and there would be no effect on additional paid-in capital.

Which of the following is an intangible asset that is subject to the recoverability test when testing for impairment? A. A patent B. A trademark with indefinite useful life C. Goodwill D. R&D costs for a patent

A. A patent The FASB requires that non-goodwill intangible assets with finite lives be amortized, whereas similar assets with indefinite lives are not amortized. Both classifications of non-goodwill intangibles are required to be reviewed for impairment. Intangible assets with finite lives are tested for recoverability whenever events or changes in circumstances indicate that the carrying value may not be recoverable. Intangible assets that have indefinite lives are not amortized and therefore are not tested for recoverability. However, they are reviewed for impairment at least annually. Patents fall into the first group, and are tested for recoverability. Goodwill and the trademark have infinite lives and fall into the second group; they are reviewed at least annually for impairment. Research and development (R&D) costs are expensed, not capitalized.

Which of the following is a criterion that must be satisfied in order for an asset or a group of assets to be classified as "held for sale"? A. All of the answer choices are criteria that must be satisfied for an asset or group of assets to be classified as held for sale. B. The asset or group of assets is being actively marketed. C. Management has committed to plan to sell the asset or group of assets. D. The asset or group of assets must be ready for immediate sale.

A. All of the answer choices are criteria that must be satisfied for an asset or group of assets to be classified as held for sale.

Which of the following transactions should be reported as a liability in the general fund financial statements? A. An amount to be paid from current financial resources B. Principal on long-term debt due 90 days after the balance sheet date C. An amount that is due within one year of the balance sheet date D. An amount set aside to pay for an unfilled contract

A. An amount to be paid from current financial resources Modified accrual accounting (revenues and expenditures reporting using the flow of current financial resources measurement focus) is used in the governmental funds, including the general fund. The General Fund is the primary governmental fund and is used to account for most routine general government operations. A government's unmatured long-term indebtedness should be reported as general long-term liabilities, rather than governmental fund liabilities.

On January 1, 20X1, Crater, Inc., purchased equipment having an estimated salvage value equal to 20% of its original cost at the end of a 10-year life. The equipment was sold December 31, 20X5, for 50% of its original cost. If the equipment's disposition resulted in a reported loss, which of the following depreciation methods did Crater use? A. Straight-line B. Composite C. Sum-of-the-years'-digits D. Double-declining balance

A. Straight-line Use of an accelerated depreciation method (e.g., double-declining balance or sum-of-the-years'-digits) would most likely produce a gain rather than a loss because of the rapid reduction in carrying value. With this in mind, it would be a good idea to try the straight-line method first.

A not-for-profit organization is exempt from reporting which of the following contributed services as gross revenue? A. An attorney solicits contributions on behalf of the organization. B. A CPA prepares the organization's tax return. C. A special education teacher tutors children with learning disabilities. D. A carpenter builds shelves for the office.

A. An attorney solicits contributions on behalf of the organization. The classification of contributions received as revenues or gains depends on whether the transactions are part of the NFP's (not-for-profit entity's) ongoing major or central activities (revenues). Events are peripheral or incidental if they are not an integral part of an NFP's usual activities, or if their gross revenues or expenses are not significant in relation to the NFP's annual budget. For the answer choices for this question, only an attorney soliciting contributions is considered peripheral, and therefore the entity is exempt from reporting those services as gross revenue.

Which of the following statements regarding foreign exchange gains and losses is correct? A. An exchange gain occurs when the exchange rate increases between the date a receivable is recorded and the date of cash receipt. B. An exchange loss occurs when the exchange rate decreases between the date a payable is recorded and the date of the cash payment. C. An exchange gain occurs when the exchange rate increases between the date a payable is recorded and the date of cash payment. D. An exchange loss occurs when the exchange rate increases between the date a receivable is recorded and the date of the cash receipt.

A. An exchange gain occurs when the exchange rate increases between the date a receivable is recorded and the date of cash receipt. Receivables denominated in a foreign currency entitle a company to a fixed number of units of the foreign currency. The U.S. dollar balance of the receivable is measured initially using the exchange rate in effect when the receivable is established. If the exchange rate is higher when the fixed number of units of the foreign currency is received (i.e., cash is received), the dollars received will exceed the balance of the receivable in dollars, and a gain will have occurred.

Which of the following is not an eligible item for the fair value measurement option under FASB ASC 825-10-15-4? A. An interest in a variable interest entity that the entity is required to consolidate B. A firm commitment that would otherwise not be recognized at commencement and that involves only financial instruments (An example is a forward purchase contract for a loan that is not readily convertible to cash. That commitment involves only financial instruments—a loan and cash—and would not otherwise be recognized because it is not a derivative instrument.) C. A recognized financial asset and financial liability, except any listed below in exceptions D. A written loan commitment

A. An interest in a variable interest entity that the entity is required to consolidate FASB ASC 825-10-15-5 lists the following items that are not eligible for the fair value election: "An investment in a subsidiary that the entity is required to consolidate "An interest in a variable interest entity that the entity is required to consolidate "Employers' and plans' obligations (or assets representing net overfunded positions) for pension benefits, other postretirement benefits (including health care and life insurance benefits), postemployment benefits, employee stock option and stock purchase plans, and other forms of deferred compensation arrangements, as defined in [FASB ASC] Topics 420; 710; 712; 715; 718; and 960. "Financial assets and financial liabilities recognized under leases as defined in [FASB ASC] Subtopic 842-10 (This exception does not apply to a guarantee of a third-party lease obligation or a contingent obligation arising from a cancelled lease.) "Deposit liabilities, withdrawable on demand, of banks, savings and loan associations, credit unions, and other similar depository institutions "Financial instruments that are, in whole or in part, classified by the issuer as a component of shareholder's equity (including "temporary equity"). An example is a convertible debt security with a noncontingent beneficial conversion feature."

The budget for the City of Goodville for the year ending December 31 was adopted and recorded on January 2 of the same year. After recording the budget, the accounting records showed a debit balance of $50,000 in the Budgetary Fund Balance account. What does this indicate? A. Appropriations are $50,000 greater than estimated revenues. B. Appropriations are $50,000 greater than revenues. C. Estimated revenues are $50,000 greater than appropriations. D. Revenues are $50,000 greater than appropriations.

A. Appropriations are $50,000 greater than estimated revenues. Annual budgets of estimated revenues and estimated expenditures are prepared for most governmental-type funds. The approved budgets of such funds are recorded in budgetary accounts in the accounting system to provide control over governmental fund revenues and expenditures. Budgetary journal entries record estimated amounts. Budgetary fund balance is the difference between estimated revenues and estimated expenditures; it can have a debit or credit balance. The following is typical of the general fund's entry to record the budget where estimated expenditures (i.e., appropriations) exceed estimated revenues: Estimated Revenue Control 40,000Budgetary Fund Balance 50,000Appropriations Control 90,000

Under current generally accepted accounting principles, which approach is used to determine income tax expense? A. Asset and liability approach B. Net of tax approach C. "With and without" approach D. Periodic expense approach

A. Asset and liability approach

Deed Co. owns 2% of Beck Cosmetic Retailers. A property dividend by Beck consisted of merchandise with a fair value lower than the listed retail price. Deed in turn gave the merchandise to its employees as a holiday bonus. How should Deed report the receipt and distribution of the merchandise in its income statement? A. At fair value for both dividend revenue and employee compensation expense B. By disclosure only C. At listed retail price for both dividend revenue and employee compensation expense D. At fair value for dividend revenue and listed retail price for employee compensation expense

A. At fair value for both dividend revenue and employee compensation expense FASB ASC 845-10-30-1 provides that "a nonmonetary asset received in a nonreciprocal transfer should be recorded at the fair value of the asset received. A transfer of a nonmonetary asset to a stockholder or to another entity in a nonreciprocal transfer should be recorded at the fair value of the asset transferred." Both receipt of the dividend and the distribution of the merchandise to employees should be recorded at fair value as dividend revenue and employee compensation expense.

Which of the following would be added back to net income when reporting operating activities' cash flows by the indirect method? A. Bond discount amortization B. Neither excess of treasury stock acquisition cost over sales proceeds (cost method) nor bond discount amortization C. Both excess of treasury stock acquisition cost over sales proceeds (cost method) and bond discount amortization D. Excess of treasury stock acquisition cost over sales proceeds (cost method)

A. Bond discount amortization FASB ASC 230-10-55-1 describes the indirect method briefly as follows: "Given sufficiently detailed information, major classes of operating cash receipts and payments may be determined indirectly by adjusting revenue and expense amounts for the change during the period in related asset and liability accounts." Thus, bond discount amortization would be deducted from interest expense to compute the amount of cash paid for bond interest. The net effect would be to add this amortization amount to net income (as a noncash expense). The excess of treasury stock acquisition cost over sales proceeds is not an income item; rather, the excess is an equity item debited to contributed capital from treasury stock transactions.

In preparing Chase City's reconciliation of the statement of revenues, expenditures, and changes in fund balances to the government-wide statement of activities, which of the following items should be subtracted from changes in fund balances? A. Book value of capital assets sold during the year B. Internal service fund increase in net position C. Payment of long-term debt principal D. Capital assets purchases

A. Book value of capital assets sold during the year The answer choice "book value of capital assets sold during the year" is correct. These amounts should be deducted because these assets were never recorded at the fund level. Capital assets are treated as expenditures within the government funds. When preparing the government-wide statement of activities, one should subtract the book value from sale price for the purpose of determining the gain or loss from the sale. "Capital assets purchases" is incorrect. This amount should be added back to fund balance to recognize the long-lived asset on a full accrual basis. "Payment of long-term debt principal" is incorrect. This amount should be added back to fund balance to recognize the balance of the long-term liability on a full accrual basis. "Internal service fund increases in net position" is incorrect. This amount should be added back to fund balance to recognize the blending of internal service fund balances within the governmental-type activities column.

Which of the following risks are inherent in an interest rate swap agreement? The risk of exchanging a lower interest rate for a higher interest rate The risk of nonperformance by the counterparty to the agreement A. Both I and II B. I only C. II only D. Neither I nor II

A. Both I and II An interest rate swap agreement is entered into in the hope of additional safety or other benefits, but it carries both the risks identified above, the potential of counterparty nonperformance or an undesirable exchange.

A city taxes merchants for various central district improvements. Which of the following accounting methods assists in assuring that these revenues are expended legally? A. Both fund accounting and budgetary accounting B. Budgetary accounting C. Neither fund accounting nor budgetary accounting D. Fund accounting

A. Both fund accounting and budgetary accounting Fund accounting segregates resources according to the purpose(s) for which they may be used. Thus, fund accounting helps ensure that the taxes required to be used for central district improvements are not expended for other purposes. Appropriations are adopted in the budget to authorize expenditure of resources during a particular budget period. Overexpenditure of appropriations—even for central district improvements—violates the budget law. Budgetary accounting maintains a record of the remaining appropriation authority available for specific purposes at any point in time. Thus, budgetary accounting helps assure that resources are expended legally by helping to prevent overexpenditure of appropriations.

In financial reporting of segment data, which of the following must be considered in determining if an industry segment is a reportable segment? A. Both sales to unaffiliated customers and intersegment sales B. Sales to unaffiliated customers C. Intersegment sales D. Neither sales to unaffiliated customers nor intersegment sales

A. Both sales to unaffiliated customers and intersegment sales

A city government levies a tax on its citizens for improvements to roads. How should the city report the tax in its statement of activities? A. By type of tax in general revenues B. By type of tax in program revenues C. As program-specific contributions in program revenues D. In special items reported separately from general revenue

A. By type of tax in general revenues All taxes of the reporting government, even if restricted to a specific program, are general revenues.

Which of the following would be reported as an investing activity in a company's statement of cash flows? A. Collection of a note receivable from a related party B. Collection of a tax refund from the government C. Collection of proceeds from a note payable D. Collection of an overdue account receivable from a customer

A. Collection of a note receivable from a related party Investing activities involve asset transactions other than those related to operating results (e.g., accounts receivables from sales and taxes).

Which of the following financial instruments issued by a public company should be reported on the issuer's books as a liability on the date of issuance? A. Common stock that contains an unconditional redemption feature B. Common stock that is issued at a 5% discount as part of an employee share purchase plan C. Cumulative preferred stock D. Preferred stock that is convertible to common stock five years from the issue date

A. Common stock that contains an unconditional redemption feature

Which of the following is the most correct statement regarding the scope of a government's MD&A? A. Governments should only present that information needed to support their analysis of financial position and results of operations prescribed by GASB Statement 34 for MD&A. B. Information that does not relate to the required topics of MD&A may be included in the MD&A, provided it replicates information contained elsewhere, such as in the letter of transmittal or in other forms of supplementary information. C. Any information presented within an ACFR in the form of an analysis of financial position must be replicated in the MD&A, even when provided elsewhere, such as in the letter of transmittal or in other forms of supplementary information. D. Governments should present any information they believe is needed to support their analysis of financial position and results of operations.

A. Governments should only present that information needed to support their analysis of financial position and results of operations prescribed by GASB Statement 34 for MD&A.

During 20X1, Brad Co. issued 5,000 shares of $100 par convertible preferred stock for $110 per share. One share of preferred stock can be converted into three shares of Brad's $25 par common stock at the option of the preferred shareholder. On December 31, 20X2, when the market value of the common stock was $40 per share, all of the preferred stock was converted. What amount should Brad credit to common stock and to additional paid-in capital common stock as a result of the conversion? A. Common stock: $375,000; Additional paid-in capital: $175,000 B. Common stock: $375,000; Additional paid-in capital: $225,000 C. Common stock: $500,000; Additional paid-in capital: $50,000 D. Common stock: $600,000; Additional paid-in capital: $0

A. Common stock: $375,000; Additional paid-in capital: $175,000 Correct Summary journal entry to record 20X1 issuance of convertible preferred stock: Dr. Cash (5,000 x $110) 550,000 CR. Convertible preferredstock (5,000 x $100) 500,000 Additional paid-in capitalpreferred ($550,000 - $500,000) 50,000 Summary journal entry to record 12/31/X2 conversion of preferred shares: Dr. Convertible preferred stock 500,000 Additional paid-in capital preferred 50,000 Cr. Common stock (5,000 x 3 x $25) 375,000 Additional paid-in capital common ($550,000 - $375,000) 175,000

Which of the following statements is correct concerning start-up costs? A. Costs of start-up activities, including organization costs, should be expensed as incurred. B. Costs of start-up activities, including organization costs, should be capitalized and amortized on a straight-line basis over the lesser of the estimated economic life of the company or 60 months. C. Costs of start-up activities should be capitalized and amortized on a straight-line basis over the lesser of the estimated economic life of the company or 60 months, while organization costs should be expensed as incurred. D. Costs of start-up activities, including organization costs, should be capitalized and expensed only if an impairment exists.

A. Costs of start-up activities, including organization costs, should be expensed as incurred.

On which of the following dates is a public entity required to measure the cost of employee services in exchange for an award of equity interests, based on the fair market value of the award? A. Date of grant B. Date of vesting C. Date of restriction lapse D. Date of exercise

A. Date of grant Both the intrinsic value method and the fair market value method use the grant date to measure the cost for stock issued to employees. At the grant date, one must compute a value for the shares used as compensation, which will be earned and taken as an expense over the service period.

How would a municipality that uses modified accrual and encumbrance accounting record the transaction of the receipt of supplies from approved purchase orders and the approval of the related invoices? A. Debit expenditures control. B. Credit other financing uses. C. Credit appropriations control. D. Debit encumbrances control.

A. Debit expenditures control.

Blythe Corp. is a defendant in a lawsuit. Blythe's attorneys believe it is reasonably possible that the suit will require Blythe to pay a substantial amount. What is the proper financial statement treatment for this contingency? A. Disclosed but not accrued B. No disclosure or accrual C. Accrued and disclosed D. Accrued but not disclosed

A. Disclosed but not accrued If the possibility that a company will be required to pay a contingent liability is reasonably possible, the liability is not required to accrue the liability. However, the nature of the liability and an estimate of the loss (or range of loss) must be disclosed.

Valley Town's public school system is administered by a separately elected board of education. The board of education is not organized as a separate legal entity and does not have the power to levy taxes or issue bonds. Valley's city council approves the school system's budget. How should Valley report the public school system's annual financial results? A. Discrete presentation, no; Blended, yes B. Discrete presentation, yes; Blended, yes C. Discrete presentation, yes; Blended, no D. Discrete presentation, no; Blended, no

A. Discrete presentation, no; Blended, yes Blending of financial results is allowed as the public school system and the city are not separate legal entities. The city is responsible for the finances of the school system (the school board has no authority to levy taxes or issue bonds). Discrete presentation is for affiliated entities whose resources are entirely for the benefit of the primary government. The school system does not operate for the sole benefit of the town.

In a statement of cash flows, which of the following would increase reported cash flows from operating activities using the direct method (Ignore income tax considerations.)? A. Dividends received from investments B. Gain on early retirement of bonds C. Gain on sale of equipment D. Change from straight-line to accelerated depreciation

A. Dividends received from investments

During the year, a city's electric utility, which is operated as an enterprise fund, rendered billings for electricity supplied to the general fund. Which of the following accounts should be debited by the general fund? A. Expenditures B. Due to electric utility enterprise fund C. Other financing uses—operating transfers out D. Appropriations

A. Expenditures This question illustrates interfund services provided and used, that is, sales and purchases of goods and services between funds for a price approximating their external exchange value. A governmental unit, like any organization, needs electric power in order to function, and so must procure and pay for that power. From the standpoint of the consuming fund, the cost of electricity used is an expenditure regardless of whether the supplier of the electricity is part of the same government or an independent supplier.

Which of the following types of funds use the economic resources measurement focus and the accrual basis of accounting? A. Fiduciary and proprietary funds B. Fiduciary, governmental, and proprietary funds C. Fiduciary and governmental funds D. Governmental funds

A. Fiduciary and proprietary funds The economic resources measurement focus and the accrual basis of accounting are used for proprietary funds and fiduciary funds. Proprietary funds consist of enterprise and internal service funds. GASB 1300.102.b indicates both enterprise and internal service funds use accrual accounting. The pension trust fund is a fiduciary fund. GASB 1300.102.b details that fiduciary funds use the accrual basis of accounting. The current financial resources measurement focus and the modified accrual basis of accounting are used for governmental funds. The general fund, capital projects fund, special revenue fund, and debt service fund are all governmental funds.

Which of the following subobjectives of accountability is "interperiod equity"? A. Financial reporting should provide information to determine whether current-year revenues were sufficient to pay for current-year services. B. Financial reporting should provide information to assist users in assessing the service efforts, costs, and accomplishments of the governmental entity. C. None of the answer choices are correct. D. Financial reporting should demonstrate whether resources were obtained and used in accordance with the entity's legally adopted budget.

A. Financial reporting should provide information to determine whether current-year revenues were sufficient to pay for current-year services. The GASB has established "accountability" as the cornerstone of financial reporting for governmental entities. Under GASB Concepts Statement 1, accountability consists of the following subobjectives: Interperiod equity: Financial reporting should provide information to determine whether current-year revenues were sufficient to pay for current-year services. Budgetary and fiscal compliance: Financial reporting should demonstrate whether resources were obtained and used in accordance with the entity's legally adopted budget; it should also demonstrate compliance with other finance-related legal or contractual requirements. Service efforts costs and accomplishments: Financial reporting should provide information to assist users in assessing the service efforts, costs, and accomplishments of the governmental entity. GASB Concepts Statement 1 Summary

Carlson City's fiscal year ends December 31. On August 1, the city issued a purchase order for new vehicles to be delivered at the rate of two per month beginning October 15. Twelve vehicles were delivered as scheduled and payments of $264,000 were made from the general fund upon delivery. If these were the only transactions made by the city, and the city started the year with just enough cash to complete these transactions, which of the following balances would appear on the balance sheet as of December 31? A. Fund balance unassigned 132,000 Fund balance--committed 132,000 B. Encumbrances 132,000 Reserve for encumbrances 132,000 C. Fund balance--committed 264,000 Fund balance--unassigned 264,000 D. Encumbrances 264,000 Reserve for encumbrances 264,000

A. Fund balance unassigned 132,000 Fund balance--committed 132,000 It appears the general fund started with a fund balance of $132,000 if it held only enough cash to pay for the six vehicles delivered in October, November, and December. The fund balance would have been debited $264,000 when expenditures were closed and outstanding encumbrances removed at year-end, leaving a debit balance of $132,000. The remaining encumbered amount would be disclosed as the fund balance category called "Fund balance—committed." (The city simply reversed encumbrances at year-end, and then displayed the ending fund balance in two amounts—committed and unassigned.) Further, the amount of fund balance set aside due to the outstanding encumbrances could be labeled "Fund balance—assigned" as well as "Fund balance—committed" depending on whether the original agreement to purchase the vehicles had been approved at the highest level of the government. The remaining fund balance, a deficit amount in this case, would be labeled "unassigned."

Which of the following is a minimum required report for the basic financial statements of a government entity? A. Fund financial statements B. Required supplementary information C. Management's discussion and analysis D. Budgetary comparison schedules

A. Fund financial statements

In the current year, Poplar City paid $5,000 interest and $20,000 principal on its outstanding general obligation bonds. The payment was made from a debt service fund using cash transferred earlier the same year from the general fund. How should the city report the expenditures? A. General fund: $0Debt service fund: $25,000Permanent fund: $0 B. General fund: $25,000Debt service fund: $5,000Permanent fund: $0 C. General fund: $0Debt service fund: $25,000Permanent fund: $20,000 D. General fund: $25,000Debt service fund: $25,000Permanent fund: $25,000

A. General fund: $0Debt service fund: $25,000Permanent fund: $0 The debt service fund is used to account for, and report payments related to, general government long-term debt. The fund includes payments related to principal on general long-term debt and associated interest charges on the debt.

In which of the following funds of a government are interfund transfers reported as "other financing sources or uses"? A. Governmental funds B. Proprietary funds C. Both governmental funds and proprietary funds D. Neither governmental funds nor proprietary funds

A. Governmental funds In governmental funds, interfund transfers should be reported as "other financing uses" in the fund making the transfer and as "other financing sources" in the fund receiving the transfer. In proprietary funds, which use full accrual accounting, transfers should be reported separately after nonoperating revenues and expenses. "Other financing sources and uses" is not an appropriate category for proprietary funds.

For which of the following funds do operating transfers affect the results of operations? A. Governmental funds, yes; Proprietary funds, yes B. Governmental funds, no; Proprietary funds, no C. Governmental funds, no; Proprietary funds, yes D. Governmental funds, yes; Proprietary funds, no

A. Governmental funds, yes; Proprietary funds, yes

For which of the following funds do operating transfers affect the results of operations? A. Governmental funds, yes; Proprietary funds, yes B. Governmental funds, no; Proprietary funds, yes C. Governmental funds, no; Proprietary funds, no D. Governmental funds, yes; Proprietary funds, no

A. Governmental funds, yes; Proprietary funds, yes Consider that if an item impacts operations, then it should be a factor in the final results of operations. Governmental and proprietary funds require a statement of revenues and expenditures; fiduciary funds do not.

Strut Co. has a payable to its parent, Plane Co. In which of the following balance sheets should this payable be reported separately? A. Strut's balance sheet B. Plane's consolidated balance sheet C. Both Strut's balance sheet and Plane's consolidated balance sheet D. Neither Strut's balance sheet nor Plane's consolidated balance sheet

A. Strut's balance sheet

FASB ASC 805-20-55-4 requires long-term customer-relationship intangible assets to be: A. subject to the same impairment loss recognition as other long-lived intangible assets that are held and used. B. not subject to impairment. C. subject to the same impairment loss recognition as other long-lived intangibles to be disposed of other than by sale. D. expensed when acquired.

A. subject to the same impairment loss recognition as other long-lived intangible assets that are held and used.

On June 30, 20X1, after paying the semiannual interest due and recording amortization of bond discount, Hake redeemed its 15-year, 8% $1,000,000 par bonds at 102. Hake has a policy to redeem bonds when it is advantageous to do so. The bonds, which had a carrying amount of $940,000 on January 1, 20X1, had originally been issued to yield 10%. Hake used the effective interest method of amortization and paid interest and recorded amortization on June 30. Compute the amount of gain or loss on the redemption of the bonds. A. Hake will record a loss of $73,000 on the redemption of the bonds. B. Hake will record a loss of $53,000 on the redemption of the bonds. C. Hake will record a loss of $80,000 on the redemption of the bonds. D. Hake will record a gain of $60,000 on the redemption of the bonds.

A. Hake will record a loss of $73,000 on the redemption of the bonds. The semiannual interest payment which was made by Hake for June 30, 20X1, was $40,000 ($1,000,000 × .08 ÷ 2). The interest expense was $47,000 ($940,000 × .10 ÷ 2) using the effective interest rate method. The amortization of bond discount was $7,000 ($47,000 - $40,000). The unamortized bond discount as of June 30, 20X1, was $53,000 given that the unamortized bond discount on January 1, 20X1, was $60,000 ($1,000,000 - $940,000). The carrying amount of the bonds as of January 1, 20X1, ($940,000) was given in the question.

Restorations of carrying value for long-lived assets are permitted if an asset's fair value increases subsequent to recording an impairment loss for which of the following? A. Held for disposal B. Held for use C. Both held for use and held for disposal D. Neither held for use nor held for disposal

A. Held for disposal A long-lived asset classified as held for sale (disposal) must be measured at the lower of its carrying amount or fair value less cost to sell. A loss should be recognized for any initial or subsequent write-down to fair value less cost to sell. A gain should be recognized for any subsequent increase in fair value less cost to sell, but not in excess of the cumulative loss previously recognized for a write-down to fair value less cost to sell.

An enterprise fund must be used when which of the following criteria are met? The activity is financed with debt that is secured solely by a pledge of the net revenues from fees of the activity. Goods or services are provided on a cost-reimbursement basis. Laws require that the cost of providing services be recovered with fees and charges, rather than with taxes. A. I and III B. II and III C. I and II D. I only

A. I and III Activities are required to be reported as enterprise funds if either I or III is true. In addition, if pricing policies of the activity establish fees and charges designed to recover costs, an enterprise fund must be used. The language describing funds as providing goods and services "on a cost-reimbursement basis" applies to internal service funds.

Greenburg's finance staff is preparing the statement of activities by checking classification of all expenses. The staff has been told that governments should report all expenses by function. They interpret this to mean that expenses that are required to be reported by function include: direct operating expenses. special and extraordinary expenses. indirect operating expenses. A. I only B. I and III C. I, II, and III D. I and II

A. I only The government-wide statement of activities should present direct operating expenses by function. Special and extraordinary items are reported separately at the bottom of the statement of activities. Extraordinary items, per GASB 2200.143, are transactions or other events that are both unusual in nature and infrequent in occurrence. Governments may allocate indirect expenses to benefiting functions, but they are not required to do so.

Which of the following statements is correct when a company applying the lower of cost or market method reports its inventory (accounted for under the LIFO method) at replacement cost? The original cost is less than replacement cost. The net realizable value is greater than replacement cost. A. II only B. I only C. Neither I nor II D. Both I and II

A. II only In order for replacement cost to be reported as the lower of cost or market inventory value: replacement cost would have to be lower than original cost (Statement I is not correct) and replacement cost would have to fall below net realizable value and above net realizable value minus a normal profit margin (Statement II is correct). Therefore, only statement II is correct. Note that inventory measured using any method other than LIFO or the retail inventory method (e.g., FIFO or average cost) is measured at the lower of cost and net realizable value (NRV), which is defined to be the estimated selling price in the ordinary course of business, less reasonably predictable costs of completion, disposal, and transportation. If the NRV of inventory is lower than its cost, the difference is recognized as a loss in earnings in the period in which it occurs.

A county acquired equipment through a lease agreement dated July 31, 20X1. The lease payments are to be financed with general government resources. Where should the noncurrent portion of the lease be reported in the June 30, 20X2, financial statements? A. In the government-wide statement of net position in the governmental activities column B. In the governmental funds balance sheet C. In the proprietary funds statement of net position D. In the government-wide statement of net position in the business-type activities column

A. In the government-wide statement of net position in the governmental activities column

A corporation issues quarterly interim financial statements and uses the lower of cost or market method to value its inventory in its annual financial statements. Which of the following statements is correct regarding how the corporation should value its inventory in its interim financial statements? A. Inventory losses generally should be recognized in the interim statements. B. Temporary market declines should be recognized in the interim statements. C. Only the cost method of valuation should be used. D. Gains from valuations in previous interim periods should be fully recognized.

A. Inventory losses generally should be recognized in the interim statements.

Which of the following transactions is included in the operating activities section of a cash flow statement prepared using the indirect method? A. Investment revenue recognized on an equity method investment B. Purchase of property, plant, and equipment C. Payment of principal on a long-term note payable D. Purchase of treasury stock

A. Investment revenue recognized on an equity method investment

Which of the following factors would not be an indicator of an investor's ability to exercise significant influence over the operating and financial policies of an investee? A. Investor recommendation for the investee to hire a specific executive B. Investor representation on the investee board of directors C. Dependence by the investee on the investor's proprietary technology D. Interchange of managerial personnel between investor and investee

A. Investor recommendation for the investee to hire a specific executive

Which of the following statements concerning patents is correct? A. Legal costs incurred to successfully defend an internally developed patent should be capitalized and amortized over the patent's remaining economic life. B. Legal fees and other direct costs incurred in registering a patent should be capitalized and amortized on a straight-line basis over a 5-year period. C. Research and development contract services purchased from others and used to develop a patented manufacturing process should be capitalized and amortized over the patent's economic life and not assessed for impairment. D. Research and development costs incurred to develop a patented item should be capitalized and amortized on a straight-line basis over 17 years.

A. Legal costs incurred to successfully defend an internally developed patent should be capitalized and amortized over the patent's remaining economic life. In general, the cost of internally developed patents should be expensed in the period incurred. An exception to this is the treatment of legal costs related to patents as required by FASB ASC 730-10-55-2. In a listing of examples of activities to be excluded from research and development treatment (i.e., expensed when incurred) is "i. Legal work in connection with patent applications or litigation, and the sale or licensing of patents." This means that legal costs related to the successful defense of internally developed patents should be capitalized and amortized over the patent's remaining economic life. FASB ASC 350-30-35-14 requires that intangibles subject to amortization also be assessed for impairment.

A company completes construction of a $400 million offshore oil platform and places it into service on January 1. State law requires that the platform be dismantled and removed at the end of its useful life, which is estimated to be 10 years. The company estimates that the cost of dismantling the platform will be $20 million. The discounted value of the liability is $9 million using the company's credit-adjusted, risk-free rate. The company has already capitalized the $400 million construction cost of the platform. What amounts should the company record as liability and expense when the asset is placed into service? A. Liability, $9,000,000; Expense, $0 B. Liability, $9,000,000; Expense, $9,000,000 C. Liability, $20,000,000; Expense, $20,000,000 D. Liability, $0; Expense, $0

A. Liability, $9,000,000; Expense, $0 The ARO liability is the discounted present value of the liability, or $9,000,000. Accretion expense is recorded at the end of the accounting period, not when the asset is place into service.

Ballard Corporation has a dispute with the IRS that will result in a probable additional payment of taxes. At the end of 20X7, Ballard recorded a liability of $480,000 related to the dispute. On February 14, 20X8, final resolution of the dispute was reached. The judgment required Ballard to pay additional taxes of $535,000 with no penalties or interest. What is the journal entry, if any, that Ballard Corporation should record for this event and include in the 20X7 financial statements that are scheduled to be issued on March 3, 20X8? A. Loss on tax settlement 55,000 Tax liability 55,000 B. Tax liability 55,000 Loss on tax settlement 55,000 C. Loss on tax settlement 535,000 Tax liability 535,000 D. No entry is needed to record the event.

A. Loss on tax settlement 55,000 Tax liability 55,000 The event occurring on February 14, 20X8, provided new information about a financial statement item that existed at the end of 20X7. Therefore, the effect of this event needs to be recorded in the financial statements to be issued in March of 20X8. Because the amount of tax Ballard must pay increased, an additional loss of $55,000 ($535,000 − $480,000) needs to be recorded along with an additional liability of $55,000. Additional information about this event should also be disclosed in the notes to the financial statements.

A U.S. publicly traded company's second fiscal quarter ends on March 31. If the company is an accelerated filer, what is the latest date that the Form 10-Q should be filed with the U.S. Securities and Exchange Commission (SEC)? A. May 10 B. May 15 C. May 30 D. June 29

A. May 10 SEC Form 10-Q is the quarterly report. Large accelerated filers ($700 million or more) and accelerated filers ($75 million or more and less than $700 million, and at least $100 million in annual revenues) must file their Form 10-Q within 40 days of quarter-end; non-accelerated filers (less than $75 million or less than $100 million in annual revenues) must file within 45 days. Therefore, an accelerated filer with a second fiscal quarter-end of March 31 must file by May 10.

Milton Co. pledged some of its accounts receivable to Good Neighbor Financing Corporation in return for a loan. Which of the following statements is correct? A. Milton will retain control of the receivables. B. Good Neighbor Financing cannot take title to the receivables if Milton does not repay the loan. Title can only be taken if the receivables are factored. C. Good Neighbor Financing will assume the responsibility of collecting the receivables. D. Good Neighbor Financing will take title to the receivables, and will return title to Milton after the loan is paid.

A. Milton will retain control of the receivables. Under a pledge, an account receivable is used as collateral for a loan. Milton continues to collect the receivables and applies the collection to the loan balance.

A company issued rights to its existing shareholders without consideration. The rights allowed the recipients to purchase unissued common stock for an amount in excess of par value. When the rights are issued, which of the following accounts will be increased? A. Neither common stock nor additional paid-in capital B. Common stock C. Additional paid-in capital D. Both common stock and additional paid-in capital

A. Neither common stock nor additional paid-in capital The issuance of rights was "without consideration" so no asset can be debited. This issuance should be recorded as a memo entry only. If, and when, the recipients exercise their rights at a later date, cash would be increased as well as common stock and additional paid-in capital. For now, however, none of the accounts would be increased.

Both Curry City and the State have a general sales tax on all merchandise. Curry City's tax rate is 2% and the State's rate is 4%. Merchants are required by law to remit all sales tax collected each month to the State by the 15th of the following month. By law, the State has 45 days to process the collections and to make disbursements to the various jurisdictions for which it acts as an agent. Sales tax collected by merchants in Curry totals $450,000 in May and $600,000 in June. Both merchants and the State make remittances in accordance with statutes. What amount of sales tax revenue for May and June is included in the June 30 year-end financial statements of the State and Curry? A. State: $700,000; Curry: $350,000 B. State: $1,050,000; Curry: $350,000 C. State: $300,000; Curry: $150,000 D. State: $1,050,000; Curry: $0

A. State: $700,000; Curry: $350,000 Distribution of the amount: The total percentage of the tax collected is 6% (4% + 2%). The amount to Curry City = $1,050,000 × 2/6 = $350,000 The amount to State = $1,050,000 × 4/6 = $700,000

Isle Co. owned a copy machine that cost $5,000 and had accumulated depreciation of $2,000. Isle exchanged the copy machine for a computer that cost $4,000. Isle's future cash flows are not expected to change significantly as a result of the exchange. What amount of gain or loss should Isle report and at what amount should it record the asset? A. No gain or loss in the income statement; $3,000 asset in the balance sheet B. No gain or loss in the income statement; $4,000 asset in the balance sheet C. $1,000 gain in the income statement; $3,000 asset in the balance sheet D. $1,000 gain in the income statement; $4,000 asset in the balance sheet

A. No gain or loss in the income statement; $3,000 asset in the balance sheet Normally, in a nonmonetary exchange, the asset received should be recorded at the fair value of the asset surrendered or the fair value of the asset received, whichever is more clearly evident. However, there are three exceptions in which the exchange is recorded at the carryover amount, not fair value. One of those exceptions is for exchange transactions that lack commercial substance, as does the exchange above. Isle should record the new copy machine at the carryover amount of $3,000 and not any gain or loss.

Which of the following is the characteristic of a perfect hedge? A. No possibility of future gain or loss B. No possibility of future gain only C. No possibility of future loss only D. The possibility of future gain and no future loss

A. No possibility of future gain or loss The purpose of a hedge is to reduce exposure to a particular type of risk. A perfect hedge would remove all of the risk—that is, remove the possibility of any future gain or loss.

Which of the following statements is correct regarding the provision for income taxes in the financial statements of a sole proprietorship? A. No provision for income taxes is required. B. The provision for income taxes should be based on business income using corporate tax rates. C. The provision for income taxes should be based on business income using individual tax rates. D. The provision for income taxes should be based on the proprietor's total taxable income, allocated to the proprietorship at the percentage that business income bears to the proprietor's total income.

A. No provision for income taxes is required. A sole proprietorship business is not a taxable entity. Business income (or loss) is "passed through" to the owner. Therefore, there would be no required provision for income taxes. Instead, the taxes would be paid by the owner on the proprietor's personal tax return.

The expenditure element "salaries and wages" is an example of which type of classification? A. Object B. Activity C. Program D. Function

A. Object In governmental accounting, expenditures should be recorded in a multiple classification scheme—typically by (1) fund, (2) function or program, (3) organizational unit (e.g., department), (4) activity, (5) character, and (6) object ("object of expenditure"). Object refers to "the type[s] of items purchased or services obtained" (GASB 1800.137) which expenditures are for—that is, "what" is acquired. Governments pay salaries and wages in order to acquire "personal services."

How should a local government's internal service fund report depreciation expense in its fund financial statements? A. Operating expense B. Nonoperating expense C. Separate from revenues and expense D. Not reported

A. Operating expense Depreciation for capital assets that are expected to be replaced through the internal service fund is recorded on the internal service fund's financial statements. Depreciation is recognized as an operating expense for the internal service fund because it relates to the services provided to the other funds.

Which of the following financial instruments may be considered a derivative financial instrument? A. Option contract B. Bank certificate of deposit C. Municipal bond D. Money market fund

A. Option contract The option contract is the only financial instrument listed that would be considered a derivative financial instrument. For an instrument to qualify as a derivative, it must have all three of the following characteristics: It has (a) one or more underlyings, (b) one or more notional amounts or payment provisions, or (c) both. It requires little or no initial net investment. Its terms require or permit net settlement, it can readily be settled net by a means outside the contract, or it provides for delivery of an asset that puts the recipient in a position not substantially different from net settlement. An option contract possesses all three characteristics. Municipal bonds, bank certificates of deposits, and money market funds at a minimum do not permit net settlement, nor do they require little or no initial net investment.

Which of the following statements is not correct? A. Performance obligations identified in a contract with a customer are limited to the goods or services explicitly stated in that contract. B. Promises implied by an entity's customary business practices can create a valid expectation by the customer that the entity will transfer goods or services to the customer. C. When determining the transaction price, an entity should consider the effects of variable consideration. D. Transaction price does not include estimates of consideration from the future exercise of options for additional goods or services.

A. Performance obligations identified in a contract with a customer are limited to the goods or services explicitly stated in that contract. Performance obligations are not limited to the goods or services explicitly stated in a contract. A performance obligation as a promise in a contract with a customer to transfer to the customer either a good or service that is distinct or a series of distinct goods or services that are substantially the same and that have the same pattern of transfer to the customer. That promise can be explicit, implicit, or based upon customary business practices. Implicit promises can create a valid expectation by the customer that the entity will transfer goods or services to the customer (e.g., free maintenance services). A portion of the transaction price must be allocated to any implied promise identified as a performance obligation.

Which of the following is a required financial statement for an investment trust fund? A. Statement of changes in fiduciary net position B. Statement of revenues, expenses, and changes in fiduciary net position C. Statement of revenues, expenditures, and changes in fiduciary net position D. Statement of activities

A. Statement of changes in fiduciary net position Investment trust funds are one of the fiduciary fund types for state and local governments. Two financial statements are required for fiduciary funds—a statement of fiduciary net position and a statement of changes in fiduciary net position.

On November 1, 20X1, Key Co. paid $3,600 to renew its only insurance policy for three years. On December 31, 20X1, Key's unadjusted trial balance showed a balance of $90 for prepaid insurance and $4,410 for insurance expense. What amounts should be reported for prepaid insurance and insurance expense in Key's December 31, 20X1, financial statements? A. Prepaid insurance: $3,400; Insurance expense: $1,100 B. Prepaid insurance: $3,300; Insurance expense: $1,200 C. Prepaid insurance: $3,490; Insurance expense: $1,010 D. Prepaid insurance: $3,400; Insurance expense: $1,200

A. Prepaid insurance: $3,400; Insurance expense: $1,100 At the end of the period, the expired insurance should be moved into expense, leaving only the unexpired portions in prepaid insurance. Prepaid insurance on November 1, 20X1 $3,600Less November and December expense2($3,600/36 months) 200Prepaid insurance on December 31, 20X1 $3,400======Insurance expense Total payments Prepaid insuranceon December 31, 20X1 = for insurance - balance= ($90 + $4,410) - $3,400= $4,500 - $3,400= $1,100 Note: Because the trial balance does not show a prepaid amount for the $3,600, and the $90 amount is too small to relate reasonably to the renewal policy, it can be assumed that the books need to be adjusted to transfer the correct prepaid amount ($3,400) from expense and expense the $90, which likely represents the expired remainder of the old policy.

U.S. Securities and Exchange Commission (SEC) regulations for the financial statement presentation and disclosure requirements of SEC filings can be found in: A. Regulation S-X. B. Regulation S-K. C. Regulation S-T. D. Regulation S-B.

A. Regulation S-X. SEC Regulation S-X prescribes the form, content, and presentation of and disclosure requirements for financial statements. SEC Regulation S-K contains the instructions for filing nonfinancial statement forms required under the Securities Act of 1933, Securities Exchange Act of 1934, and Energy Policy and Conservation Act of 1975. Regulation S-T governs the electronic submission of required forms. Due to the fact that the SEC requires electronic filing now, the instructions set forth in Regulation S-T may supersede those in Regulation S-K. Regulation S-B outlines the disclosure requirements that small business issuers must follow under the Securities Act of 1933 and Securities Exchange Act of 1934.

Which of the following accounts of a governmental unit is credited when taxpayers are billed for property taxes? A. Revenues B. Taxes receivable—current C. Appropriations D. Estimated revenues

A. Revenues Property taxes of a governmental unit can be accrued and recorded as a receivable and revenue when the property taxes are measurable and available. These revenues are considered measurable at the time the levy is approved by the legislative body. (The billing may occur immediately upon approval of the levy.) The availability criterion requires that the taxes be collected within the current period or soon enough thereafter to be used to pay liabilities of the current period. The length of time used to define available, often 60 days, should be disclosed in the notes to the financial statements. This problem assumes that the availability criterion is met. Therefore, the entry to record property taxes billed would credit revenues.

Perez, Inc., owns 80% of Senior, Inc. During 20X1, Perez sold goods with a 40% gross profit to Senior. Senior sold all of these goods in 20X1. For 20X1 consolidated financial statements, how should the summation of Perez and Senior income statement items be adjusted? A. Sales and cost of goods sold should be reduced by the intercompany sales. B. No adjustment is necessary. C. Sales and cost of goods sold should be reduced by 80% of the intercompany sales. D. Net income should be reduced by 80% of the gross profit on intercompany sales.

A. Sales and cost of goods sold should be reduced by the intercompany sales.

Where is the management's discussion and analysis (MD&A) required to be presented in the annual comprehensive financial report (ACFR)? A. Section 2 - Financial B. Section 1 - Introductory C. Section 3 - Statistical D. Any section; it is at the discretion of the governmental unit.

A. Section 2 - Financial The answer choice "Section 2 - Financial" is correct. GASB Statement 34 requires that the basic financial statements be preceded by management's discussion and analysis (MD&A).

The lower of cost or market rule for inventories may be applied to total inventory, to groups of similar items, or to each item. Which application generally results in the lowest inventory amount? A. Separately to each item B. Total inventory C. All applications result in the same amount. D. Groups of similar items

A. Separately to each item

Which of the following statements about the refinancing of short-term obligations is incorrect? A. Short-term obligations that are paid with cash before the balance sheet is released to the public can be refinanced and reclassified as long term. B. The amount that can be refinanced is limited to the amount actually refinanced or the amount specified in a refinancing agreement even though an actual refinancing did not occur. C. The ability to refinance the short-term obligations is a required criterion to reclassify current liabilities as long term. D. The intent to refinance the short-term obligations is a required criterion to reclassify current liabilities as long term.

A. Short-term obligations that are paid with cash before the balance sheet is released to the public can be refinanced and reclassified as long term.

Which of the following statements is correct regarding valuation allowances in accounting for income taxes? A. The effect of a change in the opening balance of a valuation allowance that results from a change of circumstances ordinarily is included in income from operations. B. Only negative evidence, not positive evidence, should be considered when determining whether a valuation allowance is needed. C. A valuation allowance is necessary when the realistic probability standard of evidence is satisfied. D. Both deferred tax assets and deferred tax liabilities can be reduced by a valuation allowance.

A. The effect of a change in the opening balance of a valuation allowance that results from a change of circumstances ordinarily is included in income from operations. GAAP provides that only deferred tax assets (not deferred tax liabilities) be reduced by a valuation allowance, but only if it is more likely than not (i.e., a likelihood of more than 50%) that some or all of the deferred tax assets will not be realized. The valuation allowance should reduce the deferred tax asset to the amount that is more likely than not to be realized, considering both positive and negative evidence. The effect of a change in the opening balance of a valuation allowance that results from a change of circumstances ordinarily is included in income from operations. All deferred tax liabilities and deferred tax assets are classified on the balance sheet as noncurrent.

An entity should estimate an amount of variable consideration in a contract by using either of the following methods, depending on which method the entity expects to better predict the amount of consideration to which it will be entitled: A. The expected value method or the most likely amount method B. The true value method or the fair value method C. The exact value method or the least likely amount method D. The estimated value method or the cost value method

A. The expected value method or the most likely amount method

Cuthbert Industrials, Inc., prepares 3-year comparative financial statements. In Year 3, Cuthbert discovered an error in the previously issued financial statements for Year 1. The error affects the financial statements that were issued in Years 1 and 2. How should the company report the error? A. The financial statements for Years 1 and 2 should be restated; the cumulative effect of the error on Years 1 and 2 should be reflected in the carrying amounts of assets and liabilities as of the beginning of Year 3. B. The financial statements for Years 1 and 2 should be restated; an offsetting adjustment to the cumulative effect of the error should be made to the comprehensive income in the Year 3 financial statements. C. The financial statements for Years 1 and 2 should not be restated; financial statements for Year 3 should disclose the fact that the error was made in prior years. D. The financial statements for Years 1 and 2 should not be restated; the cumulative effect of the error on Years 1 and 2 should be reflected in the carrying amounts of assets and liabilities as of the beginning of Year 3.

A. The financial statements for Years 1 and 2 should be restated; the cumulative effect of the error on Years 1 and 2 should be reflected in the carrying amounts of assets and liabilities as of the beginning of Year 3. A correction of an accounting error must be reported by restating the financial statements for all prior years. The carrying amounts for assets, liabilities, and beginning retained earnings must be restated for the earliest year presented in the financial statements presented in the year the error is discovered.

FASB ASU 2016-10 enhanced the guidance on the concept of distinctiveness. Which of the following factors is not accurate related to this concept? A. The goods or services are not interdependent or interrelated. B. One or more of the goods or services significantly modifies one or more of the other goods or services promised in the contract. C. An entity should consider whether the promise to transfer the good or service is separately identifiable. D. The entity is using the goods or services as inputs to produce or deliver the combined output or outputs specified by the customer.

A. The goods or services are not interdependent or interrelated. The following factors address the concept of distinctiveness: (1) the entity provides a significant service of integrating the goods or services with other goods or services into a bundle that represents the combined output or outputs for which the customer has contracted (i.e., the entity is using the goods or services as inputs to produce or deliver the combined output(s) specified by the customer; (2) one or more of the goods or services significantly modifies or customizes one or more of the other goods or services promised in the contract; and (3) the goods or services are highly interdependent or highly interrelated.

A U.S. company owns 80% of a non-U.S. company located in a foreign country with a very unstable political and economic climate. In which of the following situations should the U.S. company not consolidate its financial statements with the non-U.S. company? A. The government of the foreign country has recently imposed a number of severe sanctions and controls on all companies that are majority-owned by U.S. companies. B. The government of the foreign country has required all companies operating within its borders to implement International Financial Reporting Standards. C. The government of the foreign country has increased the tax rate for all companies that are majority-owned by U.S. companies. D. The foreign country has currency exchange rate fluctuations on a daily basis.

A. The government of the foreign country has recently imposed a number of severe sanctions and controls on all companies that are majority-owned by U.S. companies. This problem is testing the exception to the general rule that all subsidiaries controlled by a U.S. company are consolidated into the parent company's books. In the rare instance when a foreign country has imposed severe sanctions and controls on the U.S. company, effectively rendering it no longer in control of the parent despite the parent's controlling ownership, the U.S. parent may indicate that it does not control the subsidiary and therefore should not consolidate. Exchange rate fluctuations, changes in tax rates, and the implementation of IFRS (International Financial Reporting Standards) within the foreign borders are not reasons to avoid consolidation. A subsidiary may use IFRS for its own purposes as long as it is converted to U.S. GAAP for domestic (U.S.) reporting.

Which of the following statements about the impairment of intangible assets is correct? A. When considering impairment for intangible assets, only finite life intangible assets are tested for recoverability. B. When considering impairment for intangible assets, only indefinite life intangible assets are tested for recoverability. C. When considering impairment for intangible assets, both finite life and indefinite intangible assets are tested for recoverability. D. When considering impairment for intangible assets, neither finite life nor indefinite intangible assets are tested for recoverability.

A. When considering impairment for intangible assets, only finite life intangible assets are tested for recoverability.

If both an asset group in a company and goodwill in one of its reporting units have to be tested for impairment, which of the following statements is correct regarding impairment testing and impairment losses? A. The other asset group should be tested for an impairment loss before goodwill is tested. B. Impairment testing may be conducted concurrently for the other asset group and goodwill. C. If the other asset group is impaired, the loss should not be recognized prior to goodwill being tested for impairment. D. If goodwill is impaired, the loss should be recognized prior to testing the other assets for impairment.

A. The other asset group should be tested for an impairment loss before goodwill is tested. Goodwill must be tested for impairment at least annually. However, it must be tested between annual tests if an event occurs or circumstances change that would more likely than not reduce the fair value of a reporting unit below its carrying amount. Additionally, if the carrying amount of a reporting unit is zero or negative, goodwill of that reporting unit shall be tested for impairment on an annual or interim basis if an event occurs or circumstances exist that indicate that it is more likely than not that a goodwill impairment exists. If any other assets are to be tested for impairment (e.g., long-lived assets tested under FASB ASC 360-10-35) at the same time as goodwill is to be tested, those assets must be tested for impairment before goodwill is tested.

Which of the following is necessary for a government to use the modified approach for infrastructure capital assets? A. The system or subsystem of assets is managed using an asset management system with specified characteristics, and the government documents that the system or subsystem is being preserved at a target condition level determined by the reporting government. B. The government documents that the system or subsystem is being preserved at a target condition level determined by the reporting government. C. The system or subsystem of assets is managed using an asset management system with specified characteristics. D. None of the answer choices are correct.

A. The system or subsystem of assets is managed using an asset management system with specified characteristics, and the government documents that the system or subsystem is being preserved at a target condition level determined by the reporting government.

For a troubled debt restructuring involving only a modification of terms, which of the following items specified by the new terms would be compared to the carrying amount of the debt to determine if the debtor should report a gain on restructuring? A. The total future cash payments B. The present value of the debt at the original interest rate C. The present value of the debt at the modified interest rate D. The amount of future cash payments designated as principal repayments

A. The total future cash payments This question relates to the debtor's gain on troubled debt restructuring. FASB ASC 310-40-40-1 has changed the treatment of creditor's losses on a restructuring to include the use of present values. Debtor's gains, however, continue to follow FASB ASC 470-60-35-6. Debtor's gains are calculated based on undiscounted amounts. The total future cash payments, including interest, are used to compute the gain on troubled debt restructuring.

Which of the following phrases best describes a Level 1 input for measuring the fair value of an asset or liability? A. Unadjusted quoted prices for identical assets or liabilities in active markets B. Inputs that are principally derived from or corroborated by observable market data C. Quoted prices for similar assets or liabilities in active markets D. Inputs for the asset or liability based on the reporting entity's internal data

A. Unadjusted quoted prices for identical assets or liabilities in active markets The FASB's fair value hierarchy prioritizes the inputs to valuation techniques used to measure fair value into three broad levels, as follows: Level 1 inputs—quoted prices (unadjusted) in active markets for identical assets or liabilities that the reporting entity has the ability to access at the measurement date Level 2 inputs—inputs other than quoted prices included within Level 1 that are observable for similar assets or liabilities, either directly or indirectly Level 3 inputs—unobservable inputs for the asset or liability

If Company C is established for the merging of Company A and Company B, the business combination is classified as: A. statutory consolidation. B. an acquisition of assets. C. statutory merger. D. an acquisition of stock.

A. statutory consolidation. The statutory consolidation classification refers to the merging of two enterprises into a newly established enterprise.

Which of the following statements is correct as it relates to changes in accounting estimates? A. Whenever it is impossible to determine whether a change in accounting estimate or a change in accounting principle has occurred, the change should be considered a change in estimate. B. Whenever it is impossible to determine whether a change in an estimate or a change in accounting principle occurred, the change should be considered a change in principle. C. It is easier to differentiate between a change in accounting estimate and a change in accounting principle than it is to differentiate between a change in accounting estimate and a correction of an error. D. Most changes in accounting estimates are accounted for retrospectively.

A. Whenever it is impossible to determine whether a change in accounting estimate or a change in accounting principle has occurred, the change should be considered a change in estimate. FASB ASC 250-10-45-18 states: "Distinguishing between a change in an accounting principle and a change in an accounting estimate is sometimes difficult. In some cases, a change in accounting estimate is effected by a change in accounting principle. One example of this type of change is a change in method of depreciation, amortization, or depletion for long-lived, nonfinancial assets (hereinafter referred to as depreciation method). The new depreciation method is adopted in partial or complete recognition of a change in the estimated future benefits inherent in the asset, the pattern of consumption of those benefits, or the information available to the entity about those benefits. The effect of the change in accounting principle, or the method of applying it, may be inseparable from the effect of the change in accounting estimate. Changes of that type often are related to the continuing process of obtaining additional information and revising estimates and, therefore, shall be considered changes in estimates for purposes of applying this Subtopic." (Emphasis added)

In accounting for a long-term construction contract, assuming revenue is recognized over time, the progress billings on contracts account is a: A. contra current asset account. B. revenue account. C. contra noncurrent asset account. D. noncurrent liability account.

A. contra current asset account. The current asset account maintaining an inventory value for the costs and profits recognized so far on the contract has a contra account of progress billings, lowering its carrying value. If the billings exceed the construction in process, then a current liability can exist instead.

An employee of Hickory was granted an option to purchase 2,000 shares of Hickory's $7 par value common stock at $35 per share. This option was granted on January 2, 20X4. The option became exercisable on December 31, 20X6, after the employee had completed three years of service, and was exercised on that date when the market price of the Hickory's stock was $42. The fair value of the option on January 2, 20X4, was $8.50 per option. Hickory's entry to record the exercise of the stock options will include a: A. credit to Additional Paid-in Capital-Common Stock for $73,000. B. credit to Common Stock for $84,000. C. debit to Additional Paid-in Capital-Common Stock for $73,000. D. credit to Additional Paid-in Capital-Stock Options for $17,000.

A. credit to Additional Paid-in Capital-Common Stock for $73,000. When stock options are exercised in full, the amount recorded to Additional Paid-in Capital—Stock Options is equal to the total compensation expense recorded for the stock options (2,000 × $8.50 = $17,000). Cash is recorded for the amount of the exercise price times the number of options exercised (2,000 × $35 = $70,000). The issuance of the new shares is also recorded, at par (2,000 × $7 = $14,000). The journal entry for the exercise of the options granted by Hickory is as follows: Cash 70,000Additional Paid-in Capital—Stock Options 17,000Common Stock 14,000Additional Paid-in Capital—Common Stock 73,000

Marta City's school district is a legally separate entity, but two of its seven board members are also city council members and the district is financially dependent upon the city. The school district's financial activity should be reported in the city's financial statements by: A. discrete presentation. B. primary government. C. note disclosure. D. blended unit.

A. discrete presentation. Because the school district is financially dependent on the city, it is considered a component unit, not a separate primary government. Component unit financial information is generally included in the financial statements of the primary government but presented separately from the financial data of the primary government. Because only two of the seven school board seats are occupied by council members, the governing body of the school board is not "substantially the same" as the city council. Thus, the blending method is not required. Discrete presentation should be used unless the financial activities of the two entities are so intertwined as to make them substantially the same entity.

Impaired long-lived assets to be disposed of by sale that are subject to the reporting requirements of FASB ASC 360-10-35 are measured at: A. lower of the fair value less costs to sell or carrying amount. B. fair value. C. carrying amount. D. historical cost.

A. lower of the fair value less costs to sell or carrying amount.

The basic accounting principle that states that income or loss is determined by a process of associating realized revenues with those expenses necessary to generate them is the principle of: A. matching principle. B. going concern. C. disclosure. D. revenue realization.

A. matching principle.

The debt service fund of a governmental unit is used to account for the accumulation of resources for, and the payment of, principal, and interest in connection with: A. neither a pension trust fund nor a proprietary fund. B. both a pension trust fund and a proprietary fund. C. a pension trust fund. D. a proprietary fund.

A. neither a pension trust fund nor a proprietary fund. Debt service funds are governmental funds used to account for the accumulation of resources for, and the repayment of, general obligation long-term debt principal and interest. Debt accounted for in a pension trust or a proprietary fund is serviced from that fund and is not general long-term debt.

The original cost of an inventory item accounted for under the LIFO method is below the net realizable value and above the net realizable value less a normal profit margin. The inventory item's replacement cost is below the net realizable value less a normal profit margin. Under the lower of cost or market method, the inventory item should be valued at: A. net realizable value less normal profit margin. B. original cost. C. replacement cost. D. net realizable value.

A. net realizable value less normal profit margin.

A company is obligated to pay a specified amount to a supplier even if it does not take delivery of the contracted goods. This type of commitment is: A. not reported on the balance sheet but disclosed in the notes to the financial statements. B. recorded and reported on the balance sheet at the fair value of the goods to be received. C. recorded and reported on the balance sheet at the present value of the future required payments. D. not reported or disclosed in the financial statements.

A. not reported on the balance sheet but disclosed in the notes to the financial statements. When a company is obligated to pay a specified amount to a supplier even if it does not take delivery of the contracted goods, it has an unconditional purchase commitment. Such an obligation is not reported on the balance sheet but is disclosed in the notes to the financial statements at the present value of the future required payments.

Basic earnings per share for income from continuing operations and for net income are reported: A. on the face of the income statement. B. in the notes to the financial statements. C. for the current period only. D. if diluted earnings per share are presented.

A. on the face of the income statement.

A municipality that uses modified accrual and encumbrance accounting would use the general fund to account for: A. property tax revenues. B. payment of interest and principal on tax supported debt. C. revenues from earmarked sources to finance designated activities. D. major construction activities.

A. property tax revenues. The general fund is defined as the fund that should be used to account for all financial resources except those that are required to be accounted for in another fund. While at first this may seem like the General Fund is "residual" in nature, actually, as its name implies, the General Fund typically is the dominant fund within a city's financial structure. The General Fund is normally used to account for many of a city's general government activities. In most jurisdictions, property tax revenues are considered "general revenues" and so should be accounted for in the General Fund. (In contrast, major construction activities would be accounted for in Capital Projects Funds, payment of interest and principal on tax-supported debt would normally be accounted for in a Debt Service Fund, and revenues from earmarked sources to finance designated activities would be accounted for in a Special Revenue Fund.)

In preparing combined financial statements for a governmental entity, interfund receivables and payables should be: A. reported as amounts due to and due from other funds. B. reported as additions to or reductions from the unassigned fund balance. C. reported as reservations of fund balance. D. eliminated.

A. reported as amounts due to and due from other funds. Interfund loans should be reported as interfund receivables in lender funds and interfund payables in borrower funds, not affect the fund balance or net position of either fund involved, and not be reported as "other financing sources or uses" in the governmental fund financial statements. The transaction should be reported as a transfer from the fund that made the loan to the fund that received the loan.

Contractual asset or liability disclosures identified in Statement of Financial Accounting Concepts 8 (SFAC 8), Chapter 8, include all of the following except: A. reporting segments. B. legal terms. C. degree of nonperformance risk. D. method used to calculate the cash flow.

A. reporting segments.

Reporting of general infrastructure assets by all public institutions that report as special-purpose governments either engaged only in governmental activities or engaged in both governmental and business-type activities is: A. required using the full governmental model. B. required beginning with fiscal years ending after June 15, 2006. C. required using the special provisions of GASB Statement 35 for phase 3 public institutions. D. encouraged but not required.

A. required using the full governmental model. Public institutions that report as special-purpose governments either engaged only in governmental activities or engaged in both governmental and business-type activities should report infrastructure using the provisions of GASB Statement 34 codified as GASB Sp20.104. These provisions include the reporting of capital assets that are defined in GASB 1400.103 to include infrastructure.

The notes to the financial statements for not-for-profit entities (NFPs) are an integral part of the: A. statement of financial position, statement of activities, and statement of cash flows. B. statement of changes in assets, statement of activities, and statement of cash flows. C. statement of financial position, statement of activities, and statement of equity. D. statement of financial position, statement of cash flows, and statement of natural expenses.

A. statement of financial position, statement of activities, and statement of cash flows. The notes to a not-for-profit entity's financial statements are considered an integral part of those statements, which include only the statement of financial position, the statement of activities, and the statement of cash flows.

Options to purchase common stock are excluded from the computation of diluted EPS if: A. their exercise price is greater than the average market price. B. they are employee compensation and the employee may not be able to sell the stock until some future date. C. their exercise price is less than the average market price. D. they are issued as part of employee compensation arrangements.

A. their exercise price is greater than the average market price.

In preparing its cash flow statement for the year ending December 31, 20X1, Trove Co. collected the following data: Loss on sale of equipment $ 4,000 Dividends declared (63,000) Sale of Bly, Inc., bonds(par value $110,000) 115,000Amortization of bond premium (3,000) Purchase of equipment (73,000) Dividends paid (81,000) Purchase of treasury stock (39,000) In its December 31, 20X1, statement of cash flows, what amount should Trove report as net cash provided (used) by financing activities? A. $(39,000) B. $(120,000) C. $(42,000) D. $(81,000)

B. $(120,000) Trove should report $(120,000) as net cash provided (used) by financing activities, calculated as follows: Cash inflows from financing activities:No listed items are inflows $ 0Cash outflows from financing activities:Dividends paid (81,000)Purchase of treasury stock (39,000)Net cash provided by financing activities $(120,000) Dividends declared created a liability, but until they are paid, no cash flows out of the corporation and they will not appear on the cash flow statement until then. Purchase of equipment and the sale of bonds held for investment are investing activities, not financing activities. Loss on sale and amortization of premium are operating activities.

Last year, Katt Co. reduced the carrying amount of its long-lived assets used in operations from $120,000 to $100,000, in connection with its annual impairment review. During the current year, Katt determined that the fair value of the same assets had increased to $130,000. What amount should Katt record as restoration of previously recognized impairment loss in the current year's financial statements? A. $20,000 B. $0 C. $10,000 D. $30,000

B. $0 After an impairment loss is recognized, the reduced carrying amount of the asset should be treated as the new cost and the restoration of the impairment is not recognized.

Green Co. was preparing its year-end financial statements. Green had a pending lawsuit against a competitor for $5,000,000 in damages. Green's attorneys indicate that obtaining a favorable judgment was probable and the amount of damages is reasonably estimated. Green incurred $100,000 in legal fees. The income tax rate was 30%. What amount, if any, should Green recognize as a contingency gain in its financial statements? A. $3,430,000 B. $0 C. $4,900,000 D. $3,500,000

B. $0 Gain contingencies are not recognized since this would be the recognition of revenue prior to realization.

A company has a long-lived asset with a carrying value of $120,000, expected future cash flows of $130,000, present value of expected future cash flows of $100,000, and a market value of $105,000. What amount of impairment loss should be reported? A. $20,000 B. $0 C. $15,000 D .$5,000

B. $0 If the sum of the estimated future cash flows (undiscounted and without interest charges) is less than the carrying amount of the asset, the entity may have to recognize an impairment loss. The impairment loss, if any, to be recognized is any excess of the asset's carrying amount over its fair value. Notice, however, that no impairment loss is to be recognized unless the asset's estimated future cash flows (ECF) are less than its carrying amount, even if the asset's carrying amount (CA) exceeds its fair value (FV). Since the estimated future cash flows ($130,000) are not less than the carrying value ($120,000), no impairment loss must be recognized.

On January 2 of the current year, Cruises, Inc. borrowed $3,000,000 at a rate of 10% for three years and began construction of a cruise ship. The note states that annual payments of principal and interest in the amount of $1,300,000 are due every December 31. Cruises used all proceeds as a down payment for construction of a new cruise ship that is to be delivered two years after start of construction. What should Cruise report as interest expense related to the note in its income statement for the second year? A. $900,000 B. $0 C. $300,000 D. $600,000

B. $0 Incorrect The cruise ship qualifies for interest capitalization. Qualifying assets, per FASB ASC 835-20-15-5, include "assets that are constructed or otherwise produced for an entity's own use (including assets constructed or produced for the entity by others for which deposits or progress payments have been made)."

Brill Co. made the following expenditures during 20X1: Costs to develop computer software for internal use in Brill's general management information system $100,000 Costs of market research activities 75,000 What amount of these expenditures should Brill report in its 20X1 income statement as research and development expenses? A. $75,000 B. $0 C. $175,000 D. $100,000

B. $0 Neither of these costs meets the definition of research and development cost: Development of software for internal use is likely excluded from the applicability of FASB ASC 730-10-15-5. Marketing research is specifically excluded from the definition of research and development by FASB ASC 730-10-15-4. Research and development costs are defined as the "planned research...for new knowledge" and "the translation of research findings...into a...design for a new product or process." (FASB ASC 730-10-20)

During 20X1, Young and Zinc maintained average capital balances in their partnership of $160,000 and $100,000, respectively. The partners receive 10% interest on average capital balances, and residual profit or loss is divided equally. Partnership profit before interest was $4,000. What amount should Zinc's capital account change for the year? A. $2,000 increase B. $1,000 decrease C. $12,000 increase D. $11,000 decrease

B. $1,000 decrease Interest awarded to the partners based on average capital account balances is added to their capital accounts, and deducted from partnership profits. The remaining amounts are divided equally among the partnership capital accounts (as agreed). Alloc. to Alloc. to Total Young Zinc AllocatedProfit before interest $ 4,000Interest allocationTo Young (10% x $160,000) $16,000 (16,000)To Zinc (10% x $100,000) $10,000 (10,000)Residual allocation (1) (22,000)To Young (50% x $22,000) (11,000) 11,000To Zinc (50% x $22,000) -- (11,000) 11,000 Increase in Young capital $ 5,000 Decrease in Zinc capital (1,000) 1 Residual is $4,000 - $16,000 - $10,000 = $(22,000)

Papillon Corp. sold goods to its 90%-owned subsidiary, Trook Corp, during 20X6. At the end of 20X6, 1/4th of these goods were included in Trook's ending inventory. In its income statement for 20X6, Papillon reported freight-out expenses of $790,000, none of which was paid on sales made to Trook. Trook reported $375,000 of freight-out expenses for the same period. Trook's freight-out expenses included $103,000 in freight costs paid to ship goods to Papillon. What amount of selling expenses should be reported in Papillon's 20X6 consolidated income statement? A. $1,165,000 B. $1,062,000 C. $1,072,300 D. $1,113,500

B. $1,062,000 Since freight-out costs between Trook and Papillon are paid by the seller (Trook), they are not included in the value of inventory by the buyer (Papillon). Also, since they were paid on an intercompany sale, these costs should be eliminated from Papillon's consolidated income statement. Thus, consolidated selling expenses for 20X6 are as follows: Papillon total + Trook's total - Trook's Intercompany ($790,000) + ($375,000 - $103,000) $790,000* + $272,000 = $1,062,000 * Papillon's full total is included in consolidated selling expenses because none of the freight-out expense is related to sales to Trook.

Blue Corp.'s December 31, 2005, balance sheet contained the following items in the long-term liabilities section: 9-3/4% registered debentures(callable in 2010 and due in 2015) $700,000 9-1/2% collateral trust bonds (convertible into common stock beginning in 2008 and due in 2018) 600,000 10% subordinated debentures ($30,000 maturing annually beginning in 2005) 300,000 What is the total amount of Blue's term bonds? A. $700,000 B. $1,300,000 C. $600,000 D. $1,000,000

B. $1,300,000 Term bonds are bonds which are scheduled to be outstanding for a fixed period of time, or term.

Sayon Co. issues 200,000 shares of $5 par value common stock to acquire Trask Co. in an acquisition-business combination. The market value of Sayon's common stock is $12. Legal and consulting fees incurred in relationship to the purchase are $110,000. Registration and issuance costs for the common stock are $35,000. What should be recorded in Sayon's additional paid-in capital account for this business combination? A. $1,545,000 B. $1,365,000 C. $1,400,000 D. $1,255,000

B. $1,365,000 Under FASB ASC 805-20-25, the acquisition will be recorded at fair value: Investment in Trask Co. 2,400,000Common stock ($5 x 200,000) 1,000,000Additional paid-in capital ($7 x 200,000) 1,400,000 Legal and consulting costs are a current expense: Professional services expenses 110,000Cash 110,000 Registration and issuance costs reduce Additional Paid-in Capital: Additional paid-in capital 35,000Cash 35,000 The effect on Additional Paid-in Capital is: Stock issue $1,400,000Registration and issuance costs (35,000)$1,365,000

On January 2, 20X1, Lem Corp. bought machinery under a contract that required a down payment of $10,000, plus 24 monthly payments of $5,000 each, for total cash payments of $130,000. The cash equivalent price of the machinery was $110,000. The machinery has an estimated useful life of 10 years and estimated salvage value of $5,000. Lem uses straight-line depreciation. In its 20X1 income statement, what amount should Lem report as depreciation for the machinery? A. $12,500 B. $10,500 C. $11,000 D. $13,000

B. $10,500 (cash equivalent - salvage value) /years

On October 1 of the current year, a U.S. company sold merchandise on account to a British company for 2,000 pounds (exchange rate: 1 pound = $1.43). At the company's December 31 fiscal year-end, the exchange rate was 1 pound = $1.45. The exchange rate was 1 pound = $1.50 on collection in January of the subsequent year. What amount would the company recognize as a gain (loss) from foreign currency translation when the receivable is collected? A. $(140) B. $100 C. $140 D. $0

B. $100 The company would recognize a $100 gain: January (2,000 pounds x $1.50) $3,000December (2,000 pounds x $1.45) 2,900Gain $ 100

Smythe Co. invested $200 in a call option for 100 shares of Gin Co. $0.50 par common stock, when the market price was $10 per share. The option expired in three months and had an exercise price of $9 per share. What was the intrinsic value of the call option at the time of initial investment? A. $200 B. $100 C. $900 D. $50

B. $100 The intrinsic method is the excess of the market price over the exercise price. Market price (100 x $10) $1,000Exercise price (100 x $9) 900Intrinsic value $ 100

On January 1, Year 1, Alpha Co. signed an annual maintenance agreement with a software provider for $15,000 and the maintenance period begins on March 1, Year 1. Alpha also incurred $5,000 of costs on January 1, Year 1, related to software modification requests that will increase the functionality of the software. Alpha depreciates and amortizes its computer and software assets over five years using the straight-line method. What amount is the total expense that Alpha should recognize related to the maintenance agreement and the software modifications for the year ended December 31, Year 1? A. $5,000 B. $13,500 C. $16,000 D. $20,000

B. $13,500 The annual expenses would be the $15,000 maintenance contract multiplied by 10/12 of the year covered, or $15,000 × 10/12 = $12,500 from March to the end of the year. Also, expenses would cover 1/5 ($1,000) of the $5,000 from the other costs for one of the five years: $12,500 + $1,000 = $13,500 total.

Kauf Co. had the following amounts related to the sale of consignment inventory: Cost of merchandise shipped to consignee $72,000Sales value for 2/3rds of inventory sold by consignee 80,000Freight cost for merchandise shipped 7,500Advertising paid for by consignee, to be reimbursed 4,50010% commission due the consignee for the sale 8,000 What amount should Kauf report as net profit(loss) from this transaction for the year? A. $32,000 B. $14,500 C. $8,000 D. $(12,000)

B. $14,500 The sale was for $80,000, but a 10% commission of $8,000 was paid, so the net revenue was $72,000. The cost of the goods sold was 2/3rds of cost and freight of $72,000 and $7,500 (2/3rds of $79,500, or $53,000). The gross profit would be $72,000 less $53,000, or $19,000. The net profit is gross profit less the advertising of $4,500, thus $14,500.

Brill Co. made the following expenditures relating to Product X: Labor and material costs incurred in producing a prototype $100,000 Cost of testing the prototype 40,000 Legal costs to file a patent 5,000 Production of Product X commenced when the patent was granted. What amount of the above costs should be expensed as research and development costs? A. $145,000 B. $140,000 C. $100,000 D. $40,000

B. $140,000 U.S. GAAP defines research and development costs as costs that will be useful in developing a new product or service and the translation of research into a plan or design for a new product. The costs relate to activities identified with the period prior to the beginning of commercial production. Labor and materials developing a prototype and the cost of testing the prototype before commercial feasibility would represent research and development costs and would be expensed as incurred in their full amounts of $140,000 ($100,000 + $40,000). The legal costs to file the successful patent would be capitalized as an intangible asset subject to amortization.

Nutmeg Corporation has a reporting unit that has goodwill with a carrying amount of $175,000 at December 31, 20X2. Nutmeg Corporation is considered the reporting unit. The carrying value of the reporting unit's net assets (including goodwill) on December 31, 20X2, is $1,065,000. At that same date, the fair value of the assets and liabilities (excluding goodwill) of the reporting unit is $785,000. The fair value of the reporting unit as a whole is $920,000 at December 31, 20X2. The goodwill impairment loss reportable is: A. $175,000. B. $145,000. C. $135,000. D. $40,000.

B. $145,000. Impairment of goodwill is a one-step process: Compare: (a) Year-end fair value of reporting unit $ 920,000(b) Carrying amount, including goodwill 1,065,000Difference $ 145,000 The fair value of the reporting unit as a whole, including goodwill, must be considered. If (b) exceeds (a), the difference is the goodwill impairment; if (a) exceeds (b), there is no impairment. In this case, (b) exceeds (a), so the goodwill impairment loss is $145,000. Note: The impairment loss cannot exceed the carrying value of goodwill.

On September 29, 20X1, Wall Co. paid $860,000 for all the issued and outstanding common stock of Hart Corp. On that date, the carrying amounts of Hart's recorded assets and liabilities were $800,000 and $180,000, respectively. Hart's recorded assets and liabilities had fair values of $840,000 and $140,000, respectively. In Wall's September 30, 20X1, balance sheet, what amount should be reported as goodwill? A. $180,000 B. $160,000 C. $20,000 D. $240,000

B. $160,000 When, in the purchase of another company, the purchase price exceeds the fair value of all the assets the purchased company owns, then the excess is goodwill.

Aqua Aquarium offers a discount on its extended warranty over the aquarium provided the warranty is purchased at the same time as the aquarium tank. The warranty costs $200, but Aqua offers it for $150 when purchased with a tank. Aqua expects an 80% chance that a customer will purchase the extended warranty with the tank. Aqua sells 500 tanks along with the extended warranty discount offer. What is the total extended warranty discount option for purposes of allocating revenue among the performance obligations in those contracts? A. $25,000 B. $20,000 C. $80,000 D. $60,000

B. $20,000 the customer receives a discount for purchasing a bundle of goods or services), an entity should allocate the discount to all of the performance obligations on a relative standalone selling price basis. The warranty costs $200, but Aqua offers it for $150 when purchased with a tank. Aqua expects an 80% chance. Aqua sells 500 tanks along with the extended warranty discount offer. The $50 discount ($200 − $150) has an 80% chance of being taken by a customer, so the discount option associated with 500 tanks is $20,000 (computed as $50 × 80% × 500).

A company owns land and a building that houses its manufacturing operations. When the company purchased the manufacturing facility 10 years ago, the purchase price allocated to the land account was $120,000. The manufacturing facility is located in an area that was once the site of many factories. The owners of many of the neighboring factories have recently sold their facilities to residential real estate developers. The company's land is also suitable for residential development. The estimated current value of the land as part of the manufacturing facility is $150,000. The estimated current value of the land as an undeveloped investment is $130,000, and the current value of the land as part of a residential development would be $180,000. What is the fair value of the land? A. $130,000 B. $180,000 C. $120,000 D. $150,000

B. $180,000 FASB ASC 820 defines fair value as "the price that would be received to sell an asset or paid to transfer a liability in an orderly transaction between market participants at the measurement date." Fair value measurement assumes that the transaction occurs in the principal market for the asset or liability. If there is no principal market for that type of asset or liability, the entity should use the most advantageous market for that asset or liability; therefore, the fair value of the land should be determined using the residential market, which is $180,000.

Kern and Pate are partners with capital balances of $60,000 and $20,000, respectively. Profits and losses are divided in the ratio of 60:40. Kern and Pate decided to form a new partnership with Grant, who invested land valued at $15,000 for a 20% capital interest in the new partnership. Grant's cost of the land was $12,000. The partnership elected to use the bonus method to record the admission of Grant into the partnership. Grant's capital account should be credited for: A. $15,000. B. $19,000. C. $16,000. D. $12,000.

B. $19,000. In the bonus-to-new-partner method, one needs to compute the total value now for the partnership, whereupon the new partner's capital account will be based on the new partner's percentage of the total just computed (even if the new partner's contribution does not seem to justify the amount). Total capital of new partnership($60,000 + $20,000 + $15,000) $95,000Times capital credit percentage to Grant x .20Equals capital credit allowed to Grant $19,000======= Note: Grant is given a "bonus" equal to $4,000, the excess of his $19,000 capital credit over the $15,000 fair value of land invested.

On January 2, 20X1, Paye Co. purchased Shef Co. at a cost that resulted in recognition of goodwill of $200,000 having an expected benefit period of 10 years. During the first quarter of 20X1, Paye spent an additional $80,000 on expenditures designed to maintain goodwill. Due to these expenditures, on December 31, 20X1, Paye estimated that the benefit period of goodwill was 40 years. For 20X1, Paye assessed impairment to be $7,000. In its December 31, 20X1, balance sheet what amount should Paye report as goodwill? A. $273,000 B. $193,000 C. $180,000 D. $195,000

B. $193,000 FASB ASC 350-20-25-3 provides that any costs of developing, maintaining, or restoring goodwill should be deducted from income when incurred. Obviously, the $80,000 expenditure falls into this category. The $200,000 of purchased goodwill should be assessed for impairment each year. Initial cost of goodwill $200,000Less: 20X1 impairment 7,000Unamortized amount on December 31, 20X1 $193,000

On November 2, 20X1, Platt Co. entered into a 90-day futures contract to purchase 50,000 Swiss francs when the contract quote was $0.70. The purchase was for speculation in price movement. The following exchange rates existed during the contract period: 30-Day Futures Spot Rate November 2, 20X1 $.62 $.63 December 31, 20X1 .65 .64 January 30, 20X2 .65 .68 What amount should Platt report as foreign currency exchange loss in its income statement for the year ended December 31, 20X1? A. $4,000 B. $2,500 C. $3,500 D. $3,000

B. $2,500 Futures contracts are a selected type of derivative instrument. All derivatives must be recognized on the balance sheet at fair value. Fair value is $0.70 on November 2, 20X1. Accounting for the changes in fair value depends on whether it has been designated as and qualifies for hedge accounting. Platt Co. has not hedged the risk of the futures contract and FASB ASC 815-20-35-1 specifies that gains and losses must be included in income for these contracts. Since this is a futures contract, the future 30-day rate ($0.65) is used to measure the gain or loss for the year ended December 31, 20X1. The foreign currency exchange loss for 20X1 is ($.70 - $.65) × 50,000 = $2,500.

Brand Co. incurred the following research and development project costs at the beginning of the current year: Equipment purchased for current and future projects $100,000 Equipment purchased for current projects only 200,000 Research and development salaries for current project 400,000 Equipment has a 5-year life and is depreciated using the straight-line method. What amount should Brand record as depreciation for research and development projects at December 31? A. $0 B. $20,000 C. $60,000 D. $140,000

B. $20,000 Only the equipment that has an alternative use is capitalized and depreciated. The depreciation is $100,000 ÷ 5 = $20,000. The other expenditures should be expensed immediately. FASB ASC 730-10-25-2 states: "Elements of costs shall be identified with research and development activities as follows (see [FASB ASC] 350-50 for guidance related to website development): "Materials, equipment, and facilities. The costs of materials (whether from the entity's normal inventory or acquired specially for research and development activities) and equipment or facilities that are acquired or constructed for research and development activities and that have alternative future uses (in research and development projects or otherwise) shall be capitalized as tangible assets when acquired or constructed. The cost of such materials consumed in research and development activities and the depreciation of such equipment or facilities used in those activities are research and development costs. However, the costs of materials, equipment, or facilities that are acquired or constructed for a particular research and development project and that have no alternative future uses (in other research and development projects or otherwise) and therefore no separate economic values are research and development costs at the time the costs are incurred."

On January 1, year 1, a company granted some of its key employees stock options for 100,000 shares of $3 par common stock when the fair value of each option was $6 per share. The options vest after 3 years of service. What is the compensation expense, if any, for the year ended December 31, year 1? A. $100,000 B. $200,000 C. $0 D. $600,000

B. $200,000 6*100000=600000 600000/3=200000

On April 1, 20X2, Hill Corp. issued 200 of its $1,000 face value bonds at 101 plus accrued interest. The bonds were dated November 1, 20X1, and bear interest at an annual rate of 9% payable semiannually on November 1 and May 1. What amount did Hill receive from the bond issuance? A. $194,500 B. $209,500 C. $202,000 D. $200,000

B. $209,500 The bonds themselves were sold for 101% of face value, but they were issued between interest payment dates, and thus five months of accrued interest was also received, and will be paid back (along with another month's interest) at the scheduled interest payment date. Sales price of bonds = 1.01 x 200 x $1,000 = $202,000Accrued interest = (5/12) (.09 x 200 x $1,000) = 7,500Total received from bond issuance $209,500

On October 1, Year 2, Park Co. purchased 200 of the $1,000 face amount, 10% bonds of Ott, Inc., for $220,000, including accrued interest of $5,000. The bonds, which mature on January 1, Year 9, pay interest semiannually on January 1 and July 1. Park used the straight-line method of amortization and appropriately recorded the bonds as a long-term investment. On Park's December 31, Year 3, balance sheet, the bonds should be reported at: A. $214,400. B. $212,000. C. $215,000. D. $214,200.

B. $212,000. Since the purchase price was $220,000 and it included the accrued interest of $5,000, the price for the bonds was $215,000 ($220,000 - $5,000). The bonds were thus sold at a premium of $15,000 (the price of $215,000 less the face of $200,000, computed as 200 × $1,000). Applying straight-line, the premium will be amortized equally over the months remaining in the bond's term. The bond was bought on October 1 of Year 2, and will mature on January 1 of Year 9. That is a remaining term to maturity of 75 months (3 months of Year 2 and all of Years 3, 4, 5, 6, 7, and 8 ((6 × 12) + 3 = 75 months)). The total premium of $15,000 divided equally by 75 months is $200 per month. Thus, the bond carrying amount at December 31, Year 3, after the bond is held for 15 months (3 months in Year 2, and all of Year 3) will be: $215,000 (Initial price) − ($200 × 15 months) = $215,000 - $3,000 = $212,000

Pine Limited owns a 15% royalty interest in a natural gas well. Pine receives royalty payments on January 31 for the gas sold between the previous June 1 and November 30, and on July 31 for oil sold between the previous December 1 and May 31. Production reports show the following oil sales: May 1, 20X7-October 31, 20X7 $650,000November 1, 20X7-December 31, 20X7 124,000November 1, 20X7-April 30, 20X8 730,000May 1, 20X8-October 31, 20X8 718,000November 1, 20X8-December 31, 20X8 137,000 What amount should Pine report as royalty revenue for 20X8? A. $217,200 B. $219,150 C. $237,750 D. $198,600

B. $219,150 In this question, one needs to convert from cash receipts to accrual earnings. Compute the amount of earnings properly allocable to 20X8, and then compute the percentage. Revenue for January 1-April 30:$730,000 − $124,000 $ 606,000Revenue for May 1-October 31 718,000Revenue for November 1-December 31 137,000Total 20X8 revenue $1,461,000Times 15% × 0.15Pine's 20X8 royalty revenue $ 219,150==========

Brighton Co. spent $500,000 developing a new idea for a product that was patented during the year. The legal cost of applying for a patent license was $80,000 and this was paid at the beginning of 20X5. The patent has a useful life of 8 years. During 20X8, Brighton spent $75,000 defending the patent in a court of law. Early in 20X9, Brighton was notified they had prevailed in their defense. Because of the results of the lawsuit, the useful life of the patent was extended by 1 year. What amount of patent amortization should Brighton record for 20X9? A. $8,000 B. $23,000 C. $10,000 D. $27,750

B. $23,000 Because the patent defense was successful, the costs are capitalized and amortized over the new remaining useful life of the patent. The amount is computed as follows. (Note that the $500,000 spent to develop the "new idea" was considered research and development cost, and not capitalized as part of the cost of the patent itself.) Cost of patent $ 80,000Patent amortization for 20X5 through 20X8(($80,000 ÷ 8 years) × 4 years) (40,000)Carrying value of patent on 01/01/X9 $ 40,000Defense costs to capitalize 75,000Total cost of patent $115,000Patent amortization for 20X6($115,000 ÷ 5 years) $ 23,000

Rollins Corporation acquired 75% of the outstanding stock of Schauer Corporation. The purchase price of the acquisition was $3,960,000. The book value of Schauer's net assets was $4,640,000. Schauer had assets whose fair values were greater than their carrying value by the following amounts: Land $120,000, Buildings $280,000. How much goodwill is implied in Rollins' acquisition of Schauer? A. $80,000 B. $240,000 C. $300,000 D. $180,000

B. $240,000 Rollins will record goodwill of $240,000 by following these steps: First, the implied total fair value of Schauer must be found by taking the purchase price and adjusting for the controlling interest purchased ($3.96M ÷ 75% = $5.28M total fair value). Now that we have 100% of the fair value, we have to subtract the fair value of the net assets of Schauer. Those can be found by starting with the book value and adjusting for any differences between book and fair value, for a total net asset value of $5,040,000 ($4,640,000 + $120,000 Land + $280,000 Building). Goodwill is the difference between the total fair value of the consideration paid and total net assets, or $240,000 ($5.28M − $5.04M). Note that when recognizing the purchase, Rollins would recognize the difference between the actual purchase price and the total fair value of the net assets in the noncontrolling interest (NCI) account.

A company reported $130,000 in income from continuing operations for its first year of operations. The tax-basis depreciation deduction for the year exceeded GAAP depreciation expense by $12,500, and the warranty accrual exceeded the amount spent for warranty repairs by $8,300. The company properly calculated a $840 increase in its deferred tax liability for the year. If the enacted tax rate for the current year is 20%, what amount of income taxes payable should be reported in the year-end balance sheet? A. $24,340 B. $25,160 C. $26,000 D. $26,840

B. $25,160 Income taxes payable is found by multiplying taxable income by the enacted tax rate. To solve, taxable income must be found first by adjusting GAAP income for temporary and permanent differences: Taxable income = GAAP income +/− Differences = $130,000 − $12,500 excess depreciation + $8,300 expense add-back = $125,800 Income taxes payable = $125,800 × 20% = $25,160

At the end of year one, Boller Co. had an ending balance in allowance for uncollectible accounts of $30,000. During year two, Boller wrote off $40,000 of accounts receivable. At the end of year two, Boller had $300,000 in accounts receivable and determined that 8% of these would be uncollectible. What amount should be reported as uncollectible accounts expense on Boller's year two income statement? A. $24,000 B. $34,000 C. $14,000 D. $64,000

B. $34,000 Uncollectible accounts expense for year two is $34,000 under the balance sheet approach. Use a T-account to determine how much expense is needed to achieve an ending balance in the Allowance account of $24,000 ($300,000 × 8%).

During January 20X1, Metro Co., which maintains a perpetual inventory system, recorded the following information pertaining to its inventory: Unit Total UnitsUnits Cost Cost on HandBalance (January 1, 20X1) 1,000 $1 $1,000 1,000Purchased (January 7, 20X1) 600 3 1,800 1,600Sold (January 20, 20X1) 900 700Purchased (January 25, 20X1) 400 5 2,000 1,100 Under the moving-average method, what amount should Metro report as inventory on January 31, 20X1? A. $2,640 B. $3,225 C. $3,300 D. $3,900

B. $3,225 Moving average is the average costing method used with a perpetual inventory system. Thus, it is updated with every purchase to get a new average unit cost. Balance on January 1, 20X1 (1,000 units at $1) $1,000Purchase on January 7, 20X1 (600 units at $3) + 1,800 Moving average inventory on January 7, 20X1 $2,800Sale of 900 units on January 20, 20X1 (900 units at $1.75)* - 1,575 Moving average inventory on January 20, 20X1(700 units at $1.75) $1,225Purchase on January 25, 20X1 (400 units at $5.00) + 2,000 Moving average inventory on January 31, 20X1 $3,225======== * The $1.75 is from the total cost divided by units available for sale ($2,800 ÷ 1,600) .

Quinn Co. reported a net deferred tax asset of $9,000 in its December 31, 20X1, balance sheet. For 20X2, Quinn reported pretax financial statement income of $300,000. Temporary differences of $100,000 resulted in taxable income of $200,000 for 20X2. At December 31, 20X2, Quinn had cumulative taxable differences of $70,000. Quinn's effective income tax rate is 30%. In its December 31, 20X2, income statement, what should Quinn report as deferred income tax expense? A. $12,000 B. $30,000 C. $60,000 D. $21,000

B. $30,000 The temporary difference generated in the year is the noncurrent part of the tax expense. Deferred income Temporary Effectivetax expense for 20X2 = differences × tax rate= $100,000 × 0.30= $30,000 Note: The temporary difference of $100,000 for 20X2 offset by the temporary asset difference ($9,000 ÷ .30 = $30,000) as of January 1, 20X2, resulted in a cumulative taxable difference of $70,000 on the December 31, 20X2, balance sheet.

Ichor Co. reported equipment with an original cost of $379,000 and $344,000, and accumulated depreciation of $153,000 and $128,000, respectively, in its comparative financial statements for the years ending December 31, 20X2 and 20X1. During 20X2, Ichor purchased equipment costing $50,000, and sold equipment with a carrying value of $9,000. What amount should Ichor report as depreciation expense for 20X2? A. $25,000 B. $31,000 C. $34,000 D. $19,000

B. $31,000 In the context of this problem accumulated depreciation is affected by the asset disposal when the carrying value of the asset sold is written off and by depreciation expense for the current period. These two items account for the net increase of $25,000 ($153,000 - $128,000) in the credit balance of the accumulated depreciation account. The debit change in accumulated depreciation caused by the asset disposal needs to be determined from the facts provided. The equipment account had a beginning balance of $344,000. The $50,000 purchase of new equipment would cause this balance to increase to $394,000. However, the ending balance was $379,000. The only other transaction affecting the equipment account was the disposal of a piece of equipment. Therefore, the original cost of the disposed equipment was $15,000 ($394,000 - $379,000). Since the disposed equipment had a cost of $15,000 and a carrying value of $9,000 (carrying value = cost - accumulated depreciation), the accumulated depreciation associated with the disposed equipment was $6,000 ($9,000 = $15,000 - accumulated depreciation). The beginning credit balance in the accumulated depreciation control account was $128,000. It would have been decreased (debited) for the $6,000 of accumulated depreciation related to the disposed equipment. That would leave a credit balance of $122,000. However, the ending balance was a credit of $153,000. Depreciation expense for the period would also change (increase or credit) the balance of accumulated depreciation. Since the ending balance was $153,000, and the balance without the effect of depreciation expense was $122,000, the depreciation expense must have been $31,000 ($153,000 - $122,000).

Alton Co. had a cash balance of $32,300 recorded in its general ledger at the end of the month, prior to receiving its bank statement. Reconciliation of the bank statement reveals the following information: Bank service charge: $15 Check deposited and returned for insufficient funds check: $120 Deposit recorded in the general ledger as $258 but should be $285 Checks outstanding: $1,800 After reconciling its bank statement, what amount should Alton report as its cash account balance? A. $30,392 B. $32,192 C. $32,138 D. $30,338

B. $32,192 Balance per books: 32300 service charge: (15) NSF check: (120) Check error: 27 Total = 32192 checks outstanding have already been deducted from book balance

Tableau Company manufactures hot water heater systems that carry a 2-year warranty against defects. Based on past experience, warranty costs are estimated at 2.5% of sales for the warranty period. During 20X9, hot-water heater system sales totaled $2,600,000, and warranty costs of $31,300 were incurred. In its balance sheet at December 31, 20X9, Tableau should report warranty liability of: A. $60,000. B. $33,700. C. $31,300. D. $65,000.

B. $33,700. Warranty liability for 20X9 is $33,700 ($65,000 - $31,300). The warranty expense is recognized in the year in which the warranted product is sold. The actual warranty expenditures may or may not be made in that same period. The $31,300 warranty expenditures incurred in 20X9 result in a reduction of the estimated warranty liability. Those expenditures may relate to products sold in 20X9, but they also may relate to products sold in a prior period. Journal entries include the following: Warranty expense 65,000Estimated warranty liability 65,000Estimated warranty liability 31,300Cash 31,300

Which of the following is a pair of values that are compared to determine the amount of a possible impairment loss on an intangible asset, with an indefinite life, other than goodwill? A. Future value, carrying value B. Fair value, carrying value C. Carrying value, book value D. Fair value, present value

B. Fair value, carrying value

Bake Co.'s trial balance included the following at December 31, 20X1: Accounts payable $ 80,000Bonds payable, due 20X2 300,000Discount on bonds payable 15,000Deferred income tax liability 25,000 The deferred income tax liability is not related to an asset for financial accounting purposes and is expected to reverse in 20X2. What amount should be included in the current liability section of Bake's December 31, 20X1, balance sheet (statement of financial position)? A. $420,000 B. $365,000 C. $395,000 D. $390,000

B. $365,000 All deferred tax liabilities (and deferred tax assets) are classified as noncurrent. All of the remaining items listed are considered current as they are due and payable within the next year.

Seafood Trading Co. commenced operations during the year as a large importer and exporter of seafood. The imports were all from one country overseas. The export sales were conducted as drop shipments and were merely transshipped at Seattle. Seafood Trading reported the following data: Purchases during the year $12.0 millionShipping costs from overseas 1.5 millionShipping costs to export customers 1.0 millionInventory at year-end 3.0 million What amount of shipping costs should be included in Seafood Trading's year-end inventory valuation? A. $625,000 B. $375,000 C. $0 D. $250,000

B. $375,000 Shipping costs from overseas (freight-in) should be included in the inventory costs. Shipping costs to export customers (freight-out) should not be included in the inventory costs. Ending inventory is 1/4th ($3,000,000 ÷ $12,000,000) of purchases. Thus, 1/4th of shipping costs from overseas ((0.25 × $1,500,000) = $375,000) should be included in ending inventory.

Large City does not use the modified approach to account for roads. At the beginning of the current year, the city spent $800,000 on new roads. The roads have a 20-year useful life. What amount should Large City report as an expense related to the new roads in the statement of activities for the current year? A. $20,000 B. $40,000 C. $0 D. $800,000

B. $40,000 Except for qualifying infrastructure capital assets, governments are required to depreciate all capital assets with limited lives. Because Large City is not electing the modified approach, standard straight-line depreciation is to be utilized over each year of the asset's life, for an annual expense of $40,000 ($800,000 ÷ 20 years).

Cole Co. began constructing a building for its own use in January 20X1. During 20X1, Cole incurred interest of $50,000 on specific construction debt, and $20,000 on other borrowings. Interest computed on the weighted-average amount of accumulated expenditures for the building during 20X1 was $40,000. What amount of interest cost should Cole capitalize? A. $50,000 B. $40,000 C. $70,000 D. $20,000

B. $40,000 For qualifying assets being constructed for an entity's own use, FASB ASC 835-20-30-2 requires interest cost to be capitalized equal to the less of (a) the avoidable interest (based on the weighted-average amount of accumulated expenditures), or (b) the actual interest cost incurred. Cole's avoidable interest is given to be $40,000. Since the $70,000 actual interest cost incurred ($50,000 + $20,000) is greater than the avoidable interest of $40,000, the amount of interest that Cole can capitalize is $40,000.

For its first year of operations, Cable Corp. recorded a $100,000 expense in its tax return that will not be recorded in its accounting records until next year. There were no other differences between its taxable and financial statement income. Cable's effective tax rate for the current year is 45%, but a 40% rate has already been passed into law for next year. In its year-end balance sheet, what amount should Cable report as a deferred tax asset (liability)? A. $40,000 asset B. $40,000 liability C. $45,000 asset D. $45,000 liability

B. $40,000 liability The recognition of the $100,000 expense for tax purposes in advance of the point it is recognized under GAAP allows the company to defer payment of taxes on $100,000 of GAAP income for the current year. The tax expense, from a GAAP perspective, is incurred because of current-year income and should be matched to the current year. This means that a current-year tax expense is not paid in the current year, but will be paid in the future. (Next year the company will have to pay tax on $100,000 more income than earned per GAAP.) Therefore, the amount is a deferred tax liability. The GAAP rules for measuring the amount of the liability are designed to report the "best" estimate of the amount of tax that actually will be paid in the future on this income. Therefore, the amount of the liability is measured using enacted tax rates. Because the 40% rate has been enacted into law by year-end, it will be used to measure the deferred tax liability. This makes the balance a $40,000 liability ($100,000 × 40%).

Park, Inc. acquired 100% of Gravel Co.'s net assets. On the acquisition date, Gravel's accounting records reflected $50,000 of costs associated with in-process research and development activities. The fair value of the in-process research and development activities was $400,000. Park's consolidated intangible assets will increase by what amount, if any, as a result of the acquisition of the in-process research and development activities? A. $50,000 B. $400,000 C. $350,000 D. $0

B. $400,000 In-process research and development results are classified as intangible assets with indefinite lives until the research and development phase is complete or the project is abandoned. These assets are originally recorded at fair value (i.e., $400,000) and will be subject to impairment tests.

The following information is relevant to one of the City of Mullins' General Fund's derived tax revenues: Fiscal year-end June 30Beginning receivables $450,000Beginning deferred revenues 100,000Beginning allowance for doubtful accounts 50,000Receipts 1,250,000Ending receivables 600,000Receivables collected 6/30 - 8/30 125,000Ending allowance for doubtful accounts 60,000 The City of Mullins considers derived tax receivables collected within 60 days after the close of the fiscal year to be "available." Furthermore, the City wrote off $30,000 of receivables as uncollectible during the year. What would be the amount of deferred revenues reported at the fund level for year-end? A. $600,000 B. $415,000 C. $540,000 D. $475,000

B. $415,000 At the fund level, derived tax revenues are reported using the modified accrual method. Using modified accrual, that portion of the ending receivable which is measurable but not available, or accounted for as an allowance, is accounted for as deferred revenue. Deferred RevenuesEnding receivable $600,000Less collections June 30 through August 30 (125,000)Less ending allowance for doubtful accounts (60,000)$415,000=========

Which of the following is not reported in the income statement? A. Foreign currency transaction gains B. Foreign currency translation gains C. Foreign currency remeasurement gains D. All of the answer choices are reported in the income statement.

B. Foreign currency translation gains Foreign currency remeasurement gains and losses are reported in the income statement, as are foreign currency transaction gains and losses. Foreign currency translation gains and losses are reported in other comprehensive income.

The following information pertains to Trenton County's liability for claims and judgments: Current liability at January 1, 20X5 $ 63,000Claims paid during 20X5 429,000Current liability at December 31, 20X5 52,000Noncurrent liability at December 31, 20X5 137,000 What amount should Trenton report for 20X5 claims and judgments expenditures? A. $566,000 B. $418,000 C. $429,000 D. $481,000

B. $418,000 Because the question asks for the amount of "expenditures," it is clear that the question refers to the governmental fund reporting and not the government-wide reporting. The current liability of $63,000 at the start of year 20X5 relates to expenditures of the prior year. The 20X5 payment of $429,000 liquidated that liability of $63,000 remaining from the prior year and recognized $366,000 ($429,000 − $63,000) of 20X5 expenditures. At the end of 20X5, another $52,000 of 20X5 claims and judgments expenditures were waiting to be liquidated with current resources. Thus, the 20X5 claims and judgments expenditures were $418,000 ($429,000 − $63,000 + $52,000).

The following items were included in Opal Co.'s inventory account on December 31, 20X1: Merchandise out on consignment at sales price, including 40% markup on selling price $40,000 Goods purchased in transit, shipped FOB shipping point 36,000 Goods held on consignment by Opal 27,000 By what amount should Opal's inventory account on December 31, 20X1, be reduced? A. $51,000 B. $43,000 C. $103,000 D. $67,000

B. $43,000 markup = 40,000 * .4 = 16000 goods held on consignment = 27,000 total to subtract = 43000

Mattes Curators is an organization that seeks to acquire and maintain historic buildings. During 20X8, Mattes was given a gift of securities with the indicated stipulations: Debt securities valued at $1,545,000 are to be held for one year and then are to be sold with the proceeds used to provide funds for acquisition of a historic residence in the community. Equity securities valued at $470,000 are to be used for whatever purposes Mattes' board of directors deems appropriate. What amount should Mattes include as net assets without donor restrictions as a result of this donation? A. $2,015,000 B. $470,000 C. $0 D. $1,545,000

B. $470,000 Mattes would record additional net assets without donor restrictions resulting from this donation of $470,000 because the equity securities were donated with no donor stipulations. The answer choice of $0 is incorrect because a portion of the donation (the equity securities) has no donor restrictions.

On January 2, 20X1, Gant Co. purchased a franchise with a useful life of five years for $60,000 and an annual fee of 1% of franchise revenues. Franchise revenues were $20,000 during 20X1. Gant projects future revenues of $40,000 in 20X2 and $60,000 per year for the following three years. Gant uses the straight-line method of amortization. What amount should Gant report as intangible asset-franchise, net of related amortization in its December 31, 20X1, balance sheet? A. $48,160 B. $48,000 C. $49,920 D. $56,000

B. $48,000 The cost of the intangible asset-franchise is $60,000. It is not affected by the continuing franchise fees incurred during the life of the franchise, which are expensed as incurred. Straight-line amortization would be $12,000 per year ($60,000 ÷ 5 years). Therefore, at the end of the first year, the carrying value of the asset is $48,000 ($60,000 - $12,000).

On January 2, 20X1, Farm Co. granted an employee an option to purchase 1,000 shares of Farm's common stock at $40 per share. The option became exercisable on December 31, 20X2, after the employee had completed two years of service, and was exercised on that date. The fair value of the option on January 2, 20X1, was $10 per option. What amount should Farm recognize as compensation expense for 20X1? A. $40,000 B. $5,000 C. $10,000 D. $0

B. $5,000 FASB ASC 718-10-30-2 requires that the fair value method be used. Except in very rare circumstances, the intrinsic value method is no longer acceptable. Accordingly, total compensation cost for Farm Co. is the $10,000 fair value of the options on the grant date (1,000 shares × $10 fair value per option on January 2, 20X1). The $10,000 total compensation cost should be amortized over Farm's 2-year service period at the rate of $5,000 per year ($10,000 ÷ 2 years = $5,000). The compensation expense for 20X1 therefore is $5,000.

During its fiscal year ended June 30, 20X1, Cliff City issued purchase orders totaling $5,000,000, which were properly recorded as encumbrances at that time. Cliff received goods and related invoices at the encumbered amounts totaling $4,500,000 before year-end. The remaining goods of $500,000 were not received until after year-end. Cliff paid $4,200,000 of the invoices received during the year. What amount of Cliff's encumbrances was outstanding at June 30, 20X1? A. $300,000 B. $500,000 C. $0 D. $800,000

B. $500,000 The actual amount of expenditures may be more or less than the estimated amount and the amount paid may differ from the encumbered amount. However, that does not affect the encumbrance or the Fund Balance—Reserved for Encumbrances amounts. Therefore, the amount outstanding at June 30, 20X1, was $500,000. In the closing process, the outstanding Encumbrances and Fund Balance—Reserved for Encumbrances of $500,000 would be removed, and $500,000 of the post-closing Fund Balance would be displayed as "committed" or "assigned."

Dove Inc. owns 100% of Flom Co. On January 2, 20X3, Dove sold equipment with an original cost of $120,000 and a carrying amount of $84,000 to Flom for $108,000. It is Dove's policy to use straight-line depreciation with a useful life of 10 years for equipment like that sold to Flom. The equipment had no residual value. Flom is using straight-line depreciation over 6 years with no residual value. In Dove's December 31, 20X3, consolidating worksheet, by what amount should depreciation expense be decreased? A. $0 B. $6,000 C. $12,000 D. $18,000

B. $6,000 When dealing with unrealized gains or losses in a consolidated financial statement setting, the objective is to defer unrealized gains to establish both historical cost balances and recognize appropriate income within the consolidated financial statement. The unrealized gain of the sale of the equipment to Flom is located in the cost of the equipment on Flom's books. Depreciation expense on a consolidated basis should be the depreciation that would have been expensed on Dove's books if the equipment had not been sold. Depreciation on Flom's books (unrealized gain)($108,000 ÷ 6) $18,000 Depreciation on Dove's books (original cost)($120,000 ÷ 10) 12,000 Difference $ 6,000

Peters Corp.'s capital structure was as follows: 12/31/X1 12/31/X2 Outstanding shares of stock: Common 110,000 145,000 Convertible preferred 10,000 0 8% convertible bonds 1,000,000 500,000 On May 1, 20X2, the preferred shares were converted into 20,000 shares of common stock. The 8% bonds are convertible into 30,000 shares of common stock. On July 1, $500,000 of the bonds were converted. Net income for 20X2 was $850,000. Assume that the income tax rate is 30%. What is the basic earnings per share for 20X2? A. $6.80 B. $6.50 C. $5.86 D. $6.54

B. $6.50 110,000 * 4/12 130,000 * 2/12 145,000 * 6/12 Sum = 130,833.33 850,000 / 130833 = 6.496

The following information pertains to property taxes levied by Oak City for the calendar year 20X1: Collections during 20X1 $500,000 Expected collections during the first 60 days of 20X2 100,000 Expected collections during the balance of 20X2 60,000 Expected collections during January 20X3 30,000 Estimated to be uncollectible 10,000 Total levy $700,000======== What amount should Oak report for 20X1 net property tax revenues? A. $700,000 B. $600,000 C. $500,000 D. $690,000

B. $600,000 Oak City's net property taxes for 20X1 would be $600,000. This is the collections during 20X1 and the amount expected to be collected in the first 60 days of 20X2. Amounts expected to be collectible during the first 60 days of the following year can be recorded as revenues of the current year if they are legally usable to pay current liabilities of the current year; that is, if they are available.

Rice Co. was incorporated on January 1, 20X1, with $500,000 from the issuance of stock and borrowed funds of $75,000. During the first year of operations, net income was $25,000. On December 15, 20X1, Rice paid a $2,000 cash dividend. No additional activities affected owner's equity in 20X1. On December 31, 20X1, Rice's liabilities had increased to $94,000. In Rice's December 31, 20X1, balance sheet (statement of financial position), total assets should be reported at: A. $600,000. B. $617,000. C. $692,000. D. $598,000.

B. $617,000. 500K + 25k -2k +94k

On January 2, 20X1, Lava, Inc., purchased a patent for a new consumer product for $90,000. At the time of purchase, the patent was valid for 15 years; however, the patent's useful life was estimated to be only 10 years due to the competitive nature of the product. On December 31, 20X4, the product was permanently withdrawn from sale under governmental order because of a potential health hazard in the product. What amount should Lava charge against income during 20X4, assuming that amortization is recorded at the end of each year? A. $9,000 B. $63,000 C. $54,000 D. $72,000

B. $63,000 The remaining unamortized cost of any asset that is totally impaired is a loss. Cost of patent $90,000Patent amortization for 20X1 through 20X3(($90,000 / 10 years) x 3 years) 27,000Carrying value of patent on 01/01/X4(amount written off in 20X4) $63,000======= Remember: Since amortization is recorded at the end of each year, no amortization for 20X4 has been recorded before 12/31/X4. The objective at the end of 20X4 is to write off all remaining investment since the product was withdrawn from sale.

Alder Corp. had the following stockholders' equity balances at the beginning of the current year: Common stock 200,000 shares authorized, $1 par; 15,000 shares issued and outstanding $15,000 Additional paid-in capital 24,000 Retained earnings 11,000 During the current year, Alder issued 2,000 shares of common stock with a fair value of $35 per share to Terry Brady on a subscription basis. Terry made a down payment of $3,500, but shortly thereafter defaulted on the subscription. What would be the debit to additional paid-in capital if Alder returned the $3,500 to Terry? A. $70,000 B. $68,000 C. $66,500 D. $73,500

B. $68,000 When Alder issued the shares, they would have recorded the issued shares at their fair value, with $2,000 going to common stock subscribed at par value ($2,000 × $1 par) and the remaining $68,000 going to subscribed additional paid-in capital (APIC) (($35 − $1) × 2,000 shares). When Terry defaulted and Alder returned the money to Terry, Alder would no longer be required to provide the subscription and the original entry of $68,000 would need to be debited to remove it from APIC.

Garcel, Inc. held unfinished inventory at a cost of $85,000 with a sales value of $125,000. The inventory will cost $10,500 to complete. The normal profit margin is 30% of sales. The replacement cost of the inventory was $75,000. What amount should Garcel report (accounted for under the LIFO method) as inventory on balance sheet? A. $114,500 B. $77,000 C. $75,000 D. $85,000

B. $77,000 A departure from the cost basis is required when the utility of goods is no longer as great as cost; for inventory, the loss should be recognized in the period in which the decline takes place. Inventory measured using any method other than LIFO or the retail inventory method (e.g., FIFO or average cost) is measured at the lower of cost and net realizable value (NRV), which is defined to be the estimated selling price in the ordinary course of business, less reasonably predictable costs of completion, disposal, and transportation. If the NRV of inventory is lower than its cost, the difference is recognized as a loss in earnings in the period in which it occurs. In this case, market value will be the replacement cost unless replacement cost exceeds net realizable value (NRV) (estimated selling price less costs of completion and disposal), in which case market will be net realizable value (the ceiling); OR replacement cost is less than net realizable value reduced by a normal profit margin (the floor), in which case market will be the floor. NRV (ceiling) $125,000 - $10,500 = $114,500Replacement cost = $ 75,000NRV - Normal profit (floor) $114,500 - (30% × $125,000) = $ 77,000 Garcel should report inventory at $77,000, the lower of cost or market. (Note: market cannot be less than the floor value of $77,000.)

When a full set of general purpose financial statements are presented, comprehensive income and its components should: A. appear in a supplemental schedule in the notes to the financial statements. B. be displayed in a financial statement that has the same prominence as other financial statements. C. be reported net of related income tax effect, in total and individually. D. appear as a part of discontinued operations and cumulative effect of a change in accounting principle.

B. be displayed in a financial statement that has the same prominence as other financial statements.

On June 30, Huff Corp. issued at 99, 1,000 of its 8%, $1,000 bonds. The bonds were issued through an underwriter to whom Huff paid bond issue costs of $35,000. On June 30, Huff should report the bond liability at: A. $1,025,000. B. $955,000. C. $990,000. D. $1,000,000.

B. $955,000. The carrying value of the debt, initially, the bond liability, is $990,000, computed as the number of bonds multiplied by the face amount per bond, multiplied by the issue percentage, reduced by the bond issue costs of $35,000: 1,000 bonds × $1,000 face × 0.99 = $990,000 $990,000 − $35,000 = $955,000

A company had 400,000 shares of common stock issued and outstanding on January 1, year 1, and had the following equity transactions for year 1: Issued 200,000 new shares for cash April 1 Issued new shares as a result of a 3-for-1 stock split July 1 Purchased 300,000 shares treasury stock for cash October 1 What should the company use as the denominator for the calculation of basic earnings per share for the year ended December 31, year 1? A. 1,650,000 B. 1,575,000 C. 1,325,000 D. 1,075,000

B. 1,575,000 January 1: 1,200,000* 3/12 = 300,000 April 1: 1,800,000* 6/12 = 900,000 July 1 - - - October 1: 1,500,000* 3/12 = 375,000 WACS 1,575,000

Album Co. issued 10-year, $200,000 debenture bonds on January 2. The bonds pay interest semiannually. Album uses the effective interest method to amortize bond premiums and discounts. The carrying value of the bonds on January 2 was $185,953. A journal entry was recorded for the first interest payment on June 30, debiting interest expense for $13,016 and crediting cash for $12,000. What is the effective interest rate for the debenture bonds? A. 12% B. 14% C. 7% D. 6%

B. 14% Effective interest = Carrying value of the bonds × Effective interest rate × Time period In this case: $13,016 = $185,953 × Unknown effective interest rate × 1/2 year Effective interest rate = ($13,016 ÷ $185,953) × 2 = 14%

On March 1, 20X0, Fine Co. borrowed $10,000 and signed a 2-year note bearing interest at 12% per annum compounded annually. Interest is payable in full at maturity on February 28, 20X2. What amount should Fine report as a liability for accrued interest on December 31, 20X1? A. $1,200 B. $2,320 C. $1,000 D. $0

B. 2,320 Accrued interest on December 31, 20X1: For 20X0: $10,000 × .12 × (10/12) = $1,000 For 20X1: ($10,000 + $1,000) × .12 = 1,320 Total $2,320

An entity authorized 500,000 shares of common stock. At January 1, Year 2, the entity had 110,000 shares of common stock issued and 100,000 shares of common stock outstanding. The entity had the following transactions in Year 2: March 1 Issued 15,000 shares of common stock June 1 Resold 2,500 shares of treasury stock September 1 Completed a 2-for-1 common stock split What is the total number of shares of common stock that the entity has outstanding at the end of Year 2? A. 230,000 B. 235,000 C. 250,000 D. 117,500

B. 235,000 At the beginning of the year, the number of shares outstanding was 100,000: 15,000 new shares were issued, giving a new total of 115,000 outstanding 2,500 of the 10,000 treasury shares were reissued; adding them to the 115,000 gives a new total of 117,500. A 2-for-1 split doubles the shares outstanding at that point, for a new total of 235,000 (117,500 × 2).

TGR Enterprises provided the following information from its statement of financial position for the year ended December 31: January 1 December 31Cash $ 10,000 $ 50,000Accounts receivable 120,000 100,000Inventories 200,000 160,000Prepaid expenses 20,000 10,000Accounts payable 175,000 120,000Accrued liabilities 25,000 30,000 TGR's sales and cost of sales for the year were $1,400,000 and $840,000, respectively. What is the accounts receivable turnover, in days? A. 26.1 B. 28.7 C. 31.3 D. 41.7

B. 28.7 Accounting receivable turnover = Net credit sales ÷ Average receivables: $1,400,000 ÷ (($120,000 + $100,000) ÷ 2) = 12.727 times in a year Turnover in days = 365 days ÷ Turnover in a year: 365 ÷ 12.727 = 28.7 (rounded)

Cardamom Limited wants to get an idea about the effectiveness of its inventory management processes. The following data have been compiled to provide a benchmark for future comparisons. Net cash sales $ 51,000Cost of goods sold 272,000Inventory at beginning of year 37,000Purchases 263,000Accounts receivable at beginning of year 47,000Accounts receivable at end of year 52,000 What was the number of days' sales in average inventory turnover for the current year? A. 37.6 days B. 43.6 days C. 45.0 days D. 49.7 days

B. 43.6 days =365/inventory turnover =365/ (COGS/Average inventory) Avg Inv = (beg inv + Purchase - COGS) / 2

A company is an accelerated filer that is required to file Form 10-K with the U.S. Securities and Exchange Commission (SEC). What is the maximum number of days after the company's fiscal year-end that the company has to file Form 10-K with the SEC? A. 60 days B. 75 days C. 90 days D. 120 days

B. 75 days

The three internal service funds of a town were presented in a single column in the basic financial statements. The town's internal service funds supplied goods and services to the various governmental functions of the town. Combining the internal service funds in this way simplified: A. the presentation of the budgetary schedules. B. the conversion of the fund-based information to the government-wide financial statement format. C. the preparation and presentation of the combining fund statements. D. the preparation of the notes to the financial statements.

B. the conversion of the fund-based information to the government-wide financial statement format.

What is the minimum budgetary information required to be reported in a city's budgetary comparison schedules? A. A schedule of unfavorable variances at the functional level B. A schedule showing the original budget, the final appropriations budget, and actual inflows, outflows, and balances on a budgetary basis C. A schedule showing the final appropriations budget and actual expenditures on a budgetary basis D. A schedule showing the proposed budget, the approved budget, the final amended budget, actual inflows and outflows on a budgetary basis, and variances between budget and actual

B. A schedule showing the original budget, the final appropriations budget, and actual inflows, outflows, and balances on a budgetary basis

The FASB amends the Accounting Standards Codification through the issuance of: A. Statements of Financial Accounting Standards. B. Accounting Standards Updates. C. Staff Accounting Bulletins. D. Technical Bulletins.

B. Accounting Standards Updates.

Which of the following is true about the statement of changes in net position for fiduciary funds other than pension/OPEB funds? A. Additions and deductions can be aggregated if the resources are normally held and disbursed within a year for custodial funds. B. Additions must be disaggregated and shown by source and deductions disaggregated and shown by type. C. Private-purpose trust funds are not required to present a statement of changes in net position in the fund financial statements. D. Revenue and expenditures must be presented in detail.

B. Additions must be disaggregated and shown by source and deductions disaggregated and shown by type. GASB Statement 84 requires additions to be disaggregated and shown by source. Deductions must be disaggregated and shown by type. Additions and deductions can be aggregated in custodial funds if the resources are normally held and disbursed within a three-month period. Specific categories include investment income, investment expense, administrative costs, and net investment income.

A nongovernmental not-for-profit entity sold a truck for $5,000. The proceeds from the sale were used to purchase two new computers for the office at a total price of $1,800. In which section of the organization's statement of cash flows should the transactions be reported? A. As a cash inflow of $1,800 from investing activities and a cash outflow of $5,000 from investing activities B. As a cash inflow of $5,000 from investing activities and a cash outflow of $1,800 from investing activities C. As a cash inflow of $5,000 from financing activities and a cash outflow of $1,800 from investing activities D. As a cash inflow of $5,000 from investing activities and a cash outflow of $1,800 from operating activities

B. As a cash inflow of $5,000 from investing activities and a cash outflow of $1,800 from investing activities

With regard to infrastructure, how should a change from depreciation to the modified approach be reported? A. As a change in accounting estimate, requiring restatement of prior periods B. As a change in accounting estimate, not requiring restatement of prior periods C. As a change in accounting principle, not requiring restatement of prior periods D. As a change in accounting principle, requiring restatement of prior periods

B. As a change in accounting estimate, not requiring restatement of prior periods According to GASB 1400.107, footnote 9, a change from depreciation to the modified approach should be reported as a change in an accounting estimate, and this change would not require a restatement of prior periods.

The controller of Pane Co. was preparing the company's financial statements. Pane had a wholly owned subsidiary in a foreign country that used the euro as its currency. At December 31, the exchange rate was $1 U.S. for 1.25 euro. The weighted-average exchange rate for the year was $1 U.S. for 1.50 euro. At December 31, the subsidiary had assets of 1 million euro and revenue for the year of 2 million euro. What amounts would assets and revenue translate for consolidation? A. Assets $800,000; Revenue $1,600,000 B. Assets $800,000; Revenue $1,333,333 C. Assets $666,666; Revenue $1,333,333 D. Assets $666,666; Revenue $1,600,000

B. Assets $800,000; Revenue $1,333,333 Because the foreign subsidiary used the local currency (the euro) as its currency, the euro is the functional currency. That requires the parent to translate the statements of the subsidiary at the current rates for most of the balance sheet and the average rates for the income statements. Revenues would be translated at $1US:1.50€ and the assets would be translated at the current rate of $1US:1.25€. Assets would then be $800,000 (1 million euro ÷ 1.25 euro) and revenues would be $1.33 million (2 million euro ÷ 1.5 euro).

The last member of Cross Corners' founding family, Ezra Cross, left his collection of early American art to the City for permanent display in city office buildings. The fair value of the collection at donation was $2,000,000. The Cross Corners city council formally accepted the collection and set a policy that the art would (a) be held for public exhibition, (b) be protected, cared for, and kept unencumbered, and (c) not be sold except for the purposes of acquiring different items for the collection. The city council agreed that the collection should not be capitalized for financial reporting purposes. The city's maximum depreciation horizon for capital assets is 40 years. How would the collection affect the government-wide financial records in the first year? A. Because the art will be displayed in general government buildings, the governmental activities would show revenue from donations of $2,000,000 and expenses of $2,050,000, as well as a capital asset of $2,000,000. B. Because the art will be displayed in general government buildings, the governmental activities would show revenue from donations of $2,000,000 and an expense of $2,000,000. C. There would be no impact on the governmental activities section of the government-wide financial statements. D. Because the art will be displayed in general government buildings, the governmental activities would show revenue from donations of $2,000,000.

B. Because the art will be displayed in general government buildings, the governmental activities would show revenue from donations of $2,000,000 and an expense of $2,000,000. The city council's formal policy for the donated collection of American art meets the three criteria that permit the city to avoid capitalization of the art collection. If the collection is not capitalized, it would not be listed among the capital assets used for governmental activities, and depreciation would not be reported. In the year of the donation, a recipient government should recognize a revenue as well as an expense in the same amount.

Garson Co. recorded goods in transit purchased FOB shipping point at year-end as purchases. The goods were excluded from ending inventory. What effect does the omission have on Garson's assets and retained earnings at year-end? A. Assets understated; no effect on retained earnings B. Both assets and retained earnings understated C. No effect on assets; retained earnings overstated D. No effect on assets; retained earnings understated

B. Both assets and retained earnings understated

On January 2, Basketville City purchased equipment with a useful life of three years to be used by its water and sewer enterprise fund. Which of the following is the correct treatment for the asset? A. Capitalize; depreciation is optional. B. Capitalize; depreciation is required. C. Capitalize; depreciation is not permitted. D. Record the purchase of the equipment as an expenditure.

B. Capitalize; depreciation is required. Proprietary funds (including enterprise funds and internal service funds) are reported using the economic resources measurement focus and the accrual basis of accounting. Essentially, this means they are reported using a revenue and expense model. In this model, equipment is recorded and reported as capital assets (the term used for fixed assets in government reporting). Because expenses are reported, depreciation expense must be calculated and reported.

Coconut Company began business on January 2, 20X4. Coconut uses the installment sales method of accounting. The following account balance information is available for 20X4: Installment sales A/R, 12/31/X4 $1,200,000 Deferred gross profit, 12/31/X4(BEFORE realization of GP for 20X4) $630,000 Gross profit on sales for 20X4 35% For the year ended December 31, 20X4, cash collections and realized gross profit should be: A. Cash collections, $400,000; Realized gross profit, $420,000. B. Cash collections, $600,000; Realized gross profit, $210,000. C. Cash collections, $600,000; Realized gross profit, $420,000. D. Cash collections, $400,000; Realized gross profit, $210,000.

B. Cash collections, $600,000; Realized gross profit, $210,000. The amount of deferred gross profit to be realized is related to the amount of installment accounts receivable. If installment accounts receivable is $1,200,000, then 35% of that amount is the amount of deferred gross profit remaining. $1,200,000 × 0.35 = $420,000 is the deferred gross profit yet to be realized, so the realized gross profit for the year is the difference between $630,000 and $420,000, or $210,000. Cash collections are computed by dividing the realized gross profit by the gross profit percentage, or $210,000 ÷ 0.35 = $600,000.

Which of the following should not be disclosed in an enterprise's statement of cash flows prepared using the indirect method? A. Interest paid, net of amounts capitalized B. Cash flow per share C. Income taxes paid D. Dividends paid on preferred stock

B. Cash flow per share

Which of the following information should be disclosed in the summary of significant accounting policies? A. Refinancing of debt subsequent to the balance sheet date B. Criteria for determining which investments are treated as cash equivalents C. Guarantees of indebtedness of others D. Adequacy of pension plan assets relative to vested benefits

B. Criteria for determining which investments are treated as cash equivalents FASB ASC 230-10-50-1, in a discussion of cash and cash equivalents, states: "An entity shall disclose its policy for determining which items are treated as cash equivalents." Note: The above requirement was embedded in a long paragraph discussing cash equivalents. The wording of the correct answer choice, particularly "criteria," indicates a relation to policy, whereas none of the other answers imply such a relationship.

Which of the following does not represent an element of other comprehensive income under current generally accepted accounting principles? A. Foreign currency adjustments B. Cumulative effect of a change in accounting estimate C. Accumulated gains and losses on available-for-sale debt investments D. Amortization of unrecognized prior service costs related to a pension plan

B. Cumulative effect of a change in accounting estimate The components of other comprehensive income are to be presented based on their nature. Under current authoritative accounting literature, three categories of elements of other comprehensive income exist: Unrealized gains and losses on available-for-sale debt investments Foreign currency items Changes in unrecognized prior service costs, unrecognized gains and losses, and unrecognized transition assets or obligations related to defined benefit pension plans and defined benefit other postretirement plans

As of December 1, Year 2, a company obtained a $1,000,000 line of credit maturing in one year on which it has drawn $250,000, a $750,000 secured note due in five annual installments, and a $300,000 3-year balloon note. The company has no other liabilities. How should the company's debt be presented in its classified balance sheet (statement of financial position) on December 31, Year 2, if no debt repayments were made in December? A. Current liabilities of $500,000; long-term liabilities of $1,550,000 B. Current liabilities of $400,000; long-term liabilities of $900,000 C. Current liabilities of $1,000,000; long-term liabilities of $1,050,000 D. Current liabilities of $500,000; long-term liabilities of $800,000

B. Current liabilities of $400,000; long-term liabilities of $900,000 400000=250000+(750000*1/5) 900000=300000+600000

Dale City is accumulating financial resources that are legally restricted to payments of general long-term debt principal and interest maturing in future years. At December 31, $5,000,000 has been accumulated for principal payments and $300,000 has been accumulated for interest payments. These restricted funds should be accounted for in the: A. Debt service fund, $300,000; General fund, $5,000,000. B. Debt service fund, $5,300,000; General fund, $0. C. Debt service fund, $5,000,000; General fund, $300,000. D. Debt service fund, $0; General fund, $5,300,000.

B. Debt service fund, $5,300,000; General fund, $0. The debt service fund is a reserve used to account for and report payments of the maturing principal and interest of general government short- and long-term debt. These liabilities are recorded in the General Capital Assets and General Long-Term Liabilities accounts, which are subaccounts of the debt service fund.

Grid Corp. acquired some of its own common shares at a price greater than both their par value and original issue price but less than their book value. Grid uses the cost method of accounting for treasury stock. What is the impact of this acquisition on total stockholders' equity and the book value per common share? A. Increase in both total stockholders' equity and book value per share B. Decrease in total stockholders' equity and increase in book value per share C. Increase in total stockholders' equity and decrease in book value per share D. Decrease in both total stockholders' equity and book value per share

B. Decrease in total stockholders' equity and increase in book value per share Treasury stock is a deduction from total stockholders' equity; therefore total stockholders' equity decreases. Since the price paid for the reacquired shares is less than their book value, the book value per share of the remaining outstanding shares must increase.

Financial statement line item explanations include which of the following? A. Inability to maintain a qualified workforce B. Degree of credit or nonperformance risk C. Segment reporting D. Potential litigation

B. Degree of credit or nonperformance risk However, other line items require varying degrees of disclosure. A summary of potential additional disclosures is as follows: For assets: the nature, quality, and location; future cash flows; relation to other line items; and significant contractual, statutory, regulatory, or judicial restrictions. For assets and liabilities resulting from financial instruments or other contracts: contractual or legal terms (e.g., timing of receipts and disbursements), degree of credit or nonperformance risk, potential effect related to inability to pay or perform, and method used to determine the cash flows. Other disclosures could include equity instrument terms or conditions, potential effects of changing accounting methods, breakdown of aggregated line items, alternative measurements, and the line item's relation to other line items.

Which of the following statements regarding footnote disclosure related to "significant estimates" is incorrect? A. The disclosure about significant estimates needs to include the nature of the uncertainty. B. Disclosures in the footnotes about significant estimates are encouraged but not required. C. Disclosure is required when the change in the estimate will have a material effect on the financial statements. D. Disclosure is required when it is reasonably possible that the financial statement estimate will change in the near term.

B. Disclosures in the footnotes about significant estimates are encouraged but not required.

If a city government is the primary reporting entity, which of the following is an acceptable method to present component units in its combined financial statements? A. Cost method B. Discrete presentation C. Consolidation D. Government-wide presentation

B. Discrete presentation The answer choice "discrete presentation" is correct. GASB standards allow for two different methods of presenting a component unit with the primary government. The organization can either blend the financial data into the primary government funds or report the component unit using discrete presentation. In discrete presentation, the financial information of the component is presented in a column apart from the primary government and not included in the totals reported for the primary government.

Goodwill should be tested for value impairment at which of the following levels? A. Each acquisition unit B. Each reporting unit C. Each identifiable long-term asset D. Entire business as a whole

B. Each reporting unit Goodwill is to be tested for impairment at the level of the reporting unit or at one level below an operating segment.

Government C sponsors a public entity risk pool in which Government C is also a participant. However, Government C is not the predominant participant in the pool. For the situation described, indicate the fund(s) Government C must use to account for the risk pool. A. Internal service B. Enterprise C. Agency D. Special revenue

B. Enterprise The proper accounting and financial reporting for this risk pool depends on whether the sponsoring government is also the predominant participant. If Government C is not the predominant participant, the pool would be treated as a stand-alone pool in substance and thus be accounted for in an enterprise fund and Government C's participation in the pool would be considered incidental.

Which of the following is correct regarding the recognition principle and classification criteria included in the FASB's ASU No. 2016-01, Financial Instruments—Overall (Subtopic 825-10): Recognition and Measurement of Financial Assets and Financial Liabilities? A. All changes in the fair value of financial liabilities that use the fair value option would be recorded through the income statement. B. Equity investments accounted for under the equity method or which result in consolidation would be out of the scope of the new accounting guidance. C. Changes in the fair value of financial liabilities that use the fair value option that result from changes in the instrument-specific credit risk would be recorded through additional paid in capital. D. The Update changed the accounting requirements for measuring impairment of all financial assets.

B. Equity investments accounted for under the equity method or which result in consolidation would be out of the scope of the new accounting guidance.

Which of the following accounts should Moon City close at the end of its fiscal year? A. Vouchers payable B. Expenditures C. Fund balance D. Fund balance—reserved for encumbrances

B. Expenditures Expenditures is a nominal (operating statement) account used in periodically measuring outflows of financial resources, and as such is closed at the end of the fiscal year. Vouchers Payable, Fund Balance, and Fund Balance—Reserved for Encumbrances are all balance sheet (real) accounts. Balance sheet accounts are not closed at fiscal year-end.

Which of the following accounting pronouncements is the most authoritative? A. FASB Technical Bulletins B. FASB Accounting Standards Codification C. FASB Statement of Financial Accounting Concepts (SFAC) D. AICPA Statement of Position

B. FASB Accounting Standards Codification

Which of the following is a component of other comprehensive income? A. Minimum accrual of vacation pay B. Foreign currency-translation adjustments C. Changes in market value of inventory D. Unrealized gain or loss on investment in equity securities

B. Foreign currency-translation adjustments The change in equity (net assets) of a business entity during a period from transactions and other events and circumstances from nonowner sources. It includes all changes in equity during a period except those resulting from investments by owners and distributions to owners. Comprehensive income comprises both of the following: All components of net income All components of other comprehensive income. Some items included in comprehensive income include the following: Foreign currency translation adjustments Unrealized holding gains and losses that result from a debt security Prior service costs or credits associated with pension or other postretirement benefits It is important to note that comprehensive income is a change amount and not a cumulative amount. The amounts reported as direct charges or credits in the equity section of a balance sheet are cumulative amounts, similar to the way that retained earnings is a cumulative amount.

Which of the following forms must be filed annually with the SEC? A. Form 1065 B. Form 10-K C. Form 990 D. Form 10-Q

B. Form 10-K

Which of the following accounts would never be included in the statements prepared for a fiduciary fund? A. Net position B. Fund balance C. Additions D. Cash

B. Fund balance Fiduciary funds use the economic resource measurement focus and the accrual basis of accounting. Revenues and expenses are not reported. The difference between assets plus deferred outflows and liabilities plus deferred inflows is referred to as "net position." A change in net position is calculated as the difference between additions and deductions.

Town City has one capital projects fund with assets of $3,000,000, liabilities of $400,000, and outstanding encumbrances of $2,000,000. On the balance sheet prepared at the end of the year, the fund balance would be displayed as: A. Fund balance—nonspendable, $2,000,000; Fund balance—unreserved, $600,000. B. Fund balance—committed, $2,000,000; Fund balance—assigned, $600,000. C. Fund balance—committed, $2,000,000; Fund balance—unassigned, $600,000. D. Fund balance—restricted, $2,000,000; Fund balance—unrestricted, $600,000.

B. Fund balance—committed, $2,000,000; Fund balance—assigned, $600,000. The outstanding encumbrance would be reflected as either committed or assigned fund balance for $2,000,000, depending on the level of government that entered into the agreement with the vendor. As the vendor in this case is probably a large construction company, it is likely the city council approved the major contract. Nonspendable fund balance would reflect resources such as inventories that cannot be spent, and restricted fund balance would reflect external constraints such as those imposed by creditors or grantors. Only the general fund can have an unassigned fund balance. Any fund balance in excess of the amount of the encumbrance would be considered assigned by being accounted for in a separate governmental fund—a capital projects fund.

During the current year, Vann County's motor pool internal service fund sold two vehicles for $5,000. The vehicles had a cost of $6,000 and a carrying value of $4,000. How should Vann County's motor pool internal service fund report this transaction in its fund financial statements? A. Other financing source of $5,000 B. Gain of $1,000 C. Revenue of $5,000 D. Special item of $1,000

B. Gain of $1,000 A government's self-supporting business-type activities are accounted for in the proprietary funds, which include enterprise funds and internal service funds. The flow of economic resources measurement focus and the accrual basis of accounting are used in proprietary funds. The accounting equation of proprietary funds is similar to that of a business corporation, which includes accounts for all related assets and liabilities—not just for current assets and current liabilities. Common examples of internal service funds are those used to account for government motor pools, central repair shops and garages, data processing departments, and photocopy and printing shops.

Which of the following transactions is included in the operating activities section of a cash flow statement prepared using the indirect method? A. Payment of cash dividend to the shareholders B. Gain on sale of plant asset C. Issuance of common stock to the shareholders D. Sale of property, plant, and equipment

B. Gain on sale of plant asset

A fixed asset with a 5-year estimated useful life and no residual value is sold at the end of the second year of its useful life. How would using the sum-of-the-years'-digits method of depreciation instead of the double-declining-balance method of depreciation affect a gain or loss on the sale of the fixed asset? A. Gain: increase; Loss: increase B. Gain: decrease; Loss: increase C. Gain: decrease; Loss: decrease D. Gain: increase; Loss: decrease

B. Gain: decrease; Loss: increase To answer this question it is best to use an example and see how the numbers work out. If an asset with a 5-year life has a $100,000 cost and a zero salvage value, then the double-declining balance depreciation for the first year would be $40,000: $100,000 × 2/5 (twice straight-line) = $40,000 For the second year, depreciation would be $24,000: $100,000 - $40,000 = $60,000 $60,000 × 2/5 = $24,000 Thus, under double-declining balance, the remaining book value of the asset would be $36,000: $100,000 - $40,000 - $24,000 = $36,000 Under the sum-of-the-years'-digits method, depreciation for the first two years would be 5/15 (5/(5 + 4 + 3 + 2 + 1)) of $100,000, and then 4/15 of $100,000, for a total depreciation for the first two years of 9/15 × $100,000, or $60,000, leaving a book value of $40,000 under the sum-of-the-years'-digits depreciation method. With a higher book value under sum-of-the-years'-digits depreciation at the end of the second year, a sale at a loss would be a higher loss under sum-of-the-years'-digits method, and a sale at a gain would be a lower gain under the sum-of-the-years'-digits method.

Jonn City entered into a lease for equipment during the year. How should the asset obtained through the lease be reported in Jonn City's government-wide statement of net position? A. Other financing use B. General capital asset C. Expenditure D. Not reported

B. General capital asset The GASB Codification clearly states that leased assets should be reported in government-wide financial statements. If used for general government purposes, the leased asset would be considered a general capital asset. Other financing use and expenditure are temporary accounts used in governmental funds to represent equity reductions.

Statement of Financial Accounting Concepts 8 (SFAC 8), Chapter 8, states that noncontractual asset disclosures could include all of the following except for: A. the nature of the asset. B. the degree of nonperformance risk. C. the quality of the asset. D. the location of the asset.

B. the degree of nonperformance risk.

Which of the following is a false statement? A. In their disclosure of significant violations of finance-related legal or contractual provisions, governments should identify actions taken to address such violations. B. Governments that present their primary government in more than a single column in their basic financial statements should disclose in their summary of significant accounting policies general definitions of each column that could be used to describe any government. C. Governments should disclose in their summary of significant accounting policies the length of time used to define available for purposes of revenue recognition in the governmental fund financial statements. D. Governments should provide details in the notes to the financial statements about short-term debt activity during the year, even if no short-term debt is outstanding at year-end.

B. Governments that present their primary government in more than a single column in their basic financial statements should disclose in their summary of significant accounting policies general definitions of each column that could be used to describe any government.

Which events are supportive of interperiod equity as a financial reporting objective of a governmental unit? I. A balanced budget is adopted. II. Transfers out equal transfers in. A. II only B. I only C. Both I and II D. Neither I nor II

B. I only The intent of balanced budget laws is to achieve interperiod equity, such that current taxpayers neither pay disproportionately for future years' services nor receive current-year services that must be paid for by future taxpayers. In contrast, transfers among funds should always be equal; however, equality of transfers in and out has no bearing on the extent to which interperiod equity is achieved in governmental external financial reporting. (GASB Concepts Statement 1)

Which of the following conditions must exist in order for an impairment loss to be recognized? The carrying amount of the long-lived asset is less than its fair value. The carrying amount of the long-lived asset is not recoverable. A. Both I and II B. II only C. I only D. Neither I nor II

B. II only If the undiscounted sum of estimated future cash flows from an asset or asset group is less that the asset's or asset group's book value, an impairment loss may need to be recognized. The impairment loss is the difference between the book value of the asset(s) and its (their) fair value. Note that there is not an impairment loss if the fair value exceeds the carrying amount.

Which of the following statements related to held-to-maturity debt securities is false? A. Held-to-maturity debt securities are carried on the balance sheet at amortized cost. B. If an entity anticipates holding the security beyond maturity, it should still be classified as held-to-maturity. C. An entity must have both the intent and ability to hold the securities to maturity. D. Held-to-maturity debt securities include both convertible debt and commercial paper.

B. If an entity anticipates holding the security beyond maturity, it should still be classified as held-to-maturity. In order for the investment to be as a held-to-maturity (HTM) investment, the reporting entity must have positive intent and ability to hold the debt security to maturity. An investment in a debt security should not be included in this category if the entity has the intent to hold the security for an indefinite period. Held-to-maturity securities are accounted for using the amortized cost method, and are carried at original cost adjusted for the amortization of any discount or premium. Examples of held-to-maturity debt securities include both convertible debt and commercial paper.

How should a U.S. publicly traded company report a change in fair value of a hedged available-for-sale security attributable to foreign exchange risk if the hedge is a fair value hedge? A. As a contra-asset related to the hedge B. In earnings C. In other comprehensive income D. As a change in the cost basis of the hedge

B. In earnings Fair value hedges that are properly reported and documented require the company to report a change in fair value of the hedged asset directly to earnings. Note this is different from a cash flow hedge, which reports the change in value in other comprehensive income. The cost basis does not change as the original price remains unchanged and a contra-asset is not appropriate. The question does not state which direction the fair value changed.

Packet Corp. is in the process of preparing its financial statements for the year ended December 31, 20X1. How would a gain on remeasuring a foreign subsidiary's financial statements from the local currency into the functional currency that occurred during 20X1 be classified in these financial statements? A. Income from continuing operations, with no separate disclosure B. Income from continuing operations, with separate disclosure (either on the face of statement or in the notes) C. Other comprehensive income D. None of the answer choices are correct.

B. Income from continuing operations, with separate disclosure (either on the face of statement or in the notes)

Alpha Hospital, a large not-for-profit entity, has adopted an accounting policy that does not imply a time restriction on gifts of long-lived assets. An accounting firm prepared Alpha's annual financial statements without charge to Alpha. Indicate the manner in which this transaction affects Alpha's financial statements. A. Increase in board-restricted net assets B. Increase in unrestricted revenues, gains, and other support C. Increase in net assets with donor restrictions D. Decrease in an expense

B. Increase in unrestricted revenues, gains, and other support

During the year, Bay Co. constructed machinery for its own use and for sale to customers. Bank loans financed these assets both during construction and after construction was complete. How much of the interest incurred should be reported as interest expense in the year-end income statement? A. Interest incurred for machinery for own use: all interest incurred; Interest incurred for machinery held for sale: interest incurred after completion B. Interest incurred for machinery for own use: interest incurred after completion; Interest incurred for machinery held for sale: all interest incurred C. Interest incurred for machinery for own use: interest incurred after completion; Interest incurred for machinery held for sale: interest incurred after completion D. Interest incurred for machinery for own use: all interest incurred; Interest incurred for machinery held for sale: all interest incurred

B. Interest incurred for machinery for own use: interest incurred after completion; Interest incurred for machinery held for sale: all interest incurred Whether an asset is constructed for a company's own use or constructed for resale, the interest incurred after the asset's construction is completed is expensed as incurred. Capitalization is not allowed once the asset is ready for use or sale. If that asset is constructed for a company's own use, interest incurred until completion may be capitalized. For a routinely produced asset to be held for sale, interest is expensed during construction and after completion. It is not capitalized.

Which of the following subsequent events must not be recognized in the financial statements? A. Shortly before financial statements are issued, the actual loss of plant or inventories as a result of fire or natural disaster that occurred before the balance sheet date is determined to be greater than the loss that was originally estimated. B. Loss of plant or inventories as a result of fire or natural disaster that occurred after the balance sheet date but before financial statements are issued or are available to be issued C. The events that gave rise to litigation took place before the balance sheet date and that litigation is settled, after the balance sheet date but before the financial statements are issued or are available to be issued, for an amount different from the liability recorded in the accounts. D. Loss on an uncollectible trade account receivable as a result of a customer's deteriorating financial condition leading to bankruptcy after the balance sheet date but before the financial statements are issued or are available to be issued

B. Loss of plant or inventories as a result of fire or natural disaster that occurred after the balance sheet date but before financial statements are issued or are available to be issued The correct answer is "loss of plant or inventories as a result of fire or natural disaster that occurred after the balance sheet date but before financial statements are issued or are available to be issued." An entity must not recognize subsequent events that provide evidence about conditions that did not exist at the date of the balance sheet but arose after the balance sheet date but before financial statements are issued or are available to be issued. The other answer choices are incorrect because an entity must recognize in the financial statements the effects of all subsequent events that provide additional evidence about conditions that existed at the date of the balance sheet.

Which of the following assumptions means that money is the common denominator of economic activity and provides an appropriate basis for accounting measurement and analysis? A. Periodicity B. Monetary unit C. Economic entity D. Going concern

B. Monetary unit

On incorporation, Dee, Inc., issued common stock at a price in excess of its par value. No other stock transactions occurred, except treasury stock was acquired for an amount exceeding this issue price. If Dee uses the par value method of accounting for treasury stock appropriate for retired stock, what is the effect of the acquisition on the following? A. Net common stock: Decrease; Additional paid-in capital: No effect; Retained earnings: Decrease B. Net common stock: Decrease; Additional paid-in capital: Decrease; Retained earnings: Decrease C. Net common stock: No effect; Additional paid-in capital: Decrease; Retained earnings: No effect D. Net common stock: No effect; Additional paid-in capital: Decrease; Retained earnings: Decrease

B. Net common stock: Decrease; Additional paid-in capital: Decrease; Retained earnings: Decrease

A 70%-owned subsidiary company declares and pays a cash dividend. What effect does the dividend have on the retained earnings and noncontrolling (minority) interest balances in the parent company's consolidated balance sheet? A. No effect on either retained earnings or noncontrolling interest B. No effect on retained earnings and a decrease in noncontrolling interest C. Decreases in both retained earnings and noncontrolling interest D. A decrease in retained earnings and no effect on noncontrolling interest

B. No effect on retained earnings and a decrease in noncontrolling interest since the noncontrolling (minority) interest (in this case, 30%) is a percentage of the stockholder equity (including retained earnings) of the subsidiary, any reduction in subsidiary retained earnings (such as dividend declaration) will decrease noncontrolling (minority) interest.

Cash receipts from grants and subsidies to decrease operating deficits should be classified in which of the following sections of the statement of cash flows for governmental not-for-profit entities? A. Operating B. Noncapital financing C. Capital and related financing D. Investing

B. Noncapital financing

How should state appropriations to a state university choosing to report as engaged only in business-type activities be reported in its statement of revenues, expenses, and changes in net position? A. Operating revenues B. Nonoperating revenues C. Capital contributions D. Other financing sources

B. Nonoperating revenues Revenues from state appropriations for other than capital-asset-related purposes are recorded as nonoperating revenues. Capital contributions and other financing sources are reported in other revenues, expenses, and transfers.

On January 1, year 1, a company purchased for $10,000 an at-the-money call option on 1,200 barrels of crude oil, which the company intends to purchase in 5 years. The company elected to exclude the time value of the option from the assessment of effectiveness, classified the option as a cash flow hedge, and applied a straight-line amortization to the initial option premium. On December 31, year 1, the time value of the option decreased by $1,200, and the change in intrinsic value increased by $1,800. The journal entry that the company should make on December 31, year 1, to record the change in value of the derivative should include which of the following as a credit? A. Derivative asset, $600 B. Other comprehensive income, $600 C. Derivative asset, $1,400 D. Other comprehensive income, $1,400

B. Other comprehensive income, $600 An option is a derivative. Because this is a cash flow hedge (as opposed to a fair value hedge) the company recognizes changes in the value of the derivative in other comprehensive income. The option has increased by $600 ($1,800 increase in intrinsic value − $1,200 decrease in time value). Therefore, the company would increase (debit) the option and increase (credit) other comprehensive income to recognize this change in value.

NuCorp. agreed to give Rand Co. a machine in full settlement of a note payable to Rand. The machine's original cost was $140,000. The note's face amount was $110,000. On the date of the agreement: the note's carrying amount was $105,000, and its present value was $96,000. the machine's carrying amount was $109,000, and its fair value was $96,000. Assuming that this trade was made as part of troubled debt restructuring, what amount of gains/losses should NuCorp. recognize, and how should these be classified in its income statement? A. Other gain/loss: $4,000 B. Other gain/loss: $(4,000) C. Other gain/loss: $(13,000) D. Other gain/loss: $0

B. Other gain/loss: $(4,000) FASB ASC 470-60-35-2 requires that a debtor in a troubled debt restructuring recognize a gain measure as "the excess of (i) the carrying amount of the payable settled...over (ii) the fair value of the assets transferred to the creditor." The total gain/loss in this case is the difference between the carrying value of the debt, $105,000, and the carrying value of the machine, $109,000 and its fair value of $96,000 conceivably could be deemed to be an impairment loss. However, we do not have enough information to know that this $13,000 part of the loss qualifies as an impairment loss. Nevertheless, even if it did, it would be included in income from continuing operations. The best answer is that the $4,000 net loss should be recognized as an ordinary loss to be included in income from continuing operations.

Parent Co. owns 90% of the 10,000 outstanding shares of Subsidiary Co.'s common stock on December 31, year 1. On that date, the stockholders' equity of Subsidiary was $150,000, consisting of $100,000 of no-par common stock and $50,000 of retained earnings. On January 2, year 2, Subsidiary issued 2,000 previously unissued shares for $24,000 to various outside investors. As a consequence of this transaction, Parent's ownership share was reduced to 75%. Which of the following correctly reports this transaction? A. The consolidated income statement reports a loss of $7,500. B. Parent's investment in Subsidiary is reduced by $4,500. C. The consolidated income statement reports a gain of $4,000. D. Parent's investment in Subsidiary is increased by $3,000.

B. Parent's investment in Subsidiary is reduced by $4,500.

Roro, Inc., paid $7,200 to renew its only insurance policy for three years on March 1, 20X1, the effective date of the policy. On March 31, 20X1, Roro's unadjusted trial balance showed a balance of $300 for prepaid insurance and $7,200 for insurance expense. What amounts should be reported for prepaid insurance and insurance expense in Roro's financial statements for the 3 months ending March 31, 20X1? A. Prepaid insurance: $7,000; Insurance expense: $300 B. Prepaid insurance: $7,000; Insurance expense: $500 C. Prepaid insurance: $7,200; Insurance expense: $300 D. Prepaid insurance: $7,300; Insurance expense: $200

B. Prepaid insurance: $7,000; Insurance expense: $500 This question indicates that on Roro's unadjusted trial balance the full 3-year premium of $7,200 for the renewal of the policy has been expensed. The prepaid insurance account still contains $300 of unamortized premiums from the old policy. The accountant must make adjusting entries to achieve the following: Expense the remaining premium of the old policy (DR Insurance Expense 300; CR Prepaid Insurance 300). Transfer the premium for the new policy to the prepaid insurance account (DR Prepaid Insurance 7,200; CR Insurance Expense 7,200). Amortize one month of expense on the new policy (DR Insurance Expense 200; CR Prepaid Insurance 200).

Which of the following is information required by GAAP that governments must disclose related to debt and lease obligations? A. Principal and interest requirements to maturity, presented separately, for the subsequent fiscal year and in 5-year increments thereafter B. Principal and interest requirements to maturity, presented separately, for each of the five subsequent fiscal years and in 5-year increments thereafter C. Principal and interest requirements to maturity, presented separately, for each of the three subsequent fiscal years and in 5-year increments thereafter D. Principal and interest requirements to maturity, presented separately, for each of the four subsequent fiscal years and in 5-year increments thereafter

B. Principal and interest requirements to maturity, presented separately, for each of the five subsequent fiscal years and in 5-year increments thereafter

Monies held in trust for the benefit of parties who are not a part of government are accounted for in which type of fund? A. Custodial funds B. Private-purpose trust funds C. Permanent funds D. Special revenue funds

B. Private-purpose trust funds

Which of the following types of entities are required to report on business segments? A. Joint ventures B. Publicly traded enterprises C. Not-for-profit enterprises D. Nonpublic business enterprises

B. Publicly traded enterprises

A company recently moved to a new building. The old building is being actively marketed for sale, and the company expects to complete the sale in four months. Each of the following statements is correct regarding the old building, except it will: A. be reclassified as an asset held for sale. B. be valued at historical cost. C. be classified as a current asset. D. no longer be depreciated.

B. be valued at historical cost. When you cease using a building in operations, you cease depreciating it. If it is to be sold soon, it will be reclassified as held for sale. However, it will still be carried at no more than its original cost less depreciation accumulated (which would be below historical cost).

Which of the following characteristics of service efforts and accomplishments is the most difficult to report for a governmental entity? A. Consistency B. Relevance C. Comparability D. Timeliness

B. Relevance The service efforts and accomplishments (SEA) performance of governmental entities is primarily measured by output, outcome, and efficiency measures. These measures report what services the entity has provided, whether those services have achieved the objectives established, and what effects they have had upon the recipients and others. SEA performance information measures should meet the characteristics of relevance, understandability, comparability, timeliness, consistency, and reliability. All of these measures, except for understandability and relevance, can be objectively examined. Understandability and relevance will be subject to the user's own interpretation. As understandability is not listed in the answer choices, only relevance can be the proper answer.

Which of the following is a governmental fund that uses the current financial resources measurement focus? A. Internal service fund B. Special revenue fund C. Private-purpose trust fund D. Enterprise fund

B. Special revenue fund Governmental funds finance and account for general government activities, such as police and fire protection, courts, inspection, and general administration, and use the flow of current financial resources measurement focus and the modified accrual basis of accounting. Governmental funds include the general fund, special revenue fund, capital projects fund, debt service fund, and permanent funds. Proprietary funds finance and account for a government's self-supporting business-type activities (e.g., utilities) and use the flow of economic resources measurement focus and the accrual basis of accounting. Proprietary funds include enterprise funds and internal service funds. Fiduciary funds account for resources (and any related liabilities) held by a government entity for the benefit of others (not to support the government's programs) in a trustee capacity (trust funds). Changes in net position of trust funds are reported as additions and deductions, using the flow of economic resources measurement focus and the accrual basis of accounting. Fiduciary funds include private-purpose trust funds, investment trust funds, pension trust funds, and custodial funds.

A Building Authority was created by the city and organized as a separate legal entity. The Authority is governed by a five-person board appointed for 6-year terms by the mayor, subject to city council approval. The authority uses the proceeds of its tax-exempt bonds to finance the construction or acquisition of general capital assets for the city only. The bonds are secured by the lease agreement with the city and will be retired through lease payments from the city. Which of the following describes the city's reporting of the Building Authority? A. The Building Authority is not a component unit of the city and will not be reported by the city. B. The Building Authority is a component unit and will be reported as blended with the city's funds. C. The Building Authority is a component unit and will be reported as discretely presented in the city's financial statements. D. The Building Authority is a component unit and the city will make a note disclosure only.

B. The Building Authority is a component unit and will be reported as blended with the city's funds. The Authority is a component unit of the city. The Authority's governing body is appointed by the city and the lease constitutes the imposition of a financial burden on the city. A component unit should be included in the reporting entity or primary government's financial statements using the blending method if the component unit's governing body is the same as the primary government; if the component unit provides services entirely or almost entirely to the primary government; or if the component unit's debt is expected to be repaid with resources provided by the primary government even if provided through a lease arrangement. The Authority will be reported using the blending method because the Authority provides services entirely to the city and its debt will be repaid with city resources over time. Only one of the three criteria needs to be met for the blending method to be used. In this case, two of the three criteria were met.

Which of the following is a required part of a local government's management's discussion and analysis (MD&A) as part of its financial statements? A. The MD&A should include an analysis for each fund. B. The MD&A should compare current-year results to the prior year with emphasis on the current year. C. The MD&A should present condensed financial information from the fund financial statements. D. The MD&A should be presented with other required supplementary information.

B. The MD&A should compare current-year results to the prior year with emphasis on the current year.

Parisian Company has bonds that have an original maturity of 10 years. The bonds will mature and be retired on May 25, Year 17. Parisian established a bond sinking fund, as required by the bond indenture (contract). The bond sinking fund will be used to retire the bonds when they mature in May of Year 17. How should the bonds be classified in the December 31, Year 16, balance sheet? A. The bonds should be classified as a current liability. B. The bonds should be classified as a long-term liability. C. Parisian can choose whether to classify the bonds as either current or noncurrent. D. Because of the sinking fund, Parisian can eliminate both the bonds and the sinking fund from the balance sheet.

B. The bonds should be classified as a long-term liability. Even though the bonds will mature in less than one year, they are classified as long-term liabilities (not current maturities of long-term liabilities) because the cash to be used to pay off the bonds is restricted and has therefore been classified as a long-term asset. Bonds in this situation should not be classified as current liabilities. Choice of classification (current vs. long-term) is not permitted. The presence of both the liability and the related asset (bond-sinking fund) are not offset on the balance sheet but are both reported in total.

Sig City used the following funds for financial reporting purposes: General fund Internal service fund Airport enterprise fund Pension trust fund Capital projects fund Special revenue fund Debt service fund How many of Sig's funds use the accrual basis of accounting? A. Two B. Three C. Four D> Five

B. Three The economic resources measurement focus and the accrual basis of accounting are used for proprietary funds and fiduciary funds. Proprietary funds consist of enterprise and internal service funds. GASB 1300.102.b indicates both enterprise and internal service funds use accrual accounting. The pension trust fund is a fiduciary fund. GASB 1300.102.b details that fiduciary funds use the accrual basis of accounting. The current financial resources measurement focus and the modified accrual basis of accounting are used for governmental funds. The general fund, capital projects fund, special revenue fund, and debt service fund are all governmental funds.

Derby Co. incurred costs to modify its building and to rearrange its production line. As a result, an overall reduction in production costs is expected. However, the modifications did not increase the building's market value, and the rearrangement did not extend the production line's life. Should the building modification costs and the production line rearrangement costs be capitalized? A. The building modification costs should be capitalized. B. The building modification costs and the production line rearrangement costs should both be capitalized. C. The production line rearrangement costs should be capitalized. D. Neither the building modification costs nor the production line rearrangement costs should be capitalized.

B. The building modification costs and the production line rearrangement costs should both be capitalized. The general rule here is that if an expenditure benefits periods other than the current period, the expenditure should be capitalized and charged (depreciated) to the present and all future periods benefited. This treatment should be applied to both the building modification costs and the production line rearrangement costs since the expenditures involved more than simple routine maintenance and are large dollar amounts, nonrecurring in nature, that increase the utility (efficiency) of the asset by decreasing production costs.

To determine the point in time when a customer obtains control of a promised asset (i.e., satisfies a performance obligation), which of the following is not an indicator of control that an entity considers? A. The customer has the significant risks and rewards of ownership of the asset. B. The customer has not accepted the asset and lacks legal title to the asset. C. The entity has transferred physical possession of the asset. D. The entity has a present right to payment for the asset.

B. The customer has not accepted the asset and lacks legal title to the asset. Transfer occurs when the customer obtains control of the goods or services, either at a point in time or over time, as applicable. Control in this context is defined as the ability to direct the use of and obtain substantially all of the remaining benefits from an asset, including the ability to prevent other entities from directing the use of and obtaining the benefits from an asset ("defensive control"). Additional indicators of control include the entity has a present right to payment for the asset; the customer has legal title to the asset; the entity has transferred physical possession of the asset; the customer has the significant risks and rewards of ownership of the asset; and the customer has accepted the asset.

The objectives of financial reporting stem from which of the following sources? A. The need for conservatism B. The needs of the external users of the information C. Reporting on management's stewardship D. Reporting on management's consistency

B. The needs of the external users of the information

Where in its financial statements should a company disclose information about its concentration of credit risks? A. No disclosure is required. B. The notes to the financial statements C. Supplementary information to the financial statements D. Management's report to shareholders

B. The notes to the financial statements

An entity offers customers a mail-in rebate when the customer purchases the product of the entity. Which of the following statements is not correct? A. The refund liability and corresponding change in the transaction price should be updated at the end of each reporting period. B. The refund liability and the contract liability should be updated annually. C. An entity should recognize a refund liability if it receives consideration from a customer and expects to refund some or all of that consideration to the customer. D. The entity would estimate the amount of rebate it expects to pay related to the rebate, reduce revenue recognized by that amount, and record a liability for the amount it expects to pay for the rebate.

B. The refund liability and the contract liability should be updated annually.

In its financial statements, Pare, Inc., uses the cost method of accounting for its 15% ownership of Sabe Co. On December 31, 20X1, Pare has a receivable from Sabe. How should the receivable be reported in Pare's December 31, 20X1, balance sheet? A. Eighty-five percent (85%) of the receivable should be reported separately, with the balance offset against Sabe's payable to Pare. B. The total receivable should be reported separately. C. The total receivable should be offset against Sabe's payable to Pare, without separate disclosure. D. The total receivable should be included as part of the investment in Sabe, without separate disclosure.

B. The total receivable should be reported separately. The total receivable should be reported separately. The equity method would be used at the 20% ownership level but would not change the requirement to report the receivable separately. At the 50%-plus level of ownership, consolidation would require elimination of the receivable as an intercompany item.

Which of the following common characteristics of derivative financial instruments distinguishes them from other types of financial instruments? A. They impose a contractual obligation by one entity to deliver cash to a second entity to convey a contractual right. B. They have a notional amount or payment provision that is based on the changes in one or more underlying variables. C. They are financial investments in stocks, bonds, or other securities that are marketable. D. Most financial instruments are valued on the balance sheet at fair value, but derivatives are valued on the balance sheet at cost.

B. They have a notional amount or payment provision that is based on the changes in one or more underlying variables. A derivative instrument must have all three of the following characteristics: It has (1) one or more underlyings and (2) one or more notional amounts or payment provisions, or both. Those terms determine the amount of the settlement or settlements and, in some cases, whether or not a settlement is required. It requires no initial net investment or an initial net investment that is smaller than would be required for other types of contracts that would be expected to have a similar response to changes in market factors. Its terms require or permit net settlement, it can readily be settled net by a means outside the contract, or it provides for delivery of an asset that puts the recipient in a position not substantially different from net settlement. Derivatives are reported at fair value, not cost. Derivatives can be net settled or settled with delivery of the item being hedged or speculated (e.g., a share of stock, barrel of oil, etc.). They are not direct investments—alternatively, stocks, bonds, or other securities often represent the underlyings.

Combined statements may be used to present the results of operations of: A. commonly controlled companies. B. both commonly controlled companies and companies under common management. C. companies under common management. D. neither commonly controlled companies nor companies under common management.

B. both commonly controlled companies and companies under common management.

What is the primary purpose of the statement of activities of a nongovernmental not-for-profit organization? A. To report assets, liabilities, and net assets as of a specific date B. To report the change in net assets for the period C. To report the liquidity of the entity as of a specific date D. To report the cash flow position of the entity for the period

B. To report the change in net assets for the period

An organization is normally considered a governmental organization if: A. the organization is exempt from federal taxation. B. a controlling majority of the members of its governing board are appointed by state government officials. C. the organization is exempt from federal taxation and a controlling majority of the members of its governing board are appointed by state government officials. D. None of the answer choices are correct.

B. a controlling majority of the members of its governing board are appointed by state government officials.

For a company to obtain a retail business license in a particular state, the company is required to pay the state the equivalent of three months of sales taxes on its projected retail sales. This amount is fully refundable after five years, provided the company has filed all required sales tax returns and paid all sales taxes due. Initially the company should report the payment related to this licensing requirement as: A. an expense. B. a noncurrent asset. C. a current asset. D. a noncurrent liability.

B. a noncurrent asset. Current assets include cash or other assets expected to be turned into cash or consumed within the year. This deposit of sales taxes will not be returned within the year.

When collectibility is reasonably assured, the excess of the subscription price over the stated value of the no par common stock subscribed should be recorded as: A. additional paid-in capital when the subscription is collected. B. additional paid-in capital when the subscription is recorded. C. additional paid-in capital when the common stock is issued. D. no par common stock.

B. additional paid-in capital when the subscription is recorded. Journal entry to record subscription for no par common stock with stated value: Dr. Stock subscriptions receivable XXX Common stock subscribed (for shares subscribed x stated value per share) XX Additional paid-in capital X

When a purchase order is issued, a commitment is made by a governmental unit to buy a computer to be manufactured to specifications for use in property tax administration. This commitment should be recorded in the general fund as: A. an appropriation. B. an encumbrance. C. an expenditure. D. a fixed asset.

B. an encumbrance. Encumbrances represent commitments to purchase goods or services. They are recorded for budgetary control purposes to prevent over expenditure of appropriations, especially in General and Special Revenue Funds.

When a snowplow purchased by a governmental unit is received, it should be recorded in the general fund as: A. a fixed asset. B. an expenditure. C. an appropriation. D. an encumbrance.

B. an expenditure.

A city council designates funds in the enterprise fund for future equipment replacement. The enterprise fund should report this as: A. a restricted component of net position. B. an unrestricted component of net position. C. a designated component of net position. D. a net investment in capital assets.

B. an unrestricted component of net position. Unrestricted net position is defined by exclusion. If an item is not classified as restricted or a capital asset, then it is unrestricted. While future equipment will be a capital asset, it is not currently a capital asset.

In the current year, a state government collected income taxes of $8,000,000 for the benefit of one of its cities that imposes an income tax on its residents. The state remitted these collections periodically to the city. The state should account for the $8,000,000 in the: A. special assessment funds. B. custodial fund. C. general fund. D. internal service funds.

B. custodial fund. When one governmental entity collects revenues for another and has the responsibility to remit the funds to the other entity, the first entity acts as a fiduciary for the second. The state must record the collected taxes in a custodial fund. This is a fund which allows the collection and subsequent remission of the monies to be recorded without affecting the funds with which the state itself operates. (GASB 1300.114)

Civic Town's basic financial statements included information for the nonmajor governmental funds in combined form. The aggregated data included expenditures summarized by major functional classifications. Narrative explanations are needed to accompany the combining statements of revenues, expenditures, and changes in fund balances to provide greater detail and assure the reader's understanding of the statements. The narrative should appear: A. in a separate section of management's discussion and analysis. B. directly on the combining statements, on divider pages or in a separate section. C. in the required supplementary information following the financial statements. D. in the notes to the financial statements.

B. directly on the combining statements, on divider pages or in a separate section. Narrative explanations of combining and individual fund statements should be presented on divider pages, directly on the statements and schedules, or in a separate section according to GASB 2200.211.

Baker Co. uses the calendar year as its accounting year. During 20X1, Congress enacted new tax legislation that changed the tax rate for 20X2 from 30% to 40%. The tax rate for 20X3 and following years remained at 30%. Baker has only one type of temporary difference or carryforward—a taxable temporary difference. Accordingly, Baker had a deferred tax liability at the beginning of 20X1 and will have a deferred tax liability at the end of 20X2. With regard to the change in tax rates, Baker should: A. include the effect of the change on the January 1, 20X1, deferred tax liability in 20X1 net income as the cumulative effect of a change in accounting principle. B. include the effect of the change on the January 1, 20X1, deferred tax liability in income from continuing operations of 20X1. C. include the effect of the change on the January 1, 20X1, deferred tax liability in income from continuing operations of 20X2. D. include the effect of the change on the January 1, 20X1, deferred tax liability in 20X2 net income as the cumulative effect of a change in accounting principle.

B. include the effect of the change on the January 1, 20X1, deferred tax liability in income from continuing operations of 20X1. The effect of the change on the deferred tax asset or liability at the beginning of the year of change should be included in income from continuing operations for the period that includes the enactment date.

A deferred tax asset of $100,000 was recognized in the Year 1 financial statements by the Chaise Company when a loss from discontinued segments was carried forward for tax purposes. A valuation allowance of $100,000 was also recognized in the Year 1 statements because it was considered more likely than not that the deferred tax asset would not be realized. Chaise had no temporary differences. The tax benefit of the loss carried forward reduced current taxes payable on Year 3 continuing operations. The Year 3 income statement would include the tax benefit from the loss brought forward in: A. gain or loss from discontinued segments. B. income from continuing operations. C. cumulative effect of accounting changes. D. extraordinary gains.

B. income from continuing operations. When a loss is recognized for financial accounting, it usually generates an income tax benefit at the same time, as long as it is true that prior income and income taxes paid could be refunded or if it is expected that there will be future income to deduct the loss against. If there was no income to deduct the loss against, then the tax benefit was valued at zero for the year (using the valuation allowance specified). When later income is earned and the tax benefit can be used, then the tax benefit is recognized in the later year when the income is earned, and is caused by the later income. (The income tax benefit is thus allocated to the later year when the income arises to allow the deduction, and is allocated to the income from continuing operations from the later year.) Note that the concept of "extraordinary" items has been eliminated from GAAP; the presentation for items that are unusual in nature or occur infrequently will be expanded to include items that are both unusual in nature and infrequently occurring.

A note receivable bearing a reasonable interest rate is sold to a bank with recourse. At the date of the discounting transaction, the notes receivable discounted account should be: A. decreased by the proceeds from the discounting transaction. B. increased by the face amount of the note. C. increased by the proceeds from the discounting transaction. D. decreased by the face amount of the note.

B. increased by the face amount of the note. A note sold with recourse is a promise to pay the financial institution if the maker dishonors the note. When receivables are sold with recourse, the entity has a contingent liability. A contingent liability is an obligation that has to be paid in the future. Therefore, the notes receivable discounted account must be increased by the face amount of the note.

Some events provide evidence regarding conditions that did not exist on the balance sheet date, but arose subsequently and do not require an adjustment of the balance sheet. Assuming that the item is material, an example of a subsequent event that requires adjustment is: A. stock splits. B. loss on account receivable resulting from customer's bankruptcy. C. loss from inventory fire. D. sale of bonds.

B. loss on account receivable resulting from customer's bankruptcy. "Subsequent events affecting the realization of assets, such as receivables and inventories or the settlement of estimated liabilities, should be recognized in the financial statements when those events represent the culmination of conditions that existed over a relatively long period of time.

Expenditures of a governmental unit for insurance extending over more than one accounting period: A. must be accounted for as expenditures of the period of acquisition. B. may be allocated between or among accounting periods or may be accounted for as expenditures of the period of acquisition. C. must be accounted for as expenditures of the periods subsequent to acquisition. D. must be allocated between or among accounting periods.

B. may be allocated between or among accounting periods or may be accounted for as expenditures of the period of acquisition. GASB 1600.127.b states: "Expenditures for insurance and similar services (prepaid items) extending over more than one accounting period need not be allocated between or among accounting periods, but may be accounted for as expenditures of the period of acquisition." (Emphasis added)

Pine City owned a vacant plot of land zoned for industrial use. Pine gave this land to Medi Corp. solely as an incentive for Medi to build a factory on the site. The land had a fair value of $300,000 at the date of the gift. This nonmonetary transaction should be reported by Medi as: A. additional paid-in capital. B. nonoperating income. C. a memorandum entry. D. a credit to retained earnings.

B. nonoperating income.

An unrestricted grant received from another government to support enterprise fund operations should be reported as: A. contributed capital. B. nonoperating revenues. C. revenues and expenditures. D. operating revenues.

B. nonoperating revenues. Proprietary funds of governments, including enterprise funds, report unrestricted grants and restricted operating grants as nonoperating revenues. Restricted capital grants are reported as capital contributions. Neither contributed capital nor expenditures are reported for proprietary funds.

Statement of Financial Accounting Concepts 8 (SFAC 8), Chapter 8, relates to: A. objectives of financial reporting. B. notes to financial statements. C. qualitative characteristics of financial reporting. D. elements of financial statements.

B. notes to financial statements.

The letter of transmittal and the statistical section are classified as: A. basic financial statement. B. other. C. required supplementary schedule. D. All of the answer choices are correct.

B. other. The letter of transmittal and the statistical section required for an ACFR (annual comprehensive financial report) and GFOA's (Government Finance Officers Association) certificate are neither basic statements nor required supplemental information.

The revenues control account of a governmental unit is increased when: A. the encumbrance account is decreased. B. property taxes are recorded. C. appropriations are recorded. D. the budget is recorded.

B. property taxes are recorded. Crediting the revenues control account signifies either that cash has been collected, or that a valid receivable exists. In practice, when property taxes are levied, a receivable is created. The debit to property taxes receivable is offset by a credit to revenues to the extent that the taxes are "susceptible to accrual;" that is, both measurable and available to pay liabilities of the fiscal period. (To the extent that receivable items do not meet the susceptible to accrual criteria, deferred revenues, a liability, is credited.) Budgetary events and accounts (the other answer choices) do not affect the revenue control account.

Darrow Limited operates under a franchise agreement that has limited useful life. On Darrow's balance sheet, the intangible asset "Franchise" has a book value of $123,500. Because of declining economic conditions, Darrow determines that the undiscounted cash flows from the franchise are $107,000. Darrow recently received an offer of $95,000 to sell the franchise for its remaining useful life. Darrow should: A. recognize an impairment loss of $16,500. B. recognize an impairment loss of $28,500. C. recognize an impairment loss of $8,000. D. not recognize an impairment loss.

B. recognize an impairment loss of $28,500. Because the franchise is a limited-life intangible asset, the two-step impairment process should be used. The recoverability test (comparison of book value to undiscounted cash flows) indicates that the book value will not be recoverable (undiscounted cash flows of $107,000 is less than the book value of $123,500) and therefore the second step needs to be carried out. Because the fair value ($95,000) of the franchise is less than the book value ($123,500), the difference between the two values ($123,500 - $95,000 = $28,500) is the amount of the impairment loss.

Instead of the usual cash dividend, Evie Corp. declared and distributed a property dividend from its overstocked merchandise. The excess of the merchandise's carrying amount over its market value should be: A. reported as a separately disclosed reduction of retained earnings. B. reported as a reduction in income. C. reported as an ordinary loss, net of income taxes. D. ignored.

B. reported as a reduction in income. FASB ASC 845-10-30-1 requires that "a transfer of a nonmonetary asset to a stockholder or to another entity in a nonreciprocal transfer should be recorded at the fair value of the asset transferred, and a gain or loss should be recognized on the disposition of the asset." Since the market value of the merchandise was less than its carrying amount, Evie Corp. should report the resulting loss as a reduction in income

The City of Smithville's Management's Discussion and Analysis is classified as: A. other. B. required supplementary information. C. essential statistical information. D. basic financial statement.

B. required supplementary information. The management's discussion and analysis, or MD&A, is a component of the required supplemental information. The MD&A should introduce the basic financial statements and provide an analytical overview of the government's financial activities.

An entity should recognize an impairment loss in profit or loss to the extent that the carrying amount of an asset recognized exceeds: A. the remaining amount of consideration that the entity expects to receive in exchange for the goods or services to which the asset relates. B. the remaining amount of consideration that the entity expects to receive in exchange for the goods or services to which the asset relates, less the costs that relate directly to providing those goods or services that have not been recognized as expenses. C. the costs that relate directly to providing those goods or services that have not been recognized as expenses. D. costs that are explicitly chargeable to the customer under the contract.

B. the remaining amount of consideration that the entity expects to receive in exchange for the goods or services to which the asset relates, less the costs that relate directly to providing those goods or services that have not been recognized as expenses.

On January 2, the City of Walton issued $500,000, 10-year, 7% general obligation bonds. Interest is payable annually, beginning January 2 of the following year. What amount of bond interest is Walton required to report in the statement of revenue, expenditures, and changes in fund balance of its governmental funds at the close of this fiscal year, September 30? A. $26,250 B. $17,500 C. $0 D. $35,000

C. $0 Principal and interest on general long-term liabilities of state and local governments must be reported in the period in which they become legally due and payable. An exception is permitted, but not required, if the due date is in the first 30 days of the next fiscal year (not the case here) and dedicated resources have been set aside in a debt service fund by the end of the year (not indicated here). In any event, even if the two conditions are met, the question asked for the required amount to report for bond interest. Because no bond interest becomes legally due and payable (matures) during the year, $0 of bond interest is required to be reported.

During 20X1, Beam Co. paid $1,000 cash and traded inventory, which had a carrying amount of $20,000 and a fair value of $21,000, for other inventory in the same line of business with a fair value of $22,000. The exchange of the inventory is to facilitate sales to Beam's customers. What amount of gain (loss) should Beam record related to the inventory exchange? A. $2,000 B. $1,000 C. $0 D. $(1,000)

C. $0 The exchange of the inventory is to facilitate sales to Beam's customers. The exchange should be recorded based on carrying amounts with no gain recognized. If the inventory's carrying amount had been in excess of the fair value of the inventory given up, the inventory given up should have been written down and the loss recognized before the exchange was recorded.

During the year, Granite Co. sold a building for $100,000, resulting in a gain of $20,000. The building has a net book value of $80,000 at the time of the sale. Granite uses the indirect method when preparing its statement of cash flows. What is the amount that would be included in Granite's financing activities section because of the building sale? A. $20,000 B. $100,000 C. $0 D. $80,000

C. $0 The proceeds from the sale of the building are classified as an investing activity; gains are an operating activity adjustment for noncash items under the indirect method of cash flow presentation. Therefore, the amount that would be included in the financing section is $0.

On January 1, Year 1, Newport Corp. purchased a machine for $100,000. The machine was depreciated using the straight-line method over a 10-year period with no residual value. Because of a bookkeeping error, no depreciation was recognized in Newport's Year 1 financial statements, resulting in a $10,000 overstatement of the book value of the machine on December 31, Year 1. The oversight was discovered during the preparation of Newport's Year 2 financial statements. What amount should Newport report for depreciation expense on the machine in the Year 2 financial statements? A. $11,000 B. $9,000 C. $10,000 D. $20,000

C. $10,000 The correct amount of depreciation for all 10 years is $100,000 ÷ 10 = $10,000. The correction of the error for Year 1 would not affect the amount of depreciation expense for Year 2.

On January 2, 20X2, Lake Mining Co.'s board of directors declared a cash dividend of $400,000 to stockholders of record on January 18, 20X2, payable on February 10, 20X2. The dividend is permissible under law in Lake's state of incorporation. Selected data from Lake's December 31, 20X1, balance sheet are as follows: Accumulated depletion $100,000Capital stock 500,000Additional paid-in capital 150,000Retained earnings 300,000 The $400,000 dividend includes a liquidating dividend of: A. $300,000. B. $0. C. $100,000. D. $150,000.

C. $100,000. The liquidating dividend is that portion of the cash dividend that exceeds the balance in retained earnings because other equity accounts must be debited.

A company incurred the following costs to complete a business combination in the current year: Issuing debt securities $30,000 Registering debt securities 25,000 Legal fees 10,000 Due diligence costs 1,000 What amount should be reported as current-year expenses, not subject to amortization? A. $1,000 B. $36,000 C. $11,000 D. $66,000

C. $11,000 A business combination occurs when two or more business enterprises are brought under common control and into one accounting entity (e.g., mergers, consolidations, or acquisitions). Acquisition costs and restructuring costs must be recognized separately from the acquisition itself in the acquirer's post-combination financial statements in accordance with GAAP—usually expensed. The legal fees and due diligence costs that total $11,000 are considered acquisition costs and should be expensed in the current year. Debt issuance and debt registration costs are part of the consideration given, and recorded in conjunction with the debt itself.

During January of the current year, Haze Corp. won a litigation award for $15,000 which was tripled to $45,000 to include punitive damages. The defendant, who is financially stable, has appealed only the $30,000 punitive damages. Haze was awarded $50,000 in an unrelated suit it filed, which is being appealed by the defendant. Counsel is unable to estimate the outcome of these appeals. In its current year financial statements, Haze should report what amount of pretax gain? A. $95,000 B. $45,000 C. $15,000 D. $50,000

C. $15,000 Both of these lawsuits are gain contingencies, which are generally not recognized until it is virtually certain that there are rights to receive the amounts involved. Thus, the amounts that are yet to be received, and subject to the uncertainty of an appeal process, should not be recognized yet. However, the $15,000 award that has not been appealed can be taken now as pretax gain.

Fireworks, Inc., had an explosion in its plant that destroyed most of its inventory. Its records show that beginning inventory was $40,000. Fireworks made purchases of $480,000 and sales of $620,000 during the year. Its normal gross profit percentage is 25%. It can sell some of its damaged inventory for $5,000. The insurance company will reimburse Fireworks for 70% of its loss. What amount should Fireworks report as loss from the explosion? A. $18,000 B. $50,000 C. $15,000 D. $35,000

C. $15,000 This problem must be solved using the gross profit method: Goods available for sale = $40,000 + $480,000 = $520,000 Gross profit = $620,000 × 0.25 = $155,000 Cost of goods sold = $620,000 - $155,000 = $465,000 Ending inventory = $520,000 - $465,000 = $55,000 Reimbursement = ($55,000 - $5,000) × 0.70 = $35,000 Loss = $55,000 - $5,000 - $35,000 = $15,000

The Dunstown County general fund received a notice of a federal grant award for an expenditure-driven (reimbursement) grant in the amount of $1,000,000. Included with the notice was an advance of $250,000. During the year, the County incurred $400,000 of qualifying eligible grant expenditures, and no additional money had been received from the grantor. What would be the amount of intergovernmental receivables reported by the general fund at the close of the fiscal year? A. $0 B. $750,000 C. $150,000 D. $600,000

C. $150,000 The receivable would equal the difference between the amount of the advance and the revenues of $400,000 (the amount spent on qualifying expenditures). Revenues $400,000Less advance received (250,000)Intergovernmental receivable $150,000

Oak Co., a newly formed corporation, incurred the following expenditures related to land and building: County assessment for sewer lines $ 2,500Title search fees 625Cash paid for land with a building to be demolished 135,000Excavation for construction of basement 21,000Removal of old building $21,000 less salvage of $5,000 16,000 At what amount should Oak record the land? A. $175,625 B. $153,500 C. $154,125 D. $138,125

C. $154,125 The cost of plant assets includes all expenditures necessary to acquire the asset and prepare it for its intended use. The cost of land includes the purchase price, costs incidental to acquisition (such as legal fees, commissions, and title insurance), and the costs of preparing the land for use (such as the costs of surveying, grading, filling, draining, and clearing). The cost of tearing down an existing building is included in the cost of the land. All of the costs presented other than the excavation should be included in the cost of the land. The excavation for the basement will be included in the cost of the building.

Carr, Inc., purchased equipment for $100,000 on January 1, Year 1. The equipment had an estimated 10-year useful life and a $15,000 salvage value. Carr uses the 200% declining balance depreciation method. In its Year 2 income statement, what amount should Carr report as depreciation expense for the equipment? A. $17,000 B. $13,600 C. $16,000 D. $20,000

C. $16,000 100000*2/10=20000 (100000-20000)*2/10=16000

A company has the following liabilities at year-end: Mortgage note payable; $16,000 due within 12 months $355,000 Short-term debt that the company is refinancing with long-term debt 175,000 Deferred tax liability arising from depreciation 25,000 What amount should the company include in the current liability section of the balance sheet (statement of financial position)? A. $41,000 B. $0 C. $16,000 D. $191,000

C. $16,000 Only the current portion of the mortgage is included in current liabilities. All deferred tax liabilities and deferred tax assets are classified as noncurrent. The refinanced loan is not included in current liabilities. The FASB states that a short-term obligation should be excluded from current liabilities if the entity intends to refinance the obligation on a long-term basis, and that intent is supported by an ability to consummate the refinancing.

In Year 6, Stuckey Corporation determined that the 6-year estimated useful life of a machine purchased for $198,000 in January of Year 3 should be extended by 2 years. The machine is being depreciated using the straight-line method and has no salvage value. In addition to the extension of the useful life, Stuckey also determines the asset will have a salvage value of $12,000 at the end of its useful life. What amount of depreciation expense should Stuckey report in its financial statements for the year ending December 31, Year 6? A. $33,000 B. $34,800 C. $17,400 D. $19,800

C. $17,400 Changes in accounting estimates are handled on a prospective basis—in the current year and future years. Note disclosure includes an explanation and justification for the change and the impact on current income and EPS (earnings per share). Stuckey should report $17,400 depreciation expense for the year ending December 31, Year 6: Original cost $198,000Depreciation Years 1-3 ($198,000 × 3 / 6 yrs.) 99,000Book value at beginning of Year 6 $ 99,000Year 6 depreciation (($99,000 - 12,000) ÷ 5) $ 17,400

The stockholders' equity section of Brown Co.'s December 31, 20X1, balance sheet consisted of the following: Common stock, $30 par, 10,000shares authorized and outstanding $300,000Additional paid-in capital 150,000Retained earnings (deficit) (210,000) On January 2, 20X2, Brown put into effect a stockholder-approved quasi-reorganization by reducing the par value of the stock to $5 and eliminating the deficit against additional paid-in capital. Immediately after the quasi-reorganization, what amount should Brown report as additional paid-in capital? A. $150,000 B. $400,000 C. $190,000 D. $(60,000)

C. $190,000

On December 1, 20X1, Money Co. gave Home Co. a $200,000, 11% loan. Money paid proceeds of $194,000 after the deduction of a $6,000 nonrefundable loan origination fee. Principal and interest are due in 60 monthly installments of $4,310, beginning January 1, 20X2. The repayments yield an effective interest rate of 11% at a present value of $200,000 and 12.4% at a present value of $194,000. What amount of income from this loan should Money report in its 20X1 income statement? A. $7,833 B. $1,833 C. $2,005 D. $0

C. $2,005 Net proceeds of the loan were $194,000 and the effective interest rate was 12.4%. The journal entry to record the December 31, 20X1, accrual of interest would be: Debit Accrued Interest Receivable 2,005Credit Interest Income 2,005(12.4% × $194,000 × 1/12) The interest income reflects the effective interest rate applied to the net proceeds received. This is an application of the "effective interest" method.

Park City uses modified accrual and encumbrance accounting and formally integrates its budget into the general fund's accounting records. For the year ending July 31, 20X1, the following budget was adopted: Estimated revenues $30,000,000Appropriations 27,000,000Estimated transfer to debt service fund 900,000 When Park's budget is adopted and recorded, Park's budgetary fund balance would be a: A. $3,000,000 credit balance. B. $3,000,000 debit balance. C. $2,100,000 credit balance. D. $2,100,000 debit balance.

C. $2,100,000 credit balance. The complete entry to record the adopted budget is: Estimated revenues 30,000,000 Appropriations control 27,000,000Estimated transfer to debt service 900,000Budgetary Fund Balance 2,100,000

For the fall semester of 20X1, State University, a government university, assessed its students $3,000,000 for tuition and fees. The net amount realized was only $2,500,000 because tuition and fee waivers of $400,000 were given and uncollectible amounts of $100,000 were estimated. What amount should State University report for the period as operating revenues from tuition and fees? A. $3,000,000 B. $2,900,000 C. $2,500,000 D. $2,600,000

C. $2,500,000 Tuition and fee revenues should be reported net of tuition discounts and allowances with the discount or allowance amount parenthetically disclosed on the face of the statement. Tuition waivers are a kind of discount that lowers net tuition received from specific students. Uncollectible accounts are recognized with the use of an allowance for government college and university accounting.

A nongovernmental, not-for-profit entity calculated a $4,000 increase in net assets with donor restrictions for the current fiscal year before consideration of the following: A cash donation designated by the donoras an endowment in perpetuity $28,000Net assets released from restrictions 12,000A donation received that was designatedas a quasi-endowment 21,000 Which of the following should be reported as the increase in net assets with donor restrictions in the current-year statement of activities? A. $41,000 B. $37,000 C. $20,000 D. $16,000

C. $20,000 The answer choice "20,000" is correct: $28,000 − $12,000 + $4,000 = $20,000. The nongovernmental, not-for-profit entity should take the $4,000 increase in net assets and add the cash donation of $28,000 as this serves as an example of a donor-imposed restriction. Then, the entity should subtract net assets released from restrictions of $12,000 since the money is now unrestricted. The donation received of $21,000 that was designated as a quasi-endowment is not factored into the analysis. These quasi-endowments are classified among net assets without donor restrictions since future governing boards could undo the designation.

Eagle and Falk are partners with capital balances of $45,000 and $25,000, respectively. They agree to admit Robb as a partner. After the assets of the partnership are revalued, Robb will have a 25% interest in capital and profits, for an investment of $30,000. What amount should be recorded as goodwill to the original partners? A. $0 B. $7,500 C. $20,000 D. $5,000

C. $20,000 When a new partner is admitted by investing into the partnership, the total capital of the partnership changes, and the purchase price (amount of new investment) can be equal to, more than, or less than book value. When the purchase price is equal to book value, no goodwill or bonuses are recorded. When the purchase price is more or less than book value, either goodwill or bonuses must be recorded. The total capital of the resulting new partnership determines whether goodwill or bonuses are recorded. Under the goodwill approach, goodwill is recognized on the basis of the total value of the new partnership implied by the new partner's investment relative to the partners' total capital. Under the bonus approach, such implied value is not considered. In this problem, the assets are revalued, suggesting that goodwill is being recorded. Implied value after new investment: $30,000 represents 25% of total value; therefore, the implied total value is $120,000 ($30,000 ÷ .25). Implied Value $120,000Total partner's capital accounts (100,000)(45,000 + $25,000 + $30,000)Goodwill to original partners $ 20,000

Mobe Co. reported the following operating income (loss) for its first three years of operations: 20X0 $ 300,00020X1 (700,000)20X2 1,200,000 For each year, there were no deferred income taxes, and Mobe's effective income tax rate was 30%. In its 20X1 income tax return, Mobe elected to carry back the maximum amount of loss possible. In 20X1, Mobe was unsure that it would earn any future taxable income, thus requiring a valuation allowance to write down the deferred tax asset to zero until it is used next year. In its 20X2 income statement, what amount should Mobe report as total income tax expense? A. $120,000 B. $150,000 C. $240,000 D. $360,000

C. $240,000 20X1 loss after carryback to 20X0: = ($700,000) − $300,000= ($400,000) 20X2 income after carryforward of remainder of 20X1 loss: = $1,200,000 − $400,000= $800,000 20X2 income tax expense: = ($800,000 × 0.30)= $240,000

The following condensed balance sheet is presented for the partnership of Alfa and Beda, who share profits and losses in the ratio of 60:40, respectively: Cash $ 45,000 Other assets 625,000 Beda (loan) 30,000 =$700,000 Accounts payable $120,000 Alfa (capital) 348,000 Beda (capital) 232,000 =$700,000 Instead of admitting a new partner, Alfa and Beda decide to liquidate the partnership. If the other assets are sold for $500,000, what amount of the available cash should be distributed to Alfa? A. $348,000 B. $255,000 C. $273,000 D. $327,000

C. $273,000 Prior to any liquidation computations, combine the partner's loans to the partnership with their capital account balances. Then pay the debts, sell the assets (and account for losses on sales), divide up the losses, and then distribute the remaining cash based on the remaining partner capital account balances. Loss from sale of other assets = $625,000 - $500,000 = $125,000Allocation of loss to Alfa = .60 x $125,000 = $ 75,000Alfa's new capital balance = $348,000 - $75,000 = $273,000

Campbell Corp. exchanged delivery trucks with Highway, Inc. Campbell's truck originally cost $23,000, its accumulated depreciation was $20,000, and its fair value was $5,000. Highway's truck originally cost $23,500, its accumulated depreciation was $19,900, and its fair value was $5,700. Campbell also paid Highway $700 in cash as part of the transaction. The transaction lacks commercial substance. What amount is the new book value for the truck Campbell received? A. $3,000 B. $5,700 C. $3,700 D. $5,000

C. $3,700 Use carrying value of asset surrendered 3000 + 700 (cash paid)

On July 1, one of Rudd Co.'s delivery vans was destroyed in an accident. On that date, the van's carrying amount was $2,500. On July 15, Rudd received and recorded a $700 invoice for a new engine installed in the van in May, and another $500 invoice for various repairs. In August, Rudd received $3,500 under its insurance policy on the van, which it plans to use to replace the van. What amount should Rudd report as gain (loss) on disposal of the van in its year-end income statement? A. $(200) B. $1,000 C. $300 D. $0

C. $300 The cost of the van prior to its destruction should include the carrying amount and the new engine installation cost of $3,200 ($2,500 + $700). The repairs should be expensed, not capitalized. Thus, the gain on the destruction of the van should be the insurance proceeds of $3,500 less the carrying cost of $3,200 for a gain of $300 ($3,500 - $3,200).

A local government recorded revenues as follows: personal income tax, $200,000; sales taxes, $100,000; and property taxes, $150,000. What should the local government report as total derived tax revenue? A. $100,000 B. $150,000 C. $300,000 D. $450,000

C. $300,000 Derived tax revenues are those revenues that result from assessments on exchange transactions. Those transactions are commonly taxes on income or sales taxes. In this case, that is the $200,000 personal income tax and the $100,000 of sales taxes for a total derived tax revenue of $300,000. Property taxes are classified as imposed nonexchange transactions.

Cy City's Municipal Solid Waste Landfill Enterprise Fund was established when a new landfill was opened January 3, 2006. The landfill is expected to close December 31, 2027. Cy's 2006 expenses would include a portion of which of the year 2027 expected disbursements? I. Cost of a final cover to be applied to the landfill II. Cost of equipment to be installed to monitor methane gas buildup A. II only B. I only C. Both I and II D. Neither I nor II

C. Both I and II GASB L10.103 states that both the costs of a final cover to be applied to the landfill, and costs of postclosing monitoring equipment should be included in the estimated total current cost of landfill closure and postclosure care.

Foster Company reported a retained earnings balance of $402,000 on December 31, 20X4. In May 20X5, Foster discovered that merchandise costing $95,000 had been erroneously included in inventory in its 20X4 financial statements. Foster has a 25% tax rate. What amount should Foster report as adjusted beginning retained earnings in its statement of retained earnings at December 31, 20X5? A. $402,000 B. $473,750 C. $330,750 D. $425,750

C. $330,750 Foster should report $330,750 as adjusted beginning retained earnings: Reported retained earnings (12/31/X4) $402,000Correction of error:DECREASE in income ($95,000)*Less income taxes (25%) 23,750Prior-period adjustment (71,250)Adjusted beginning retained earnings $330,750======== * Inventory was overstated in 20X4, meaning inventory was reported as ending inventory when it should have been included in cost of goods sold, leading to a lower net income in 20X4. Therefore, 20X4 was overstated, causing beginning retained earnings to also be overstated.

Cyan Corp. issued 20,000 shares of $5 par common stock at $10 per share. On December 31, 20X1, Cyan's retained earnings were $300,000. In March 20X2, Cyan reacquired 5,000 shares of its common stock at $20 per share. In June 20X2, Cyan sold 1,000 of these shares to its corporate officers for $25 per share. Cyan uses the cost method to record treasury stock. Net income for the year ending December 31, 20X2, was $60,000. On December 31, 20X2, what amount should Cyan report as retained earnings? A. $380,000 B. $365,000 C. $360,000 D. $375,000

C. $360,000 Note: The re-issuance of the 5,000 shares of treasury stock at $5 more per share than acquisition cost ($25 sales price − $20 acquisition cost) resulted in a credit to additional paid-in capital. Retained earnings were not affected. The journal entry to record the reissuance would be as follows:

A company that uses the accrual method of accounting started the fiscal year with assets of $600,000 and liabilities of $400,000. During the fiscal year the company recorded credit sales of $250,000, of which $8,000 remained to be collected at year-end, and incurred expenses of $90,000, of which $72,000 was paid in cash. A stock dividend valued at $10,000 was declared and issued to stockholders during the year. What is the year-end balance of total equity? A. $350,000 B. $370,000 C. $360,000 D. $380,000

C. $360,000 Stockholders' equity is equal to the net assets of the balance sheet (assets minus liabilities) and primarily includes retained earnings and stock issued and repurchased by the company. Retained earnings, and therefore stockholders' equity, is increased by net income and decreased by cash dividends declared. Stock dividends have no net impact on equity. The beginning equity balance is equal to the net assets of $600,000 less liabilities of $400,000, or $200,000. Year-end equity is $360,000, computed as follows:

On January 2, Vole Co. issued bonds with a face value of $480,000 at a discount to yield 10%. The bonds pay interest semiannually. On June 30, Vole paid bond interest of $14,400. After Vole recorded amortization of the bond discount of $3,600, the bonds had a carrying amount of $363,600. What amount did Vole receive upon issuing the bonds? A. $476,400 B. $367,200 C. $360,000 D. $480,000

C. $360,000 When bonds are issued at a discount, the carrying value of the bonds is less than the face value. The initial carrying value is the issue price (proceeds received upon issuance). When you pay interest, you amortize the discount, making it smaller. As discount is amortized, the carrying value of the bond comes closer to face value. After the initial interest payment, therefore, the amortization of the bond discount on the first payment date was from the issue price to the present carrying amount. Subtract the discount amortization just added to get the present book value, the $3,600, to get the original book value, the issue price. So, the bond carrying cost after the first payment less the amortization of the first payment is the issue price: $363,600 - $3,600 = $360,000.

On January 2, Rafa Co. purchased a franchise with a useful life of 5 years for $45000. An additional franchise fee of 3% of franchise operation revenues must be paid each year to the franchisor. Revenues from franchise operations amounted to $240000 during the year. In its December 31 balance sheet, what amount should Rafa report as an intangible asset-franchise? A. $9000 B. $45000 C. $36000 D. $43200

C. $36000 A franchise is an intangible asset and, if it has a finite useful life, it is amortized over that useful life. Here the carrying amount of the franchise is $45000, and amortizing that cost equally over its useful life of 5 years would make for an amortization expense of $9000 each year. After one year, the franchise will be carried at $36000 ($45000 - $9000).

At the beginning of the fiscal year, Turmeric Corp. purchased 30% of Basil Co. In its balance sheet at the end of the fiscal year, Turmeric reported $482,500 as its investment in Basil. During the current year, Basil reported net income of $570,000 and declared and paid cash dividends of $190,000. Turmeric uses the equity method of accounting. How much did Turmeric pay when it purchased its investment in Basil at the beginning of the year? A. $539,500 B. $482,500 C. $368,500 D. $254,500

C. $368,500 Since the information given is the ending balance and not the beginning balance, one way to solve for the beginning balance is to use the following equation: Beginning balance + (0.30 × $570,000) - (0.30 × $190,000) = $482,500 Beginning balance = $482,500 - $171,000 + $57,000 Beginning balance = $368,500

Simm Co. has determined its December 31 inventory on a LIFO basis to be $400,000. Information pertaining to the inventory follows: Estimated selling price $408,000Estimated cost of disposal 20,000Normal profit margin 60,000Current replacement cost 390,000 At December 31, what should be the amount of Simm's inventory? A. $400,000 B. $390,000 C. $388,000 D. $328,000

C. $388,000 When computing the market price, first determine the floor and the ceiling. The ceiling is net realizable value less the sales price less costs to complete the inventory item and to dispose of it ($408,000 - $20,000 = $388,000). The floor is net realizable value less normal profit ($388,000 - $60,000 = $328,000). Provided the replacement cost falls within this range, it would be the designated market; however, in this case, it exceeds the ceiling amount, so the designated market value would be $388,000. Given that cost is $400,000, the lower of market or cost is market of $388,000.

On January 2, 20X1, Smith purchased the net assets of Jones's Cleaning, a sole proprietorship, for $350,000, and commenced operations of Spiffy Cleaning, a sole proprietorship. The assets had a carrying amount of $375,000 and a market value of $360,000. In Spiffy's cash-basis financial statements for the year ending December 31, 20X1, Spiffy reported revenues in excess of expenses of $60,000. Smith's drawings during 20X1 were $20,000. In Spiffy's financial statements, what amount should be reported as Capital-Smith? A. $400,000 B. $415,000 C. $390,000 D. $410,000

C. $390,000 Under the cash method of accounting, revenue is recognized when received in the form of cash, and expenses are recognized when paid in cash. The owner's capital account maintains a record of the owner's investment and the earnings for the owner, and is lowered by drawings. Capital-Smith balance January 2, 20X1 $350,000Add: Net income 60,000Subtotal $410,000Deduct: Withdrawals (20,000)Capital-Smith balance December 31, 20X1 $390,000

On March 4, 20X1, Evan Co. purchased 1,000 shares of LVC common stock at $80 per share. On September 26, 20X1, Evan received 1,000 stock rights to purchase an additional 1,000 shares at $90 per share. The stock rights had an expiration date of February 1, 20X2. On September 30, 20X1, LVC's common stock had a market value, ex-rights, of $95 per share and the stock rights had a market value of $5 each. What amount should Evan report on its September 30, 20X1, balance sheet for investment in stock rights? A. $5,000 B. $10,000 C. $4,000 D. $15,000

C. $4,000 The investor receiving stock rights must allocate a portion of the purchase price of the investment that "carried" the rights. Here, although the stock rights were received 6 months after the purchase, the rights still "ride" with the purchase. Thus, the stock rights are allocated a portion of the purchase price based on the market value of the rights as a percentage of the total market value of the stock investment plus the rights at the balance sheet date: Cost of shares acquired = 1,000 x $80 = $80,000Cost allocated to stock rights = $80,000 ($5 / ($95 + $5))= $80,000 ($5 / $100)= $4,000

Webb Co. has outstanding a 7%, 10-year $100,000 face-value bond. The bond was originally sold to yield 6% annual interest. Webb uses the effective interest rate method to amortize bond premium. On June 30, Year 2, the carrying amount of the outstanding bond was $105,000. What amount of unamortized premium on bond should Webb report in its June 30, Year 3, balance sheet? A. $1,050 B. $3,950 C. $4,300 D. $4,500

C. $4,300 The interest paid for the year from June 30, Year 2. to June 30, Year 3. is based on the face amount ($100,000) multiplied by the stated 7% payment rate: $100,000 × 0.07 = $7,000 The interest expense using the interest method is based on the carrying amount of the debt multiplied by the yield of the debt: $105,000 × 0.06 = $6,300 The premium amortized from June 30, Year 2, to June 30, Year 3, is the difference of these two amounts: $7,000 - $6,300 = $700 Thus, the premium of $5,000 on June 30, Year 2, ($105,000 - $100,000, carrying amount less face amount) is lowered by the $700 premium amortization down to $4,300: $5,000 - $700 = $4,300

State University received two contributions during the year that must be used to provide scholarships. Contribution A for $10,000 was collected during the year, and $8,000 was spent on scholarships. Contribution B is a pledge for $30,000 to be received next fiscal year. What amount of contribution revenue should the university report in its statement of activities? A. $38,000 B. $10,000 C. $40,000 D. $8,000

C. $40,000 State University would report the full amount of the pledges as revenue because the availability criterion does not apply. The Statement of Activities is a government-wide statement using the full accrual basis of accounting. The contributions are restricted to scholarships but this purpose restriction by the donor is not a condition that must be met before the donation is made. It does not function like an "expenditure driven" government grant (the recipient must incur allowable costs before it recognizes any grant revenue).

XL Software Company is developing a new software product. During 20X1, monthly costs of the project were $100,000 per month. A detailed program design was completed on August 31. How much of the development costs would be capitalized? A. $0 B. $1,200,000 C. $400,000 D. $800,000

C. $400,000 Computer software costs to be sold, leased, or otherwise marketed are charged to expense as research and development until technological feasibility has been established for the product. Technological feasibility is established on completion of a detailed program design or completion of a working model. Since a technological feasibility was established on August 31, all of the costs up to that date (8 × $100,000) would be expensed as research and development expenses. The costs after August 31 (4 × $100,000) would be capitalized.

At the beginning of the current year, Paxx County's enterprise fund had a $125,000 balance for accrued compensated absences. At the end of the year, the balance was $150,000. During the year, Paxx paid $400,000 for compensated absences. What amount of compensated absences expense should Paxx County's enterprise fund report for the year? A. $550,000 B. $400,000 C. $425,000 D. $375,000

C. $425,000 An enterprise fund is a proprietary fund that uses the economic resources measurement focus and accrual accounting. Therefore, the compensated absences long-term liability would be decreased when employees use their promised leave time and increased when employees accrue additional leave. The newly accrued leave would be recognized as an expense of the period. Therefore, analysis of the liability account indicates that $400,000 of compensated absence liability was extinguished during the year and $425,000 of additional liability incurred to explain the ending liability balance of $150,000.

During year 2, Pipp Co. incurred the following costs to develop and produce a routine, low-risk computer software product: Completion of detailed program design $10,000 Costs incurred for coding and testing to establish technological feasibility 15,000 Other coding and testing costs incurred after establishment of technological feasibility 44,000 In Pipp's December 31, year 2, balance sheet, what amount should be capitalized as software costs? A. $25,000 B. $10,000 C. $44,000 D. $59,000

C. $44,000 The only cost that is associated with a future benefit is the $44,000 in coding and testing costs incurred after feasibility was established. This amount would be capitalized as software costs on the balance sheet. The remaining $25,000 costs represent research and development costs. These costs relate to activities identified with the period prior to the beginning of commercial production and feasibility and are expensed as incurred because their ability to provide future benefits is still uncertain. Once feasibility is reached, the costs are capitalized.

On January 1, year 1, a company capitalized $100,000 of costs for software that is to be sold. The company amortizes the software costs on a straight-line basis over five years. The carrying value of the software costs on January 1, year 3, was $60,000. As of December 31, year 3, the estimated future gross revenue to be generated from the sale of the software is $23,000, and the estimated future cost of disposing of the software is $8,000. What amount should the company expense related to the software costs for the year ended December 31, year 3? A. $20,000 B. $18,400 C. $45,000 D. $37,000

C. $45,000 Software production costs are capitalized and reported at the lower of unamortized cost or net realizable value (NRV) once technological feasibility has been met. The unamortized cost is $60,000 and the NRV is $15,000 ($23,000 − $8,000); therefore, the software should be written down by $45,000 (i.e., expensed) to the NRV of $15,000.

Trans Co. had the following balances at December 31, 20X1: Cash in checking account $ 35,000Cash in money market account 75,000U.S. Treasury bill,purchased 11/01/20X1, maturing 01/31/20X2 350,000U.S. Treasury bill,purchased 12/01/20X1, maturing 03/31/20X2 400,000 Trans' policy is to treat as cash equivalents all highly liquid investments with a maturity of three months or less when purchased. What amount should Trans report as cash and cash equivalents in its December 31, 20X1, balance sheet? A. $110,000 B. $385,000 C. $460,000 D. $860,000

C. $460,000 FASB ASC 210-10-20 defines cash equivalents as highly liquid investments with original maturities of less than or equal to three months. Cash in a money market account is a cash equivalent, as is the U.S. Treasury bill purchased on November 1, 20X1, maturing January 31, 20X2. The U.S. Treasury bill, purchased December 1, 20X1, maturing March 31, 20X2, has an original maturity of greater than three months, and is therefore not a cash equivalent. To calculate the cash and cash equivalents balance, add the cash in money market account and the U.S. Treasury bill purchased on November 1, 20X1, to the cash in checking account ($35,000 + $75,000 + $350,000 = $460,000). Balance at December 31, 20X1 = $460,000.

A company holds a financial asset that is actively traded in two different markets. The company transacts in both markets equally. The price of the asset in market A is $50. If the company sells the asset in market A, it incurs a transaction cost of $4. The price of the asset in market B is $48. If the company sells the asset in market B, it incurs a transaction cost of $1. What is the fair value of the financial asset? A. $47 B. $46 C. $48 D. $50

C. $48 Fair value measurement requires the company to measure the asset in its most advantageous market if no principal market exists. The most advantageous market is the market where the company could extract the highest value. Market A would return a value of $46 ($50 − $4) and market B would return a value of $47 ($48 − $1). Therefore, market B is the most advantageous market and the asset will be reported at $48. Transaction costs are relevant to determining the most advantageous market. They do not impact the recognition of the asset at fair value. Fair value is the selling price of the asset in the most advantageous market.

Jensen performed legal services to assist Balm Co. in accomplishing its initial organization. Jensen accepted 1,000 shares of $5 par common stock in Balm as payment for his services. The Balm shares were not yet publicly traded, but they had a book value of $4 per share. Jensen provided 48 hours of service, which is normally billed at $125 per hour. By what amount should the common stock account increase? A. $1,000 B. $6,000 C. $5,000 D. $4,000

C. $5,000 In general, accounting for nonmonetary exchanges is based on fair value. The asset received should be recorded at the fair value of the asset surrendered OR the fair value of the asset received, whichever is more clearly evident. Since Balm's stock is not yet trading, the fair value of the services provided (48 hours × $125/hour = $6,000) is more readily determinable. The common stock account will increase by its par value of $5,000; the remaining $1,000 will be an increase to Additional Paid-in Capital.

On January 1, Fonk City approved the following general fund resources for the new fiscal period: Property taxes $5,000,000Licenses and permits 400,000Intergovernmental revenues 150,000Transfers in from other funds 350,000 What amount should Fonk record as estimated revenues for the new fiscal year? A. $5,750,000 B. $5,900,000 C. $5,550,000 D. $5,400,000

C. $5,550,000 Property taxes ($5,000,000), licenses and permits ($400,000), and intergovernmental revenues ($150,000) will all be represented in the budget as "estimated revenues" ($5,550,000). The expected transfers in ($350,000) will be represented by "estimated other financing sources" in the budget.

At December 31, Year 1, Gasp Co.'s allowance for uncollectible accounts had a credit balance of $30,000. During Year 2, Gasp wrote off uncollectible accounts of $45,000. At December 31, Year 2, an aging of the accounts receivable indicated that $50,000 of the December 31, Year 2, receivables may be uncollectible. What amount of allowance for uncollectible accounts should Gasp report in its December 31, Year 2, balance sheet? A. $25,000 B. $35,000 C. $50,000 D. $20,000

C. $50,000

Puff Co. acquired 40% of Straw, Inc.'s, voting common stock on January 2, 20X1, for $400,000. The carrying amount of Straw's net assets at the purchase date totaled $900,000. Fair values equaled carrying amounts for all items except equipment, for which fair values exceeded carrying amounts by $100,000. The equipment has a 5-year life. During 20X1, Straw reported net income of $150,000. What amount of income from this investment should Puff report in its 20X1 income statement? A. $40,000 B. $56,000 C. $52,000 D. $60,000

C. $52,000 FASB ASC 323-10-05-5 requires the use of the equity method when a company acquires 20% or more of the outstanding stock of another company. Under the equity method, Puff should report 40% of the $150,000 income of Straw, or $60,000. Because Straw's equipment has a fair market value exceeding its carrying value, Puff should amortize the difference over the equipment's 5-year life. Puff should record 40% of $100,000 ($40,000) as equipment subject to amortization (depreciation). Straight-line amortization of $40,000 over five years yields an expense of $8,000. Puff has income of $60,000 less $8,000, or $52,000 for 20X1

Hilltop Co.'s monthly bank statement shows a balance of $54,200. Reconciliation of the statement with company books reveals the following information: Bank service charge: $10 Insufficient funds check: $650 Checks outstanding: $1,500 Deposits in transit: $350 Check deposited by Hilltop and cleared by the bank for $125, but improperly recorded by Hilltop as $152 What is the net cash balance after the reconciliation? A. $52,363 B. $53,023 C. $53,050 D. $53,077

C. $53,050 We have no way of finding the unadjusted ending month balance per books in the cash account, but we can find the corrected ending cash balance from the information given by starting with the ending bank balance. The ending bank balance is $54,200 and when we add the deposits in transit and subtract outstanding checks from this balance, we should have the corrected ending balance. Thus, starting with $54,200 and adding the $350 deposit in transit, and then subtracting the $1,500 checks outstanding, we get $53,050. The other items would be adjustments to unadjusted ending cash balance, which we do not have, the check error being the company's error, not the bank's error.

Yellow Co. spent $12,000,000 during the current year developing its new software package. Of this amount, $4,000,000 was spent before it was at the application development stage and the package was only to be used internally. The package was completed during the year and is expected to have a 4-year useful life. Yellow has a policy of taking a full-year's amortization in the first year. After the development stage, $50,000 was spent on training employees to use the program. What amount should Yellow report as an expense for the current year? A. $2,000,000 B. $6,012,500 C. $6,050,000 D. $1,600,000

C. $6,050,000 Costs incurred to develop software for internal use are capitalized after the application development stage is reached (in accordance with FASB ASC 350-40-35-4). The costs are amortized over the benefited period—four years in this case. Costs incurred prior to the application development stage are expensed, as are training costs incurred after the development stage. Therefore, the amount expensed is: Pre-development stage costs $4,000,000Amortization of capitalized costs:$8,000,000 / 4 years 2,000,000Training costs 50,000Total expenses $6,050,000

For the fall semester of 20X2, a state-operated college sent tuition notices to enrolled students that totaled $6,720,000 for tuition and fees. Selected students were granted tuition waivers of $420,000 and fee waivers of $52,400. The net amount of tuition and fees realized was $6,184,200, because estimated uncollectible tuition and fees amounted to $63,400. What amount should the college report for the period as operating revenues from tuition and fees? A. $6,247,600 B. $6,720,000 C. $6,184,200 D. $6,300,000

C. $6,184,200 Tuition and fee revenues should be reported net of tuition discounts and allowances with the discount or allowance amount parenthetically disclosed on the face of the statement. Tuition waivers are a kind of discount that lowers net tuition received from specific students. Uncollectible accounts are recognized with the use of an allowance for government college and university accounting. Operating revenue from tuition and fees is $6,184,200 ($6,720,000 - $420,000 - $52,400 - $63,400).

On July 1, 20X1, Casa Development Co. purchased a tract of land for $1,200,000. Casa incurred additional costs of $300,000 during the remainder of 20X1 in preparing the land for sale. The tract was subdivided into residential lots as follows: Lot Number Sales PriceClass of Lots Per Lot A 100 $24,000B 100 16,000C 200 10,000 Using the relative sales value method, what amount of costs should be allocated to the Class A lots? A. $300,000 B. $375,000 C. $600,000 D. $720,000

C. $600,000 In a basket purchase, one price is paid for many separable assets. The price is allocated among these assets in accordance with their relative fair values.

Azim Services, a nongovernmental not-for-profit organization, received dues of $100 from its members. Azim provided its members with a newsletter that had a $25 value. All other services were valued at $10 per member. What is the amount of contribution made to Azim by each member? A. $100 B. $10 C. $65 D. $25

C. $65 Contribution revenues for not-for-profit organizations must be distinguished from exchange transactions. In this case, the members are exchanging $25 of services for a newsletter and $10 for other services. Contribution revenue would then be $65 ($100 − $25 − $10).

On April 1, Aloe, Inc., factored $80,000 of its accounts receivable without recourse. The factor retained 10% of the accounts receivable as an allowance for sales returns and charged a 5% commission on the gross amount of the factored receivables. What amount of cash did Aloe receive from the factored receivables? A. $72,000 B. $68,400 C. $68,000 D. $76,000

C. $68,000 Factoring a receivable without recourse is, in effect, a sale of the receivable. 80k - 8k- 4k=68k

On September 1, 20X1, Cano & Co., a U.S. corporation, sold merchandise to a foreign firm for 250,000 francs. Terms of the sale require payment in francs on February 1, 20X2. On September 1, 20X1, the spot exchange rate was $.20 per franc. At December 31, 20X1, Cano's year-end, the spot rate was $.19, but the rate increased to $.22 by February 1, 20X2, when payment was received. How much should Cano report as foreign exchange gain or loss in its 20X2 income statement? A. $5,000 gain B. $0 C. $7,500 gain D. $2,500 loss

C. $7,500 gain Gains and losses arising from foreign currency transactions (transactions denominated in a currency other than the functional currency) arise when monetary assets and liabilities (such as the account receivable in this question) are denominated in a currency other than the functional currency and the exchange rate between the currencies changes. These changes increase or decrease expected functional currency cash flows and shall be included in determining net income for the period in which the exchange rate changes (i.e., the receivable is revaluated at the spot rate at year-end and again when the payment is received). Thus, Cano would revalue the receivable at $.19/franc and report a $2,500 ($250,000 × ($.19 − $.20)) foreign transaction loss at December 31, 20X1. Then, in its 20X2 income statement, Cano again revalues the receivable (from $.19 to $.22) and reports a $7,500 gain ($250,000 × ($.22 − $.19)).

Vale City legally adopts a cash-basis budget. What basis should be used in Vale's combined statement of revenues, expenditures, and changes in fund balances—budget and actual? A. Modified cash B. Accrual C. Cash D. Modified accrual

C. Cash Budgetary comparisons must be presented on the same basis of accounting as the adopted budget. For this question, the preparation of the budget is on the cash basis, so budgetary comparisons will be prepared on the cash basis. Budgetary comparison schedules should be accompanied by information (either in a separate schedule or in notes to required supplementary information (RSI)) that reconciles budgetary information to GAAP information.

On October 15, 20X7, Felis Limited, a U.S. corporation, sold merchandise to a Belgian firm for 130,000 euros. Terms of the sale require payment in euros on February 16, 20X8. On October 15, 20X7, the spot exchange rate was $1.17 per euro. At December 31, 20X7, Felis' year-end, the spot rate was $1.15, but the rate increased to $1.21 by February 16, 20X8, when payment was received. How much should Felis report as foreign exchange gain or loss in its 20X8 income statement? A. $7,800 loss B. $2,600 loss C. $7,800 gain D. $2,600 gain

C. $7,800 gain Gains and losses arising from foreign currency transactions (transactions denominated in a currency other than the functional currency) arise when monetary assets and liabilities (such as the account receivable in this question) are denominated in a currency other than the functional currency and the exchange rate between the currencies changes. These changes increase or decrease expected functional currency cash flows and shall be included in determining net income for the period in which the exchange rate changes (i.e., the receivable is revalued at the spot rate at year-end and again when the payment is received). Thus, Felis would revalue the receivable at $1.15/euro and report a $2,600 (€130,000 × ($1.17 - $1.15)) foreign transaction loss at December 31, 20X7. Then, in its 20X8 income statement, Felis again revalues the receivable (from $1.15 to $1.21) and reports a $7,800 gain (€130,000 × ($1.21 - $1.15)).

Hoyt Corp.'s current balance sheet reports the following stockholders' equity: 5% cumulative preferred stock, par value $100 per share;2,500 shares issued and outstanding $250,000 Common stock, par value $3.50 per share;100,000 shares issued and outstanding 350,000 Additional paid-in capital in excess of par value of common stock 125,000 Retained earnings 300,000 Dividends in arrears on the preferred stock amount to $25,000. If Hoyt were to be liquidated, the preferred stockholders would receive par value plus a premium of $50,000. The book value per share of common stock is: A. $7.25. B. $7.75. C. $7.00. D. $7.50.

C. $7.00. The book value of common stock is $7.00/share, calculated as follows: Book value/sh = Total Equity - Liquidation value to preferredNo. shares common stock outstanding= $1,025,000(a) - $325,000(b) / 100,000 shares= $700,000 100,000 shares= $7.00/share a Total SHE = $250,000 + $350,000 + $125,000 + $300,000 = $1,025,000 b Preferred value = Par + Premiums + Dividends in arrears = $250,000 + $50,000 + $25,000 = $325,000

At the acquisition date, July 2, 20X1, reporting unit R has a fair value of $370,000 and a carrying amount (including goodwill of $100,000) of $470,000. On December 31, 20X1, the fair value of the assets and liabilities assigned to reporting unit R is $330,000, and the fair value of R is $400,000. The goodwill impairment loss reportable is: A. $30,000. B. $100,000. C. $70,000. D. $0.

C. $70,000. Impairment of goodwill is a one-step process: Compare: (a) Year-end fair value of reporting unit $400,000(b) Carrying amount, including goodwill 470,000Difference $ 70,000

Frame Co. has an 8% note receivable dated June 30, 20X0, in the original amount of $150,000. Payments of $50,000 in principal plus accrued interest are due annually on July 1, 20X1, 20X2, and 20X3. In its June 30, 20X2, balance sheet, what amount should Frame report as a current asset for interest on the note receivable? A. $0 B. $4,000 C. $8,000 D. $12,000

C. $8,000 150k-50k (paid) = 100k 100k * 8% = 8000 (the accrued amount)

Gold Co. purchased equipment from Marshall Co. on July 1. Gold paid Marshall $10,000 cash and signed a $100,000 noninterest-bearing note payable, due in three years. Gold recorded a $24,868 discount on notes payable related to this transaction. What is the acquired cost of the equipment on July 1? A. $75,132 B. $100,000 C. $85,132 D. $110,000

C. $85,132 The cost of equipment is the price paid to acquire it, or the value paid out (or liability taken on) in the purchase. In this case, the cost of the equipment is the $10,000 paid, plus the value of the note payable, which is the face value of $100,000, less the discount of $24,868. This gives a total cost of $10,000 + $100,000 - $24,868 = $85,132.

Hunt Co. purchased merchandise for 300,000 British pounds from a vendor in London on November 30, 20X1. Payment in British pounds was due on January 30, 20X2. The exchange rates to purchase one pound were as follows: 11/30/X1 12/31/X1 Spot-rate $1.65 $1.62 30-day rate 1.64 1.59 60-day rate 1.63 1.56 In its December 31, 20X1, income statement, what amount should Hunt report as foreign exchange gain? A. $6,000 B. $12,000 C. $9,000 D. $0

C. $9,000 FASB ASC 830-20 (Foreign Currency Transactions) provides that a gain or loss on a forward contract is computed by multiplying the foreign currency amount of the forward contract by the difference between the spot rate at the balance sheet date and the spot rate at the date of inception of the forward contract. Hunt's reported foreign exchange gain= 300,000 British pounds x ($1.65 - $1.62) per British pound= $9,000

The premium on a 3-year insurance policy expiring on December 31, Year 3, was paid in total on January 2, Year 1. If the company has a 6-month operating cycle, then on December 31, Year 1, the prepaid insurance reported as a current asset would be for: A. 6 months. B. 24 months. C. 12 months. D. 18 months.

C. 12 months. Current items cover a period which is the company operating cycle (6 months) or a year, whichever is longer, and a year is longer than six months. Thus, for this company, items covering a 12-month period going forward are current.

On January 2 of the current year, Peace Co. paid $310,000 to purchase 75% of the voting shares of Surge Co. Peace reported retained earnings of $80,000, and Surge reported contributed capital of $300,000 and retained earnings of $100,000. The purchase differential was attributed to depreciable assets with a remaining useful life of 10 years. Peace used the equity method in accounting for its investment in Surge. Surge reported net income of $20,000 and paid dividends of $8,000 during the current year. Peace reported income, exclusive of its income from Surge, of $30,000 and paid dividends of $15,000 during the current year. What amount will Peace report as dividends declared and paid in its current year's consolidated statement of retained earnings? A. 23,000 B. 8,000 C. 15,000 D. 21,000

C. 15,000 Only dividends paid to Peace shareholders will be reported as dividends paid. Dividends paid to Peace by Surge will be eliminated in consolidation. Dividends paid to shareholders other than Peace will be reported as an adjustment to the noncontrolling interest account.

When should a conditional pledge to a nongovernmental not-for-profit organization be recognized as revenue? A. When the cash is received B. At the beginning of the next fiscal period C. When the pledge conditions are met D. Immediately

C. When the pledge conditions are met

Nest Co. issued 100,000 shares of common stock. Of these, 5,000 were held as treasury stock on December 31, 20X1. During 20X2, transactions involving Nest's common stock were as follows: May 3: 1,000 shares of treasury stock were sold. August 6: 10,000 shares of previously unissued stock were sold. November 18: a 2-for-1 stock split took effect. Laws in Nest's state of incorporation protect treasury stock from dilution. On December 31, 20X2, how many shares on Nest's common stock were issued and outstanding? A. 220,000 shares issued and 216,000 shares outstanding B. 222,000 shares issued and 218,000 shares outstanding C. 220,000 shares issued and 212,000 shares outstanding D. 222,000 shares issued and 214,000 shares outstanding

C. 220,000 shares issued and 212,000 shares outstanding Shares issued include: Original shares issued 100,000Sale of shares on August 6 10,000Additional shares from stock split 110,000Total 220,000 Shares outstanding: Change OutstandingOriginal issue 100,000 100,000Treasury shares held in 20X1 (5,000) 95,000Sales of Treasury shares in 20X2 1,000 96,000Sale of unissued shares 10,000 106,000Stock split 106,000 212,000 The difference of 8,000 shares is the 4,000 treasury shares plus the additional 4,000 issued treasury shares in the stock split.

On December 31, 20X1, Bit Co. had capitalized costs for a new computer software product with an economic life of 5 years. Sales for 20X2 were 30% of expected total sales of the software. On December 31, 20X2, the software had a net realizable value equal to 90% of the capitalized cost. What percentage of the original capitalized cost should be reported as the net amount on Bit's December 31, 20X2, balance sheet? A. 80% B. 90% C. 70% D. 72%

C. 70% FASB ASC 985-20-35-1 provides: "The annual amortization shall be the greater of the amount computed using (a) The ratio that current gross revenues for a product bear to the total of current and anticipated future gross revenues for that product, or (b) The straight-line method over the remaining estimated economic life of the product including the period being reported on." Ratio of current to total revenues (given) = 30% Straight-line rate = 1/5 = 20% The greater of these, 30%, would be used in computing 20X2 amortization, leaving a net amount of 70% (100% − 30%) to be shown on Bit's December 31, 20X2, balance sheet.

On December 31, 20X4, Jefferson Corporation had capitalized costs for a new computer software product with an economic life of 4 years. Sales for 20X5 were 27% of expected total sales of the software. On December 31, 20X5, the software had a net realizable value equal to 80% of the capitalized cost. What percentage of the original capitalized cost should be reported as the net amount on Jefferson's December 31, 20X5, balance sheet? A. 75% B. 80% C. 73% D. 50%

C. 73% FASB ASC 985-20-35-1 provides: "The annual amortization shall be the greater of the amount computed using (a) The ratio that current gross revenues for a product bear to the total of current and anticipated future gross revenues for that product, or (b) The straight-line method over the remaining estimated economic life of the product including the period being reported on." Ratio of current to total revenues (given) = 27% Straight-line rate = 1/4 = 25% The greater of these, 27%, would be used in computing 20X5 amortization, leaving a net amount of 73% (100% − 27%) to be shown on Jefferson's December 31, 20X5, balance sheet.

Which of the following would represent an option with a material right offered to a customer? A. An offer to accept a returned product for a full refund that is offered to all customers B. A coupon offering 20% off the next purchase that is given to all customers C. A coupon offering 50% off of a customer's next purchase if the customer spends over a specified amount, with customers not spending over that amount getting no such offer D. An online advertisement that highlights an upcoming holiday sale

C. A coupon offering 50% off of a customer's next purchase if the customer spends over a specified amount, with customers not spending over that amount getting no such offer Certain options that grant a material right to a customer are considered as separate performance obligations with a portion of the transaction price allocated to them and recognized when the option is exercised. If an option provides a material right to the customer that it would not have received without entering into the contract, it should be considered as a separate performance obligation. For example, offering a 50% discount on future purchases over and above one offered to all customers would be considered a material right. Offering a free item when a customer purchases other products would be another example of an option granting a material right. A coupon offering 20% off the next purchase that is given to all customers, an offer to accept a returned product for a full refund that is offered to all customers, and an online advertisement that highlights an upcoming holiday sale are not examples of an option with a material right.

Which of the following statements regarding reporting for discontinued operations is incorrect? A. In order for a sale to qualify as a discontinued operation, it needs to represent a strategic shift for the entity. B. In order to qualify to be reported as a discontinued operation, the portion of the business being sold must have separate and identifiable cash flows. C. A strategic shift is the sale of a product line that represents 25% or more of the entity's total revenues. D. The resulting gain or loss from discontinued operations should be reported net of applicable taxes.

C. A strategic shift is the sale of a product line that represents 25% or more of the entity's total revenues. In order for the sale of a portion of an entity to qualify for discontinued operations, it must represent a strategic shift for the entity, meaning that the sale will have a significant effect on the entity's operations and financial results. A strategic shift is the sale of a product line that represents 15% (not 25%) or more of the entity's total revenues. Discontinued operations are reported after income from continuing operations and therefore need to be reported net of tax. The cash flows of the portion of the business to be sold must be clearly distinguishable from the other operations of the portion to be sold.

The provisions of FASB ASC 718-10-25-2, "Recognition Principle for Share-Based Payment Transactions," apply to all of the following transactions except those related to: A. common stock granted to employees. B. transfer of other equity instruments to employees. C. employee stock ownership plan instruments. D. stock options awarded to employees.

C. employee stock ownership plan instruments.

How may an entity elect to treat sales and other similar taxes which it collects from customers from its revenue transactions? A. An entity may elect to include sales taxes in its state of domicile as part of the transaction price but should record other such taxes on a net basis. B. An entity, on a transaction-by-transaction basis, may elect to record such taxes as either gross or net. C. An entity may elect, as an accounting policy, to exclude all amounts collected from customers for sales and other similar taxes from the transaction price. D. An entity must present revenue gross of all such taxes received, with the corresponding tax recorded as part of cost of goods sold.

C. An entity may elect, as an accounting policy, to exclude all amounts collected from customers for sales and other similar taxes from the transaction price.

A publicly traded corporation reported a $10,000 deduction in its current-year tax return for an item it expects to be disallowed. The tax rate is 40%. How should the corporation report this tax position in the financial statements? A. As a temporary difference disclosed in the notes to the financial statements that is not recognized B. As a $10,000 deferred tax asset C. As a $4,000 income tax expense and a $4,000 liability for an unrecognized tax benefit D. As a $4,000 deferred tax asset and a $4,000 income tax benefit

C. As a $4,000 income tax expense and a $4,000 liability for an unrecognized tax benefit If the company expects the item to be disallowed, it believes it does not meet the "more likely than not" threshold for recognizing a potential tax benefit. Following accounting conservatism, the company must treat the deduction as if it was disallowed from the start by recognizing tax expense and the associated liability. The company would have $4,000 ($10,000 × 40%) more in tax expense and liability because it expects the deduction to be disallowed.

During 20X1, Leader Corp. sued Cape Co. for patent infringement. On December 31, 20X1, Leader was awarded a $500,000 favorable judgment in the suit. On that date, Cape offered to settle out of court for $300,000 and not appeal the judgment. In February 20X2, after the issuance of its 20X1 financial statements, Leader agreed to the out-of-court settlement and received a certified check for $300,000. In its 20X1 financial statements, how should Leader have reported these events? A. As a receivable and deferred credit of $300,000 B. It should not be reported in the financial statements. C. As a disclosure in the notes to the financial statements only D. As a gain of $300,000

C. As a disclosure in the notes to the financial statements only As of December 31, 20X1, the gain from the infringement action (regardless of whether the amount is $500,000 or $300,000) is a gain contingency because the settlement offer could be revoked or rejected and an appeal could still occur. Leader should report this situation as a disclosure in the notes to Leader's financial statements as provided for in FASB ASC 450-30-50-1. This treatment is confirmed in FASB ASC 855-10-25-3. A contingency must be recognized under the following conditions: "If the events that gave rise to litigation had taken place before the balance sheet date and that litigation is settled, after the balance sheet date but before the financial statements are issued or are available to be issued, for an amount different from the liability recorded in the accounts, then the settlement amount should be considered in estimating the amount of liability recognized in the financial statements at the balance sheet date." (FASB ASC 855-10-55-1) Since the settlement occurred after the financial statements were issued, only disclosure is required

Payne Co. prepares its statement of cash flows using the indirect method. Payne's unamortized bond discount account decreased by $25,000 during the year. How should Payne report the change in unamortized bond discount in its statement of cash flows? A. As a financing cash inflow B. As a financing cash outflow C. As an addition to net income in the operating activities section D. As a subtraction from net income in the operating activities section

C. As an addition to net income in the operating activities section The amortization of a bond discount is the difference between cash interest and interest expense. Cash paid for interest is reported in operating activities. Amortization of a discount on bonds payable results in interest expense greater than cash interest. Because more expense has been deducted in computing income than the amount of cash paid for interest, the difference (captured in the change in the bond discount account) must be added to income to reconcile to the cash provided or used for operating activities.

Cliff Hospital, a private not-for-profit institution, has an endowment fund, the income from which is required to be used for library acquisitions. How should the income be reported in the statement of activities if there have not yet been any library acquisitions using such income? A. As an increase in permanently restricted net assets B. As an increase in deferred revenues C. As an increase in net assets with donor restrictions D. As an increase in net assets without donor restrictions

C. As an increase in net assets with donor restrictions

In a compensatory stock option plan for which the grant, vesting, and exercise dates are all different, the additional paid-in capital—stock options account should be reduced at the: A. vesting date. B. beginning of the service period. C. exercise date. D. date of grant.

C. exercise date.

On April 1, 20X1, Ivy began operating a service proprietorship with an initial cash investment of $1,000. The proprietorship provided $3,200 of services in April and received full payment in May. The proprietorship incurred expenses of $1,500 in April which were paid in June. During May, Ivy drew $500 against her capital account. What was the proprietorship's income for the two months ending May 31, 20X1, under the following methods of accounting? A. Cash basis: $1,200; Accrual basis: $1,200 B. Cash basis: $2,700; Accrual basis: $1,200 C. Cash basis: $3,200; Accrual basis: $1,700 D. Cash basis: $1,700; Accrual basis: $1,700

C. Cash basis: $3,200; Accrual basis: $1,700

On April 30, 20X1, Algee, Belger, and Ceda formed a partnership by combining their separate business proprietorships. Algee contributed cash of $50,000. Belger contributed property with a $36,000 carrying amount, a $40,000 original cost, and $80,000 fair value. The partnership accepted responsibility for the $35,000 mortgage attached to the property. Ceda contributed equipment with a $30,000 carrying amount, a $75,000 original cost, and $55,000 fair value. The partnership agreement specifies that profits and losses are to be shared equally but is silent regarding capital contributions. Which partner has the largest April 30, 20X1, capital account balance? A. All capital account balances are equal. B. Algee C. Ceda D. Belger

C. Ceda Initial partnership capital account balances are based on the net fair value invested by the partner into the partnership. Partner Fair Value - Liability = Capital of Contribution Assumed Balance Algee $50,000 $50,000Belger 80,000 - 35,000 = 45,000Ceda 55,000 55,000 Ceda's net capital contribution, $55,000, is the largest.

Which of the following best describes the general disclosure principle? A. Disclosure in the notes to the financial statements is needed only when management feels it is necessary to supplement information presented on the face of the financial statements. B. Disclosure in the notes to the financial statements is needed only when the meaningful information is not provided elsewhere therein. C. Certain information may be presented either on the face of the financial statements or in the notes to the financial statements. Disclosure in the notes to the financial statements is needed only when the information required to be disclosed is not displayed on the face of the financial statements. D. Certain information may be presented either on the face of the financial statements or in the notes to the financial statements. Disclosure in the notes to the financial statements is needed only when the information required to be disclosed is not displayed on the face of the financial statements and the MD&A.

C. Certain information may be presented either on the face of the financial statements or in the notes to the financial statements. Disclosure in the notes to the financial statements is needed only when the information required to be disclosed is not displayed on the face of the financial statements.

Imez Industries received a $30,000 prepayment from Aztec Associates for the sale of new equipment. Imez will bill Aztec an additional $70,000 upon delivery of the equipment. Upon receipt of the $30,000 prepayment, how much should Imez recognize for a contract asset, a contract liability, and accounts receivable (A/R)? A. Contract asset: $30,000; contract liability: $0, A/R: $0 B. Contract asset: $30,000; contract liability: $0, A/R: $70,000 C. Contract asset: $0; contract liability: $30,000, A/R: $0 D. Contract asset: $0; contract liability: $30,000, A/R: $70,000

C. Contract asset: $0; contract liability: $30,000, A/R: $0 A contract asset is an entity's right to consideration in exchange for goods or services that the entity has transferred to a customer. If the contract asset represents an unconditional right to receive consideration (no future performance obligation), the asset should be presented as a receivable separately from contract assets. A contract liability is an entity's obligation to transfer goods or services to a customer for which the entity has received consideration (or an amount of consideration is due) from the customer. The $30,000 is a prepayment and is a contract liability. The $70,000 owed upon delivery is neither a contract asset nor an account receivable, because Imez has not fulfilled its performance obligation and therefore has no right to receive payment.

How would a municipality that uses modified accrual and encumbrance accounting record the condition of an excess of estimated inflows over estimated outflows? A. Credit appropriations control B. Debit appropriations control C. Credit budgetary fund balance D. Credit other financing sources

C. Credit budgetary fund balance Encumbrance accounting is a feature of the accounting practices used by municipalities for governmental funds. Accounting for governmental funds is done on the modified accrual basis. Annual budgets of estimated inflows and estimated outflows are prepared for the general fund and most other governmental funds. The budget is so important to a city's financial operations for a fiscal year that "budgetary accounting," a method of integrating budgeted amounts into a city's accounting system, is used. Adoption of the operating budget is recorded by debiting appropriate budgetary accounts for estimated inflows ("estimated revenues") and crediting the appropriate budgetary accounts for estimated and approved outflows ("appropriations"). "Budgetary fund balance" is debited or credited for the difference. In this problem, since estimated inflows exceed estimated outflows, budgetary fund balance is credited for the difference. (Note: This budgetary entry is reversed at the end of the period as the first closing entry.)

How would a municipality that uses modified accrual and encumbrance accounting record the transaction of short-term financing received from a bank, secured by the city's taxing power? A. Debit deferred revenues B. Credit expenditures control C. Credit tax anticipation notes payable D. Credit other financing sources

C. Credit tax anticipation notes payable In this problem, a city obtained short-term bank financing secured by the city's taxing power. This is interpreted to mean that (1) the General Fund is involved, since some or all of a city's tax revenues are normally recorded in that fund, and (2) future tax proceeds will be used to repay the loan. The journal entry to record the transaction will include a debit to cash, of course. In this problem, the loan is short-term and there is no information to suggest that the loan will be refinanced with long-term borrowing. Therefore, the loan represents establishment of a fund liability (not an increase in "other financing sources" as is the case when the General Fund accounts for the proceeds from long-term borrowing). The credit side of the entry then must be to some liability account. The only available response in this problem that increases a liability is "credit tax anticipation notes payable." Moreover, in view of the information given, this is the ideal response.

A company has 10,000 shares of common stock issued and 2,000 shares of treasury stock. The par value of the stock is $10 per share. On January 1, year 1, the company declared a 5% dividend to be paid in cash on June 30, year 1. What journal entry should the company record on the declaration date? A. Debit retained earnings for $5,000 and credit dividends payable for $5,000. B. Debit dividends expense for $5,000 and credit dividends payable for $5,000. C. Debit retained earnings for $4,000 and credit dividends payable for $4,000. D. Debit dividends expense for $4,000 and credit dividends payable for $4,000.

C. Debit retained earnings for $4,000 and credit dividends payable for $4,000. The company only has 8,000 shares of stock outstanding (10,000 shares issued - 2,000 shares in treasury stock) because treasury stock is considered issued but not outstanding. When the company pays the 5% dividend, it is paid only to shares outstanding. Since the company owns 2,000 of the 10,000 shares, it wouldn't make sense to pay a dividend to itself. The value of the dividend is $4,000 (8,000 shares × $10 par × 5%).

Which of the following is false regarding the reporting of capital assets at the entity-wide perspective? A. General capital assets, including infrastructure capital assets, should be reported at known or estimated historical cost less any accumulated depreciation. B. Depreciation of general capital assets, including infrastructure capital assets, should be reported by function. C. Depreciation on general capital assets, including infrastructure, is always required. D. Depreciation is not taken on infrastructure assets accounted for using the modified approach.

C. Depreciation on general capital assets, including infrastructure, is always required. Under GASB 1400.105, certain infrastructure capital assets are not required to be depreciated under the modified approach. The other three statements are true.

Which one of the following statements is correct regarding comparability of governmental financial reports? A. Similarly designated governments perform the same functions. B. Selection of different alternatives in accounting procedures or practices account for the differences between financial reports. C. Differences between financial reports should be due to substantive differences in underlying transactions or the governmental structure. D. Comparability is not relevant in governmental financial reporting.

C. Differences between financial reports should be due to substantive differences in underlying transactions or the governmental structure. GASB Concepts Statement 1 states at paragraph 68: "Comparability implies that differences between financial reports should be due to substantive differences in the underlying transactions or the governmental structure rather than due to selection of different alternatives in accounting procedures or practices."

Town, Inc., is preparing its financial statements for the year ended December 31, 20X1. On January 5, 20X2, prior to the issuance of the financial statements, Town redeemed its outstanding bonds and issued new bonds with a lower rate of interest. The reacquisition price was in excess of the carrying amount of the bonds. What is the appropriate reporting requirement? A. Both accrual and disclosure B. Neither accrual nor disclosure C. Disclosure only D. Accrual only

C. Disclosure only Information which becomes known after the balance sheet date, but before the financial statements are issued, should be disclosed to keep the financial statements from being misleading. The gains or losses associated with the redemption and issuance of the bonds would be reported and disclosed in the 20X2 financial statements.

One of the elements of a financial statement is comprehensive income. Comprehensive income excludes changes in equity resulting from which of the following? A. Loss from discontinued operations B. Prior-period error correction C. Dividends paid to stockholders D. Unrealized loss on investments in equity securities

C. Dividends paid to stockholders Comprehensive income per SFAC 6, Elements in Financial Statements, encompasses all changes in equity of a business resulting from transactions with nonowners. Specifically: "It includes all changes in equity during a period except those resulting from investments by owners and distributions to owners." Based on this, dividends paid to stockholders would not be included in computation of comprehensive income.

On March 21, Year 2, a company with a calendar year-end issued its Year 1 financial statements. On February 28, Year 2, the company's only manufacturing plant was severely damaged by a storm and had to be shut down. Total property losses were $10 million and determined to be material. The amount of business disruption losses is unknown. How should the impact of the storm be reflected in the company's Year 1 financial statements? A. Accrue and disclose the property loss with no accrual or disclosure of the business disruption loss B. Accrue and disclose the property loss and additional business disruption losses in the financial statements C. Do not accrue the property loss or the business disruption loss, but disclose them in the notes to the financial statements D. Provide no information related to the storm losses in the financial statements until losses and expenses become fully known

C. Do not accrue the property loss or the business disruption loss, but disclose them in the notes to the financial statements An entity must not recognize events that arose after the balance sheet date but before the financial statements are issued. One of the events specifically mentioned in FASB ASC 855-10-55-2 is the loss of plant as a result of fire or other natural disaster. However, this event must be disclosed in the notes to the financial statements.

Which of the following statements concerning the acquisition of assets is false? A. Assets donated by entities other than governmental units are included in revenue in the period of receipt. B. If several dissimilar assets are purchased for a lump sum, the total amount paid should be allocated to each individual asset on the basis of its relative fair value. C. Donated assets should be recorded at book value along with any incidental costs incurred. D. When an asset is received from a governmental entity, no income is recognized, and the offsetting credit is to an owners' equity account, "Additional Paid-In Capital: Donated Assets."

C. Donated assets should be recorded at book value along with any incidental costs incurred. According to FASB ASC 845-10-30-1, donated assets should be valued at fair value, not book value, so "donated assets should be recorded at book value along with any incidental costs incurred" is false. The other three answer choices are acceptable ways to account for donated assets.

The City of Windemere decided to construct several large windmills to generate electrical power. The construction was financed through a general residential property tax levy for the next 10 years. Utility revenues are intended to offset all expenses associated with the windmills. The land for the windmills was donated to the city by a local farmer. The land from the farmer should be reported in which fund type? A. Capital projects B. Permanent C. Enterprise D. Special revenue

C. Enterprise

Which of the following types of events must be recognized in the financial statements? A. Both events that provide evidence about conditions that did not exist at the date of the balance sheet and events or transactions that provide additional evidence about conditions that existed at the date of the balance sheet, including the estimates inherent in the process of preparing financial statements B. Neither events that provide evidence about conditions that did not exist at the date of the balance sheet but arose after that date nor events or transactions that provide additional evidence about conditions that existed at the date of the balance sheet, including the estimates inherent in the process of preparing financial statements C. Events or transactions that provide additional evidence about conditions that existed at the date of the balance sheet, including the estimates inherent in the process of preparing financial statements D. Events that provide evidence about conditions that did not exist at the date of the balance sheet but arose after that date

C. Events or transactions that provide additional evidence about conditions that existed at the date of the balance sheet, including the estimates inherent in the process of preparing financial statements Subsequent events are events or transactions that occur after the balance sheet date but before financial statements are issued or are available to be issued. Under FASB ASC 855-10, there are two types of subsequent events: The first type consists of events or transactions that provide additional evidence about conditions that existed at the date of the balance sheet, including the estimates inherent in the process of preparing financial statements (that is, recognized subsequent events). The second type consists of events that provide evidence about conditions that did not exist at the date of the balance sheet but arose after that date (that is, nonrecognized subsequent events).

An entity having which of the following characteristics may not be a governmental organization? A. Power to enact and enforce a tax levy B. Body corporate and politic C. Exempt from federal taxation D. Body corporate and politic or exempt from federal taxation

C. Exempt from federal taxation Entities having status as bodies corporate and politic and entities with the power to enact and enforce a tax levy are each considered governmental organizations (joint FASB/GASB definition of governmental organizations included in several AICPA Audit and Accounting Guides, including State and Local Governments, paragraph 1.01). However, many nongovernmental organizations are exempt from federal taxation.

At what value should a nongovernmental not-for-profit organization record shares of stock when received? A. Fair value at end-of-year B. Average of donor's basis and fair value on the date of donation C. Fair value on the date of donation D. Donor's basis

C. Fair value on the date of donation

Which of the following characteristics means that information is reasonably free from error and bias? A. Consistency B. Predictive value C. Faithful representation D. Relevance

C. Faithful representation

For governmental fund types, which of the following does not identify the primary characteristics of the structure? A. The representative form of government and the separation of powers B. The relationship of taxpayers to services received C. Flows and balances of financial resources D. The federal system of government and the prevalence of intergovernmental revenues

C. Flows and balances of financial resources According to the summary of GASB Concept Statement 1, the primary characteristics of governmental structure are as follows: "(1) The representative form of government and the separation of powers "(2) The federal system of government and the prevalence of intergovernmental revenues "(3) The relationship of taxpayers to services received "These are the primary characteristics that affect financial reporting of governmental-type activities and must be considered in establishing financial reporting objectives."

A U.S. public company with a worldwide public float of $800 million at the end of the second quarter of the fiscal year is required to file its annual report with the U.S. SEC on: A. Form 10-Q within 40 days after the end of the reporting period. B. Form 10-Q within 45 days after the end of the reporting period. C. Form 10-K within 60 days after the end of the reporting period. D. Form 10-K within 75 days after the end of the reporting period.

C. Form 10-K within 60 days after the end of the reporting period.

A company should recognize goodwill in its balance sheet at which of the following points? A. The company expects a future benefit from the creation of goodwill. B. The fair market value of the company's assets exceeds the book value of the company's assets. C. Goodwill has been created in the purchase of a business. D. Costs have been incurred in the development of goodwill.

C. Goodwill has been created in the purchase of a business.

Land and other real estate held as investments by endowments in a government's permanent fund should be reported at: A. fair value less costs of disposal. B. the lower of cost and net realizable value. C. fair value. D. historical cost.

C. fair value. Land and other real estate held as investments by endowments in a government permanent fund should be reported at fair value, not historical cost or the lower of cost and NRV (net realizable value). There is no requirement to deduct the cost of disposal from fair value.

New Town's review of payroll records indicates that employees providing governmental services have accrued $250,000 of vacation pay and employees of the proprietary funds have accrued $100,000 of vacation pay. It is anticipated that 5% of the accrued vacation pay will be claimed by employees within the first 60 days of 20X1. How would the vacation pay liability be recognized on the financial statements issued at December 31, 20X0? A. Governmental fund liability: $12,500; Proprietary fund liability: $5,000; Governmental activities liability: $250,000; Business-like activities liability: $100,000 B. Governmental fund liability: $12,500; Proprietary fund liability: $100,000; Governmental activities liability: $12,500; Business-like activities liability: $100,000 C. Governmental fund liability: $12,500; Proprietary fund liability: $100,000; Governmental activities liability: $250,000; Business-like activities liability: $100,000 D. Governmental fund liability: $250,000; Proprietary fund liability: $100,000; Governmental activities liability: $250,000; Business-like activities liability: $5,000

C. Governmental fund liability: $12,500; Proprietary fund liability: $100,000; Governmental activities liability: $250,000; Business-like activities liability: $100,000 As employees earn the right to claim vacation pay, a compensated absence, the liability is accrued and reported in full in the proprietary fund and government-wide financial statements (governmental activities and business-like activities). The portion reported in the government-wide financial statements as governmental activities is a general long-term liability. The governmental funds, using the modified accrual method, report only the portion of the liability expected to be claimed by employees in the first 60 days of the new fiscal year.

A company's foreign subsidiary operation maintains its financial statements in the local currency. The foreign operation's capital accounts would be translated to the functional currency of the reporting entity using which of the following rates? A. Current exchange rate at the balance sheet date B. Weighted-average exchange rate C. Historical exchange rate D. Functional exchange rate

C. Historical exchange rate When translating the capital accounts of a subsidiary, the historical exchange rate is used for the capital stock account and additional paid-in capital. This date cannot be earlier than the date the parent acquired the investment in the subsidiary.

A special-purpose unit of government such as a school board is considered a primary government if it meets which of the following criteria? It has a separately elected governing body. It is legally separate. It is fiscally independent of other state and local governments. A. I and II or III B. I or II and III C. I , II, and III D. I or II or III

C. I , II, and III

A statement of activities prepared by a nongovernmental not-for-profit organization is most similar to which of the following financial statements prepared by a for-profit entity? A. Balance sheet B. Statement of cash flows C. Income statement D. Statement of changes in stockholders' equity

C. Income statement A statement of activities most closely resembles an income statement. It covers a period of time (from date 1 to date 2) instead of being a point-in-time statement (at date 1) like a balance sheet. It is derived from changes in net assets. Increases in net assets are similar to revenues while decreases are similar to expenses, both of which can be found on the income statement.

Alpha Hospital, a large not-for-profit entity, has adopted an accounting policy that does not imply a time restriction on gifts of long-lived assets. Alpha received investments subject to the donor's requirement that investment income be used to pay for outpatient services. Indicate the manner in which this transaction affects Alpha's financial statements. A. Increase in unrestricted revenues, gains, and other support B. No required reportable event C. Increase in net assets with donor restrictions D. Increase in net assets without donor restrictions

C. Increase in net assets with donor restrictions

In 20X1, Lee Co. acquired Enfield, Inc., 10-year bonds at a premium as a long-term investment. On December 31, 20X2, Enfield's bonds were quoted at a small discount. Which of the following situations is the most likely cause of the decline in the bonds' market value? A. Enfield issued a stock dividend. B. Interest rates have declined since Lee purchased the bonds. C. Interest rates have increased since Lee purchased the bonds. D. Enfield is expected to call the bonds at a premium, which is less than Lee's carrying amount.

C. Interest rates have increased since Lee purchased the bonds.

Which of the following funds of a governmental unit records depreciation? A. Capital projects fund B. Debt service fund C. Internal service fund D. Special revenue fund

C. Internal service fund The internal service fund, a proprietary fund, is the only answer choice in which capital assets and related depreciation are recorded. The other answer choices are governmental funds in which neither capital assets nor depreciation expense are recorded.

On January 15, 20X1, Rico Co. declared its annual cash dividend on common stock for the year ended January 31, 20X1. The dividend was paid on February 9, 20X1, to stockholders of record as of January 28, 20X1. On what date should Rico decrease retained earnings by the amount of the dividend? A. January 31, 20X1 B. February 9, 20X1 C. January 15, 20X1 D. January 28, 20X1

C. January 15, 20X1 At the date of dividend declaration (January 15, 20X1), an entry to reduce retained earnings and record a dividend liability is made. No entry is necessary on the date of record (January 28, 20X1). On the payment date (February 9, 20X1), both cash and the dividend liability are reduced.

What measurement focus and basis of accounting should be used by a local government's private-purpose trust fund? A. Measurement focus: current financial resources; Basis of accounting: modified accrual B. Measurement focus: economic resources; Basis of accounting: modified accrual C. Measurement focus: economic resources; Basis of accounting: accrual D. Measurement focus: current financial resources; Basis of accounting: accrual

C. Measurement focus: economic resources; Basis of accounting: accrual The private-purpose trust fund is a type of fiduciary fund. Fiduciary funds follow the economic resources measurement focus and accrual basis of accounting. The current financial resources measurement focus and modified accrual basis of accounting are used for governmental funds, not fiduciary funds.

Which of the following statements is false with respect to fiduciary (trust) funds? A. Net position as opposed to fund balance is used. B. Revenue and expenses do not exist. C. Modified accrual accounting is applied. D. The fund statements are not consolidated in the government-wide statements.

C. Modified accrual accounting is applied. The correct answer is "modified accrual accounting is applied" because it is a false statement. The fiduciary funds utilize an economic measurement perspective and apply full accrual accounting.

Which of the following statements is false with respect to fiduciary (trust) funds? A. The fund statements are not consolidated in the government-wide statements. B. Revenue and expenses do not exist. C. Modified accrual accounting is applied. D. Net position as opposed to fund balance is used.

C. Modified accrual accounting is applied. The correct answer is "modified accrual accounting is applied" because it is a false statement. The fiduciary funds utilize an economic measurement perspective and apply full accrual accounting.

Which of the following fund types of a governmental unit have the income determination orientation? A. Both general funds and fiduciary funds B. Fiduciary funds C. Neither general funds nor fiduciary funds D. General funds

C. Neither general funds nor fiduciary funds The measurement focus of the general fund and other governmental-type funds is flow of current financial resources. This focus provides a measure of the change in current financial resources. The measurement focus of trust funds, including fiduciary funds, is economic resources. The purpose of measurement is change in trust net position reported as additions and deductions. Net income is not determined for governmental or fiduciary fund types.

At the end of Year 1, Lane Co. held equity securities that cost $86,000 and which had a year-end market value of $92,000. During Year 2, all of these securities were sold for $104,500. At the end of Year 2, Lane had acquired additional equity securities that cost $73,000 and which had a year-end market value of $71,000. What is the net impact of these stock activities on Lane's Year 2 income statement? A. Net gain of $16,500 B. Net loss of $2,000 C. Net gain of $10,500 D. Net gain of $18,500

C. Net gain of $10,500 Investments in equity securities are recognized on the balance sheet at fair value. Unrealized holding gains and losses are included in earnings. Realized gains and losses are also included in earnings. Gain on sale of the first securities was$104,500 − $92,000 $12,500Unrealized loss of the second securities was $73,000 − $71,000 (2,000)Net gain on income statement $10,500

A company entered into a loan with a lender for $100,000 and pledged $120,000 of the company's accounts receivable as collateral. The lender does not have the right to sell or repledge the accounts receivable. When the company receives the cash for the loan proceeds, what entry, if any, should be made to accounts receivable? A. Credit accounts receivable $20,000 B. Credit accounts receivable $100,000 C. No entry is made to accounts receivable. D. Credit accounts receivable $120,000

C. No entry is made to accounts receivable. Even though the company has offered $120,000 in accounts receivable, no entry is made because the lender does not have the rights to the assets. The rights to the assets remain with the company, meaning they control what to do with the accounts receivable. Note that the company would likely be required to include a note to the financial statements that some accounts receivable balances are pledged as collateral but the company retains their rights. If the lender had rights to the assets immediately or the company transferred receipt of the receivables to the lender, then an adjustment would be necessary.

Which of the following fiduciary funds typically entails significant disclosure within the notes to the financial statements due to its relationship to the general fund? A. Private-purpose trust fund B. Custodial fund C. Pension trust fund D. Investment trust fund

C. Pension trust fund "Pension trust fund" is the correct answer choice. While activities within any one of these fiduciary funds could become a note disclosure, the pension trust fund is more likely to require necessary disclosures. Calculating pension funding requirements involves making many assumptions about future events, which can significantly affect the determination of the liability. Actuarial assumptions are among the leading factors resulting in GASB to prescribe rather extensive note disclosure of the pension trust fund. Custodial fund, investment trust fund, and private-purpose trust fund are incorrect. While these items often warrant disclosure within the notes to the financial statements, they are less likely to be disclosed in comparison to the pension fund. Further, the scope of pension disclosures will be far more extensive.

Company J acquired all of the outstanding common stock of Company K in exchange for cash. The acquisition price exceeds the fair value of net assets acquired. How should Company J determine the amounts to be reported for the plant and equipment and long-term debt acquired from Company K? A. Plant and equipment, K's carrying amount; Long-term debt, K's carrying amount B. Plant and equipment, K's carrying amount; Long-term debt, fair value C. Plant and equipment, fair value; Long-term debt, fair value D. Plant and equipment, fair value; Long-term debt, K's carrying amount

C. Plant and equipment, fair value; Long-term debt, fair value This must be accounted for under the acquisition method. Assets and liabilities are recorded at fair value. Any excess of acquisition price over fair value is recorded as goodwill.

Which of the following items is recognized for governmental activities in the government-wide statement of activities and not the statement of revenues, expenditures, and changes in fund balance for governmental funds? A. Transfers between governmental funds B. Salaries payable at the end of the current year that will be paid at the beginning of the subsequent year C. Property tax revenue for an amount deferred because it was not available D. A state grant awarded and received for road repairs that were completed this fiscal year

C. Property tax revenue for an amount deferred because it was not available The link between government-wide and governmental fund statements requires a reconciliation to convert the governmental funds (which use the modified accrual basis of accounting) to the economic resources measurement and accrual basis of accounting used by the government-wide funds. Adjustments usually include moving transactions for general capital assets and general long-term liabilities from the operating statement to the balance sheet. Other reconciling items may include revenue and expense accruals or adding the internal service fund residual net assets. Transfers between governmental funds, a state grant awarded and received for road repairs that were completed this fiscal year, and salaries payable at the end of the current year that will be paid at the beginning of the subsequent year are reported in both statements and would therefore not be reconciling items. Property tax revenue must be available to be recognized under modified accrual accounting; since the revenue is not available, there is no revenue recognition. However, property tax revenue would be recognized in the government-wide statement of activities, which uses full accrual accounting and is not constrained by the "availability" criteria; thus, it is the only reconciling item.

Past events and current conditions potentially impacting the entity's future cash flow include which of the following? A. Segment reporting B. Related party transactions C. Subsequent events D. Timing of asset cash flows

C. Subsequent events

Which of the following does not affect an internal service fund's revenues earned or expenses incurred? A. Depreciation expense on its fixed assets B. Operating transfer sources C. Temporary transfers D. Residual equity transfers

C. Temporary transfers Internal service funds are used to account for in-house business enterprise activities (i.e., to account for the financing of goods or services provided by one government department or agency to other departments or agencies of the government and perhaps to other governments also) on a cost-reimbursement basis. By the very nature of "temporary," the implication is to undo the transfer at some point in time, and it should not impact the revenues or expenses of the fund.

Which of the following is one of the criteria an entity must meet in order to recognize an asset for the costs incurred to fulfill a contract? A. The costs relate directly to a contract or to an anticipated contract that the entity can specifically identify, and include direct labor, materials, overhead, and general and administrative-type costs. B. The costs have been approved for asset recognition by the entity's independent accountants. C. The costs generate or enhance resources of the entity that will be used in satisfying or in continuing to satisfy performance obligations in the future. D. The costs are expected to be recovered within one year of the transaction date.

C. The costs generate or enhance resources of the entity that will be used in satisfying or in continuing to satisfy performance obligations in the future. An entity should recognize an asset for the costs incurred to fulfill a contract only if those costs meet all of the following criteria: the costs relate directly to a contract that the entity can specifically identify (e.g., costs relating to services to be provided under renewal of an existing contract or costs of designing an asset to be transferred under a specific contract that has not yet been approved); the costs generate or enhance resources of the entity that will be used in satisfying or in continuing to satisfy performance obligations in the future; and the costs are expected to be recovered.

A company is required to file quarterly financial statements with the U.S. Securities and Exchange Commission on Form 10-Q. The company operates in an industry that is not subject to seasonal fluctuations that could have a significant impact on its financial condition. In addition to the most recent quarter-end, for which of the following periods is the company required to present balance sheets on Form 10-Q? A. The end of the corresponding fiscal quarter of the preceding fiscal year B. The end of the preceding fiscal year and the end of the corresponding fiscal quarter of the preceding fiscal year C. The end of preceding fiscal year D. The end of the preceding fiscal year and the end of the prior two fiscal years

C. The end of preceding fiscal year Form 10-Q is used to file quarterly reports with the SEC. Required financial statements include a quarterly and end of the preceding fiscal year balance sheet. If the company is subject to seasonal fluctuations, a balance sheet for the corresponding quarter of the prior fiscal year is required

Opto Co. is a publicly traded, consolidated enterprise reporting segment information. Which of the following items is a required enterprise-wide disclosure regarding external customers? A. The identity of any external customer considered to be "major" by management B. The identity of any external customer providing 10% or more of a particular operating segment's revenue C. The fact that transactions with a particular external customer constitute more than 10% of the total enterprise revenues D. Information on major customers is not required in segment reporting.

C. The fact that transactions with a particular external customer constitute more than 10% of the total enterprise revenues

Which of the following statements is false concerning the notes to the financial statements of state and local governments? A. The notes are found within Part 2, the Financial Section of the ACFR. B. The notes follow the presentation of the financial statements. C. The notes are not considered to be essential to fully understanding the financial statements. D. The notes are subject to audit procedures.

C. The notes are not considered to be essential to fully understanding the financial statements. Notes to the financial statements are essential to a reader's understanding of a government's financial position or resource inflows and outflows.

If both an asset group in a company and goodwill in one of its reporting units have to be tested for impairment, which of the following statements is correct regarding impairment testing and impairment losses? A. Impairment testing may be conducted concurrently for the other asset group and goodwill. B. If goodwill is impaired, the loss should be recognized prior to testing the other assets for impairment. C. The other asset group should be tested for an impairment loss before goodwill is tested. D. If the other asset group is impaired, the loss should not be recognized prior to goodwill being tested for impairment.

C. The other asset group should be tested for an impairment loss before goodwill is tested.

Which of the following information should be included in Gold Corporation's 20X7 summary of significant accounting policies? A. The specific amounts of raw material inventory, work-in-process inventory, and finished goods shown in aggregate on the balance sheet B. The valuation model used to determine the value of stock options granted to upper-level managers C. The policies regarding inventory valuation and the methods used for inventory cost determination D. The specific amounts of the components of pension expense

C. The policies regarding inventory valuation and the methods used for inventory cost determination FASB ASC 235-10-50-4 requires a description of all significant accounting policies when financial statements are issued. A listing of required policy disclosures by this pronouncement includes basis of consolidation, depreciation methods, amortization of intangibles, inventory pricing, and several other items. Gold should include information concerning how inventory is valued and the inventory cost flow assumptions used in its summary of significant accounting policies. Note: While the other three information items in the answer choices should be disclosed in the notes to the financial statements, they should not be included in the summary of significant accounting policies.

Which of the following statements is the most significant characteristic in determining the classification of an activity of an enterprise fund? A. Laws or regulations require that the activity's costs of providing services including capital costs be recovered with taxes or similar revenues. B. The activity is financed by debt that is secured partially by a pledge of the net revenues from fees and charges of the activity. C. The pricing policies of the activity establish fees and charges designed to recover its cost. D. The predominant customer is the primary government.

C. The pricing policies of the activity establish fees and charges designed to recover its cost. That the pricing policies of the activity establish fees and charges designed to cover its cost is the most important criterion from this list to use an enterprise fund to account for the activity. If the predominant customer of an activity is the primary government, an internal service fund, a different proprietary fund, would be used. An enterprise fund must be used if the activity is financed with debt solely, not partially, secured by a pledge of the activity's revenues. An enterprise fund would be required if the activity's costs must, by law, be covered by fees and charges, not only by "taxes or similar revenues."

A firm's ending inventory balance was overstated by $1,000. Which of the following statements is correct according to a periodic inventory system? A. The cost of goods available for sale was overstated by $1,000. B. The gross margin was understated by $1,000. C. The retained earnings were overstated by $1,000. D. The cost of goods sold was overstated by $1,000.

C. The retained earnings were overstated by $1,000. To solve this question, try using the inventory formula: Cost of goods sold (COGS) = Beginning inventory (BI) + Purchases (P) - Ending inventory (EI) If EI is overstated, then COGS is understated. This results in overstating net income (NI). Since NI closes into retained earnings (RE) at year-end, RE would also be overstated (by $1,000). Cost of goods available for sale (CGAS) is BI + P and would not be affected by the amount in EI.

Which of the following is a true statement regarding FASB ASC 825-10-25-1? A. The fair value option established by FASB ASC 825-10 requires all entities to measure eligible items at fair value at specified dates. B. A business entity shall report unrealized gains and losses on items for which the fair value option has been elected as an adjustment to retained earnings. C. The statement permits election of fair value measurement on a contract-by-contract basis. D. None of the answer choices are true statements regarding FASB ASC 825-10-25-1.

C. The statement permits election of fair value measurement on a contract-by-contract basis. FASB ASC 825-10-25-1 permits the fair value election but does not require it. Unrealized gains and losses on these items are reported in earnings, not directly to retained earnings: "This Subtopic permits all entities to choose, at specified election dates, to measure eligible items at fair value (the 'fair value option')." FASB ASC 825-10-25-2 requires that the fair value option be applied contract by contract: "The decision about whether to elect the fair value option: "Shall be applied instrument by instrument, except as discussed in [FASB ASC] 825-10-25-7 "Shall be irrevocable (unless a new election date occurs, as discussed in [FASB ASC] 825-10-25-4) "Shall be applied only to an entire instrument and not to only specified risks, specific cash flows, or portions of that instrument."

Which of the following statements about accounting standard setting in the United States is true? A. While the SEC is primarily responsible for setting accounting standards in the United States, the FASB is primarily responsible for enforcing the standards. B. The FASB and SEC must seek approval of the U.S. Congress before a new accounting standard can be approved. C. While the FASB is primarily responsible for setting accounting standards in the United States, the SEC is primarily responsible for enforcing the standards. D. The FASB and the SEC are both deeply involved in the day-to-day process of setting accounting standards.

C. While the FASB is primarily responsible for setting accounting standards in the United States, the SEC is primarily responsible for enforcing the standards.

A public business entity has a December 31 year-end reporting period and is aware that a goodwill impairment test must be performed at least once during each reporting period. The entity's controller has compiled a list of four potential dates in year 1 and year 2 to test for goodwill impairment. Assuming that there are no events or circumstances requiring impairment testing between the two scheduled dates, which of the following dates in year 1 and year 2, when viewed together, would comply with appropriate guidance? A. Year 1, June 20; year 2, December 20 B. Year 1, December 31; year 2, March 31 C. Year 1, April 15; year 2, April 15 D. Year 1, December 31; year 2, June 30

C. Year 1, April 15; year 2, April 15 U.S. GAAP requires public entities to assess their goodwill accounts for impairment at least annually. The April 15 testing dates fall within that timeframe. The June 20 to December 20 period is longer than annually. The other two options are at least annually but would require a third testing date to cover the full-year period and therefore are not the best answers as they are missing a more recent testing date.

A company acquired a building, paying a portion of the purchase price in cash and issuing a mortgage note payable to the seller for the balance. In a statement of cash flows, what amount is included in financing activities for the transaction? A. Cash payment B. Acquisition price C. Zero D. Mortgage amount

C. Zero The only cash involved in this transaction is the cash paid. It would be included in cash flows from investing activities. The cash purchase of the building is an investing activity. The issuance of the note payable has no impact on cash as we are issuing debt for the remaining purchase price. When the company pays down the note payable in cash, that would be a financing activity. Any interest paid on the note would be an operating activity. Helpful Hint: For cash flow problems, write down a hypothetical journal entry. If cash isn't in your journal entry, it has no cash flow impact. If it does, ask yourself, what is the cash going towards?

Fogg Co., a U.S. company, contracted to purchase foreign goods. Payment in foreign currency was due one month after the goods were received at Fogg's warehouse. Between the receipt of goods and the time of payment, the exchange rates changed in Fogg's favor. The resulting gain should be included in Fogg's financial statements as: A. a separate component of stockholders' equity. B. a component of other comprehensive income. C. a component of income from continuing operations. D. a deferred credit.

C. a component of income from continuing operations. Fogg should include the gain as a component of income from continuing operations according to the provisions of FASB ASC 830-20-35-1: "A change in exchange rates between the functional currency and the currency in which a transaction is denominated increases or decreases the expected amount of functional currency cash flows upon settlement of the transaction. That increase or decrease in expected functional currency cash flows is a foreign currency transaction gain or loss that generally shall be included in determining net income for the period in which the exchange rate changes."

Newt Co. sold a warehouse and used the proceeds to acquire a new warehouse. The excess of the proceeds over the carrying amount of the warehouse sold should be reported as: A. a component of other comprehensive income. B. a gain from discontinued operations, net of income taxes. C. a part of continuing operations. D. a reduction of the cost of the new warehouse.

C. a part of continuing operations. The sale and purchase should be recorded separately. The gain on the sale is reported as other income and is a component of income from continuing operations.

Some events provide evidence regarding conditions that did not exist on the balance sheet date but arose subsequently and do not require an adjustment of the balance sheet. Other types of subsequent events do require an adjustment to the financial statements. Assuming that the item is material, an example of a subsequent event that requires adjustment is: A. the issuance of additional shares of common stock. B. a business combination. C. a settlement of an IRS judgment in which the final amount due is determined. D. a loss from the damage caused by an earthquake.

C. a settlement of an IRS judgment in which the final amount due is determined. "An entity shall recognize in the financial statements the effects of all subsequent events that provide additional evidence about conditions that existed at the date of the balance sheet, including the estimates inherent in the process of preparing financial statements...Subsequent events affecting the realization of assets, such as receivables and inventories or the settlement of estimated liabilities, should be recognized in the financial statements when those events represent the culmination of conditions that existed over a relatively long period of time.

A summary reconciliation between fund financial statements and government-wide financial statements is required at the bottom of the fund statements or in an accompanying schedule. Assume that internal service funds provide goods and services for governmental functions. For the business-type activities portion of the government-wide statement of net position, the reconciliation should tie with the fund balance(s) of: A. all enterprise funds and discretely presented component units. B. all proprietary funds both enterprise and internal service. C. all enterprise funds. D. all enterprise and fiduciary funds that provide support services to the business-type activities.

C. all enterprise funds. The business-like activities portion of the government-wide financial statements report the functions also reported in the enterprise funds and internal service funds providing goods and services to the enterprise funds. In this problem, the internal service funds provide goods and services only for governmental functions. Fiduciary fund information is not shown within the government-wide financial statements. Discretely presented component unit information is shown separate from the governmental and business-like activities portions on the government-wide financial statements.

The measurement focus of governmental-type funds is the determination of: A. flow of financial resources. B. financial position. C. both flow of financial resources and financial position. D. neither flow of financial resources nor financial position.

C. both flow of financial resources and financial position. The measurement focus of governmental type funds is on both: the changes in financial position and financial position. The flow of financial resources refers to the changes in financial position from the sources and uses of financial resources (GASB 1300.102). By contrast, the measurement focus of a proprietary fund is on determining "operating income, changes in net position (or cost recovery), financial position, and cash flows"—similar to a commercial entity.

A city has a number of open purchases remaining at year-end. These purchase orders are represented in the general fund records as both Encumbrances (debit balance) and Fund balance—reserved for encumbrances (credit balance). Encumbrances outstanding at year-end represent: A. liabilities of the general fund. B. intentions of the general fund. C. budgetary control for the general fund. D. expenditures of the general fund.

C. budgetary control for the general fund. Encumbrance accounting is a method of budgetary control for governmental funds, including the general fund. Encumbrances do not represent expenditures as they are a memorandum of commitments that will eventually lead to expenditures. They do not represent liabilities as the goods and services are yet to be delivered. Governmental intent to commit resources for specific uses in the future is indicated by an assignment of fund balance.

The initial test in FASB ASC 360-10-35 for determining whether an impairment of the carrying amount of a long-lived asset is indicated is: A. carrying amount exceeds the fair value. B. fair value exceeds carrying amount. C. carrying amount exceeds undiscounted future cash flows. D. carrying amount exceeds book value.

C. carrying amount exceeds undiscounted future cash flows. FASB ASC 360-10-35-17 indicates that an impairment loss exists when the asset's carrying amount exceeds its undiscounted future cash flows. Carrying amount exceeding the fair value is used to measure the amount of the impairment loss, not to identify the existence of such a loss. Assets subject to assessment for impairment under FASB ASC 805-20-55-4 are also subject to the same undiscounted cash flow recoverability test.

By definition, a fund is: A. interchangeable and interdependent with the general fund. B. a single entity reporting on all government activities. C. composed of a self-balancing set of accounts. D. a "government-type" entity through which all governmental functions are financed.

C. composed of a self-balancing set of accounts. Under fund accounting, each fund is a separate "fiscal and accounting entity with a self-balancing set of accounts recording cash and other financial resources, together with all related liabilities and residual equities and balances, and changes therein, which are segregated for the purpose of carrying on specific activities or attaining certain objectives in accordance with special regulations, restrictions, or limitations." (Emphasis added) (GASB 1100.102) A government's activities are not captured in a single fund but are divided into several separate funds to account for specific revenue sources and activities. A fund can be one of three categories: (1) governmental, (2) proprietary, or (3) fiduciary. A fund is not interchangeable and interdependent with the general fund. Each fund is a separate fiscal and accounting entity.

Park City uses encumbrance accounting and formally integrates its budget into the general fund's accounting records. For the current year ending July 31, the following budget was adopted: Estimated revenues $30,000,000Appropriations 27,000,000Estimated transfer to debt service fund 900,000 Park should record budgeted appropriations by a: A. debit to estimated expenditures, $27,000,000. B. credit to appropriations control, $27,900,000. C. credit to appropriations control, $27,000,000. D. debit to estimated expenditures, $27,900,000.

C. credit to appropriations control, $27,000,000.

South City School District has a separately elected governing body that administers the public school system. The district's budget is subject to the approval of the city council. The district's financial activity should be reported in the City's financial statements by: A. inclusion as a footnote only. B. either blending or inclusion as a footnote. C. discrete presentation. D. blending only.

C. discrete presentation. Per GASB 2100.109, financial information for separate organizations for which the primary government's elected officials are financially accountable must be included in the primary government's financial statements even though the organization is a separate legal entity. These separate organizations are called component units. Discrete presentation should be used for this presentation of financial information unless the financial activities of the two entities are so intertwined as to make them substantially the same entity. Since this does not appear to be the case in this question, discrete presentation of the information is required.

An impairment loss for assets to be held and used is reported as: A. other comprehensive income. B. discontinued operations. C. element of income from operations. D. cumulative effect of change in accounting principle.

C. element of income from operations.

Chase City uses an internal service fund for its central motor pool. The assets and liabilities account balances for this fund that are not eliminated normally should be reported in the government-wide statement of net position as: A. note disclosures only. B. business-type activities. C. governmental activities. D. fiduciary activities.

C. governmental activities. The governmental activity which is the predominant user of the internal service funds absorbs and reports the assets and liabilities of an internal service fund that are not eliminated. In most situations, this will be the governmental activities. (GASB 2200.147-.150)

For purposes of consolidating financial interests, a majority voting interest is deemed to be: A. 50% of the directly or indirectly owned outstanding voting shares of another entity. B. 50% of the directly or indirectly owned outstanding voting shares and at least 50% of the directly or indirectly owned outstanding nonvoting shares of another entity. C. greater than 50% of the directly or indirectly owned outstanding voting shares of another entity. D. greater than 50% of the directly or indirectly owned outstanding voting shares and at least 50% of the directly or indirectly owned outstanding nonvoting shares of another entity.

C. greater than 50% of the directly or indirectly owned outstanding voting shares of another entity.

Bay Manufacturing Co. purchased a 3-month U.S. Treasury bill. In preparing Bay's statement of cash flows, this purchase would: A. be treated as an outflow from investing activities. B. be treated as an outflow from operating activities. C. have no effect. D. be treated as an outflow from financing activities.

C. have no effect.

A note receivable bearing a reasonable interest rate is sold to a bank with recourse. At the date of the discounting transaction, the notes receivable discounted account should be: A. increased by the proceeds from the discounting transaction. B. decreased by the proceeds from the discounting transaction. C. increased by the face amount of the note. D. decreased by the face amount of the note.

C. increased by the face amount of the note. A note sold with recourse is a promise to pay the financial institution if the maker dishonors the note. When receivables are sold with recourse, the entity has a contingent liability. A contingent liability is an obligation that has to be paid in the future. Therefore, the notes receivable discounted account must be increased by the face amount of the note.

Each of the following statements is correct regarding the Financial Accounting Standards Board, except: A. it establishes accounting concepts and standards for financial accounting and reporting, and provides guidance on implementation of standards. B. it is recognized as authoritative by the United States Securities and Exchange Commission and the American Institute of Certified Public Accountants. C. it develops principles and attributes that allow organizations to understand the necessary elements to ensure a robust system of internal control. D. it provides a conceptual framework that helps to increase understanding of, and confidence in, financial information on the part of users of financial reports.

C. it develops principles and attributes that allow organizations to understand the necessary elements to ensure a robust system of internal control.

A town's fund financial statements are prepared following major fund reporting requirements, and combining fund statements detailing nonmajor funds are also provided. In reviewing the annual comprehensive financial report (ACFR), you notice that all the internal service funds were combined and reported in a single column in the basic financial statements. The combining financial statements for internal service funds included a column for each of the town's four internal service funds. The explanation of this presentation is: A. the internal service funds provided service specifically for governmental activities. B. none of the internal service funds accounted for 10% of the total assets, liabilities, revenues, or expenditures/expenses of the proprietary funds or 5% of those elements of all funds. C. major fund reporting requirements do not apply to internal service funds. D. management did not feel that any internal service fund was important enough for separate presentation in the basic financial statements, although the officials had the option of including any fund felt to be important.

C. major fund reporting requirements do not apply to internal service funds. Major fund reporting requirements do not apply to internal service funds. Therefore, they are usually combined for financial reporting and presented individually with combining statements. The other answer choices are not sufficient explanations for the town's presentation of the internal service fund financial information.

State and local governments must report budgetary comparisons showing both the original and final appropriated budgets for the reporting period as well as actual inflows, outflows, and balances stated on the government's budgetary basis. If the budgetary perspective does not significantly differ from the fund reporting perspective, this budgetary comparison statement: A. may be included in the management's discussion and analysis (MD&A), the basic financial statements, or the required supplementary information (RSI). B. must be included in the basic financial statements. C. may be included in the basic financial statements or in the required supplementary information (RSI). D. must be included in required supplementary information (RSI).

C. may be included in the basic financial statements or in the required supplementary information (RSI).

An investor purchased a bond classified as a long-term investment between interest dates at a discount. At the purchase date, the carrying amount of the bond is more than: A. the cash paid to the seller. B. the face amount of the bond. C. neither the cash paid to the seller nor the face amount of the bond. D. both the cash paid to the seller and the face amount of the bond.

C. neither the cash paid to the seller nor the face amount of the bond. If the investor buys a bond at a discount, then the bond will be carried at the discount price initially, which is below the face amount of the bond. However, if the investor buys a bond between interest payment dates, the investor will pay (in part) for the already accrued interest that the investor will soon receive back. Thus, the carrying amount of the bond will actually be less than the total the investor pays to acquire the bond, both its discount price plus the amount paid for the interest receivable.

The billings for transportation services provided to other governmental units are recorded by the internal service fund as: A. nonoperating revenues. B. transportation appropriations. C. operating revenues. D. other financing sources.

C. operating revenues. Internal service funds are established to account for activities that one department within a government undertakes for the benefit of (1) other departments within that same government (usual case) and (2) (sometimes) other governments, at prices approximating their external exchange value. This question illustrates an exchange-like reciprocal interfund activity. Interfund services provided and used should be reported as revenues in seller funds and expenditures or expenses in purchaser funds (GASB 1800.102). Other financing sources are used to record operating transfers-in for governmental funds. Nonoperating revenues include earnings tangential to the purpose of the fund, e.g., interest and miscellaneous revenue. Transportation appropriations is a budgetary account.

On January 1, 20X2, to better reflect the variable use of its only machine, Holly, Inc., elected to change its method of depreciation from the straight-line method to the units of production method. The original cost of the machine on January 2, 20X0, was $50,000, and its estimated life was 10 years. Holly estimates that the machine's remaining total life is 50,000 machine hours as of January 1, 20X2. Machine hours usage was 8,500 during 20X1 and 3,500 during 20X0. Holly's income tax rate is 30%. Holly should report the accounting change in its 20X2 financial statements as: A. a cumulative effect of a change in accounting principle of $1,400 in its income statement. B. an adjustment to beginning retained earnings of $2,000. C. prospectively. D. an adjustment to beginning retained earnings of $1,400.

C. prospectively. FASB ASC 250-10-45-5 requires that changes in depreciation methods be accounted for prospectively. Accordingly, the carrying amount (book value) of the machine at January 1, 20X2, should be depreciated over the remaining life of the machine. At January 1, 20X2, the carrying amount is $40,000 (i.e., $50,000 acquisition cost less $10,000 accumulated depreciation), which should be depreciated over the remaining 50,000 machine hours at a rate of $.80 per machine hour (i.e., $40,000 ÷ 50,000 hours = $.80). The depreciation amount to be recognized in 20X2 is the number of machine hours used in 20X2 × $.80.

Civic Town has a number of enterprise funds, some reported as major funds in the basic financial statements and some considered as nonmajor funds, reported in aggregated form. One of these nonmajor funds, the Airport fund, accounts for the operations of a small airport used intermittently by hobbyists. Due the requirements of a grant, Airport fund expenses must be reported in more detail than the other nonmajor enterprise funds with which the Airport fund is aggregated. This information can best be shown in the annual comprehensive financial report by: A. including the Airport fund in the combining fund statements of governmental funds. B. including the Airport fund in the combining funds statements of the enterprise funds. C. providing schedules supporting Airport fund information. D. adding information about the Airport fund to the required supplementary information.

C. providing schedules supporting Airport fund information. The question makes it clear that the expense categories in the combining enterprise funds statements are not detailed enough for the grant reporting. The additional detail is not one of the items required as supplementary information. The Airport fund would not be included in the combining fund statements of governmental funds because it is an enterprise fund. The additional detail should be shown in a schedule in order to show compliance with finance-related legal and contractual provisions.

Darrow Limited operates under a franchise agreement that has limited useful life. On Darrow's balance sheet, the intangible asset "Franchise" has a book value of $123,500. Because of declining economic conditions, Darrow determines that the undiscounted cash flows from the franchise are $107,000. Darrow recently received an offer of $95,000 to sell the franchise for its remaining useful life. Darrow should: A. recognize an impairment loss of $8,000. B. not recognize an impairment loss. C. recognize an impairment loss of $28,500. D. recognize an impairment loss of $16,500

C. recognize an impairment loss of $28,500. Because the franchise is a limited-life intangible asset, the two-step impairment process should be used. The recoverability test (comparison of book value to undiscounted cash flows) indicates that the book value will not be recoverable (undiscounted cash flows of $107,000 is less than the book value of $123,500) and therefore the second step needs to be carried out. Because the fair value ($95,000) of the franchise is less than the book value ($123,500), the difference between the two values ($123,500 - $95,000 = $28,500) is the amount of the impairment loss.

Clover City's government-wide financial statements should: A. not distinguish between governmental and business-type activities. B. be prepared using the modified accrual basis of accounting. C. report information about the overall government without displaying individual funds or fund types. D. include information about fiduciary activities.

C. report information about the overall government without displaying individual funds or fund types. The government-wide financial statements consist of a statement of net position and a statement of activities. Those statements should "report information about the overall government without displaying individual funds or fund types" (GASB 2200.110). The statements should be prepared using the accrual basis of accounting and distinguish between governmental and business-type activities. Information about fiduciary funds should not be included in the statements.

During 20X1, Orca Corp. decided to change from the FIFO method of inventory valuation to the weighted-average method. Inventory balances under each method were as follows: FIFO Weighted-AverageJanuary 1, 20X1 $71,000 $77,000December 31, 20X1 79,000 83,000 Orca's income tax rate is 30%. Orca should report the cumulative effect of this accounting change as: A. a component of income from continuing operations. B. a component of income after discontinued operations. C. retrospectively as an adjustment of the beginning-of-period balance of retained earnings of the earliest year presented. D. a component of accumulated other comprehensive income.

C. retrospectively as an adjustment of the beginning-of-period balance of retained earnings of the earliest year presented. FASB ASC 250-10-45-5 mandates that voluntary changes in accounting principle be recognized using the retrospective approach, in which the cumulative effect is reported as an adjustment of the beginning-of-year retained earnings of the earliest year presented. The only exception is when the FASB issues a new pronouncement and mandates in that pronouncement that a change in accounting principle made to comply with that pronouncement should be made by including the cumulative effect in net income of the year of change.

A town's basic financial statements include information for major and nonmajor governmental funds. There were no internal service or enterprise funds. One of the nonmajor funds is the Road Tax fund, which accounts for a share of tax moneys remitted by the state on a prorated basis. Individual fund statements with prior-year comparative data would have to be presented for the Road Tax fund if: A. the Road Tax fund is the town's only special revenue fund. B. every governmental fund must be reported individually. C. the town has opted to present budgetary data as required supplementary information rather than as part of the basic financial statements. C. state law requires prior-year comparative data for any individual fund receiving a prorated share of state tax collections.

C. state law requires prior-year comparative data for any individual fund receiving a prorated share of state tax collections. As a nonmajor fund, the financial information for the Road Tax fund is combined with other nonmajor funds in the basic financial statements. Individual fund financial statements would be required to demonstrate compliance with state law in this case. Whether shown in RSI or as part of the basic financial statements, the budgetary data would not otherwise show the details of individual, nonmajor funds. In combining nonmajor funds, it does not matter if only one of the governmental funds was a special revenue fund. Because the focus of fund financial reporting is on major funds, nonmajor funds should be aggregated and do not need to be shown individually.

During periods of rising prices, when the FIFO inventory method is used, a perpetual inventory system results in an ending inventory cost that is: A. higher than in a periodic inventory system. B. higher or lower than in a periodic inventory system, depending on whether physical quantities have increased or decreased. C. the same as in a periodic inventory system. D. lower than in a periodic inventory system.

C. the same as in a periodic inventory system. The FIFO perpetual inventory method will produce the same ending inventory as the FIFO periodic method. This is due to the fact that the "first-in" units are removed first under both methods. The only difference is that the units sold are removed immediately under the perpetual approach but only at the end of the period under the periodic approach. The flow and amounts are the same. This is not true for any other inventory method (other than specific identification).

Bren Co.'s beginning inventory at January 1 was understated by $26,000, and its ending inventory was overstated by $52,000. As a result, Bren's cost of goods sold for the year was: A. understated by $26,000. B. overstated by $26,000. C. understated by $78,000. D. overstated by $78,000.

C. understated by $78,000.

At December 31, Year 3, Wee World held the following available-for-sale debt securities portfolio: UnrealizedSecurity Cost Fair Value Gain (Loss)BCD $12,200 $11,800 ($400)FGH 60,300 62,000 1,700JKL 25,700 25,000 (700) Total $98,200 $98,800 $600======= ======= ======Previous fair value adjustment - Dr. ($500)====== Which of the following would be included on the Income Statement? A. Unrealized holding gain of $1,100 B. Unrealized holding gain of $600 C. Unrealized holding loss of $500 D. $0

D. $0 Available-for-sale (AFS) debt securities are carried on the balance sheet at fair value. Unrealized gains and losses from changes in fair value are reported in Other Comprehensive Income (OCI) for the period. OCI is then added to (subtracted from) Accumulated Other Comprehensive Income (AOCI), which is shown as a separate component of stockholders' equity until realized. Therefore, $0 would be reflected on the income statement.

Aln Co. incurred the following expenses during the current period: Routine ongoing efforts to improve an existing product $ 50,000 Troubleshooting in connection with breakdowns during commercial production 75,000 Routine testing of products during commercial production for quality-control purposes 100,000 What is the total amount of research and development expense incurred by Aln during the current period? A. $125,000 B. $75,000 C. $175,000 D. $0

D. $0 U.S. GAAP defines research and development costs as costs that will be useful in developing a new product or service and the translation of research into a plan or design for a new product. The costs relate to activities identified with the period prior to the beginning of commercial production and are expensed as incurred because their ability to provide future benefits is still uncertain. Ongoing efforts to improve an existing product already in production, troubleshooting during commercial production, and routine testing during commercial production are all activities that have occurred after the production phase has begun and therefore would not quality as research and development expenses. The company would report $0 in research and development as none of its activities relate to the preproduction period.

Bort Co. purchased 2,000 shares of Crel Co. common stock on March 5, Year 1, for $72,000. Bort received a $1,000 cash dividend on the Crel stock on July 15, Year 1. Crel declared a 10% stock dividend on December 15, Year 1, to stockholders of record as of December 31, Year 1. The dividend was distributed on January 15, Year 2. The market price of the stock was $38 on December 15, Year 1, $40 on December 31, Year 1, and $42 on January 15, Year 2. What amount should Bort record as dividend revenue for the year ended December 31, Year 1? A. $9,000 B. $9,400 C. $8,600 D. $1,000

D. $1,000 Cash dividends ($1,000) are recorded as income (revenue) when the dividend is declared for investments in equity securities not accounted for under the equity method. Stock dividends are not reflected in earnings (i.e., income statement).

On January 2 of the current year, Otto Co. purchased 40% of Penn Co.'s outstanding common stock. The carrying amount of Penn's depreciable assets was $1,000,000 on January 2. Penn's depreciable assets had an original useful life of 10 years, and a remaining useful life of 5 years. Otto recognized $8,000 amortization for the current year ending December 31 related to its investment in Penn due to the excess of fair value over book value on these assets. What was the fair value of Penn's depreciable assets on January 2 of the current year? A. $100,000 B. $900,000 C. $1,000,000 D. $1,100,000

D. $1,100,000 When a purchase is made of the stock of another company, and the equity method is applicable to it (generally from 20% to 50% ownership acquired), if the depreciable assets have a fair value in excess of the book value, then the acquiring corporation may need to use its purchase price (the higher fair value) for those assets to figure out its depreciation when recognizing its earnings from the equity method holdings. The owned corporation, Penn, continues to depreciate its assets at their book value, but the owning corporation, Otto, will use the fair value of those assets at the time of purchase to figure out income. Thus, Otto must adjust its income (from Penn) as Penn's income less the additional depreciation, which is part of the purchase price. The additional depreciation ($8,000) is computed as the total fair value of the assets multiplied by the percentage of ownership (40%), divided by the remaining useful life of the assets. Working backwards: $8,000 × 5 years (Remaining useful life) ÷ 0.40 (Ownership) = $100,000 Thus, when Otto bought in, the fair value of Penn must have been $100,000 over book value, or $1,100,000.

Jones Co. exchanged a machine with a fair value of $25,000 for a new machine with a fair value of $20,000, and received $5,000 in cash. The old machine cost $80,000 and had accumulated depreciation of $64,000. The exchange transaction lacked commercial substance. What amount of gain should Jones recognize on the exchange? A. $9,000 B. $7,200 C. $0 D. $1,800

D. $1,800 When a nonmonetary exchange occurs that lacks commercial substance, the typical accounting records the received asset at the book value of the old asset. However, if boot (typically cash) is received along with the new asset, the company will recognize a gain. If the boot is less than 25% of the total consideration (Asset + Boot) received, the gain will be recognized proportionally. The boot represented 20% (($5,000 ÷ ($20,000 + $5,000)) of the total consideration. The old machine had a book value of $16,000 ($80,000 cost − $64,000 accumulated depreciation). The total gain that would be recognized usually is $9,000 ($25,000 value received − $16,000 book value given up). Since boot is less than 25%, Jones gets a proportional recognition of $1,800 ($9,000 total gain × 20%).

During the year, Smith University's board of trustees established a $100,000 fund to be retained and invested for scholarship grants. The fund earned $6,000 which had not been disbursed at December 31. What amount should Smith report in a quasi-endowment fund's net assets at December 31? A. $0 B. $100,000 C. $6,000 D. $106,000

D. $106,000 Since the principal of the endowment and the income from investment of endowment funds are both restricted to the purpose of funding scholarships, and the investment income remained undisbursed at the end of the fiscal year, the principal and income both contribute to the net assets of this specific fund.

A company with a June 30 fiscal year-end entered into a $3,000,000 construction project on April 1 to be completed on September 30. The cumulative construction-in-progress balances at April 30, May 31, and June 30 were $500,000, $800,000, and $1,500,000, respectively. The interest rate on company debt used to finance the construction project was 5% from April 1 through June 30 and 6% from July 1 through September 30. Assuming that the asset is placed into service on October 1, what amount of interest should be capitalized to the project on June 30? A. $18,750 B. $75,000 C. $90,000 D. $11,666

D. $11,666 For qualifying assets, the interest cost incurred during the period of construction of an asset is a part of the historical cost of acquiring the asset. In general, the capitalization rate shall be a weighted average of the rates applicable to borrowings (debt) outstanding during the accounting period for which the capitalizable interest is being calculated. Cumulative CWIP # of Months Capitalized Balance Interest Rate Outstanding Interest $ 500,000 5% 1/12 $ 2,083800,000 5% 1/12 3,3331,500,000 5% 1/12 6,250TOTAL $11,666

When equipment is retired, accumulated depreciation is debited for the original cost less any residual recovery under which of the following depreciation methods? A. Composite depreciation: yes; Group depreciation: no B. Composite depreciation: no; Group depreciation: no C. Composite depreciation: no; Group depreciation: yes D. Composite depreciation: yes; Group depreciation: yes

D. Composite depreciation: yes; Group depreciation: yes When applying either group or composite depreciation, when an asset is sold, the cost of the asset is credited, cash proceeds are debited, and accumulated depreciation is also debited as a plug figure to balance the entry.

In Year 2, the Nord Association, a nongovernmental not-for-profit entity, received a $100,000 contribution to fund scholarships for medical students. The donor stipulated that only the interest earned on the contribution be used for the scholarships. Interest earned in Year 2 of $15,000 was used to award scholarships in Year 3. What amount should Nord report as net assets with donor restrictions in the statement of financial position at the end of Year 2? A. $0 B. $100,000 C. $15,000 D. $115,000

D. $115,000 The $100,000 contribution is meant by the donor to remain invested, so the balancing equity in the accounting equation would be net assets with donor restrictions. The $15,000 of interest earned is meant by the donor to be used for a specific purpose, to fund scholarships. The scholarships will be awarded in Year 3 although the interest was earned in Year 2. For the statement of financial position, therefore, the $15,000 of interest earnings is also considered net assets with donor restrictions since the restriction on use will not be released until Year 3. The total amount of net assets with restrictions is $115,000.

In September 20X1, West Corp. made a dividend distribution of one right for each of its 120,000 shares of outstanding common stock. Each right was exercisable for the purchase of 1/100 of a share of West's $50 variable rate preferred stock at an exercise price of $80 per share. On March 20, 20X5, none of the rights had been exercised, and West redeemed them by paying each stockholder $0.10 per right. As a result of this redemption, West's stockholders' equity was reduced by: A. $36,000. B. $2,400. C. $120. D. $12,000.

D. $12,000. Summary journal entries for issuance and redemption of rights: Dr. Cr. September 20X1 Memo entry onlyMarch 20, 20X5, Retained Earnings 12,000(120,000 x $.10)Cash 12,000 The result is a reduction of $12,000 in West's stockholders' equity. Since the warrants were not exercised, equity was not increased, and because the rights were paid for, assets and equity decreased.

East Corp. manufactures stereo systems that carry a 2-year warranty against defects. Based on past experience, warranty costs are estimated at 4% of sales for the warranty period. During 20X1, stereo system sales totaled $3,000,000, and warranty costs of $67,500 were incurred. In its income statement for the year ending December 31, 20X1, East should report warranty expense of: A. $52,500. B. $60,000. C. $67,500. D. $120,000.

D. $120,000. Warranty expense for 20X1: 4% of sales = .04 × $3,000,000 = $120,000 Dr. Cr.Warranty expense 120,000Estimated warranty liability 120,000 The warranty expense is recognized in the year in which the warranted product is sold. The actual warranty expenditures may or may not be made in that same period. The $67,500 warranty expenditures incurred in 20X1 result in a reduction of the estimated warranty liability. Those expenditures may relate to products sold in 20X1 but they also may relate to products sold in a prior period. The entry for the warranty expenditures made in 20X1 is: Dr. Cr.Estimated warranty liability 67,500Cash 67,500

On June 1 of the current year, Cross Corp. issued $300,000 of 8% bonds payable at par with interest payment dates of April 1 and October 1. In its income statement for the current year ended December 31, what amount of interest expense should Cross report? A. $6,000 B. $8,000 C. $12,000 D. $14,000

D. $14,000 The interest is calculated as: Principal ($300,000) × Interest Rate (8%) × Time (7 months ÷ 12 months) = $14,000

Jorge sells $150,000 of product to Wilson, and also purchases $25,000 of advertising services from Wilson. The advertising services have a fair value of $20,000. Jorge should record revenue on its sale of product to Wilson of: A. $130,000. B. $125,000. C. $150,000. D. $145,000.

D. $145,000. An entity should recognize a refund liability if it receives consideration from a customer and expects to refund some or all of that consideration to the customer. A refund liability is measured at the amount of consideration received or receivable for which the entity does not expect to be entitled; that is, amounts not included in the transaction price. The refund liability and corresponding change in the transaction price, and therefore the contract liability, should be updated at the end of each reporting period for changes in circumstances. Jorge is paying more for advertising services than the fair value of those services, so the excess of $5,000 ($25,000 price paid - $20,000 fair value) is a refund of part of the $150,000 sale. Therefore, Jorge records revenue of $145,000 ($150,000 - $5,000).

North Corp. has an employee benefit plan for compensated absences that gives employees 10 paid vacation days and 10 paid sick days. Both vacation and sick days can be carried over indefinitely. Employees can elect to receive payment in lieu of vacation days; however, no payment is given for sick days not taken. At December 31, 20X1, North's unadjusted balance of liability for compensated absences was $21,000. North estimated that there were 150 vacation days and 75 sick days available at December 31, 20X1. North's employees earn an average of $100 per day. In its December 31, 20X1, balance sheet, what amount of liability for compensated absences is North required to report? A. $22,500 B. $36,000 C. $21,000 D. $15,000

D. $15,000 FASB ASC 710-10-25-7 provides that "an employer is not required to accrue a liability for nonvesting accumulating rights to receive sick pay benefits..." Thus, North Corp.'s liability for compensated absences at December 31, 20X1, is $15,000 for the 150 vacation days (at $100 per day).

The following are held by Smite Co.: Cash in checking account $20,000 Cash in bond sinking fund account 30,000 Postdated check from customer dated 1 month from balance sheet date 250 Petty cash 200 Commercial paper (matures in 2 months) 7,000 Certificate of deposit (matures in 6 months) 5,000 What amount should be reported as cash and cash equivalents on Smite's balance sheet? A.$57,200 B. $32,200 C. $27,450 D. $27,200

D. $27,200 Cash includes cash in the checking account and the petty cash. Cash equivalents are short-term, highly liquid investments. They must be convertible to known amounts of cash and generally have maturities when purchased of not more than three months. The commercial paper qualifies as a cash equivalent because it is highly liquid and matures within three months of the balance sheet date. The sinking fund would be restricted cash, and the postdated check is still a receivable.

Estimates of price-level changes for specific inventories are required for which of the following inventory methods? A. Weighted-average cost B. Average cost retail C. Conventional retail D. Dollar-value LIFO

D. Dollar-value LIFO

Dahl Co. traded a delivery van and $5,000 cash for a newer van owned by West Corp. The following information relates to the values of the vans on the exchange date: Carrying Value Fair ValueOld van $30,000 $45,000New van 40,000 50,000 Dahl's income tax rate is 30%. What amounts should Dahl report as gain on exchange of the vans? A. $1,000 B. $700 C. $0 D. $15,000

D. $15,000 FASB ASC 845-10-30-1 specifies that the accounting for nonmonetary exchanges generally should be accounted based on fair values, which is the same basis as that used for monetary transactions. FASB ASC 845-10-30-3 provides three exception cases in which a nonmonetary exchange should be recorded based on the recorded amount (book value) of the assets surrendered: Fair value is not determinable. Exchange transaction is to facilitate sales for customers. Exchange transaction lacks commercial substance. In determining if a nonmonetary exchange has commercial substance, the key issue is to determine if the exchange is expected to significantly change the entity's future cash flows. FASB ASC 845-10-30-4 specifies that the entity's future cash flows are expected to change significantly if either of the following criteria is met: The configuration (risk, timing and amount) of the future cash flows of the asset(s) received differs significantly from the configuration of the future cash flows of the asset(s) transferred. The entity-specific value of the asset(s) received differs from the entity-specific value of the asset(s) transferred, and the difference is significant in relation to the fair values of the assets exchanged. In Dahl's case, criterion (1) is met because the configuration of the future cash flows associated with the asset received (a new van) differs significantly from the future cash flows of the asset surrendered (the old van and cash). Therefore, the exchange has commercial substance and must be accounted for based on fair value. Dahl should record the new van at the $50,000 fair value of the assets surrendered ($5,000 cash × $45,000 fair value of old van). The $15,000 difference between the $30,000 carrying amount of the old van and its fair value of $45,000 should be recognized as a gain. The entry is: Van (new) 50,000Van (old) 30,000Cash 5,000Gain 15,000

Frame Construction Company's contract requires the construction of a bridge in 3 years. The expected total cost of the bridge is $2,000,000, and Frame will receive $2,500,000 for the project. The actual costs incurred to complete the project were $500,000, $900,000, and $600,000, respectively, during each of the 3 years. Progress payments received by Frame were $600,000, $1,200,000, and $700,000, respectively. Assuming revenue is recognized over time, what amount of gross profit would Frame report during the last year of the project? A. $120,000 B. $125,000 C. $140,000 D. $150,000

D. $150,000 Incorrect The first step is to figure the total profit on the contract, as follows: Total revenue was $2,500,000. Total actual costs are known (since the project has been completed) to be $2,000,000 (made up of $500,000 + $900,000 + $600,000 from the 3 years of work). This gives us a profit of $500,000 on the contract ($2,500,000 - $2,000,000). At the beginning of the third year, Frame had expended a total cost of $1,400,000 ($500,000 from the first year, and $900,000 more from the second year combined). At the start of Year 3, Frame was thus 70% complete (based on total cost expended so far, $1,400,000, divided by total cost estimated to finish, $2,000,000). Frame would have already recognized 70% of the total contract profit so far ($350,000, or 0.7 × $500,000 total profit). Thus, Frame has only $150,000 profit remaining to be recognized in Year 3 (Total profit of $500,000 - Profit already recognized of $350,000). Since the total expected cost was the total actual cost, in this particular case the percentage completed in Year 3 times the total contract profit will also give the correct answer: $600,000 ÷ $2,000,000 = 0.3 0.3 × $500,000 = $150,000

Pear Co.'s income statement for the year ended December 31, 20X1, as prepared by Pear's controller, reported income before taxes of $125,000. The auditor questioned the following amounts that had been included in income before taxes: Equity in earnings of Cinn Co. $40,000Dividends received from Cinn 8,000Adjustments to profits of prior yearsfor arithmetical errors in depreciation (35,000) Pear owns 40% of Cinn's common stock. Pear's December 31, 20X1, income statement should report income before taxes of what amount? A. $117,000 B. $85,000 C. $120,000 D. $152,000

D. $152,000 The income before taxes amount is wrong, since the dividends received from an equity method investment are not income, and neither is the correction from the prior year part of this year's net income. Both need to be corrected. Since Pear owns 40% of Cinn's common stock, Pear has the ability to exercise significant influence over Cinn by virtue of its investment and should account for its investment in Cinn by the equity method. The correction of the errors should be reported as an adjustment to the opening balance of retained earnings, net of the related income tax effect.

Wolf Co.'s grant of 30,000 stock appreciation rights enables key employees to receive cash equal to the difference between $20 and the market price of the stock on the date each right is exercised. The service period is 20X1 through 20X3, and the rights are exercisable in 20X4 and the following year. The market price of the stock was $25 and $28 on December 31, 20X1 and 20X2, respectively. Assuming that the fair value of the stock appreciation rights was $5 at December 31, 20X1, and $8 at December 31, 20X2, what amount should Wolf report as the liability under the stock appreciation rights plan in its December 31, 20X2, balance sheet? A. $240,000 B. $130,000 C. $0 D. $160,000

D. $160,000 The measurement date for a liability award, such as Wolf's, is the date of settlement. Liabilities incurred under share-based payment arrangements, such as Wolf's SARs, are remeasured at the end of each reporting period until settlement. Wolf's liability at December 31, 20X2, would be measured as follows: Fair value of SARs at 12/31/X2 (30,000 x $8 fair value) $240,000Percentage to service period through 12/31/X2 2/3Liability at 12/31/X2 $160,000

A company obtained a $300,000 loan with a 10% interest rate on January 1, Year 1, to finance the construction of an office building for its own use. Building construction began on January 1, Year 1, and the project was not completed as of December 31, Year 1. The following payments were made in Year 1 related to the construction project: January 1 Purchased land for $120,000September 1 Progress payment to contractor for $150,000 What amount of interest should be capitalized for the year ended December 31, Year 1? A. $30,000 B. $15,000 C. $13,500 D. $17,000

D. $17,000 The capitalized interest is based on the payments made for the land and progress payments to the contractor based on the amount of the year that these payments were outstanding. The land payment of $120,000 was outstanding the entire year for an expenditure amount of $120,000 × 12/12, or $120,000. The progress payment to the contractor was for $150,000 and was made in September, for only the last four months of the year, thus an expenditure of $150,000 × 4/12, or $50,000. The total weighted-average expenditures was thus $120,000 and $50,000, for a total of $170,000. A larger ($300,000) specifically construction-related debt was outstanding all year, so there is enough principal to capitalize the interest at the 10% rate of the loan. The capitalized interest is thus $170,000 × 0.10, or $17,000.

Duke Co. reported cost of goods sold of $270,000 for 20X1. Additional information is as follows: December 31 January 1 Inventory $60,000 $45,000 Accounts payable 26,000 39,000 If Duke uses the direct method, what amount should Duke report as cash paid to suppliers in its 20X1 statement of cash flows? A. $242,000 B. $268,000 C. $272,000 D. $298,000

D. $298,000 2021 COGS: 270,000 Increase Inventory: 15,000 Decrease AP: 13,000 total = 298,000

Clove Corporation had the following liabilities at the fiscal year end of 20X6: Accounts payable $ 31,0006% bonds payable, due on July 1, 20X7 126,000Notes payable, due in 20X9 75,000Payroll liabilities 11,000Unearned revenue to be earned in 20X8 10,000Accrued vacation liabilities 14,0004% bonds payable, due on December 1, 20X7 132,000 The 4% bonds have a related sinking fund that has been classified as restricted cash and is classified as a noncurrent asset on the balance sheet. How much should Clove Corporation report as current liabilities in its December 31, 20X6, balance sheet? A. $389,000 B. $314,000 C. $192,000 D. $182,000

D. $182,000 Current liabilities are those obligations that will be settled by the use of current assets. The items in the list that will be settled by the use of current assets (cash) are accounts payable, 6% bonds payable, payroll liabilities, and accrued vacation liabilities, for a total of $182,000 ($31,000 + $126,000 + $11,000 + $14,000). The notes payable are not due until 20X9, and the unearned revenue will not be earned until 20X8. Even though the 4% bonds are due within the next year, they are not included in current liabilities because the related bond sinking is classified as a noncurrent asset. The 4% bonds classification in the balance sheet needs to be the same (current vs. noncurrent) as the bond sinking fund.

On March 31, 20X1, the Winn company traded in an old machine that had a carrying amount of $16,800, and a fair value of $14,500. Winn paid a cash difference of $6,000 for a new machine having a total cash price of $20,500. The exchange should include recording: A. no gain or loss. B. $3,700. C. $2,300 loss on exchange. D. $2,300 impairment loss.

D. $2,300 impairment loss. According to FASB ASC 360-10-40-4, an impairment loss is recognized on an exchange of similar productive assets if the carrying amount of the asset exceeds its fair value on the date of exchange. For Winn, an impairment loss of $2,300 ($16,800 carrying amount less $14,500 fair value) should be recognized.

A company manufactured 1,000 units of product during the year and sold 800 units. Costs incurred during the current year are as follows: Direct materials and direct labor $7,000Indirect materials and indirect labor 2,000Insurance on manufacturing equipment 3,000Advertising 1,000 What amount should be reported as inventory in the company's year-end balance sheet? A. $2,600 B. $1,800 C. $1,400 D. $2,400

D. $2,400 Total costs incurred related to the manufacturing of inventory is $12,000 ($7,000 + $2,000 + $3,000). The $1,000 in advertising costs is expensed as incurred and is not capitalized to inventory. There were 1,000 units produced and 800 units sold, so 200 units remain in inventory at a cost of $2,400 ($12,000 total inventory × 200/1,000 units). Note that because no cost assumptions (i.e., LIFO, FIFO, weighted average) were provided, we can assume that all inventory was manufactured at the same cost.

Eagle and Falk are partners with capital balances of $45,000 and $25,000, respectively. They agree to admit Robb as a partner. After the assets of the partnership are revalued, Robb will have a 25% interest in capital and profits, for an investment of $30,000. What amount should be recorded as goodwill to the original partners? A. $0 B. $5,000 C. $7,500 D. $20,000

D. $20,000 When a new partner is admitted by investing into the partnership, the total capital of the partnership changes, and the purchase price (amount of new investment) can be equal to, more than, or less than book value. When the purchase price is equal to book value, no goodwill or bonuses are recorded. When the purchase price is more or less than book value, either goodwill or bonuses must be recorded. The total capital of the resulting new partnership determines whether goodwill or bonuses are recorded. Under the goodwill approach, goodwill is recognized on the basis of the total value of the new partnership implied by the new partner's investment relative to the partners' total capital. Under the bonus approach, such implied value is not considered. In this problem, the assets are revalued, suggesting that goodwill is being recorded. Implied value after new investment: $30,000 represents 25% of total value; therefore, the implied total value is $120,000 ($30,000 ÷ .25). Implied Value $120,000Total partner's capital accounts (100,000)(45,000 + $25,000 + $30,000)Goodwill to original partners $ 20,000

On July 1, Year 1, Pell Co. purchased Green Corp. 10-year, 8% bonds with a face amount of $500,000 for $420,000. The bonds mature on June 30, Year 9, and pay interest semiannually on June 30 and December 31. Using the interest method, Pell recorded bond discount amortization of $1,800 for the six months ended December 31, Year 1. From this long-term investment, Pell should report Year 1 revenue of: A. $16,800. B. $18,200. C. $20,000. D. $21,800.

D. $21,800. If bonds are purchased at a discount, then the discount is immediately recorded as a credit in the acquiring corporation's books. As the discount is amortized, it is thus debited, to decrease it. When cash is received as interest, the additional debit to amortize the discount adds to the debit to cash to increase the total credit to recognized revenue. Therefore, the total revenue for the year will be the cash received as interest over the semiannual period, $500,000 (face amount) × 0.08 (8%) × 6/12, or $20,000, plus the $1,800 discount amortized, for a total revenue for the year of $21,800.

Chatham Co. owned 25% of the voting stock of Boyrum Co. Chatham applied the equity method to account for this investment. Boyrum reported income of $100,000 and paid $30,000 in cash dividends during the period. What amount should Chatham report as investment income? A. $0 B. $7,500 C. $17,500 D. $25,000

D. $25,000 The equity method requires the investor to recognize its share of net income but not dividend income. Dividends received by equity method investors are treated as reductions in their investment accounts and are not recognized as income. Chatham would recognize $25,000 in investment income ($100,000 income reported × 25% ownership).

Thyme, Inc. owns 16,000 of Sage Co.'s 20,000 outstanding common shares. The carrying value of Sage's equity is $500,000. Sage subsequently issues an additional 5,000 previously unissued shares for $200,000 to an outside party that is unrelated to either Thyme or Sage. What is the total noncontrolling interest after the additional shares are issued? A. $140,000 B. $172,000 C. $300,000 D. $252,000

D. $252,000 After Sage issued the additional $200,000 in equity, its total equity was $700,000 ($500,000 + $200,000). Thyme now owns 16,000 shares out of the total 25,000 (20,000 + 5,000) shares outstanding, or 64% (16,000 ÷ 25,000). The remaining 36% (100% − 64%) accounts for a noncontrolling interest of $252,000 ($700,000 × 36%).

Standard Co. spent $10,000,000 on its new software package that is to be used only for internal use. The amount spent is for costs after the application development stage. The economic life of the product is expected to be three years. The equipment on which the package is to be used is being depreciated over five years. What amount of expense should Standard report on its income statement for the first full year? A. $10,000,000 B. $2,000,000 C. $0 D. $3,333,333

D. $3,333,333 Costs incurred to develop software for internal use are capitalized after the application development stage is reached (in accordance with FASB ASC 350-40-35-4). The costs are amortized over the benefited periods—three years in this case.

Norsk Corporation has convertible bonds as part of its capital structure. The bonds have a face value of $100,000 and were issued at $105,417. The stated interest on the bonds is 8%. The amount of premium amortization for the first year was $1,700. If Norsk has an income tax rate of 40%, what is the amount that will be added to the numerator in the computation of diluted earnings per share? A. $8,000 B. $4,800 C. $6,300 D. $3,780

D. $3,780 The amount to be added back is the after-tax amount of the interest expense on the bonds. The annual interest expense on the bonds is equal to the cash payment less the premium amortization, since the bonds were issued at a premium. The annual cash payment is $8,000 ($100,000 × .08). The net annual interest expense is $8,000 - $1,700 = $6,300. The net annual interest expense is multiplied by the tax rate and then that amount is subtracted from the annual interest expense to arrive at after-tax interest expense ($6,300 × .40 = $2,520; $6,300 - $2,520 = $3,780). The amount can be more efficiently computed in the following manner: $6,300 × (1 - .40) = $3,780.

On December 7, 20X5, Pecan Restaurants granted permission to BF Enterprises to operate a Pecan restaurant. The initial franchise fee is $75,000 and BF Enterprises paid $30,000 on the date permission was granted. The remaining three payments of $15,000 to be made in 20X6 have a present value of $36,000. The services to be provided by Pecan Restaurants will begin in 20X6 and their performance obligation will be completed in 20X6, when the restaurant is expected to open. In Pecan's December 31, 20X5, balance sheet, how much unearned franchise revenue should be reported in relation to its agreement with BF Enterprises? A. $36,000 B. $75,000 C. $66,000 D. $30,000

D. $30,000 Pecan has received $30,000 as of the balance sheet date for a performance obligation to provide services in 20X6. Because Pecan will not begin or complete their performance obligations until 20X6, all of the franchise fee received to date via the down payment represents unearned franchise revenue for Pecan. The remaining payments and the upfront $30,000 will be recognized in earnings over the 20X6 year. Thus, the total unearned franchise revenue at December 31, 20X5, is $30,000.

On April 30, Deer approved a plan to dispose of a segment of its business. For the period January 1 through April 30, the segment had revenues of $500,000 and expenses of $800,000. The assets of the segment were sold on October 15, at a loss for which no tax benefit is available. In its income statement for the calendar year, how should Deer report the segment's operations from January 1 to April 30? A. $300,000 reported as a net loss, as part of continuing operations B. $300,000 reported as an extraordinary loss C. $500,000 and $800,000 included with revenues and expenses, respectively, as part of continuing operations D. $300,000 reported as a loss from discontinued operations

D. $300,000 reported as a loss from discontinued operations

On October 1, Year 1, Gold Co. borrowed $900,000 to be repaid in three equal, annual installments. The note payable bears interest at 5% annually. Gold paid the first installment of $300,000 plus interest on September 30, Year 2. What amount should Gold report as a current liability on December 31, Year 2? A. $300,000 B. $303,750 C. $330,000 D. $307,500

D. $307,500 The last payment on September 30, Year 2, included all interest to date; thus, we only need to accrue interest from September 30, Year 2, until December 31, Year 2, which is a total of $7,500 based on principal of $600,000 × 0.05 × 3/12 (last 3 months of the year). So, the total current liability as of December 31, Year 2, is $300,000 + $7,500, for a total of $307,500.

West, Inc., made the following expenditures relating to Product Y: Legal costs to file a patent on Product Y—$10,000. Production of the finished product would not have been undertaken without the patent. Special equipment to be used solely for development of Product Y—$60,000. The equipment has no other use and has an estimated useful life of four years. Labor and material costs incurred in producing a prototype model—$200,000. Cost of testing the prototype—$80,000. What is the total amount of costs that will be expensed when incurred? A. $280,000 B. $350,000 C. $295,000 D. $340,000

D. $340,000 The $10,000 patent cost will be capitalized as an intangible asset in accordance with the provisions of FASB ASC 350-30-25-1. The remaining costs are considered to be research and development costs, as defined in FASB ASC 730-10-55-1. Special developmental equipment and prototypes are addressed in FASB ASC 730-10-55-1. They are considered to be research and development costs which "shall be charged to expense when incurred." Thus the entire $340,000 ($60,000 + $200,000 + $80,000) would be expensed.

Hudson Corp. operates several factories that manufacture medical equipment. The factories have a historical cost of $200 million. Near the end of the company's fiscal year, a change in business climate related to a competitor's innovative products indicated to Hudson's management that the $170 million carrying amount of the assets of one of Hudson's factories may not be recoverable. Management identified cash flows from this factory and estimated that the undiscounted future cash flows over the remaining useful life of the factory would be $150 million. The fair value of the factory's assets is reliably estimated to be $135 million. The change in business climate requires investigation of possible impairment. Which of the following amounts is the impairment loss? A. $20 million B. $15 million C. $65 million D. $35 million

D. $35 million Since the estimated future cash flow of the asset ($150 million) is less than the carrying value of the asset ($170 million), an impairment loss must be recognized. The impairment loss for long-lived assets to be held and used is the excess of the asset's carrying amount over its fair value. Carrying amount $170 millionFair value 135 millionImpairment loss $ 35 million

During the current year, Cooley Co. had an unrealized gain of $100,000 on a debt investment classified as available-for-sale. Cooley's corporate tax rate is 25%. What amount of the gain should be included in Cooley's net income and other comprehensive income at the end of the current year? A. Net income $25,000Other comprehensive income $75,000 B. Net income $100,000Other comprehensive income $0 C. Net income $75,000Other comprehensive income $25,000 D. Net income $0Other comprehensive income $75,000

D. Net income $0Other comprehensive income $75,000

Noting that interest rates are declining, Blue Township opted to retire an existing, callable general obligation bond and replace it with a new bond issue with lower interest. A $4,000,000, 5% bond originally issued at par value with 15 years remaining was retired for $4,100,000. A new $4,000,000, 2%, 30-year bond was issued. The new bond issue was sold at 104 and printing, legal, and administrative costs for the transactions were paid in an amount of $5,000. On the fund financial statements, this refunding would result in: A. $4,100,000 of other financing uses and $4,160,000 of other financing sources to the debt service fund. B. $4,100,000 of expenditures and $4,160,000 of revenues to the debt service fund. C. $4,100,000 of expenditures, $5,000 of other financing uses, and $4,160,000 of revenues to the debt service fund. D. $4,100,000 of other financing uses, $5,000 of expenditures, and $4,160,000 of other financing sources to the debt service fund.

D. $4,100,000 of other financing uses, $5,000 of expenditures, and $4,160,000 of other financing sources to the debt service fund. This transaction is recognizable as a "current" refunding in that the proceeds of new debt are used to repay old debt in its entirety. Usually, the debt service fund is used for refunding transactions. The proceeds of the new debt are not considered revenues and payment of the old debt is not considered expenditures. If underwriter fees or attorney costs had been paid out of the proceeds, the smaller net proceeds would have been recognized as other financing sources.

Financing for the renovation of Fir City's municipal park, begun and completed during 20X1, came from the following sources: Grant from state government $400,000Proceeds from general obligationbond issue 500,000Transfer from Fir's general fund 100,000 In its 20X1 capital projects fund operating statement, Fir should report these amounts as: A. $1,000,000 revenues and $0 other financing sources. B. $900,000 revenues and $100,000 other financing sources. C. $0 revenues and $1,000,000 other financing sources. D. $400,000 revenues and $600,000 other financing sources.

D. $400,000 revenues and $600,000 other financing sources. The financing of the renovation of Fir City's municipal park is accounted for in the capital projects fund of the city. The resources from the state grant would be recorded as revenue. However, the proceeds from the bond issue and the interfund transfer would be classified as "other financing sources." If the grant funds had not yet been used according to the grant guidelines, they would be recognized as deferred revenues, a liability, but not as other financing sources. Revenues are defined as inflows of financial resources from other than issuance of debt, interfund reimbursement, and interfund transfers.

Rory Co.'s prepaid insurance was $50,000 at December 31, Year 2, and $25,000 at December 31, Year 1. Insurance expense was $20,000 for Year 2 and $15,000 for Year 1. What amount of cash disbursements for insurance would be reported in Rory's Year 2 net cash flows from operating activities presented on a direct basis? A. $55,000 B. $20,000 C. $30,000 D. $45,000

D. $45,000 Insurance expense - Year 2 $20,000 Increase in prepaid insurance ($50,000 - $25,000) 25,000 Total $45,000

On December 31, 20X1, Byte Co. had capitalized software costs of $600,000 with an economic life of four years. Sales for 20X2 were 10% of expected total sales of the software. At December 31, 20X2, the software had a net realizable value of $480,000. In its December 31, 20X2, balance sheet, what amount should Byte report as net capitalized cost of computer software? A. $480,000 B. $540,000 C. $432,000 D. $450,000

D. $450,000 When software costs are capitalized, yearly amortization of these costs is based on the greater of the ratio of current sales to expected total sales or the straight-line method over the useful life of the asset (four years). Sales ratio: 10% (0.10) × $600,000 = $60,000 Straight-line: 25% (0.25) × $600,000 = $150,000 Since straight-line amortization is larger and is used, the remaining capitalized cost is $600,000 less $150,000, or $450,000. Since the net realizable value, $480,000, is greater than the $450,000, there is no need for an additional write-off.

On March 1, 20X1, Rya Corp. issued 1,000 shares of its $20 par value common stock and 2,000 shares of its $20 par value convertible preferred stock for a total of $80,000. At this date, Rya's common stock was selling for $36 per share, and the convertible preferred stock was selling for $27 per share. What amount of the proceeds should be allocated to Rya's convertible preferred stock? A. $60,000 B. $44,000 C. $54,000 D. $48,000

D. $48,000 Incorrect This is a basket purchase transaction with one total price paid for two items. The proceeds must be allocated based on the relative fair values of each item separately. Type Market TotalStock Shares x Price = Market = Ratio x Proceeds = AllocationCommon 1,000 x $36 = $36,000 = 40% x $80,000 = $32,000$90,000*Preferred 2,000 x $27 = $54,000 = 60% x $80,000 = $48,000$90,000

Zarr Town levied property taxes of $500,000, of which 1% is expected to be uncollectible. During the year, Zarr Town collected $450,000. What amount of property tax revenue should Zarr Town report in its government-wide statement of activities for the current year? A. $500,000 B. $450,000 C. $0 D. $495,000

D. $495,000 Governmental fund revenues, including taxes such as property taxes, are recorded net of estimated uncollectible taxes. Zarr Town has levied $500,000 of taxes, of which 1% are deemed uncollectible. Therefore, Zarr can recognize $495,000 of property tax revenue ($500,000 × (1 − .01)), the amount it expects to collect, even though it has currently only collected $450,000.

Elm City issued a purchase order for supplies with an estimated cost of $5,000. When the supplies were received, the accompanying invoice indicated an actual price of $4,950. What amount should Elm debit (credit) to the reserve for encumbrances after the supplies and invoice were received? A. $(50) B. $50 C. $4,950 D. $5,000

D. $5,000 When the purchase order is approved by Elm City, the estimated amount is recorded in the journal entry: Encumbrances 5,000Fund Balance--Reservedfor Encumbrances 5,000 When the purchase order is filled for $4,950, the entry is reversed, for the original estimated amount of the purchase order: Fund Balance--Reservedfor Encumbrances 5,000Encumbrances 5,000

Leaf Co. purchased from Oak Co. a $20,000, 8%, 5-year note that required five equal annual year-end payments of $5,009. The note was discounted to yield a 9% rate to Leaf. At the date of purchase, Leaf recorded the note at its present value of $19,483. What should be the total interest revenue earned by Leaf over the life of this note? A. $5,045 B. $8,000 C. $9,000 D. $5,562

D. $5,562 The total amount of interest revenue one earns on a note is related to the total payments but also to the present value of the note, with the discount recognized here initially. The total amount to be received on this note is 5 years multiplied by $5,009, as specified, for a total of $25,045. Interest is generally the amount returned over and above the amount originally recognized, which was $19,483 originally. Thus, the total interest revenue is $25,045 − $19,483, or $5,562. (Alternatively, a payment amortization schedule could be utilized.)

Foy Corp. failed to accrue warranty costs of $50,000 in its December 31, 20X1, financial statements. In addition, a change from straight-line to accelerated depreciation made at the beginning of 20X2 resulted in a cumulative effect of $30,000 on Foy's retained earnings. Both the $50,000 and the $30,000 are net of related income taxes. What amount should Foy report as prior-period adjustments in 20X2? A. $80,000 B. $0 C. $30,000 D. $50,000

D. $50,000 FASB ASC 250-10-45-22 notes, "Any error in the financial statements of a prior period discovered after the financial statements are issued or are available to be issued (as discussed in Section 855-10-25) shall be reported as an error correction, by restating the prior-period financial statements." Foy Corp.'s failure to accrue $50,000 of warranty cost in 20X1 is an error which should be reported in 20X2 as a prior-period adjustment. (The change in depreciation is a change in accounting principle, shown on the income statement.)

Delar Co. completed its year-end physical count of inventory. The inventory was valued at first-in, first-out (FIFO) costs and totaled $500,000. Delar subsequently noted the following two items: 1,000 units of inventory with a FIFO cost of $10 each were shipped and billed to a customer, FOB destination. These items were included in the physical count. 6,000 units at a FIFO cost of $5 each were held on consignment for one of its suppliers, but were excluded from the physical count. What amount should Delar report as inventory at year-end? A. $530,000 B. $490,000 C. $520,000 D. $500,000

D. $500,000 correct FOB destination would not be included nor the items held on consignment

On August 1, 20X7, Remy signed a two-year contract to provide house painting services on an as-needed basis to Cox Homebuilding Inc., with the contract to start immediately. Cox agreed to pay Remy $2,400 for the two-year period. Payment is not scheduled to occur until completion of the contract, in 20X9. Remy should recognize revenue in 20X7 in the amount of: A. $2,400. B. $1,200. C. $0. D. $500.

D. $500. Revenue for long-term contracts is recognized over time when any one of the following criteria is met: the customer simultaneously receives and consumes the benefits provided by the entity's performance as the entity performs; the entity's performance creates or enhances an asset, such as work-in-process (WIP), that the customer controls as the asset is created or enhanced; or the entity's performance does not create an asset with an alternative use to the entity, and the entity has an enforceable right to payment for performance completed to date. This arrangement qualifies for revenue recognition over time because the customer consumes the benefit of the seller's service as the seller provides it. Therefore, Remy would recognize revenue of $500 ($2,400 × 5/24 of the contract duration) in 20X7.

During 20X1, Kam Co. began offering its goods to selected retailers on a consignment basis. The following information was derived from Kam's 20X1 accounting records: Beginning inventory $122,000Purchases 540,000Freight-in 10,000Transportation to consignees 5,000Freight-out 35,000Ending inventoryHeld by Kam 145,000Held by consignees 20,000 In its 20X1 income statement, what amount should Kam report as cost of goods sold? A. $547,000 B. $527,000 C. $507,000 D. $512,000

D. $512,000 Cost of goods sold would include all inventoriable costs less ending inventory. Beginning inventory $122,000Purchases 540,000Freight-in 10,000Cost of goods acquired 672,000Add transportation to consignees 5,000Cost of goods available $677,000Less ending inventoryHeld by Kam $145,000Held by consignees 20,000 165,000Cost of goods sold $512,000

Hutch, Inc., uses the conventional retail inventory method to account for inventory. The following information relates to current-year operations: Average Cost Retail Beginning inventory and purchases $600,000 $920,000 Net markups 40,000 Net markdowns 60,000 Sales 780,000 What amount should be reported as cost of sales for the current year? A. $480,000 B. $487,500 C. $520,000 D. $525,000

D. $525,000 600000/(920000+40000)= .625 960000-60000-780000=120000 120000*.625=75000 600000-75000=525000

Walt Co. adopted the dollar-value LIFO inventory method as of January 1, 20X1, when its inventory was valued at $500,000. Walt's entire inventory constitutes a single pool. Using a relevant price index of 1.10, Walt determined that its December 31, 20X1, inventory was $577,500 at current-year cost, and $525,000 at base-year cost. What was Walt's dollar-value LIFO inventory on December 31, 20X1? A. $525,000 B. $577,500 C. $552,500 D. $527,500

D. $527,500 When applying dollar-value LIFO (last in, first out), one must first compute the ending inventory total using base-year prices, to tell how much the total inventory (at base-year prices) has increased or decreased. If this is an increase, then add the increase amount (multiplied by the year's price level) to the beginning inventory total.

Glass Co. had net income of $70,000 during the year. Depreciation expense was $10,000. The following information is available: Accounts receivable increase $20,000 Equipment gain on sale (sale price $100,000) 10,000 increase Nontrade notes payable increase 50,000 Equipment purchases 40,000 increase Accounts payable increase 30,000 What amount should Glass report as net cash provided by investing activities in its statement of cash flows for the year? A. $(40,000) B. $10,000 C. $50,000 D. $60,000

D. $60,000 Cash flows from investing activities involve the use and receipt related to nonoperating assets (i.e., property, plant, and equipment; equity and debt securities; notes receivable; etc.). Cash provided by investing activities is found in an identical manner under the direct or indirect method of preparing the statement of cash flows. In this example, only two items would be classified as investing activities: the equipment purchase and equipment sale. The sale resulted in a $100,000 cash inflow and the purchase resulted in a $40,000 cash outflow, for a net cash provided amount $60,000 ($100,000 inflow − $40,000 outflow). The change in accounts receivable and accounts payable balances would be captured in the operating section of the statement of cash flows. The change in notes payable would be captured in the financing section.

At June 30, Almond Co.'s cash balance was $10,012 before adjustments, while its ending bank statement balance was $10,772. Check number 101 was issued June 2 in the amount of $95, but was erroneously recorded in Almond's general ledger balance as $59. The check was correctly listed in the bank statement at $95. The bank statement also included a credit memo for interest earned in the amount of $35, and a debit memo for monthly service charges in the amount of $50. What was Almond's adjusted cash balance at June 30? A. $10,462 B. $9,598 C. $10,048 D. $9,961

D. $9,961 Book balance: 10,012 Interest: 35 Service: 50 Check error: (36) Adjusted balance =9961

Gottfredsen and Sibbett are partners with capital balances of $120,000 and $80,000, respectively. Profits and losses are divided in the ratio of 60:40. Gottfredsen and Sibbett decided to form a new partnership with Tolman, who invested land valued at $40,000 for a 25% capital interest in the new partnership. Tolman's cost of the land was $34,000. The partnership elected to use the bonus method to record the admission of Tolman into the partnership. Tolman's capital account should be credited for: A. $40,000. B. $50,000. C. $34,000. D. $60,000.

D. $60,000. In the bonus-to-new-partner method, one needs to compute the total value now for the partnership, whereupon the new partner's capital account will be based on the new partner's percentage of the total just computed (even if the new partner's contribution does not seem to justify the amount). Total capital of new partnership($120,000 + $80,000 + $40,000) $240,000Times capital credit percentage to Tolman × 0.25Equals capital credit allowed to Tolman $ 60,000 Note: Tolman is given a "bonus" equal to $20,000, the excess of his $60,000 capital credit over the $40,000 fair value of land invested.

Pardelle, Inc. acquired 80% of Soran Co.'s outstanding common stock on December 31, year 1. Pardelle's retained earnings total $600,000 and Soran's retained earnings total $400,000 on the acquisition date. What amount should be reported for consolidated retained earnings in the consolidated statement of financial position on the acquisition date? A. $920,000 B. $1,000,000 C. $680,000 D. $600,000

D. $600,000 When a company obtains control of a subsidiary, typically evidenced by greater than 50% ownership, only the parent (Pardelle) company's retained earnings are carried through in consolidation. The subsidiary's (Soran) retained earnings are eliminated in consolidation and the 20% Pardelle does not own would be eliminated and reallocated to the noncontrolling interest. Pardelle has retained earnings of $600,000, which equals consolidated retained earnings at year-end after Soran's retained earnings are eliminated.

During the current year, Beta Motor Co. incurred the following costs related to a new solar-powered car: Salaries of laboratory employees researching how to build the new car $250,000 Legal fees for the patent application for the new car 20,000 Engineering follow-up during the early stages of commercial production (the follow-up occurred during the current year) 50,000 Marketing research to promote the new car 30,000 Design, testing, and construction of a prototype 400,000 What amount should Beta Motor report as research and development expense in its income statement for the current year? A. $720,000 B. $250,000 C. $750,000 D. $650,000

D. $650,000 Research and development is defined as a planned search aimed at discovery of new knowledge and translation of the research findings into a plan or design. The research and development costs are incurred prior to commercial production of the product. R&D costs are reported as an expense, not an asset. Salaries $250,000 + Design $400,000 = Total $650,000

Riverview Co. has a stock portfolio in equity securities currently valued at $7,000. Its cost was $7,200. Prior to the current-year adjustment, the Fair Value Adjustment account has a debit balance of $500. Which of the following is the correct adjustment for the current year? A, $200 debit to the Fair Value Adjustment account B. $900 credit to the Fair Value Adjustment account C. $300 credit to the Fair Value Adjustment account D. $700 credit to the Fair Value Adjustment account

D. $700 credit to the Fair Value Adjustment account Prior to any current year adjustment, the carrying amount is $7,700 (cost of $7,200 increased by the Fair Value Adjustment of $500). In order to reflect a year-end fair value of $7,000, the Fair Value Adjustment account must be a credit of $700.

Ralph entered into a contract offering variable consideration; the contract pays $600/month for six months of identity theft monitoring services. There is an 80% chance the contract will pay an additional $1,200 and a 50% chance the contract will pay an additional $1,800, depending on the outcome of the monitoring contract. Ralph recognizes the contract revenue over time. If Ralph estimates variable consideration as the most likely amount, what amount of revenue would Ralph recognize for the second month of the contract? A. $760 B. $910 C. $600 D. $800

D. $800 The amount of consideration in a contract may vary due to discounts, rebates, refunds, credits, price concessions, incentives, performance bonuses, penalties, or other similar items, or if an entity's entitlement to the consideration is contingent on the occurrence (or nonoccurrence) of a future event. The amount of variable consideration should be estimated by using either of the following methods: a. Expected value method: The sum of probability-weighted amounts in a range of possible consideration amounts. This method is appropriate when an entity has a large number of contracts with similar characteristics. b. Most likely amount method: The single most likely amount in a range of possible consideration amounts; the single most likely outcome of the contract. This method is appropriate when the contract has only two possible outcomes: an entity either achieves a performance bonus or does not. The most likely outcome is that Ralph receives the $1,200 bonus (likelihood = 80%), in which case Ralph would be paid a total of $3,600 ($600 × 6 months) + $1,200, or $4,800. Therefore, Ralph would recognize $4,800 ÷ 6 months ($800) each month.

Crossroads Co. chooses to report a financial asset at its fair value. The asset trades in two different markets; however, neither market is the principal market for the financial asset. In the first market, sales proceeds are $76, which is net of transaction costs of $6. In the second market, the sales proceeds are $80, which is net of transaction costs of $1. What amount should Crossroads report as the fair value of the asset? A. $82 B. $80 C. $76 D. $81

D. $81 Fair value measurement assumes that the transaction to sell the asset or transfer the liability occurs in the principal market for the asset or liability. If there is no principal market, the entity should use the most advantageous market for that asset or liability. The FASB defines the most advantageous market as "the market that maximizes the amount that would be received to sell the asset or minimizes the amount that would be paid to transfer the liability, after taking into account transaction costs and transportation costs" (FASB ASC 820-10-20). The price used to measure fair value should not be adjusted for transaction costs, which are the incremental direct costs to sell the asset or transfer the liability. In other words, in determining the most advantageous market, adjust for transaction costs. But to compute the actual fair value amount, ignore the transaction costs. The first market yields $76 and the second market yields $80, so the second market is the most advantageous. The fair value in the second market is $81 ($80 with the $1 added back).

Dayne County's general fund had the following disbursements during the year: Payment of principal on long-term debt $100,000Payments to vendors 500,000Purchase of a computer 300,000 What amount should Dayne County report as expenditures in its governmental funds statement of revenues, expenditures, and changes in fund balances? A. $500,000 B. $300,000 C. $800,000 D. $900,000

D. $900,000 Because the governmental funds have the measurement focus of current financial resources, all three disbursements ($900,000) would be reported as expenditures. Because governmental funds do not account for noncurrent assets or liabilities, the purchase of equipment ($300,000) and the payment of principal on long-term debt ($100,000) are fund expenditures, as well as the payments to vendors ($500,000).

A local government established a new special revenue fund during the current year. The fund incurred the following transactions: Purchased new machines: 5-year life $140,000 Paid interest debt 300,000 Paid debt principal 200,000 Paid management salaries 100,000 Purchased office equipment: 4-year life 100,000 Paid utilities 50,000 Purchased office supplies (1/2 used up) 20,000 If the unit expects the assets to have no salvage value, what amount would be recognized as expenditures for the current year? A. $513,000 B. $450,000 C. $713,000 D. $910,000

D. $910,000 All of the items listed represented actual outgoing cash payments (i.e., expenditures) during the year. The total is $910,000. The salvage value of the asset or any depreciation (not an expenditure) are irrelevant to this problem.

The following information is relevant to the computation of Chan Co.'s earnings per share to be disclosed on Chan's income statement for the year ending December 31: Net income for the year is $600,000. $5,000,000 face value 10-year convertible bonds outstanding on January 1. The bonds were issued four years ago at a discount that is being amortized in the amount of $20,000 per year. The stated rate of interest on the bonds is 9%, and the bonds were issued to yield 10%. Each $1,000 bond is convertible into 20 shares of Chan's common stock. Chan's corporate income tax rate is 25%. Chan has no preferred stock outstanding and no other convertible securities. What amount should be used as the numerator in the fraction used to compute Chan's diluted earnings per share assuming that the bonds are dilutive securities? A. $247,500 B. $130,000 C. $1,070,000 D. $952,500

D. $952,500 The numerator will be Net income + Interest (net of tax). Net income $600,000 Interest: Cash ($5,000,000 x .09) = $450,000 Discount amortization 20,000 Tax ($470,000 x .25) = (117,500) SUBTOTAL 352,500 Numerator $952,500

The following data pertain to Cowl, Inc., for the year ending December 31, 20X1: Net sales $ 600,000Net income 150,000Total assets (January 1, 20X1) 2,000,000Total assets (December 31, 20X1) 3,000,000 What was Cowl's rate of return on assets for 20X1? A. 5% B. 20% C. 24% D. 6%

D. 6% The return on assets ratio relates net income to average total assets. Rate of return on assets = Net income / Average assets = $150,000 / .5 ($2,000,000 + $3,000,000) = $150,000 / .5 ($5,000,000) = $150,000 / $2,500,000 = .06= 6%

Bee Co. uses the direct write-off method to account for uncollectible accounts receivable. During an accounting period, Bee's cash collections from customers equal sales adjusted for the addition or deduction of the following amounts: A. Accounts written off: addition; Increase in accounts receivable balance: addition B. Accounts written off: addition; Increase in accounts receivable balance: deduction C. Accounts written off: deduction; Increase in accounts receivable balance: addition D. Accounts written off: deduction; Increase in accounts receivable balance: deduction

D. Accounts written off: deduction; Increase in accounts receivable balance: deduction This question is asking to convert from sales during the period to cash collections. Since the company uses the direct write-off method, there is no need to consider any allowance account balance. Thus, the sales for the period only have to be adjusted downwards for any accounts written off, and also downwards for any increases in deferred receipts (in the form of additions to accounts receivables).

Which of the following is included on a statement of changes in equity? A. The impact of the transactions on the number of shares of stock, if any, is presented in the descriptions to the left. B. Column headings identify individual stockholders' equity accounts. C. Events changing stockholders' equity accounts are listed chronologically to the left. D. All of the items listed are included on a statement of changes in equity.

D. All of the items listed are included on a statement of changes in equity.

A financial instrument that embodies an unconditional obligation that the issuer must or may settle by issuing a variable number of its equity shares must be classified as a liability if, at inception, the monetary value of the obligation is based solely or predominantly on which of the following:? A. Variations in something other than the fair value of the issuer's equity shares B. Variations inversely related to changes in the fair value of the issuer's equity shares C. A fixed monetary amount known at inception D. Any one of the conditions listed

D. Any one of the conditions listed

During 20X1, Brad Co. issued 5,000 shares of $100 par convertible preferred stock for $110 per share. One share of preferred stock can be converted into three shares of Brad's $25 par common stock at the option of the preferred shareholder. On December 31, 20X2, when the market value of the common stock was $40 per share, all of the preferred stock was converted. What amount should Brad credit to common stock and to additional paid-in capital common stock as a result of the conversion? A. Common stock: $600,000; Additional paid-in capital: $0 B. Common stock: $375,000; Additional paid-in capital: $225,000 C. Common stock: $500,000; Additional paid-in capital: $50,000 D. Common stock: $375,000; Additional paid-in capital: $175,000

D. Common stock: $375,000; Additional paid-in capital: $175,000 Summary journal entry to record 20X1 issuance of convertible preferred stock: Dr. Cr.Cash (5,000 x $110) 550,000Convertible preferredstock (5,000 x $100) 500,000Additional paid-in capitalpreferred ($550,000 - $500,000) 50,000 Summary journal entry to record 12/31/X2 conversion of preferred shares: Dr. Cr.Convertible preferred stock 500,000Additional paid-in capital preferred 50,000Common stock (5,000 x 3 x $25) 375,000Additional paid-in capitalcommon ($550,000 - $375,000) 175,000

During 20X1, Smith Co. filed suit against West, Inc., seeking damages for patent infringement. On December 31, 20X1, Smith's legal counsel believed that it was probable that Smith would be successful against West for an estimated amount in the range of $75,000 to $150,000, with all amounts in the range considered equally likely. In March 20X2, Smith was awarded $100,000 and received full payment thereof. In its 20X1 financial statements, issued in February 20X2, how should this award be reported? A. As a disclosure of a contingent gain of $100,000 B. As a receivable and deferred revenue of $100,000 C. As a receivable and revenue of $100,000 D. As a disclosure of a contingent gain of an undetermined amount in the range of $75,000 to $150,000

D. As a disclosure of a contingent gain of an undetermined amount in the range of $75,000 to $150,000 According to FASB ASC 450-30, gain contingencies should not be reflected in the accounts but should be given adequate disclosure. FASB ASC 450-30 reads: "A contingency that might result in a gain usually should not be reflected in the financial statements because to do so might be to recognize revenue before its realization." (FASB ASC 450-30-25-1) "Adequate disclosure shall be made of contingencies that might result in gains, but care shall be exercised to avoid misleading implications as to the likelihood of realization." (FASB ASC 450-30-50-1) Smith Co.'s financial statements should include a disclosure of a contingent gain with a range of $75,000 to $150,000.

Band Co. uses the equity method to account for its investment in Guard, Inc., common stock. How should Band record a 2% stock dividend received from Guard? A. As a dividend revenue at Guard's carrying value of the stock B. As dividend revenue at the market value of the stock C. As a reduction in the total cost of Guard stock owned D. As a memorandum entry reducing the unit cost of all Guard stock owned

D. As a memorandum entry reducing the unit cost of all Guard stock owned A company using the equity method to account for an investment does not recognize dividends received as revenue. When a cash dividend is received, the receipt of cash is treated as a liquidation of the investment and the carrying amount of the investment is reduced by the amount of the dividend. However, when additional stock shares are received in lieu of cash, no liquidation of the investment has occurred. Instead, the investment carrying value now applies to a larger number of shares held by the investor. Therefore, the investor needs only to note that the value per share of its investment has decreased and the number of shares has increased.

Which characteristic of information in the statistical tables included in an annual comprehensive financial report differentiates it from information contained in the financial statements? A. Fiscal years B. Neither data covering financial trends nor fiscal years C. Data covering financial trends D. Both data covering financial trends and fiscal years

D. Both data covering financial trends and fiscal years An annual comprehensive financial report (ACFR) includes an introductory section, financial section, and statistical section. The financial section includes the financial statements, auditor's report, and required supplementary information for the financial statements prepared for the required fiscal years. The statistical section includes financial, economic, and demographic information and their associated trends over the fiscal years presented in the financial statements and data not confined to the fiscal-year approach required of the financial statements.

During 20X2, Solomon Co. purchased equipment for cash of $128,000, and sold equipment with a $38,000 carrying value for a loss of $14,000. How should these transactions be reported in Solomon's 20X2 statement of cash flows? A. Cash outflow of $104,000 B. Cash outflow of $142,000 C. Cash outflow of $152,000 D. Cash inflow of $24,000 and cash outflow of $128,000

D. Cash inflow of $24,000 and cash outflow of $128,000 Cash inflows are not to be netted against cash outflows. There is a cash outflow of $128,000 to purchase equipment given directly in the problem. The cash inflow must be determined from the facts as given. A loss on disposal indicates that the cash received is lower than the carrying value.

On December 31, a company has the following bank accounts and corresponding cash balances: California Bank Operating - Summit Ridge ($400,000) Operating - Bakersville 300,000 Operating - Smithville 50,000 Savings 500,000 Sedona Bank Checking ($375,000) How should the company report the above bank account balances in the balance sheet at December 31? A. Cash of $800,000 and a liability of $725,000 B. Cash of $850,000 and a liability of $775,000 C. Cash of $75,000 D. Cash of $450,000 and a liability of $375,000

D. Cash of $450,000 and a liability of $375,000 Cash bank accounts from the same bank can generally be offset so that accounts that are overdrawn can be netted against accounts with natural positive balances. In this problem the total amount of cash from the California bank is $450,000 ($500,000 + $50,000 + $300,000 - $400,000). This net positive amount cannot be offset or combined with the balances in other banks, such as Sedona Bank. Sedona Bank has an overdrawn balance of $375,000, representing a liability to the company. Negative cash balances are liabilities because companies must repay the bank the overdrawn balance. Therefore, the bank account balances would be cash of $450,000 and a liability of $375,000.

ASU 2016-14 stipulates a number of disclosures required for underwater endowment funds. Which of the following is not one of the required disclosures? A. The aggregate amount of the fair value of underwater endowment funds B. Total original endowment gifts or levels required to be maintained by donor stipulation or law C. The NFP's policy, and any actions taken during the period, concerning the appropriation of underwater endowment funds D. Comparison of underfunded versus overfunded endowment funds

D. Comparison of underfunded versus overfunded endowment funds

Sun Co. is a wholly owned subsidiary of Star Co. Both companies have separate general ledgers and prepare separate financial statements. Sun requires stand-alone financial statements. Which of the following statements is correct? A. After consolidation, the accounts of both Star and Sun should be combined together into one general-ledger accounting system for future ease in reporting. B. Consolidated financial statements should be prepared for both Star and Sun. C. After consolidation, the accounts of both Star and Sun should be changed to reflect the consolidated totals for future ease in reporting. D. Consolidated financial statements (including the financial information of both corporations) should only be prepared by Star and not by Sun.

D. Consolidated financial statements (including the financial information of both corporations) should only be prepared by Star and not by Sun. A company that controls another company must prepare consolidated financial statements. Star owns 100% of Sun and must consolidate the financial statements of the two companies.

In year 1, a donor promised to give $100,000 to a nongovernmental, not-for-profit kitchen if it provides 20,000 meals by March 31, year 2. At the end of year 1, the kitchen had provided 20,000 meals. In which line item, if any, should the contribution be reported in the kitchen's statement of financial position at the end of year 1? A. Cash B. The contribution should not be reported in the statement of financial position. C. Deferred revenue D. Contributions receivable

D. Contributions receivable The statement of financial position presents the assets and liabilities of a not-for-profit entity. At year-end this contribution would be earned as the kitchen has completed their performance obligation, removing deferred revenue as an option. The answer choice "cash" is not correct as the kitchen has not received the cash yet. Because they have earned the revenue and have yet to receive it, the appropriate classification is contributions receivable as they have earned the right to receive $100,000 through providing 20,000 meals.

Generally, which inventory costing method approximates most closely the current cost for each of the following? A. Cost of goods sold: LIFO; Ending inventory: LIFO B. Cost of goods sold: FIFO; Ending inventory: FIFO C. Cost of goods sold: FIFO; Ending inventory: LIFO D. Cost of goods sold: LIFO; Ending inventory: FIFO

D. Cost of goods sold: LIFO; Ending inventory: FIFO Because LIFO (last in, first out) counts the most recent purchases as sold items, its cost of goods sold will be closest to current costs. Since FIFO (first in, first out) counts the most recent purchases as still in inventory, the ending inventory under FIFO will be closest to current costs.

How would a municipality that uses modified accrual and encumbrance accounting record the transaction of an interfund billing in the internal service accounts? A. Debit deferred revenues B. Credit deferred revenues C. Credit other financing sources D. Credit interfund revenues

D. Credit interfund revenues Modified accrual and encumbrance accounting are features of governmental fund accounting. Internal service funds, one of the two kinds of enterprise funds that use accrual accounting, account for those activities that one department within a government undertakes for the benefit of (1) other departments within that same government (usual case) and (2) (sometimes) other governments, at predetermined prices. Billings to departments within the same government are considered "interfund services provided and used." Thus, the normal, ongoing activities of an internal service fund result in interfund/intergovernmental receivables (debits) and operating revenues (credits). The receivable is often recorded as a "due from XXXX fund." The operating revenues may be subclassified, as in the case of this problem, as "interfund" or "intergovernmental."

Which of the following should be reported as a stockholders' equity contra account? A. Discount on convertible bonds that are common stock equivalents B. Premium on convertible bonds that are common stock equivalents C. Organization costs D. Cumulative foreign exchange translation loss

D. Cumulative foreign exchange translation loss FASB ASC 830-30-45-12 requires that foreign currency translation adjustments "not be included in determining net income but shall be reported separately and accumulated in a separate component of equity." If these cumulative adjustments netted out to a loss, this loss would effectively be reported as a stockholders' equity contra account. Discounts or premiums on convertible bonds are reported as contra accounts to the bond liability. Organization costs are an amortizable asset.

Jole Co. lent $10,000 to a major supplier in exchange for a noninterest-bearing note due in three years and a contract to purchase a fixed amount of merchandise from the supplier at a 10% discount from prevailing market prices over the next three years. The market rate for a note of this type is 10%. On issuing the note, Jole should record: A. Discount on note receivable: No; Prepaid asset: Yes B. Discount on note receivable: Yes; Prepaid asset: No C. Discount on note receivable: No; Prepaid asset: No D. Discount on note receivable: Yes; Prepaid asset: Yes

D. Discount on note receivable: Yes; Prepaid asset: Yes If a noninterest-bearing note due in three years is issued in connection with a contract to purchase merchandise, a discount on notes receivable for the imputed interest should be established as a valuation account and amortized over the 3-year period using the effective interest method. The contract to purchase should be shown as a prepaid asset (prepaid purchases) for the amount of the 10% discount on the fixed amount of purchases. The deferred charge will be written off in the ratio of purchases made to the fixed amount of merchandise required to be purchased.

Which of the following resources increases the net assets with donor restrictions of a nongovernmental, not-for-profit voluntary health and welfare entity? A. Membership fees to fund general operations B. Refundable advances for purchasing playground equipment C. Participants' deposits for an entity-sponsored trip D. Donor contributions to fund a resident camp program

D. Donor contributions to fund a resident camp program Of the answer choices presented, only the donor contributions for a specific purpose, to fund a resident camp program, would increase the net assets with donor restrictions. The refundable advances represent an asset of the organization as it had given cash to an equipment provider before the purchase had taken place. The participants' deposits represent a receipt of cash and a liability to provide the promised activity for participants, a trip. Membership fees are exchange revenues considered unrestricted.

Encumbrances would not appear in which fund? A. General B. Special revenue C. Capital projects D. Enterprise

D. Enterprise

King City Council will be establishing a library fund. Library fees are expected to cover 55% of the library's annual resource requirements. King has decided that an annual determination of net income is desirable in order to maintain management control and accountability over the library. What type of fund should King establish in order to meet their measurement objectives? A. Special revenue fund B. Internal service fund C. General fund D. Enterprise fun

D. Enterprise fun Enterprise funds are used to report any activities for which a fee is charged, and 55% of the library's needs are covered by fees.

Which of the following accounts would appear in the plant fund of a not-for-profit private college? A. Neither Fuel inventory for power plant nor Equipment B. Fuel inventory for power plant C. Both Fuel inventory for power plant and Equipment D. Equipment

D. Equipment

When Rolan County adopted its budget for the current year ending June 30, $20,000,000 was recorded for estimated revenues control. Actual revenues for the fiscal year amounted to $17,000,000. In closing the budgetary accounts at June 30: A. Revenues Control should be debited for $3,000,000. B. Estimated Revenues Control should be debited for $3,000,000. C. Revenues Control should be credited for $20,000,000. D. Estimated Revenues Control should be credited for $20,000,000.

D. Estimated Revenues Control should be credited for $20,000,000. As the amounts were estimated in the beginning, it is proper to reverse the estimate and to record the actual revenue. The estimated revenue control acts as a contra account to the estimated revenues actually credited. To reverse the control account, a revenue contra account, would be to credit the account for the full balance and then properly record the correct actual amount.

A company's research department incurred $1,000,000 in material, labor, and overhead costs to construct a prototype of a new product and $100,000 to test and modify the prototype. Which of the following statements correctly describes the accounting treatment of prototype costs incurred by the company? A. Capitalize $1,000,000 and amortize it over the life of the prototype and expense $100,000 as incurred. B. Capitalize $1,100,000 and amortize it over the life of the prototype. C. Capitalize $1,100,000 and amortize it over the expected sales life of the new product. D. Expense $1,100,000 as incurred.

D. Expense $1,100,000 as incurred.

Which of the following expenses related to the business combination should be included, in total, in the determination of net income of the combined corporation for the period in which the expenses are incurred? A. Fees of finders and consultants: Yes; Issuance fees for equity securities issues: Yes B. Fees of finders and consultants: No; Issuance fees for equity securities issues: No C. Fees of finders and consultants: No; Issuance fees for equity securities issues: Yes D. Fees of finders and consultants: Yes; Issuance fees for equity securities issues: No

D. Fees of finders and consultants: Yes; Issuance fees for equity securities issues: No Business combinations accounted for as an acquisition should treat expenses related to the combination as follows: Out-of-pocket costs such as fees of finders and consultants are expensed. Issuance costs such as SEC filing fees are charged to the paid-in-capital account.

At the end of the year, Town City's general fund included, among the assets, an inventory of office supplies worth $1,500 and prepaid fire insurance covering the first two months of the subsequent year of $600. As a result of these assets, the fund balance presented in the balance sheet of the general fund would include which of the following? A. Fund balance—nonspendable: $1,500; Fund balance—committed: $600 B. Fund balance—restricted: $2,100 C. Fund balance—assigned: $1,500; Fund balance—restricted: $600 D. Fund balance—nonspendable: $2,100

D. Fund balance—nonspendable: $2,100 The portion of fund balance that reflects equity for amounts that cannot be spent because they are not in spendable form would be termed "nonspendable." Both supplies inventory and prepaid expenses are not spendable. Committed fund balance would result from an action of the highest level of the government and assigned fund balance would reflect action by a government designee. Restrictions of fund balance would reflect specific purposes externally imposed by creditors, grantors, or enabling legislation.

Which of the following items is not subject to the application of intraperiod income tax allocation? A. Prior-period adjustments B. Discontinued operations C. Income from continuing operations D. Gross profit

D. Gross profit Gross profit is a subtotal well before income tax expense. Income tax expenses during the period are specifically allocated to the other three answer choices (discontinued operations, income from continuing operations, and prior-period adjustments). The following items are subject to the application of intraperiod income tax allocation: Discontinued operations Cumulative effects of accounting changes Prior-period adjustments Direct adjustments to capital accounts

Metro General is a municipally owned and operated hospital and a component unit of Metro City. Where should the information for Metro General be presented in the government-wide financial statements? A. It should not appear in the government-wide financial statements. B. In the business-type activities column C. In the governmental activities column D. In the component units column after the primary government totals

D. In the component units column after the primary government totals Data of discretely presented component units should be displayed in the government-wide statement of activities and statement of net position in columns to the right or after the primary government information. The focus of the government-wide financial statements should be on the primary government. Separate columns distinguish the primary government and its discretely presented component units. Discretely presented component units are not in substance part of the primary government. This contrasts with the presentation of blended component unit data, which is included within the primary government information because blended component units are, in substance, part of the primary government.

Which of the following is considered part of the required supplementary information for general purpose external financial reporting of a local government? A. Combining nonmajor fund statement B. Fund financial statement C. Notes to the financial statements D. Management discussion

D. Management discussion GASB Statement 34 requires governments to present the management's discussion and analysis (MD&A) section as required supplementary information (RSI); since the purpose of MD&A is to introduce the basic financial statements and provide an analytical overview of the government's financial activities, it precedes the basic financial statements in the ACFR (annual comprehensive financial report). In addition to MD&A, this statement requires budgetary comparison schedules to be presented as RSI and also requires RSI for governments that use the modified approach for reporting infrastructure assets.

In a company's notes to its financial statements, the first note described significant changes in accounting policies related to valuations of inventory and plant assets. Subsequent notes included a separate note detailing inventories and a separate note detailing plant assets. For which of these subsequent notes, if any, should the company duplicate a description of its changes to significant accounting policies? A. The plant assets note, but not the inventory note B. The inventory note, but not the plant assets note C. Both the plant assets note and the inventory note D. Neither the inventory note nor the plant assets note

D. Neither the inventory note nor the plant assets note The summary of significant accounting policies receives its own note at the beginning of the financial statements. It is admissible to cross-reference the significant accounting policy notes in other sections. However, the description itself should not be duplicated across notes to avoid redundancy. Study Hint: You are told both the inventory and plant assets experienced a policy change. Therefore, you can immediately eliminate the options that only one of the two listed notes would experience a change. That leaves you with the choice of "both" or "neither."

Janna Association, a nongovernmental not-for-profit entity, received a cash gift with the stipulation that the principal be held for at least 20 years. How should the cash gift be recorded? A. An advance B. Net assets without donor restrictions C. A temporary liability D. Net assets with donor restrictions

D. Net assets with donor restrictions

In 20X1, Wildlife Rescue, a not-for-profit entity that works with injured wild animals, received donations of $5,200 for supplies. In 20X2, $600 was spent for this purpose and another $2,100 was spent and all supplies used by the end of 20X3. What should Wildlife Rescue report in the statement of financial position for 20X3 regarding the donation? A. Net assets without donor restrictions $2,700, net assets with donor restrictions $2,500 B. Net assets without donor restrictions $0, net assets with donor restrictions $0 C. Net assets without donor restrictions $600, net assets with donor restrictions $2,100 D. Net assets without donor restrictions $0, net assets with donor restrictions $2,500

D. Net assets without donor restrictions $0, net assets with donor restrictions $2,500 The donation was earmarked for a specific purpose. As of the date of the statement of financial position for 20X3, the unspent donation amounted to $2,500 that would be reflected in net assets with donor restrictions. The 20X3 supplies purchase and use of supplies would be reported in the statement of activities as both an increase at the time of purchase (reclassification from net assets with donor restrictions to net assets without donor restrictions) and a decrease (expense) at the time of use with a zero net effect on net assets without donor restrictions by the end of 20X3.

On November 1 of the current year, Fruita Corporation entered into an option contract to purchase 800 shares of Silt Company stock for $19 per share (the same as the current market price) by the end of the next three months. The option is purchased for speculative purposes. The time value of the option contract is $500 at purchase. At the end of December, Silt's stock was selling for $23, and the time value of the option is now $120. If Fruita does not exercise its option until January of the subsequent year, which of the following changes would reflect the proper accounting treatment for this transaction on Fruita's December 31, year-end financial statements? A. Current assets will decrease by $380. B. The option value will be disclosed in the footnotes only. C. Other comprehensive income will increase by $3,200. D. Net income will increase by $2,820.

D. Net income will increase by $2,820. The options in this situation are being used as a tool to earn a return on investment, not for hedging purposes. For that reason, the options do not qualify for hedge accounting. The gain or loss must be currently recognized in net income. The value of an option is equal to time value plus intrinsic value. At issuance the option had intrinsic value of $0 (800 shares × ($23 − $23)) and its time value was $500 (given) for a total value of $500. At December 31, the option had the following changes: Intrinsic value (($23 − $19) × 800 sh.) $3,200Time value (given) 120Total value $3,320Less: Beginning value (500)Gain (loss) $2,820

During the year, Hauser Co. wrote off a customer's account receivable. Hauser used the allowance method for uncollectable accounts. What impact would the write-off have on net income and total assets? A. Net income: No effect; Total assets: Decrease B. Net income: Decrease; Total assets: Decrease C. Net income: Decrease; Total assets: No effect D. Net income: No effect; Total assets: No effect

D. Net income: No effect; Total assets: No effect When the allowance method of accounting for bad debts is applied, the accounts that will be eventually written off are in both the accounts receivable and the allowance account balances. When the account is written off, it is taken out of both at the same time, and the bad debt expense had already been taken as an estimated expense when the sale was made. The write-off lowers accounts receivable with a credit, and the allowance account with a debit of the same amount. The write-off entry does not affect expenses, and leaves the net realizable amount of accounts receivable the same.

Peel Co. received a cash dividend from a common stock investment. Peel should report an increase in the investment account if it uses which method of accounting? A. The cost method B. The equity method C. Either the cost method or the equity method D. None of the answer choices are correct.

D. None of the answer choices are correct. Under the cost method the receipt of a cash dividend is recorded as income from investments. The investment account remains unchanged. The receipt of a cash dividend is treated very differently under the equity method. Since income of the investor is recorded by the investor when reported, cash dividends are treated as a reduction in the investment account.

Roy City received a gift, the principal of which is to be invested in perpetuity with the income to be used to support the local library. In which type of fund should this gift be recorded? A. Private-purpose trust fund B. Special revenue fund C. Investment trusts fund D. Permanent fund

D. Permanent fund The GASB Codification directs that resources that must be held as investment principal with earnings restricted to support the reporting government's programs for a specific purpose must be accounted for in a permanent fund. The gift received by Roy City meets this description. Investment trust funds and private-purpose trust funds are both fiduciary funds that should be used to hold resources held in trust for others and not for gifts received by the government itself. A special revenue fund could be used to account for the income from the gift, but not for the investment of the gift itself.

Cross Corp. has outstanding 2,000 shares of 11% preferred stock, $50 par. On August 8, 20X1, Cross redeemed and retired 25% of these shares for $22,500. On that date, Cross' additional paid-in capital from preferred stock totaled $30,000. To record this transaction, Cross should debit (credit) its capital accounts as follows: A. Preferred stock: $25,000; Additional paid-in capital: $7,500; Retained earnings: $(10,000) B. Preferred stock: $25,000; Additional paid-in capital: $0; Retained earnings: $(2,500) C. Preferred stock: $22,500; Additional paid-in capital: $0; Retained earnings: $0 D. Preferred stock: $25,000; Additional paid-in capital: $(2,500); Retained earnings: $0

D. Preferred stock: $25,000; Additional paid-in capital: $(2,500); Retained earnings: $0 The entry to record the redemption and retirement of the preferred stock: Dr. Cr.11% Preferred stock (.25 x 2,000 x $50) 25,000Additional paid-in capital frompreferred stock ($25,000 - $22,500) 2,500Cash 22,500 Note: Retained earnings would be debited if the balance in additional paid-in capital from preferred stock was less than the amount by which the cash paid to redeem the stock exceeded the par value.

In Sloan County's general fund statement of revenues, expenditures, and changes in fund balances, which of the following has an effect on the excess of revenues over expenditures? A. Proceeds from the sale of capital assets B. Special items C. Payment to a debt service fund D. Purchase of fixed assets

D. Purchase of fixed assets "Purchase of fixed assets" is correct. This transaction would be classified as a capital expenditure and would not be depreciated. The full amount of the purchase would be deducted from fund revenues when calculating the excess of revenues over expenditures. "Payment to a debt service fund" is incorrect. This would be classified as an other financing use and would be listed after the calculation of revenues over expenditures. "Special items" and "proceeds from the sale of capital assets" are both incorrect. These are both nonoperating activities that would be listed after the calculation of revenues over expenditures.

Which account should Spring Township credit when it issues a purchase order for supplies? A. Appropriations control B. Voucher payable C. Encumbrance control D. Reserve for encumbrances

D. Reserve for encumbrances Issuing purchase orders and contracts represents commitment of a certain amount of the appropriation for the fiscal year. The commitment is formally recorded by debiting the "encumbrances" budgetary account and crediting the "reserve for encumbrances" or "Fund balance—reserved for encumbrances" budgetary account. The debit-balance encumbrances account can be used in a report as contra to the credit-balance appropriations account so as to reduce the balance of the appropriation for operational control. In essence, encumbrances "hold the place" of expenditures that are in process but will not be recorded until the liability is formally recognized upon receipt of goods or services.

LLA, Inc. was capitalized through the issuance of 10,000 shares of $30 par common stock that was sold at $50 per share. LLA had net income as follows: Year 1 $100,000 Year 2 200,000 If, during year 2, LLA paid dividends to its shareholders at $2.50 per share, what amount was LLA's retained earnings balance and shareholders' equity balance at the end of year 2? A. Retained earnings $50,000; Shareholders' equity $550,000 B. Retained earnings $275,000; Shareholders' equity $550,000 C. Retained earnings $50,000; Shareholders' equity $775,000 D. Retained earnings $275,000; Shareholders' equity $775,000

D. Retained earnings $275,000; Shareholders' equity $775,000 Retained earnings in this example would be increased through year 1 and year 2 net income, $300,000, and decreased by the $2.50 per share dividend payment of $25,000, for an ending balance of $275,000 ($300,000 − $25,000). Stockholders' equity would have been initially recorded at the stock issuance value of $500,000 ($50* price per share × 10,000) shares. At the end of year 2 it would include the initial value and retained earnings, for a total balance of $775,000 ($500,000 common stock and additional paid-in capital + $275,000 retained earnings). * On issuance, the $50 per share price would be split between common stock ($30 par value) and additional paid-in capital ($20 difference between par and fair value).

A company whose stock is trading at $10 per share has 1,000 shares of $1 par common stock outstanding when the board of directors declares a 30% common stock dividend. Which of the following adjustments should be made when recording the stock dividend? A. Common stock is debited for $3,000. B. Treasury stock is debited for $300. C. Additional paid-in capital is credited for $2,700. D. Retained earnings is debited for $300.

D. Retained earnings is debited for $300. When the 30% dividend is issued, 1,000 shares are outstanding. Thus, 30% of 1,000 shares (300 shares) will be issued in the stock dividend. A 30% stock dividend is a large stock dividend that lowers retained earnings only by the par value of the newly issued shares. No additional paid-in capital is recorded, and common stock is credited for 300 × $1 par for $300, and retained earnings is debited by the same amount. The fair value of the stock is not used.

On a nongovernmental, not-for-profit entity's statements of activities, which of the following amounts should not be netted together under any circumstances? A. Gains and losses from exchange rates or other foreign currency translations B. Revenues and expenditures from the sale of used equipment C. Investment income, custodial fees, and other advisory expenditures D. Revenues and expenditures from an annual fundraising campaign

D. Revenues and expenditures from an annual fundraising campaign The only option listed where net reporting is disallowed is revenues and expenditures from annual fundraising events because fundraising represents a major ongoing activity for not-for-profit entities.

Which of the following statements is correct concerning the appearance of noncontrolling interest on the income statement? A. Revenues, expenses, gains, losses, net income or loss, and other comprehensive income are reported in the consolidated financial statements at the owners' amounts with disclosure of the noncontrolling interest only in the footnotes. B. None of the answer choices are appropriate disclosure of the noncontrolling interest on the income statement. C. Revenues, expenses, gains, losses, net income or loss, and other comprehensive income are reported in the consolidated financial statements as the amounts attributable to the owners of the parent, followed by a separate disclosure of the revenues, expenses, gains, losses, net income or loss, and other comprehensive income attributable to the noncontrolling interest. D. Revenues, expenses, gains, losses, net income or loss, and other comprehensive income are reported in the consolidated financial statements at the consolidated amounts, which include the amounts attributable to the owners of the parent and the noncontrolling interest.

D. Revenues, expenses, gains, losses, net income or loss, and other comprehensive income are reported in the consolidated financial statements at the consolidated amounts, which include the amounts attributable to the owners of the parent and the noncontrolling interest.

A customer is considering buying a television set with a retail price of $2,000. The customer asks the store manager if the store will consider paying the sales tax so that the total cash payment is $2,000. The sales tax is 8%. The store manager agrees to accept $2,000 cash. What should the accountant credit in this transaction? A. Sales $2,000; Sales tax payable $0 B. Sales $1,840; Sales tax payable $160 C. Sales $2,000; Sales tax payable $148 D. Sales $1,852; Sales tax payable $148

D. Sales $1,852; Sales tax payable $148 This question is testing critical thinking skills. The quickest way to solve this problem is to understand that sales revenue must be less than $2,000 for the customer to pay a total of $2,000, as sales taxes are applied to the new sales price, regardless of price. From there, the easiest approach is to quickly apply 8% to both remaining sales prices ($1,840 and $1,852) to determine which leads to a total transaction cost of $2,000. The only possible answer is sales of $1,852 and sales tax payable of $148 ($1,852 × 8%). Sales of $1,840 is incorrect because $160 is not 8% of $1,840.

Which of the following are included in a local government's government-wide financial statements? A. Statement of activities B. Statement of net position; statement of revenues, expenditures, and changes in fund balances; and statement of activities C. Statement of net position and statement of revenues, expenditures, and changes in fund balances D. Statement of net position and statement of activities

D. Statement of net position and statement of activities The government-wide financial statements consist of a statement of net position and a statement of activities. A statement of revenues, expenditures, and changes in fund balances is a required financial statement for the governmental funds, but not a government-wide financial statement.

Lano Corp.'s forest land was condemned for use as a national park. Compensation for the condemnation exceeded the forest land's carrying amount. Lano purchased similar, but larger, replacement forest land for an amount greater than the condemnation award. As a result of the condemnation and replacement, what is the net effect on the carrying amount of forest land reported in Lano's balance sheet? A. No effect, because the condemned forest land's carrying amount is used as the replacement forest land's carrying amount. B. The amount is increased by the excess of the replacement forest land's cost over the condemnation award. C. The amount is increased by the excess of the condemnation award over the condemned forest land's carrying amount. D. The amount is increased by the excess of the replacement forest land's cost over the condemned forest land's carrying amount.

D. The amount is increased by the excess of the replacement forest land's cost over the condemned forest land's carrying amount. The receipt of a condemnation award is considered an involuntary conversion of a nonmonetary asset (the land) for monetary assets (cash). FASB Interpretation 30 requires that Lano Corp., as recipient of such an award, recognize a gain even though it reinvests the award in new land. The new land is then recorded at its acquisition cost. Thus, the net effect on the carrying amount of forest land reported in Lano's balance sheet is that the amount is increased by the excess of the replacement forest land's cost over the condemned forest land's carrying amount.

Which of the following items is a required disclosure regarding fair value hedges? A. The net amount of gains or losses included in the cumulative translation adjustment during the reporting period B. The estimated net amount of the existing gains or losses at the reporting date that is expected to be reclassified into earnings within the next 12 months C. A description of the transactions or other events that will result in the reclassification into earnings of gains and losses that are reported in accumulated other comprehensive income D. The amount of net gain or loss recognized in earnings when a hedged firm commitment no longer qualifies as a fair value hedge

D. The amount of net gain or loss recognized in earnings when a hedged firm commitment no longer qualifies as a fair value hedge Of the answer choices listed, only "the amount of net gain or loss recognized in earnings when a hedged firm commitment no longer qualifies as a fair value hedge" is a disclosure requirement for a fair value hedge. The other answer choices are disclosure requirements for a cash flow hedge. FASB ASC 815-10-50-4C

With respect to budgets in the public sector, which of the following statements is correct? A. The documents do not need to be strictly followed. B. The documents are used as internal planning tools only. C. The documents must be prepared on a GAAP basis. D. The documents, as an expression of public will, are a matter of law.

D. The documents, as an expression of public will, are a matter of law.

A corporation issuing stock should charge retained earnings for the market value of the shares issued in: A. a purchase of a subsidiary. B. a 2-for-1 stock split. C. an employee stock bonus. D. a 10% stock dividend.

D. a 10% stock dividend. FASB ASC 505-20-30-3 provides that for issuances of additional shares less than 20% or 25%, the issuing corporation should transfer from earned surplus (retained earnings) "an amount equal to the fair value of the additional shares issued." Thus, retained earnings should be charged for an amount equal to the market value of the shares issued in a 10% stock dividend.

How would an entity account for an implicit promise made to a customer in a contract that meets the definition of a performance obligation? A. The entity would combine the implicit promise with other promises made in the contract in all instances, as the implicit promise was not formally written into the contract, recognizing revenue over the term of the written promises. B. The entity would recognize the entire amount of the contract revenue at the point of sale and record as an expense the cost of the implicit promise as it is provided to the customer. C. The entity would defer all revenue on the contract until all promises provided in the contract have been performed. D. The entity would allocate a portion of the contract price to the implicit promise, recognizing revenue and the related cost of the promise, as it is provided to the customer.

D. The entity would allocate a portion of the contract price to the implicit promise, recognizing revenue and the related cost of the promise, as it is provided to the customer. Performance obligations identified in a contract with a customer may not always be limited to the goods or services explicitly stated in that contract. Promises implied by an entity's customary business practices, published policies, or specific statements at the time of entering into the contract can also create a valid expectation by the customer that the entity will transfer goods or services to the customer. As such, the entity identifies the implicit promise as a performance obligation to which it allocates a portion of the transaction price, recognizing revenue and the related cost of the promise, as it is provided to the customer.

In which of the following circumstances would trademarks acquired by an entity most likely be deemed to have an indefinite useful life? A. The entity pays substantial amounts of money to renew the trademarks. B. The entity plans to use the trademark until the planned phaseout date of the underlying asset. C. The entity operates in an industry with a rapidly changing regulatory environment governing trademarks. D. The entity's trademark has a remaining legal life of five years but is renewable at very little cost.

D. The entity's trademark has a remaining legal life of five years but is renewable at very little cost. If an asset's legal life is easily renewable (evidenced by low cost and/or low effort to renew) it is assumed that the company will take the easy steps to renew it, and its life is considered indefinite until the facts and circumstances change materially. The paying of substantial amounts would support a definite, not indefinite, life. Planning to use the trademark for only a fixed time would support a definite life as the useful life appears fixed. A rapidly changing industry is indicative of forced obsolescence and indicates a fixed, definite life, not indefinite life.

A company issued bonds with detachable common stock warrants. The issue price exceeded the sum of the warrants' fair value and face value of the bonds. The fair value of the bonds cannot be determined. What value, if any, should be assigned to the warrants? A. No amount, because the total proceeds should be assigned to the bonds B. The excess of the proceeds over the face value of the bonds C. The proportion of the proceeds that the warrants' fair value bears to the face value of the bonds D. The fair value of the warrants

D. The fair value of the warrants Warrants give the holder the right (but not the obligation) to purchase a set number of shares of stock from the issuing company for a fixed price on or before the warrants' expiration date. Because a warrant holder can receive issuer shares, the issuer usually classifies warrants as equity instruments and carries their value in the warrants paid-in capital account in the stockholders' equity section of the balance sheet. When detachable warrants are issued, the total proceeds must be allocated between the bond and the warrants based on their freestanding relative fair values on the issuance date. Since the fair value of the bonds cannot be determined, the warrants would be assigned their fair value with the remainder amount being assigned to the bonds.

What is the major difference between an exchange transaction and a nonexchange transaction for governmental units? A. Time requirements and whether the transaction is required by law B. Whether resources acquired can be further exchanged C. Purpose restrictions placed upon fund balances D. The relationship between the amount of value given and received

D. The relationship between the amount of value given and received

Accumulated other comprehensive income is reported in which of the following financial statements? A. The income statement B. The statement of comprehensive income C. The statement of cash flows D. The statement of financial position

D. The statement of financial position

Which of the following statements about group and composite depreciation is true? A. When a company has a large number of the same kind of assets, it is appropriate to use either the group or the composite depreciation method. B. When a company has a large number of the same kind of assets, it is not appropriate to use either the group or composite method of depreciation. C. When a company has a large number of the same kind of assets, it is appropriate to use the composite depreciation method but not the group depreciation method. D. When a company has a large number of the same kind of assets, it is appropriate to use the group depreciation method but not the composite depreciation method.

D. When a company has a large number of the same kind of assets, it is appropriate to use the group depreciation method but not the composite depreciation method. Group depreciation is used when many of the same type of assets are used by a company (e.g., telephone poles). Composite depreciation is used for assets that are similar but not the same (e.g., cars and trucks). Therefore, the only correct statement indicates that group depreciation will be used when there are many of the same type of asset.

It is inappropriate to record depreciation expenses in: A. an internal service fund. B. a private-purpose trust fund. C. an enterprise fund. D. a capital projects fund.

D. a capital projects fund. Depreciation expense is not recorded in any governmental fund since governmental funds account for neither depreciable assets nor expenses (the measurement focus of governmental funds is on expenditures, not expenses). Capital assets and related depreciation expenses are accounted for in proprietary funds and fiduciary funds. The only governmental fund type listed in the responses to this question is the capital projects fund, so this is the appropriate response.

Management's discussion and analysis, MD&A, to be issued with the financial statements of a state or local government should provide the user with each of the following, except: A. an objective and easily readable analysis based on currently known facts, decisions, or conditions. B. information about the primary government and matters related to a component unit if deemed appropriate in the managers' professional judgment. C. a fact-based analysis providing positive and negative aspects of comparisons with prior years. D. a careful pro forma presentation of the implications of current year's decisions for future financial statements.

D. a careful pro forma presentation of the implications of current year's decisions for future financial statements. While managers may wish to discuss implications and contingent outcomes of current decisions, these matters should be presented elsewhere in the annual comprehensive financial report (ACFR), such as the letter of transmittal or in other supplementary information. The other answer choices consist of MD&A requirements discussed in GASB 2200.106-.109.

A derivative financial instrument is best described as: A. a contract that conveys to a second entity a right to future collections on accounts receivable from a first entity. B. a contract that conveys to a second entity a right to receive cash from a first entity. C. evidence of an ownership interest in an entity such as shares of common stock. D. a contract that has its settlement value tied to an underlying notional amount.

D. a contract that has its settlement value tied to an underlying notional amount. A derivative instrument has three characteristics: There is an underlying or notional amount. There is little or no initial net investment. Its term requires or permits net settlement.

On December 31, 20X1 and 20X2, Apex Co. had 3,000 shares of $100 par, 5% cumulative preferred stock outstanding. No dividends were in arrears as of December 31, 20X0. Apex did not declare a dividend during 20X1. During 20X2, Apex paid a cash dividend of $10,000 on its preferred stock. Apex should report dividends in arrears in its 20X2 financial statements as: A. an accrued liability of $15,000. B. a disclosure of $15,000. C. an accrued liability of $20,000. D. a disclosure of $20,000.

D. a disclosure of $20,000. Since the preferred stock is cumulative, Apex must pay dividends to preferred shareholders for every year before any dividend can be paid to common shareholders. Thus Apex should report dividends in arrears computed as follows: However, dividends in arrears are not an accrued liability until actually declared. Thus, the reporting of dividends in arrears is achieved through disclosure either on the face of the balance sheet or in the notes.

Blue City has a major garage facility used by the Public Works department to maintain the streets and roads equipment. The garage was built 10 years ago and was expected to meet the city's needs for 30 years. The City has been updating its equipment fleet and unexpectedly discovered that the service bays are no longer adequate for many of the new vehicles, which are much larger. The sudden obsolescence of the building has been evaluated as an impairment cost. This impairment should be reported in the financial statements as: A. not as an expense in the statement of activities, but as an expenditure in the general fund (Public Works). B. a program expense (Public Works) in the statement of activities and an expenditure in the general fund (Public Works). C. neither as an expense in the statement of activities nor as an expenditure in the general fund. D. a program expense (Public Works) in the statement of activities, but not as an expenditure in the general fund.

D. a program expense (Public Works) in the statement of activities, but not as an expenditure in the general fund. The general fund, as a governmental fund, is accounted for from the current financial resources perspective. Therefore, capital assets and revaluations of capital assets are not reported. In the government-wide statements, impairment is a revaluation of a capital asset that is reported as an expense in the statement of activities. Depending on circumstances, an expense could be reported in the statement of activities as a program expense, special item, or extraordinary item.

A balance in the Fund Balance—Reserved for Encumbrances account in excess of a balance of encumbrances account indicates: A. an excess of vouchers payable over encumbrances. B. an excess of purchase orders over invoices received. C. an excess of appropriations over encumbrances. D. a recording error.

D. a recording error. A recording error must have been made if the balance in the Fund Balance—Reserved for Encumbrances account exceeds the amount of encumbrances. For example, when a purchase order is approved, the estimated amount is recorded in the journal entry: Encumbrances XXFund Balance--Reservedfor Encumbrances XX When the purchase order is filled, the entry is reversed, for the amount estimated, and the actual expenditure is recorded. The actual amount of expenditures may be more or less than the estimated amount, but that would not affect the encumbrance accounts.

A fund that exists when the government is the sponsor of a multi-government investment pool and accounts for the external portion of the trust assets is: A. a private-purpose trust fund. B. a pension trust fund. C. a custodial fund. D. an investment trust fund.

D. an investment trust fund. An investment trust fund is correct. While all the funds listed are considered fiduciary funds, the investment trust fund is the only one used to account for the external portion of investment pools reported by a sponsoring government.

A storm broke glass windows in Lea Meditators' building. Lea is a not-for-profit religious organization. A member of Lea's congregation, a professional glazier, replaced the windows at no charge. In Lea's statement of activities, the breakage and replacement of the windows should: A. be reported by note disclosure only. B. not be reported. C. be reported as an increase in both net assets and contributions. D. be reported as an increase in both expenses and contributions.

D. be reported as an increase in both expenses and contributions. Contributions received shall be recognized as revenues or gains in the period received and as assets, decreases of liabilities, or expenses depending on the form of the benefits received. Contributions of services shall be recognized if the services received (a) create or enhance nonfinancial assets or (b) require specialized skills, are provided by individuals possessing those skills, and would typically need to be purchased if not provided by donation.

South City School District has a separately elected governing body that administers the public school system. The district's budget is subject to the approval of the city council. The district's financial activity should be reported in the City's financial statements by: A. blending only. B. inclusion as a footnote only. C. either blending or inclusion as a footnote. D. discrete presentation.

D. discrete presentation. Per GASB 2100.109, financial information for separate organizations for which the primary government's elected officials are financially accountable must be included in the primary government's financial statements even though the organization is a separate legal entity. These separate organizations are called component units. Discrete presentation should be used for this presentation of financial information unless the financial activities of the two entities are so intertwined as to make them substantially the same entity. Since this does not appear to be the case in this question, discrete presentation of the information is required.

If determining the actual historical cost of general infrastructure assets is not practical because of inadequate records, public institutions that report as special-purpose governments either engaged only in governmental activities or engaged in both governmental and business-type activities should report major general infrastructure assets using: A. fair market value. B. current replacement value. C. current replacement value less an allowance for estimated accumulated depreciation. D. estimated historical cost.

D. estimated historical cost.

Regarding stock-based compensation, FASB ASC 718-10-30-10 requires that the total amount of compensation cost recognized in a stock-based employee compensation award be based on the total number of instruments that: A. have a cash market. B. have a fair value greater than their par value. C. are allowed under federal securities statutes. D. eventually vest.

D. eventually vest.

SEC Regulation S-X provides guidance for the issuer regarding: A. nonfinancial forms and disclosures required by the SEC. B. the use of EDGAR by SEC registrants. C. instructions on electronically filing the forms required by the SEC. D. format and content of financial information submitted to the SEC.

D. format and content of financial information submitted to the SEC.

To achieve the objective of providing information to assist users in assessing the level of services that can be provided by the entity and its ability to meet its obligations as they become due, financial reporting should provide information about all of the following, except: A. the financial position and condition of the governmental entity. B. legal or contractual restrictions of resources and risks of potential loss of resources. C. the governmental entity's physical and other nonfinancial resources. D. how the governmental entity met its cash requirements.

D. how the governmental entity met its cash requirements. To achieve the objective of providing information to assist users in assessing the level of services that can be provided by the entity and its ability to meet its obligations as they become due, financial reporting should provide information about: the financial position and condition of the governmental entity, the governmental entity's physical and other nonfinancial resources, and legal or contractual restrictions of resources and risks of potential loss of resources. Information about how the governmental entity met its cash requirements is related to the objective of providing information to assist users in evaluating the operating results of the governmental entity. (GASB Concepts Statement 1)

XYZ Museum, a not-for-profit entity, received a very important painting three years ago as a donation to its permanent collection. At the time of receipt, the painting was appropriately valued. The museum does not capitalize its collections. Disclosure would be handled by: A. disregarding the painting entirely because XYZ Museum opted not to capitalize. B. including the value of the painting in the net assets without donor restrictions. C. including the value of the painting in the net assets with donor restrictions. D. including a line item on the face of the financial statement with disclosures regarding XYZ Museum's permanent art collection, which includes the painting.

D. including a line item on the face of the financial statement with disclosures regarding XYZ Museum's permanent art collection, which includes the painting. A not-for-profit entity may opt not to capitalize works of art, historical treasures, and the like as long as the items meet the definition of collection items: held for exhibition, protected, and preserved, with a policy that requires sales for acquisition of other collection items, direct care, or both. Capitalization means an addition to both sides of the accounting equation, assets and equities. If the collection is not capitalized, there is no amount included in equities or net assets. However, the existence of the collection, including the donated painting, cannot be disregarded and a note describing the collection must be referenced on a line on the face of the financial statement.

When the allowance method of recognizing uncollectible accounts is used, the entries at the time of collection of a small account previously written off would: A. have no effect on the allowance for uncollectible accounts. B. increase net income. C. decrease the allowance for uncollectible accounts. D. increase the allowance for uncollectible accounts.

D. increase the allowance for uncollectible accounts. The original entry for the write off decreases AR and the Allowance accounts. In the collection of the write off you increase these accounts once again. Finally the collection is marked as a decrease of AR and increase of Cash.

Long-term debt often has covenants in the debt contract. Debt covenants are standards for the financial strength and performance of the borrower. These covenants are intended to protect the interest of the: A. stockholders. B. employees. C. company's management. D. lending institution.

D. lending institution.

Posy Corp. acquired treasury shares at an amount greater than their par value, but less than their original issue price. Compared to the cost method of accounting for treasury stock, the par value method reports a greater amount for: A. both additional paid-in capital and retained earnings. B. additional paid-in capital. C. retained earnings. D. neither additional paid-in capital nor retained earnings.

D. neither additional paid-in capital nor retained earnings.

Posy Corp. acquired treasury shares at an amount greater than their par value, but less than their original issue price. Compared to the cost method of accounting for treasury stock, the par value method reports a greater amount for: A. retained earnings. B. additional paid-in capital. C. both additional paid-in capital and retained earnings. D. neither additional paid-in capital nor retained earnings.

D. neither additional paid-in capital nor retained earnings. Assume: Par value of shares = $1,000Original issue price = $1,200 ($1,000 par, $200 additionalpaid-in capital)Reacquisition price = $1,100 (1) Reacquisition using cost method: Dr. Treasury shares 1,100 Cash 1,100 (2) Reacquisition using par value method: Dr.Treasury shares 1,000 Additional paid-in capital 100 Cash 1,100 The entry under the par value method reduces additional paid-in capital (i.e., the amount is not "greater"), while retained earnings are not affected. Note: The question asks if the amount is "greater," not just if the account is affected.

A company performing its long-lived asset impairment testing is reviewing the fair value of equipment. Each of the following valuation techniques may be appropriate for measuring the fair value of the equipment, except the: A. cost approach. B. market approach. C. income approach. D. net realizable value approach.

D. net realizable value approach. The net realizable value approach is not an acceptable fair valuation method. The determination of fair value may require the use of one or more valuation techniques. The valuation technique used should be consistent with the market approach, income approach, and/or cost approach, as appropriate.

In preparing consolidated financial statements of a U.S. parent company with a foreign subsidiary, the foreign subsidiary's functional currency is the currency: A. of the country in which the subsidiary is located. B. of the country in which the parent is located. C. in which the subsidiary maintains its accounting records. D. of the environment in which the subsidiary primarily generates and expends cash.

D. of the environment in which the subsidiary primarily generates and expends cash.

A major exception to the general rule of expenditure accrual for governmental units relates to unmatured: A. neither principal of general long-term debt nor interest on general long-term debt. B. interest on general long-term debt. C. principal of general long-term debt. D. principal of general long-term debt and interest on general long-term debt.

D. principal of general long-term debt and interest on general long-term debt. Governmental units use a modified accrual basis of accounting. Expenditures should be recorded as fund liabilities are incurred or assets are expended. However, the expenditure rules will not apply to the principal or to the interest on debt as specified by the GASB.

According to the FASB conceptual framework, for financial reporting to be useful, it must: A. be understandable to those who have a limited knowledge of business activities. B. be in accordance with generally accepted accounting principles. C. directly measure the value of the entity being reported on. D. provide information useful for making business and investment decisions.

D. provide information useful for making business and investment decisions.

A company's accounts receivable decreased from the beginning to the end of the year. In the company's statement of cash flows (direct method), the cash collected from customers would be: A. the same as sales revenues. B. sales revenues less the decrease in accounts receivable from the beginning to the end of the year. C. sales revenues plus accounts receivable at the beginning of the year. D. sales revenues plus the decrease in accounts receivable from the beginning to the end of the year.

D. sales revenues plus the decrease in accounts receivable from the beginning to the end of the year. Cash collected from customers includes: sales revenue, plus collections of accounts receivable from the prior year, less recorded sales not yet received in cash. A decrease in accounts receivable would indicate less collections than recorded sales not yet collected.

Consolidated financial statements are typically prepared when one company has a controlling financial interest in another, unless: A. the fiscal year-ends of the two companies are more than three months apart. B. the subsidiary is a finance company. C. the two companies are in unrelated industries, such as manufacturing and real estate. D. such control does not rest with the majority owner because the subsidiary is in bankruptcy.

D. such control does not rest with the majority owner because the subsidiary is in bankruptcy.

A company that wishes to disclose information about the effect of changing prices should report this information in: A. the notes to the financial statements. B. management's report to shareholders. C. the body of the financial statements. D. supplementary information to the financial statements.

D. supplementary information to the financial statements. FASB ASC 255-10-50-1 provides that "a business entity that prepares its financial statements in U.S. dollars and in accordance with U.S. generally accepted accounting principles is encouraged, but not required, to disclose supplementary information on the effects of changing prices." This information would be supplementary information to the financial statements.

All of the following statements regarding notes to the basic financial statements of governmental entities are true except: A. it is acceptable to present notes in a very extensive format. B. notes that are considered essential to the basic financial statements need to be presented. C. some notes presented by governments are identical to notes presented in business financial statements. D. the notes contain disclosures related only to required supplementary information.

D. the notes contain disclosures related only to required supplementary information.

Gem City's internal service fund received $50,000 cash from the general fund that does not need to be repaid. This should be reported in Gem's internal service fund as a credit to: A. revenues. B. accounts payable. C. other financing sources. D. transfers.

D. transfers. Inflows of assets from other funds without a requirement for repayment are considered interfund transfers. In proprietary funds (which include internal service funds), transfers should be reported separately after nonoperating revenues and expenses in the statement of revenues, expenses and changes in fund net position. An interfund transfer to/from a governmental fund should be reported as "other financing sources or uses" in the governmental fund.

In general, an enterprise preparing interim financial statements should: A. defer recognition of seasonal revenue. B. disregard permanent decreases in the market value of its inventory. C. allocate revenues and expenses evenly over the quarters, regardless of when they actually occurred. D. use the same accounting principles followed in preparing its latest annual financial statements.

D. use the same accounting principles followed in preparing its latest annual financial statements.

On February 5, 20X1, an employee filed a $2,000,000 lawsuit against Steel Co. for damages suffered when one of Steel's plants exploded on December 29, 20X0. Steel's legal counsel expects the company will lose the lawsuit and estimates the loss to be between $500,000 and $1,000,000. The employee has offered to settle the lawsuit out of court for $900,000, but Steel will not agree to the settlement. In its December 31, 20X0, balance sheet, what amount should Steel report as liability from lawsuit? A. $1,000,000 B. $2,000,000 C. $900,000 D. $500,000

FASB ASC 450-20-25 provides the following guidance when the estimated loss is in a range of values: When no amount within the range is a better estimate than any other amount, it is required that the minimum amount in the range shall be accrued. Thus, Steel Co. should report a liability from the lawsuit in the amount of $500,000, the minimum amount in the range.

GASB Statement 28 states that a government that (1) loans securities to a broker-dealer and (2) receives collateral in the form of other securities that the government cannot pledge or sell without borrower default should report: the securities lent as assets. the collateral received as assets. a liability for the government's obligation to return the collateral securities. I, II, and III I only II and III I and III

I only GASB Statement 28 states: "Governmental entities should report securities lent (the underlying securities) as assets in their balance sheets." Further, GASB Statement 28 states that "securities lending transactions collateralized by letters of credit or by securities that the governmental entity does not have the ability to pledge or sell unless the borrower defaults should not be reported as assets and liabilities in the balance sheet." (Emphasis added)

Simm Co. has determined its December 31 inventory on a LIFO basis to be $400,000. Information pertaining to the inventory follows: Estimated selling price $408,000 Estimated cost of disposal 20,000 Normal profit margin 60,000 Current replacement cost 390,000 At December 31, what should be the amount of Simm's inventory? A. $388,000 B. $328,000 C. $400,000 D. $390,000

When computing the market price, first determine the floor and the ceiling. The ceiling is net realizable value less the sales price less costs to complete the inventory item and to dispose of it ($408,000 - $20,000 = $388,000). The floor is net realizable value less normal profit ($388,000 - $60,000 = $328,000). Provided the replacement cost falls within this range, it would be the designated market; however, in this case, it exceeds the ceiling amount, so the designated market value would be $388,000

The Turtle Society, a nongovernmental not-for-profit entity, receives numerous contributed hours from volunteers during its busy season. Chris, a clerk at the local tax collector's office, volunteered 10 hours per week for 24 weeks transferring turtle food from the port to the turtle shelter. His rate of pay at the tax office is $10 per hour, and the prevailing wage rate for laborers is $6.50 per hour. What amount of contribution revenue should Turtle Society record for this service? A. $1,560 B. $0 C. $2,400 D. $840

B. $0 Contributions of services are only recognized if they create or enhance nonfinancial assets --> require special training or skills --> would need to be purchased if not donated

A nongovernmental not-for-profit college has a portfolio of bond investments that had an original cost of $2,000,000. The college's board of trustees voted to hold the principal of this fund intact in perpetuity and designated the earnings to reimburse faculty for travel to academic conferences. During the year, interest of $50,000 was earned in cash. The fair value of the bonds was $1,980,000. What amount should the college report as net assets with donor restrictions at year-end? A. $2,000,000 B. $0 C. $2,030,000 D. $1,980,000

B. $0 Only the donor can restrict assets; the board of trustees does not hold that power. Therefore, the amount of net assets with donor restrictions at year-end is $0.

Reporting entity disclosures include which of the following? A. Accounting method change impact B. Related party reporting C. Asset nature, location and quality D. Potential litigation

B. Related party reporting

A company reports the following information as of December 31: Sales revenue $800,000 Cost of goods sold 600,000 Operating expenses 90,000 Unrealized holding gain on available-for-sale debt securities, net of tax 30,000 What amount should the company report as comprehensive income as of December 31? A. $200,000 B. $110,000 C. $30,000 D. $140,000

D. $140,000 800,000 - 600,000 -90,000 = 110,000 NI 30,000 unreal hold gain 110,000 + 30,000 = 140,000

On December 30, 20X1, Hale Corp. paid $400,000 cash and issued 80,000 shares of its $1 par value common stock to its unsecured creditors on a pro rata basis pursuant to a reorganization plan under Chapter 11 of the bankruptcy statutes. Hale owed these unsecured creditors a total of $1,200,000. Hale's common stock was trading at $1.25 per share on December 30, 20X1. As a result of this transaction, Hale's total stockholder's equity had a net increase of: A. $800,000 B. $1,200,000 C. $80,000 D. $100,000

A. $800,000 Total Equity = CS + APIC + RE CS = 80,000 APIC = 20,000 RE = 1,200,000 - (400,000 + (80,000 * 1.25)) = 700,000 Total equity = 80,000 + 20,000 + 700,000 = 800,000

A company's activities for Year 2 included the following: Gross sales $3,600,000 Cost of goods sold 1,200,000 Selling and administrative expense 500,000 Adjustment for a prior-year understatement of amortization expense 59,000 Sales returns 34,000 Gain on sale of investment in equity securities 8,000 Gain on disposal of a discontinued business segment 4,000 The company has a 30% effective income tax rate. What is the company's net income for Year 2? A. $1,314,600 B. $1,316,000 C. $1,267,700 D. $1,273,300

A. $1,314,600 NI Includes 3,600,000 (34,000) (1,200,000) (500,000) 8,000 =1,874,000 (1,874,000*30%) TAXES = 1,311,800 income from continuing operations (4,000*30%) = 2800 discontinued operations 2800 + 1,311,800 = 1,314,600

Jane Co. owns 90% of the common stock of Dun Corp. and 100% of the common stock of Beech Corp. On December 30, Dun and Beech each declared a cash dividend of $100,000 for the current year. What is the total amount of dividends that should be reported in the December 31 consolidated financial statements of Jane and its subsidiaries, Dun and Beech? A. $10,000 B. $200,000 C. $190,000 D. $100,000

A. $10,000 Intercompany dividends are eliminated in consolidation. The only dividends that remain after the eliminating entries are dividends paid to noncontrolling shareholders: 10% of Dun's dividend of $100,000, or $10,000.

For the 8 months ended August 31, year 5, the carpet division of a flooring company, which is considered a major line of business, had an operating loss of $115,000 from operations. On September 1, year 5, the board of directors voted to discontinue the division's operations. On December 31, year 5, the division was sold for a pretax loss of $135,000. The division's operating loss for year 5 was $240,000. The company's income tax rate is 30%. What amount of loss should the company report as discontinued operations in the December 31, year 5, income statement? A. $262,500 B. $260,000 C. $168,000 D. $182,000

A. $262,500 Discontinued operations include all previously unrecognized gains or losses from the sale of the discontinued component and the results of operations for the discontinued component during the period, among other gains and losses. During year 5, the division lost $240,000 and was sold for a loss of $135,000, creating a total gross loss of $375,000 ($240,000 + $135,000). On the income statement the total loss is reduced by the tax benefit associated with the loss of $112,500 ($375,000 × 30%) for a net loss of $262,500 ($375,000 − $112,500). The $115,000 loss from the first 8 months of the year is already included in the $240,000 operating loss and is not needed to solve this problem.

Dannon Co. mistakenly reported its expenses of $35,200 on the cash basis. Corporate records revealed the following information: Beginning prepaid expense $1,300 Beginning accrued expense 1,650 Ending prepaid expense 1,800 Ending accrued expense 1,200 What amount of expense should Dannon report on its books under the accrual basis? A. $34,250 B. $36,150 C. $35,150 D. $35,300

A. $34,250 Prepaid expenses increased $500, so debit Prepaid and credit Cash for $500. That $500 represents excessive recognition of expense under the cash basis. Accrued expenses decreased $450, so debit Accrued Expenses and credit Cash $450. The credit to cash represents excessive expensive recognition under the cash basis, for a total excess expense recognition of $950. Dannon should report accrued expenses of $34,250 ($35,200 − $950).

During the first quarter of the current year, Tech Co. had income before taxes of $200,000, and its effective income tax rate was 15%. Tech's previous-year effective annual income tax rate was 30%, but Tech expects its current-year effective annual income tax rate to be 25%. In its first-quarter interim income statement, what amount of income tax expense should Tech report? A. $50,000 B. $30,000 C. $0 D. $60,000

A. $50,000 The current-year effective annual tax rate is used for interim reports:

Glass Co. had net income of $70,000 during the year. Depreciation expense was $10,000. The following information is available: Accounts receivable increase $20,000 Equipment gain on sale (sale price $100,000) 10,000 increase Nontrade notes payable increase 50,000 Equipment purchases 40,000 increase Accounts payable increase 30,000 What amount should Glass report as net cash provided by investing activities in its statement of cash flows for the year? A. $60,000 B. $(40,000) C. $10,000 D. $50,000

A. $60,000 Investing activities relate to nonoperating assets (PPE; equity and debt securities; notes receivable; etc) 100,000 from sale - 40,000 purchase = 60,000 Accounts receivable/payable would be accounted in operating, notes payable in financing

A partial listing of a company's accounts is presented below: Revenues $80,000Operating expenses 50,000Foreign currency translation adjustment gain,net of tax 4,000Income tax expense 10,000 What amount should the company report as net income? A. 20,000 B. 24,000 C. 30,000 D. 34,000

A. 20,000 Net income is $20,000 ($80,000 − $50,000 − $10,000). Foreign currency translation adjustment gain is an item of Other Comprehensive Income (OCI), and is included in comprehensive income but not net income; OCI is reported as a direct charge or credit to equity.

Which of the following financial categories are used in a nongovernmental not-for-profit entity's statement of financial position? A. Assets, liabilities and net assets B. Net assets, income and expenses C. Changes in net assets with donor restrictions and net assets without donor restrictions D. Income, expenses and changes in net assets

A. Assets, liabilities and net assets The statement of financial position is the term for the balance sheet reported by not-for-profit (NFP) entities. The NFP account equation uses the term "net assets" for equity. Its accounting equation is assets equal liabilities plus net assets.

When a parent-subsidiary relationship exists, consolidated financial statements are prepared in recognition of the accounting concept of: A. Economic entity B. Materiality C. Faithful representation D. Legal entity

A. Economic entity

How should a nongovernmental, not-for-profit entity report donor-restricted cash contributions for long-term purposes in its statement of cash flows? A. Financing activity inflow B. Investing activity inflow C. Operating activity inflow D. As a noncash transaction

A. Financing activity inflow

During the current year, Ace Co. amortized a bond discount. Ace prepares its statement of cash flows using the indirect method. In which section of the statement should Ace report the amortization of the bond discount? A. Operating activities B. Investing activities C. Supplemental disclosures D. Financing activities

A. Operating Activities

According to the FASB conceptual framework, which of the following correctly pairs a primary qualitative characteristic of accounting information with one of its components? A. Relevance and predictive value B. Relevance and verifiability C. Faithful representation and predictive value D. Faithful representation and feedback value

A. Relevance and predictive value Relevance and faithful representation are the primary qualitative characteristics of accounting information. The components of relevance are predictive value, confirmatory value, and materiality. The characteristics of faithful representation are completeness, neutrality, and being free from error.

Wagner, a holder of a $1,000,000 Palmer, Inc., bond, collected the interest due on March 31, 20X1, and then sold the bond to Seal, Inc., for $975,000. On that date, Palmer, a 75% owner of Seal, had a $1,075,000 carrying amount for this bond. What was the effect of Seal's purchase of Palmer's bond on the retained earnings and noncontrolling (minority) interest amounts reported in Palmer's March 31, 20X1, consolidated balance sheet? A. Retained earnings: $100,000 increase; Noncontrolling interest: $0 B. Retained earnings: $0; Noncontrolling interest: $25,000 increase C. Retained earnings: $75,000 increase; Noncontrolling interest: $25,000 increase D. Retained earnings: $0; Noncontrolling interest: $100,000 increase

A. Retained earnings: $100,000 increase; Noncontrolling interest: $0 carrying value of bond 1,075,000 - acquisition cost 975,000 = 100,000 gain --> RE This gain would, of course, increase consolidated retained earnings. The gain is identified with the issuer of the bonds, which is Palmer in this case. Therefore, the gain has no effect on noncontrolling (minority) interest.

What is the purpose of SFAC 4 as stated in that concepts statement? A. To provide a basis for establishing detailed accounting and reporting standards for nonbusiness entities B. To provide detailed accounting and reporting standards for nonbusiness entities C. All of the answer choices are correct. D. To provide the methods for preparing financial statements for nonbusiness entities

A. To provide a basis for establishing detailed accounting and reporting standards for nonbusiness entities SFAC 4, Objectives of Financial Reporting by Nonbusiness Organizations, represents the most recent expression of the overall purposes and related objectives of financial reporting by nonbusiness organizations. The purposes and related accounting and reporting objectives set forth in SFAC 4 are concepts—not standards—and are designed to provide a basis for establishing detailed accounting and reporting standards.

During 20X1, Jones Foundation received the following support: A cash contribution of $875,000 to be used at the board of directors' discretion A promise to contribute $500,000 in 20X2 from a supporter who has made similar contributions in prior periods Contributed legal services with a value of $100,000, which Jones would have otherwise purchased At what amounts would Jones classify and record these transactions? A. Unrestricted revenue: $975,000; Restricted revenue: $500,000 B. Unrestricted revenue: $875,000; Restricted revenue: $500,000 C. Unrestricted revenue: $975,000; Restricted revenue: $0 D. Unrestricted revenue: $1,375,000; Restricted revenue: $0

A. Unrestricted revenue: $975,000; Restricted revenue: $500,000 The foundation should report unrestricted revenue of $975,000. This is the $875,000 contribution as well as the $100,000 in contributed services. The value of volunteer services is recognized as contributions insofar as the services require specialized skills, were provided by persons possessing those skills, and would typically have been purchased if not provided by donation. A promise to contribute a specified amount to an organization should be recorded as income immediately upon receipt of the promise. Because the promised contribution ($500,000 in this scenario) will not be collected until the subsequent year, it should be considered restricted. Because the promised contribution is expected to be collected within one year of the financial statement date, it may be measured at net realizable value.

Compared to the accrual basis of accounting, the cash basis of accounting understates income by the net decrease during the accounting period of: A. accrued expenses. B. neither accounts receivable nor accrued expenses. C. both accounts receivable and accrued expenses. D. accounts receivable.

A. accrued expenses. The cash basis of accounting, as compared to accrual-basis accounting, will understate income when accrued expenses decrease because that decrease must have been the result of paying out more cash than expenses incurred. A decrease in accounts receivable would have the opposite effect. This decrease would occur as more cash was collected than sales made, producing a higher cash basis income. In summary, cash basis income is: higher when accounts receivable decrease. lower when accrued expenses decrease.

Financial statement line item explanations which may require additional information for full disclosure purposes include all of the following except: A. nature of primary activities. B. line item relation to other line items. C. potential effect related to inability to perform or pay. D. degree of credit or nonperformance risk.

A. nature of primary activities. A summary of potential additional disclosures is as follows: For assets: the nature, quality, and location; future cash flows; relation to other line items; and significant contractual, statutory, regulatory, or judicial restrictions. For assets and liabilities resulting from financial instruments or other contracts: contractual or legal terms (e.g., timing of receipts and disbursements), degree of credit or nonperformance risk, potential effect related to inability to pay or perform, and method used to determine the cash flows. Other disclosures could include equity instrument terms or conditions, potential effects of changing accounting methods, breakdown of aggregated line items, alternative measurements, and the line item's relation to other line items.

All NFPs are required to disclose an analysis of expenses by both functional and natural classifications to be presented in: A. one location in the financial statements, whether in the notes or in a separate financial statement B. one location in the financial statements: a separate financial statement C. one location in the financial statements: the notes D. two location in the financial statements: the notes and in a separate financial statement

A. one location in the financial statements, whether in the notes or in a separate financial statement

Each of the following events is required to be reported to the United States Securities and Exchange Commission on Form 8-K, except: A. the quarterly results of operations and financial condition of a registrant B. the creation of an obligation under an off-balance sheet arrangement of a registrant C. the unregistered sale of equity securities D. a change in a registrant's certifying account

A. the quarterly results of operations and financial condition of a registrant Form 8-K reports current events, allowing investors to obtain information in a timely manner about events material to a company's performance (including a change in a registrant's certifying accountant, notification about entering into material agreements and entering into debt or other direct financial obligations)

A nongovernmental not-for-profit animal shelter receives contributed services from the following individuals valued at their normal billing rate: Veterinarian provides volunteer animal care $8,000 Board members volunteer to prepare books for audit 4,500 Registered nurse volunteers as receptionist 3,000 Teacher provides volunteer dog walking 2,000 What amount should the shelter record as contribution revenue? A. $14,500 B. $12,500 C. $11,000 D. $8,000

B. $12,500 Contribution revenues and assets or expenses should be reported for donated services if: special skills are required to perform the service, the individual providing the service has those special skills, and the organization would have to buy the services if they were not donated. Therefore, $12,500 would be recorded as contribution revenue ($8,000 + $4,500 = $12,500).

Reid Partners, Ltd., which began operations on January 1, 20X1, has elected to use cash-basis accounting for tax purposes and accrual-basis accounting for its financial statements. Reid reported sales of $175,000 and $80,000 in its tax returns for the years ending December 31, 20X2 and 20X1, respectively. Reid reported accounts receivable of $30,000 and $50,000 in its balance sheets as of December 31, 20X2 and 20X1, respectively. What amount should Reid report as sales in its income statement for the year ending December 31, 20X2? A. $205,000 B. $155,000 C. $145,000 D. $195,000

B. $155,000 Sales = $175,000 + $30,000 - $50,000 175,000 = collections 30,000 = ending AR 50,000 = beg. AR

Young & Jamison's modified cash-basis financial statements indicate cash paid for operating expenses of $150,000, end-of-year prepaid expenses of $15,000, and accrued liabilities of $25,000. At the beginning of the year, Young & Jamison had prepaid expenses of $10,000, while accrued liabilities were $5,000. If cash paid for operating expenses is converted to accrual-basis operating expenses, what would be the amount of operating expenses? A. $125,000 B. $165,000 C. $175,000 D. $135,000

B. $165,000 In converting expenses from a cash basis to an accrual basis, a prepaid expense increase of $5,000 must be deducted because it is an expense on a cash basis and an asset on an accrual basis. An increase of $20,000 in accrued liabilities must be added because it is an expense incurred but not paid. $150,000 - $5,000 + $20,000 = $165,000

On January 2, 20X1, Pare Co. purchased 75% of Kidd Co.'s outstanding common stock. Selected balance sheet data on December 31, 20X1, is as follows: PARE Total Assets: $420,000 Liabilities: $120,000 Common Stock: $200,000 Retained Earnings: $200,000 KIDD Total Assets: $180,000 Liabilities: $60,000 Common Stock: $50,000 Retained Earnings: $70,000 During 20X1, Pare and Kidd paid cash dividends of $25,000 and $5,000, respectively, to their shareholders. There were no other intercompany transactions. The combination is accounted for as an acquisition. In Pare's December 31, 20X1, consolidated balance sheet, what amount should be reported as noncontrolling (minority) interest in net assets? A. $0 B. $30,000 C. $45,000 D. $105,000

B. $30,000 Noncontrolling Interest = Noncontrolling Holding * Net Assets of Kidd Noncontrolling Interest = (1-.75) * ($50,000 + $70,000) Noncontrolling Interest = .25 * 120,000 Noncontrolling Interest = 30,000 (Net assets can be book value of SHE or Assets - Liabilities)

Abbott Co. is preparing its statement of cash flows for the year. Abbott's cash disbursements during the year included the following: Payment of interest on bonds payable $500,000 Payment of dividends to stockholders 300,000 Payment to acquire 1,000 shares of Marks Co. common stock 100,000 What should Abbott report as total cash outflows for financing activities in its statement of cash flows? A. $0 B. $300,000 C. $800,000 D. $900,000

B. $300,000 Interest payments are included in operating activities. Cash payments to acquire equity instruments are included in investing activities. Only payments of dividends to stockholders are included in financing activities.

King, Inc., owns 70% of Simmon Co.'s outstanding common stock. King's liabilities total $450,000, and Simmon's liabilities total $200,000. Included in Simmon's financial statements is a $100,000 note payable to King. What amount of total liabilities should be reported in the consolidated financial statements? A. $520,000 B. $550,000 C. $650,000 D. $590,000

B. $550,000 450,000 + 200,000 - 100,000 = 550,000

Clear Co.'s trial balance has the following selected accounts: Cash (includes $10,000 in bond-sinking fund for long-term bond payable) $50,000 Accounts receivable 20,000 Allowance for doubtful accounts 5,000 Deposits received from customers 3,000 Merchandise inventory 7,000 Unearned rent 1,000 Prepaid expenses 2,000 What amount should Clear report as total current assets in its balance sheet? A. $72,000 B. $64,000 C. $74,000 D. $67,000

B. $64,000 Cash (net of $10,000 in bond-sinkingfund classified as Other Asset) $40,000 Accounts receivable 20,000 Allowance for doubtful accounts (5,000) Merchandise inventory 7,000 Prepaid expenses 2,000 TOTAL $64,000 Deposits received from customers and unearned rent are both liabilities

Mirr, Inc., was incorporated on January 1, 20X0, with proceeds from the issuance of $750,000 in stock and borrowed funds of $110,000. During the first year of operations, revenues from sales and consulting amounted to $82,000, and operating costs and expenses totaled $64,000. On December 15, 20X0, Mirr declared a $3,000 cash dividend, payable to stockholders on January 15, 20X1. No additional activities affected owners' equity in 20X0. Mirr's liabilities increased to $120,000 by December 31, 20X0. On Mirr's December 31, 20X0, balance sheet (statement of financial position), total assets should be reported at: A. $878,000. B. $885,000. C. $875,000. D. $882,000.

B. $885,000. Since (1) assets equals liabilities plus equity and (2) the $120,000 amount of liabilities is a given, the key is to compute the December 31, 20X0, balance of equity. The transactions described affected equity as follows: $750,000 + 82,000 + (64,000) + (3,000) = 765,000 765,000 + 120,000 = 885,000 total assets

On January 16, Tree Co. paid $60,000 in property taxes on its factory for the current calendar year. On April 2, Tree paid $240,000 for unanticipated major repairs to its factory equipment. The repairs will benefit operations for the remainder of the calendar year. What amount of these expenses should Tree include in its third-quarter interim financial statements for the three months ended September 30? A. $75,000 B. $95,000 C. $15,000 D. $0

B. $95,000 property taxes = 3/12 * 60,000 = 15,000 major repairs = 3/9 * 240,000 = 80,000 80,000 + 15,000 = 95,000

A nongovernmental not-for-profit organization may report on which of the following basis and remain in compliance with generally accepted accounting principles (GAAP)? A. Cash B. Accrual C. Modified Cash D. Modified Accrual

B. Accrual Nongovernmental not-for-profits (NFPs) follow U.S. GAAP, particularly FASB ASC 958. They most closely follow guidance established for business (for-profit) entities and therefore the only answer that closely matches for-profit enterprises is accrual accounting. Modified cash, modified accrual, and cash accounts are not acceptable for for-profit or NFP institutions that are nongovernmental organizations in nature.

According to the FASB conceptual framework, which of the following does not relate to both relevance and faithful representation? A. Comparability B. Confirmatory Value C. Verifiability D. Timeliness

B. Confirmatory Value "Comparability, verifiability, timeliness, and understandability are qualitative characteristics that enhance the usefulness of information that is relevant and faithfully represented." Thus, comparability, verifiability and timeliness all relate to both relevance and faithful representation. Confirmatory value is a component of relevance.

Which of the following should be disclosed as supplemental information in the statement of cash flows? A. Cash flow per share B. Conversion of debt to equity C. Both cash flow per share and conversion of debt to equity D. Neither cash flow per share nor conversion of debt to equity

B. Conversion of debt to equity "financial statements shall not report an amount of cash flow per share." Also, "information about all investing and financing activities of an enterprise during a period that affects recognized assets or liabilities, but that does not result in cash receipts or cash payments in the period, shall be reported in related disclosures."

When a nongovernmental not-for-profit entity has board-designated net assets, where should the amounts and purposes of these net assets be disclosed? A. In the notes to the financial statements but not on the face of statement of financial position B. Either on the face of the statement of financial position or in the notes to the financial statements C. Neither on the face of the statement of financial position nor in the notes to the financial statements D. On the face of the statement of financial position but not in the notes to the financial statements

B. Either on the face of the statement of financial position or in the notes to the financial statements Nongovernmental not-for-profit entities have the option to report board designated net assets either on the face of the statement of financial position or in the notes to the financial statements.

Terra Co.'s total revenues from its three business segments were as follows: Segment L sales to unaffiliated: $70,000 intersegment sales: 30,000 total revenues: 100,000 Segment M sales to unaffiliated: $22,000 intersegment sales: 4,000 total revenues: 26,000 Segment N sales to unaffiliated: $8,000 intersegment sales: 16,000 total revenues: 24,000 Combined sales to unaffiliated: $100,000 intersegment sales: 50,000 total revenues: 150,000 Elimination sales to unaffiliated: - intersegment sales: (50,000) total revenues: (50,000) Consolidated sales to unaffiliated: $100,000 intersegment sales: - total revenues: 100,000 Which business segments are deemed to be reportable segments? A. L and M only B. L, M and N C. None D. L only

B. L, M and N If an operating segment's revenue (sales to unaffiliated customers and intersegment sales) is "10% or more of the combined revenue...of all the enterprise's industry segments," it is a reportable segment. L% = 100,000/150,000 = 67% M% = 26,000/150,000 = 17% N% = 24,000/150,000 = 16%

Fleming Co.'s five operating segments have revenues and profits/losses as shown below: R revenue: 64 S revenue: 12 T revenue: 26 U revenue: 8 V revenue: 28 138 total revenue R profit/(loss): 13 S profit/(loss): (25) T profit/(loss): 21 U profit/(loss): 4 V profit/(loss): (17) (4) total profit/(loss) Which of these operating segments are deemed to be reportable operating segments? A. R, T, V B. R, S, T, V C. R, S, T, U, V D. R, S

B. R, S, T, V must report information separately for each operating segment that meets any of the following three tests: Revenue test: If its revenue is 10% or more of the combined revenue of all operating segments Profitability test: If the absolute amount of its reported profit or loss is 10% or more of the greater, in absolute terms, of: a. the combined reporting profit of all operating segments that did not report a loss or b. the combined reported loss of all operating segments that did report a loss Asset test: If its assets are 10% or more of the combined assets of all operating segments Following these rules, segments must be reported if: Revenue > = $13.8 ($138 × 10%)Segments meeting requirement: Rust, Tov, and Vin Profit or loss > = $4.2 ($42 combined losses × 10%) > ($38 combined profit)Segments meeting requirement: Rust, Slad, Tov, and Vin

The net asset reclassifications of a nongovernmental not-for-profit organization would be reported on which of the following? A. Statement of financial position B. Statement of activities C. Statement of cash flows D. Statement of functional expenses

B. Statement of activities The statement of activities provides information about the change in amount and nature of net assets by reporting on changes in net assets with donor restrictions and net assets without donor restrictions for a period of time. The statement of financial position is comparable to a balance sheet, and reports assets, liabilities, and net assets (rather than equity) at a point in time. Cash receipts and cash payments are reported on the statement of cash flows.

What is the primary objective of financial reporting? A. To provide forecasts for future cash flows and financial performance B. To provide information that is useful for economic decision making C. To provide economic information that is comprehensible to all users D. To provide management with an accurate evaluation of their financial performance

B. To provide information that is useful for economic decision making SFAC 5 states that one of the characteristics of financial statements is to provide financial information (not managerial or economic) that is useful to decision making. SFAC 5 states that financial statements individually and collectively contribute to meeting the objectives of financial reporting, with each financial statement providing a different kind of information. Financial reporting is based upon past performance, not future performance or future cash flows.

How should a nongovernmental not-for-profit entity classify gains and losses on investments purchased with net assets with donor restrictions? A. Unless explicitly restricted by donor or law, gains and losses should be reported in the statement of activities as increases or decreases in net assets with donor restrictions. B. Unless explicitly restricted by donor or law, gains and losses should be reported in the statement of activities as increases or decreases in net assets without donor restrictions. C. Gains may not be netted against losses in the statement of activities D. Gains and losses can only be reported net of expenses in the statement of activities

B. Unless explicitly restricted by donor or law, gains and losses should be reported in the statement of activities as increases or decreases in net assets without donor restrictions.

A company acquired a building, paying a portion of the purchase price in cash and issuing a mortgage note payable to the seller for the balance. In a statement of cash flows, what amount is included in financing activities for the transaction? A. Cash payment B. Zero C. Acquisition price D. Mortgage amount

B. Zero The only cash involved in this transaction is the cash paid. It would be included in cash flows from investing activities.

When a CPA is applying the enhancing qualitative characteristics of useful financial information, it is important for the CPA to remember that: A. each of the four enhancing qualitative characteristics should be given equal priority. B. applying the enhancing qualitative characteristics is an iterative process that does not follow a prescribed order C. cost is a secondary consideration when applying the enhancing qualitative characteristics. D. the enhancing qualitative characteristics could compensate for unfaithful representation.

B. applying the enhancing qualitative characteristics is an iterative process that does not follow a prescribed order Enhancing qualitative characteristics enhance the usefulness of information that is relevant and faithfully stated (e.g., accurate and truthful). Applying the characteristics to financial reporting is an adaptive process that changes as the facts, information, and method of presentation change. There is no standard or predefined order that must be followed when applying the enhancing qualitative characteristics. There is no requirement that the four characteristics be given equal priority—it depends on the information being presented. Characteristics are only relevant for faithful representations of facts. Cost is considered in conjunction with the characteristics and is not subordinate to them.

During a period when an enterprise is under the direction of a particular management, its financial statements will directly provide information about: A. neither enterprise performance nor management performance. B. enterprise performance but not directly provide information about management performance. C. management performance but not directly provide information about enterprise performance. D. both enterprise performance and management performance.

B. enterprise performance but not directly provide information about management performance. Financial statements provide direct information about enterprise performance because the primary focus of the statements is to provide information about the financial performance of that enterprise by providing information about earnings. The same cannot be said, however, in regard to management performance. The financial statements depict only indirect information concerning management performance. (Direct information related to management performance would be provided in internal managerial performance reports but not in the external financial statements.)

Materiality and relevance are both defined by: A. quantitative criteria set by the Financial Accounting Standards Board. B. what influences or makes a difference to a decision maker. C. the perceived benefits to be denied that exceed the perceived costs associated with it. D. the consistency in the application of methods over time.

B. what influences or makes a difference to a decision maker. Relevance must be material—capable of influencing decision makers. These are not quantitative characteristics. Consistency helps to achieve comparability—one of the enhancing qualitative characteristics. Benefits exceeding costs is the cost constraint on useful financial information.

Beni Corp. purchased 100% of Carr Corp.'s outstanding capital stock for $430,000 cash. Immediately before the purchase, the balance sheets of both corporations reported the following: Beni Assets: $2,000,000 liabilities: 750,000 common stock: 1,000,000 retained earnings: 250,000 liabilities and SHE: 2,000,000 Carr Assets: $750,000 liabilities: 400,000 common stock: 310,000 retained earnings: 40,000 liabilities and SHE: 750,000 At the date of purchase, the fair value of Carr's assets was $50,000 more than the aggregate carrying amounts. In the consolidated balance sheet prepared immediately after the purchase, the consolidated stockholders' equity should amount to: A. $1,680,000 B. $1,600,000 C. $1,250,000 D. $1,650,000

C. $1,250,000 Carr's stockholders' equity is eliminated for consolidated financial statement purposes, but Beni's stockholder's equity was not increased. Beni simply used its assets to acquire the net assets of Carr. Actual entry: DR. Investment in Carr 430,000 CR. Cash 430,000 Consolidated adjustment: DR. Assets (at fair value) 800,000 DR. Goodwill 30,000 CR. Liabilities 400,000 CR. Investments in Carr 430,000

Selected information from the separate and consolidated balance sheets and income statements of Para, Inc., and its subsidiary, Shel Co., as of December 31, 20X1, and for the year then ended is as follows: PARE AR: 52,000 Inventory: 60,000 Revenue: 400,000 COGS: 300,000 Gross Profit: 100,000 SHEL AR: 38,000 Inventory: 50,000 Revenue: 280,000 COGS: 220,000 Gross Profit: 60,000 Consolidated PARE AR: 78,000 Inventory: 104,000 Revenue: 616,000 COGS: 462,000 Gross Profit: 154,000 Additional information: During 20X1, Pare sold goods to Shel at the same markup on cost that Pare uses for all sales. On December 31, 20X1, what was the amount of Shel's payable to Pare for intercompany sales? A. $6,000 B. $64,000 C. $12,000 D. $58,000

C. $12,000 Separate A/R = 52,000 + 38,000 = 90,000 Consolidated A/R = 78,000 90,000 - 78,000 = 12,000 Intercompany receivables and payables are always removed in the consolidation process, thus the difference shows intercompany transaction.

Sussman Co. prepared cash-basis financial statements for the month ended January 31. A summary of Sussman's January activities follows: Credit Sales of $5,600 Collections of $1,900 relating to January credit sales Accrued salaries of $1,200 By what amount will Sussman's cash-basis income for the month ended January 31 increase as a result of restating these activities to the accrual basis of accounting? A. $4,400 B. $4,900 C. $2,500 D. $3,700

C. $2,500 Sussman's cash-basis accounting income would be $1,900 ($1,900 Cash in − $0 Cash out). Sussman's accrual basis income would be $4,400 ($5,600 Credit sales − $1,200 Accrued salary expense). Restating the income from cash to accrual would result in net income increasing $2,500 ($4,400 Accrual income − $1,900 Cash-basis income).

Oz, a nongovernmental not-for-profit entity, received $50,000 from Ame Company to sponsor a play given by Oz at the local theater. Oz gave Ame 25 tickets, which generally cost $100 each. Ame received no other benefits. What amount of ticket sales revenue should Oz record? A. $47,500 B. $0 C. $2,500 D. $50,000

C. $2,500 This payment is partially an exchange transaction and partially a contribution and the two parts should be accounted for separately. Oz would recognize ticket sales revenue for the 25 tickets ($2,500) and recognize the balance as contribution revenue.

Ute Co. had the following capital structure during 20X1 and 20X2: Preferred stock, $10 par, 4% cumulative, 25,000 shares issued and outstanding $ 250,000 Common stock, $5 par, 200,000 shares issued and outstanding 1,000,000 Ute reported net income of $500,000 for the year ended December 31, 20X2. Ute paid no preferred dividends during 20X1 and paid $16,000 in preferred dividends during 20X2. In its December 31, 20X2, income statement, what amount should Ute report as basic earnings per share? A. $2.48 B. $2.42 C. $2.45 D. $2.50

C. $2.45 The $16,000 dividends paid in 20X2 included only the $10,000 (i.e., 25,000 × $10 × .04) preferred dividend requirement for 20X2. Dividends in arrears should have been included in the previous year's computation of earnings per share. (NI - preferred stock dividend)/ Weighted average common shares 500,000 - (25% * 10 * 4%) / 200,000 490,000 / 200,000 = 2.45

Baker Co. began its operations during the current year. The following is Baker's balance sheet at December 31: Assets Cash $192,000 Accounts receivable 82,000 Total assets $274,000 Liabilities and stockholders' equity Accounts payable $ 24,000 Common stock 200,000 Retained earnings 50,000 Total liabilities and stockholders' equity $274,000 Baker's net income for the current year was $78,000 and dividends of $28,000 were declared and paid. Common stock was issued for $200,000. What amount should Baker report as cash provided by operating activities in its statement of cash flows for the current year? A. $250,000 B. $50,000 C. $20,000 D. $192,000

C. $20,000 NI 78,000 - 82,000 increase in AR + 24,000 increase in AP = 20,000 net cash from operating activities

For the purpose of determining that no additional segments should be reported, the total of external revenues of all reportable segments must make up at least what percentage of total consolidated revenues? A. 50% B. 60% C. 75% D. 85%

C. 75% Additional operating segments are reported unless the total of external revenues of all reportable segments make up at least 75% of total consolidated revenues OR a segment that previously met the criteria as a reportable segment (but not currently) may still be reportable if management sees it to be of continuing significance

Which of the following statements is correct regarding reporting comprehensive income? A. A separate statement of comprehensive income is required. B. Comprehensive income must include all changes in stockholders' equity for the period. C. Accumulated other comprehensive income is reported in the stockholders' equity section of the balance sheet. D. Comprehensive income is reported in the year-end statements but not in the interim statements.

C. Accumulated other comprehensive income is reported in the stockholders' equity section of the balance sheet. accumulated other comprehensive income be reported in the stockholders' equity section of the balance sheet: "The total of other comprehensive income for a period shall be transferred to a component of equity that is presented separately from retained earnings and additional paid-in capital in a statement of financial position at the end of an accounting period. A descriptive title such as accumulated other comprehensive income shall be used for that component of equity."

FASB ASC 270-10-45-1 concluded that interim financial reporting should be viewed primarily in which of the following ways? A. As useful only if activity is spread evenly throughout the year. B. As if the interim period were an annual accounting period. C. As reporting for an integral part of an annual period. D. As reporting under a comprehensive basis of accounting other than GAAP

C. As reporting for an integral part of an annual period.

Town, Inc., is preparing its financial statements for the year ending December 31, 20X1. On December 1, 20X1, Town was awarded damages of $75,000 in a patent infringement suit it brought against a competitor. The defendant did not appeal the verdict, and payment was received in January 20X2, prior to the issuance of the financial statements. What is the reporting requirement? A. Neither accrual nor disclosure B. Disclosure only C. Both accrual and disclosure D. Accrual only

C. Both accrual and disclosure Town, Inc., has a legally enforceable right to the settlement from the lawsuit on December 1, 20X1, so it would be reported in 20X1. The circumstances of the accrual of the gain should be disclosed in the interest of full disclosure.

Which of the following disclosures should prospective financial statements include? A. Summary of significant assumptions B. Neither summary of significant accounting policies nor summary of significant assumptions C. Both summary of significant accounting policies and summary of significant assumptions D. Summary of significant accounting policies

C. Both summary of significant accounting policies and summary of significant assumptions accountants provide summaries of the significant accounting policies and the assumptions used to prepare these forward-looking statements. The same full disclosure principle that guides the preparation of historical financial statements applies to the reporting of prospective financial statements. error_outline

According to the FASB conceptual framework, which of the following situations violates the concept of reliability? A. Management reports to stockholders regularly refer to new projects undertaken, but the financial statements never report project results. B. Data on segments having the same expected risks and growth rates are reported to analysts estimating future profits. C. Financial statements include property with a carrying amount increased to management's estimate of market value. D. Financial statements are issued nine months late.

C. Financial statements include property with a carrying amount increased to management's estimate of market value. "To be reliable, information about an item must be representationally faithful, verifiable, and neutral. To be reliable, information must be sufficiently faithful in its representation of the underlying resource, obligation, or effect of events and sufficiently free of error and bias to be useful to investors, creditors, and others in making decisions." Clearly, values assigned to property based on management's estimate of fair value are not representationally neutral and would not be considered reliable as defined by the SFAC 5. Late financial statements violate timeliness (relevance), not reliability. Note: The term "reliability" is still used in SFAC 5, even though it has changed to "faithful representation" in SFAC 8.3.

Which of the following statements is true? A. GAAP applicable to nonbusiness organizations differ significantly from those applicable to business enterprises. B. Applicable GAAP differ significantly among the various types of nonbusiness organizations. C. GAAP applicable to nonbusiness organizations differ significantly from those applicable to business enterprises and among the various types of nonbusiness organizations. D. None of the answer choices are correct.

C. GAAP applicable to nonbusiness organizations differ significantly from those applicable to business enterprises and among the various types of nonbusiness organizations. GAAP applicable to nonbusiness organizations differ significantly from those applicable to business enterprises. Moreover, applicable GAAP differ significantly among the various types of nonbusiness organizations (e.g., between government hospitals and nongovernment hospitals and between government educational institutions and nongovernment educational institutions).

How are discontinued operations that occur at midyear initially reported? A. Included in net income and disclosed in the notes to the year-end financial statements B. Disclosed only in the notes to the year-end financial statements C. Included in net income and disclosed in the notes to interim financial statements D. Disclosed only in the notes to interim financial statements

C. Included in net income and disclosed in the notes to interim financial statements Discontinued operations should be reported separately, net-of-tax, on the income statement for the interim period. Disclosure in the notes to the interim statements is required.

The per-share amount must be reported on the face of a public company's income statement for which of the following items? A. Compensation effect of fair value on stock options B. U.S. Treasury Stock C. Income from continuing operations D. Preferred stock dividend

C. Income from continuing operations EPS is not required for preferred stock dividends, U.S. Treasury stock, or for the compensation effect of fair value on stock options.

The presentation of financial statements must be applied within an identifiable framework (AU-C 800). Normally, the framework is provided by generally accepted accounting principles (GAAP). However, in some circumstances, a different framework may be used. Which of the following would not be indicative of an acceptable framework for the presentation of financial statements? A. Ignoring accrued income and expenses B. Presenting the financial statements under the same basis that the entity uses to file its income tax return C. Modifying select items on the cash flow statement based on definite criteria D. Reporting on cash basis but capitalizing fixed assets and recording depreciation

C. Modifying select items on the cash flow statement based on definite criteria Modifications to some items on the cash flow statement without including all material items on the other financial statements would not qualify as a special purpose framework. Reporting on a cash basis but capitalizing fixed assets and ignoring accrued income and expenses would represent an acceptable cash basis or modified cash basis reporting framework. Filing statements based on the framework under which the entity prepares its tax returns would be an appropriate tax basis reporting framework.

In the preparation of the statement of activities for a nongovernmental not-for-profit entity, all expenses are reported as decreases in which of the following net asset classes? A. Expenses are not reported on the statement of activities. B. Net asset with donor restrictions C. Net assets without donor restrictions D. Total net assets

C. Net assets without donor restrictions The FASB states that a statement of activities shall report expenses as decreases in net assets without donor restrictions.

Alpha Hospital, a large not-for-profit entity, has adopted an accounting policy that does not imply a time restriction on gifts of long-lived assets. Alpha's board designates $1 million of assets already on hand to purchase investments whose income will be used for capital improvements. Indicate the manner in which this transaction affects Alpha's financial statements. A. Increase in net assets with donor restrictions B. Increase in net assets without donor restrictions C. No required reportable event D. Increase in unrestricted revenues, gains and other support

C. No required reportable event The designation by the board does not cause a change in restrictions (those are placed by donors), designation is not change in revenue/expenses...Non-reportable

Which of the following qualifies as a reportable operating segment? A. South American segment, whose results of operations are reported directly to the board of directors, and has 5% of the company's assets, 9% of revenues, and 8% of the profits B. Corporate headquarters, which oversees $1 billion in sales for the entire company C. North American segment, whose assets are 12% of the company's assets of all segments, and management reports to the chief operating officer D. Eastern Europe segment, which reports its results directly to the manager of the European division, and has 20% of the company's assets, 12% of revenues, and 11% of profits

C. North American segment, whose assets are 12% of the company's assets of all segments, and management reports to the chief operating officer "Not every part of an enterprise is necessarily a reportable operating segment or part of an operating segment. For example, a corporate headquarters or certain functional departments may not earn revenues or may earn revenues that are only incidental to the activities of the enterprise and would not be operating segments." "Generally, an operating segment has a segment manager who is directly accountable to and maintains regular contact with the chief operating decision maker to discuss operating activities, financial results, forecasts, or plans for the segment."

Mend Co. purchased a 3-month U.S. Treasury bill. Mend's policy is to treat as cash equivalents all highly liquid investments with an original maturity of three months or less when purchased. How should this purchase be reported in Mend's statement of cash flows? A. As an outflow from financing activities B. AS an outflow from investing activities C. Not reported D. As an outflow from operating activities

C. Not reported Since Mend's policy is to treat these investments as cash equivalents, the purchase would not be reported in the statement of cash flows.

Which of the following characteristics of accounting information primarily allows users of financial statements to generate predictions about an organization? A. Timeliness B. Neutrality C. Relevance D. Reliability

C. Relevance The qualitative characteristics of useful accounting information are found in SFAC 8, chapter 3. The primary qualitative characteristics are relevance and faithful representation. Relevance to the users of financial information means that the information is capable of making a difference when the user is making a financial decision. The characteristics that have the potential to make a difference are: predictive value, confirmatory value, and materiality.

A public entity sells steel for use in construction. One of its customers accounts for 43% of sales, and another customer accounts for 40% of sales. What should the entity disclose in its annual financial statements about these two customers? A. The payment terms of accounts receivable due from each of the two customers B. The financial condition of the two customers C. The amount of the entity's revenue from each of the two customers D. The names of the two customers

C. The amount of the entity's revenue from each of the two customers If revenues from transactions with a single external customer amount to 10% or more of an entity's revenues, the business must disclose that fact, the total amount of revenues from each such customer, and the identity of the segments reporting the revenues. A major customer is a single customer, or a group of entities, known to a reporting enterprise to be under common control. The identity, payment terms, and financial condition of the customers do not need to be disclosed.

Wood Co.'s dividends on noncumulative preferred stock have been declared but not paid. Wood has not declared or paid dividends on its cumulative preferred stock in the current or the prior year and has reported a net loss in the current year. For the purpose of computing basic earnings per share, how should the income available to common stockholders be calculated? A. The current-year dividends and the dividends in arrears on the cumulative preferred stock should be added to the net loss, but the dividends on the noncumulative preferred stock should not be included in the calculation. B. Neither the dividends on the noncumulative preferred stock nor the current-year dividends and the dividends in arrears on cumulative preferred stock should be included in the calculation. C. The dividends on the noncumulative preferred stock and the current-year dividends on the cumulative preferred stock should be added to the net loss. D. The dividends on the noncumulative preferred stock should be added to the net loss, but the current-year dividends and the dividends in arrears on the cumulative preferred stock should not be included in the calculation.

C. The dividends on the noncumulative preferred stock and the current-year dividends on the cumulative preferred stock should be added to the net loss.

Which of the following statements about the cash basis of determining taxable income is true? A. A deduction can be recognized when all the events have occurred to create the liability, and the amount of the liability can be determined with reasonable accuracy. B. An item is included in gross income for the year in which it is earned. C. There is no current deduction for capital expenditures. D. None of the answer choices is a true statement regarding the cash method.

C. There is no current deduction for capital expenditures. Under any basis of accounting for income taxes, expenses are deductible only when paid or accrued. There is no current deduction for capital expenditures. The expense for capital expenditures will be recognized in the form of depreciation, amortization or depletion.

A portfolio of equity securities that are traded on a national exchange is donated to a private, not-for-profit college as an endowment fund. How should the equity portfolio be valued in the college's year-end financial statements three years after the donation? A. Using the fair value at the time of donation B. Using the donor's original cost basis C. Using fair value at the date of the financial statements D. Using the lower of fair value at donation and fair value at the date of the financial statements

C. Using fair value at the date of the financial statements The equity portfolio should be valued in the college's year-end financial statements three years after the donation at fair value at the date of the statements. Donated equity securities with readily determinable market values are reported at market value; the length of the holding period is irrelevant

Pahn, a nongovernmental not-for-profit organization, received an unconditional promise to give $50,000. The donor stipulated that the donation must be used in the next fiscal year. Pahn received and spent the $50,000 in the next year. For the current fiscal year, what element of Pahn's statement of financial position will increase as a result of the unconditional promise to give? A. cash and cash equivalents B. unrestricted support C. contribution receivables D. deferred contributions

C. contribution receivables A promise to give (sometimes referred to as a "pledge," though this term is discouraged by the FASB) is recognized as receivable when it is unconditional or any conditions on the promise are met. Contributions receivables and contributions revenues are recognized in the period that unconditional contributions are made. Unrestricted contributions to be collected in the subsequent period or periods are reported as changes in net assets without donor restrictions.

Birdlovers, a not-for-profit community foundation, incurred $5,000 in management and general expenses in 20X1. In Birdlovers statement of activities for the year ended December 31, 20X1, the $5,000: A. is presented as a program expense. B. decreases net assets with donor restrictions. C. decreases net assets without donor restrictions. D. is presented as a contra account offsetting revenue and gains.

C. decreases net assets without donor restrictions. Management and general expenses are reported in a separate functional classification from program expenses. They are shown as expenses and not as an offset to revenues. All expenses in the statement of activities are classified as changes in net assets without donor restrictions.

In a not-for-profit entity, which of the following should be included in total expenses? A. depreciation B. grants to other organizations C. grants to other organizations and depreciation D. Neither grants to other organizations nor depreciation

C. grants to other organizations and depreciation contributions made by a business are considered expenses of the period. Not-for-profit entities recognize expenses the same way as businesses, so the contribution would be considered an expense with the other expenses of the period. FASB ASC 958-720-45-15 lists depreciation as an expense.

NFPs that allocate joint costs disclose all of the following in the notes to the financial statements except: A. the types of activities for which joint costs have been incurred B. a statement that such costs have been allocated C. the amount of joint costs for each kind of joint activity D. the total amount allocated during the period and the portion allocated to each functional expense category

C. the amount of joint costs for each kind of joint activity This is OPTIONAL

A company is required to file quarterly financial statements with the U.S. Securities and Exchange Commission on Form 10-Q. The company operates in an industry that is not subject to seasonal fluctuations that could have a significant impact on its financial condition. In addition to the most recent quarter-end, for which of the following periods is the company required to present balance sheets on Form 10-Q? A. the end of the preceding fiscal year and the end of the corresponding fiscal quarter of the preceding fiscal year B. the end of the preceding fiscal year and the end of the prior two fiscal years C. the end of the preceding fiscal year D. The end of the corresponding fiscal quarter of the preceding fiscal year

C. the end of the preceding fiscal year Required financial statements include a quarterly and end of the preceding fiscal year balance sheet. If the company is subject to seasonal fluctuations, a balance sheet for the corresponding quarter of the prior fiscal year is required.

Each of the following is a component of the changes in the net assets available for benefits of a defined benefit pension plan trust, except: A. benefits paid to participants B. the net change in fair value of each significant class of investments C. the net change in the actuarial present value of accumulated plan benefits D. contributions from the employer and participants

C. the net change in the actuarial present value of accumulated plan benefits changes in net assets must include: -the change in fair value of each significant type of investment -investment income -contributions from employers/participants -benefits paid to participants -payments to insurance entities to purchase contracts -admin expenses

RST Charities received equities securities valued at $100,000 as an unrestricted gift. During the year, RST received $5,000 in dividends from these securities; at year-end, the securities had a fair market value of $110,000. By what amount did these transactions increase RST's net assets? A. $105,000 B. $100,000 C. $110,000 D. $115,000

D. $115,000 Investments are initially recorded at fair value if received as a contribution or gift. Unrealized gains on investments carried at fair value also increase net assets. As the investments themselves were an unrestricted gift, the unrealized gain would increase net assets without donor restrictions. Investment income includes dividends that increase net assets without donor restrictions unless there are donor stipulations.

At the beginning of the year, Stam Co. had 200,000 shares of common stock issued and outstanding. On March 31, the company issued 40,000 additional shares. On July 1, it declared and distributed a 50% stock dividend and on September 30 repurchased 10,000 shares as treasury stock. What amount of shares should Stam use to calculate basic earnings per share? A. 345,000 B. 287,500 C. 360,000 D. 342,500

D. 342,500 Weighted Avg. Common Shares (200,000 + 100,000 from stock dividend) * 3/12 = 75,000 (240,000 + 120,000 from stock dividend) * 6/12 = 180,000 (360,000 - 10,000) * 3/12 = 87,500 Weighted Avg. = 75,000 + 180,000 + 87,500 = 342,500

The Cats and Dogs League was organized as a nongovernmental not-for-profit organization. The League received a pledge of $10,000 to be used to build an addition to the kennel. This donation will not be received for three years. How should this pledge be recorded? A. As a conditional promise to give of $10,000 B. It should not be accounted for until it is received C. As restricted support of $10,000 D. As restricted support of the present value of $10,000

D. As restricted support of the present value of $10,000 Contribution revenues are reported when unconditionally promised or received at present value, or net realizable value for contributions expected to be received within a year after the promise was made.

To be relevant, information should have which of the following? A. Verifiability B. Understandability C. Costs and benefits D. Confirmatory value

D. Confirmatory value QC6. Relevant financial information is capable of making a difference in the decisions made by users. Information may be capable of making a difference in a decision even if some users choose not to take advantage of it or already are aware of it from other sources. "QC7. Financial information is capable of making a difference in decisions if it has predictive value, confirmatory value, or both. "QC8. Financial information has predictive value if it can be used as an input to processes employed by users to predict future outcomes. Financial information need not be a prediction or forecast to have predictive value. Financial information with predictive value is employed by users in making their own predictions. "QC9. Financial information has confirmatory value if it provides feedback (confirms or changes) about previous evaluations. "QC10. The predictive value and confirmatory value of financial information are interrelated. Information that has predictive value often also has confirmatory value. For example, revenue information for the current year, which can be used as the basis for predicting revenues in future years, also can be compared with revenue predictions for the current year that were made in past years. The results of those comparisons can help a user to correct and improve the processes that were used to make those previous predictions."

The FASB's conceptual framework explains both financial and physical capital maintenance concepts. Which capital maintenance concept is applied to currently reported net income, and which is applied to comprehensive income? A. Financial capital is applied to currently reported net income; physical capital is applied to comprehensive income. B. Physical capital is applied to both currently reported net income and comprehensive income. C. Physical capital is applied to currently reported net income; financial capital is applied to comprehensive income. D. Financial capital is applied to both currently reported net income and comprehensive income.

D. Financial capital is applied to both currently reported net income and comprehensive income. Capital maintenance concept: the recovery of cost; separation of return on capital from return of capital. Financial capital concept: The effects of price changes on assets held and liabilities owed are recognized as "holding gains and losses" and included in return on capital. Physical capital concept: The effect of price changes are recognized as "capital maintenance adjustments" as a separate element of equity and would not be included in return on capital. The financial capital concept is the traditional view and is generally the capital maintenance concept in present primary financial statements. Comprehensive income as defined in paragraph 70 is a return on financial capital." Thus, the financial capital maintenance approach is applied to both currently reported net income and comprehensive income.

Due to a decline in market price in the second quarter, Petal Co. incurred an inventory loss. The market price is expected to return to previous levels by the end of the year. At the end of the year the decline had not reversed. When should the loss be reported in Petal's interim income statements? A. Ratably over the second, third and fourth quarters B. Ratably over the third and fourth quarters C. In the second quarter only D. In the fourth quarter only

D. In the fourth quarter only Temporary market declines expected to reverse are not recognized in interim financial statements. The decline should not be recognized until year-end.

Which of the following is not a special-purpose framework? A. Basis of accounting used by an entity to comply with the financial reporting requirements of a government regulatory agency B. Basis of accounting used by an entity to file its income tax return C. Cash receipts and disbursements basis of accounting D. Modified cash basis of accounting where some fixed assets are capitalized while others are expensed

D. Modified cash basis of accounting where some fixed assets are capitalized while others are expensed For the modified cash basis to be an acceptable form, the modifications must be applied logically and consistently. Capitalizing some fixed assets while expensing others would not represent a consistent or logical application of the framework.

According to the FASB conceptual framework, which of the following is not an essential characteristic of a liability? A. Based on a past event B. Arising from a present obligation C. Probable future sacrifice of economic benefits D. Requires future transfer of assets

D. Requires future transfer of assets SFAC 6, Elements of Financial Statements, defines a liability as a "probable future sacrifice of economic benefits arising from present obligations of a particular entity to transfer assets or provide services to other entities in the future as a result of past transactions or events." Because liabilities can be reduced via providing services, a future transfer of assets is not required.

Which of the following statements is correct concerning the appearance of noncontrolling interest on the income statement? A. None of the answer choices are appropriate disclosure of the noncontrolling interest on the income statement. B. Revenues, expenses, gains, losses, net income or loss, and other comprehensive income are reported in the consolidated financial statements as the amounts attributable to the owners of the parent, followed by a separate disclosure of the revenues, expenses, gains, losses, net income or loss, and other comprehensive income attributable to the noncontrolling interest. C. Revenues, expenses, gains, losses, net income or loss, and other comprehensive income are reported in the consolidated financial statements at the owners' amounts with disclosure of the noncontrolling interest only in the footnotes. D. Revenues, expenses, gains, losses, net income or loss, and other comprehensive income are reported in the consolidated financial statements at the consolidated amounts, which include the amounts attributable to the owners of the parent and the noncontrolling interest.

D. Revenues, expenses, gains, losses, net income or loss, and other comprehensive income are reported in the consolidated financial statements at the consolidated amounts, which include the amounts attributable to the owners of the parent and the noncontrolling interest. consolidated amounts of these items (revenues, expenses, gains, losses, net income or loss, and other comprehensive income) be reported on the income statement. The amount of consolidated net income attributable to the parent and to the noncontrolling interest must be clearly identified and presented on the face of the consolidated statement of income.

Pharm, a nongovernmental not-for-profit entity, is preparing its year-end financial statements. Which of the following statements is required? A. Statement of revenue, expenses, and changes in fund balance B. Statement of changes in fund balance C. Statement of changes in financial position D. Statement of cash flows

D. Statement of cash flows The basic financial statements for a not-for-profit entity are statement of financial position (like a balance sheet), statement of activities, statement of cash flows, and for voluntary health and welfare entities, a statement of functional expenses.

Which of the following adjustments is necessary to convert cash receipts to revenues as reported on an accrual basis? A. Add beginning accounts receivable to cash receipts from customers. B. Subtract ending accounts receivable from cash receipts from customers. C. Subtract beginning contract liability from cash receipts from customers. D. Subtract ending contract liability from cash receipts from customers.

D. Subtract ending contract liability from cash receipts from customers. Contract liability is a deferred revenue account representing cash received for work not yet performed. Therefore, if we subtract the ending liability from cash receipts, we can determine the amount of revenue earned. We would need to add the ending accounts receivable to cash receipts, not subtract it, as accounts receivable represents revenue earned but not yet received in cash. It must be combined with the cash received to determine accrual revenue.

According to the FASB conceptual framework, which of the following is not an essential characteristic of a gain? A. There is an increase in net assets. B. They can result from all other transactions and other events and circumstances affecting an entity. C. They can result from peripheral or incidental transactions. D. They can result from revenues or investment by owners.

D. They can result from revenues or investment by owners. SFAC 6, Elements of Financial Statements, defines gains as "increases in equity (net assets) from peripheral or incidental transactions of an entity and from all other transactions and other events and circumstances affecting the entity during a period except those that result from revenues or investments by owners." Because gains are distinct from revenues, transactions that result in revenues are not gains. Investments by owners are contributions, not gains.

According to the FASB conceptual framework, an entity's revenue may result from: A. a decrease in an asset from primary operations. B. an increase in an asset from incidental transactions. C. an increase in a liability from incidental transactions. D. a decrease in a liability from primary operations.

D. a decrease in a liability from primary operations. SFAC 6, Elements of Financial Statements, notes: "In concept, revenues increase assets rather than decrease liabilities, but a convenient shortcut is often to directly record reduction of liabilities." An example would be the earning of currently unearned revenue. (Unearned revenue would decrease while earned revenue would increase.) Revenues (and expenses) relate to primary operations rather than incidental transactions. Incidental transactions result in gains and losses.

Unrestricted earnings on specific-purpose fund investments that are part of a hospital's central operations are reported as: A. specific-purpose fund restricted revenues B. general fund deferred revenues C. specific-purpose fund unrestricted revenues D. general fund unrestricted revenues

D. general fund unrestricted revenues Unless specifically restricted, earnings on restricted investments are recorded as an increase in net assets without donor restrictions. Restrictions would only apply in this case to the investment but not the income from the investment.

Income tax-basis financial statements differ from those prepared under GAAP in that income tax-basis financial statements: A. do not include nontaxable revenues and nondeductible expenses in determining income. B. contain no disclosures about finance and operating lease transactions. C. include detailed information about current and deferred income tax liabilities. D. recognize certain revenues and expenses in different reporting periods.

D. recognize certain revenues and expenses in different reporting periods. Both income tax-basis and GAAP-basis financial statements recognize all the financial activities of a company's business. However, it is the timing of this recognition that differs between the two methods. For income tax-basis financial statements, taxable revenues and tax-deductible expenses are recognized in the financial statements in the same period they are reported in the tax return. For financial statements prepared in accordance with GAAP, all revenues and expenses are recognized using the accrual method of accounting.

According to the FASB's conceptual framework, the objectives of financial reporting for business enterprises are based on: A. the need for conservatism B. generally accepted accounting principles C. reporting on managements stewardship D. the needs of the users of the information

D. the needs of the users of the information

Accumulated other comprehensive income is reported in which of the following financial statements? A. the income statement B. the statement of comprehensive income C. the statement of cash flows D. the statement of financial position

D. the statement of financial position OCI --> AOCI --> equity on balance sheet/statement of financial position


Related study sets

Pediatric Nurse Practitioner Exam

View Set

English II Frankenstein Chapter 11-15

View Set

Data Analytics C756 / CIW Data Analyst

View Set

HESI 2d semester Fundamental Skills practice questions

View Set

BISC 102 Take Home Exam Questions - Johnson Ole Miss

View Set

Physical Science - Chapter 3: Forces

View Set